Vous êtes sur la page 1sur 317

Vasile Cîrtoaje

C j

L
MATHEMMATICAL

.M
INEQUA
Q ALITIES

D
A
Volume 1
PI
M
LY

SYMM
METRIC
O

POLYNOMIAL INEQUALITIES
POLYNOMIAL
.M
W
W
W

UNIVERSITY OF PLOIESTI, ROMANIA


2015
W
W
W
.M
O
LY
M
PI
A
D
.M
L
About the author

“The simpler and sharper, the more beautiful.”

L
Vasile Cîrtoaje

D
.M
Vasile Cîrtoaje is a Professor at the Department of Automatic Control and Computers

A
from Petroleum-Gas University of Ploiesti, Romania, where he teaches university courses
such as Control System Theory and Digital Control Systems.
PI
Since 1970, he published many mathematical problems, solutions and articles in the
M
Romanian journals Gazeta Matematica-B, Gazeta Matematica-A and Mathematical Re-
view of Timisoara. In addition, from 2000 to present, Vasile Cîrtoaje has published
LY

many interesting problems and articles in Art of Problem Solving website, Mathemati-
cal Reflections, Crux with Mayhem, Journal of Inequalities and Applications, Journal
O

of Inequalities in Pure and Applied Mathematics, Mathematical Inequalities and Ap-


plications, Banach Journal of Mathematical Analysis, Journal of Nonlinear Science and
.M

Applications, Journal of Nonlinear Analysis and Application, Australian Journal of Math-


ematical Analysis and Applications, British Journal of Mathematical and Computer Sci-
ence,International Journal of Pure and Applied Mathematics, A.M.M.
W

He collaborated with Titu Andreescu, Gabriel Dospinescu and Mircea Lascu in writing
W

the book Old and New Inequalities, with Vo Quoc Ba Can and Tran Quoc Anh in writ-
ing the book Inequalities with Beautiful Solution, and he wrote his own books Algebraic
W

Inequalities - Old and New Methods and Mathematical Inequalities (Volume 1 ... 4).
Notice that Vasile Cîrtoaje is the author of some well-known results and strong meth-
ods for proving and creating discrete inequalities, such as:
- Half convex function method (HCF method) for Jensen type discrete inequalities;
- Partial convex function method (PCF method) for Jensen type discrete inequalities;
- Jensen type discrete inequalities with ordered variables;
- Equal variable method (EV method) for real or nonnegative variables;
- Arithmetic compensation method (AC method);
- Best lower and upper bounds for Jensen’s inequality;
- Necessary and sufficient conditions for symmetric homogeneous polynomial inequal-
ities of degree six in real variables;

i
ABOUT THE AUTHOR

- Necessary and sufficient conditions for symmetric homogeneous polynomial inequal-


ities of degree six in nonnegative variables;
- Highest coefficient cancellation method (HCC method) for symmetric homogeneous
polynomial inequalities of degree six and eight in real variables;
- Highest coefficient cancellation method for symmetric homogeneous polynomial in-
equalities of degree six, seven and eight in nonnegative variables;
- Necessary and sufficient conditions for cyclic homogeneous polynomial inequalities
of degree four in real variables;
- Necessary and sufficient conditions for cyclic homogeneous polynomial inequalities
of degree four in nonnegative variables;
- Strong sufficient conditions for cyclic homogeneous polynomial inequalities of de-

L
gree four in real or nonnegative variables;

.M
- Inequalities with power-exponential functions.

D
A
PI
M
LY
O
.M
W
W
W
Foreword

The author, Vasile Cîrtoaje, professor at University of Ploiesti-Romania, has become

L
well-known for his excellent creations in the mathematical inequality field, ever since

.M
the time when he was student in high school (in Breaza city, Prahova Valley). As a
student (quite some time ago, oh yes!), I was already familiar with the name of Vasile

D
Cîrtoaje. For me, and many others of my age, it is the name of someone who helped
me to grow in mathematics, even though I never met him face to face. It is a name

A
synonymous to hard and beautiful problems involving inequalities. When you say Vasile
PI
Cîrtoaje (Vasc username on the site Art of Problem Solving), you say inequalities. I
remember how happy I was when I could manage to solve one of the problems proposed
M
by professor Cîrtoaje in Gazeta Matematica or Revista Matematica Timisoara.
The first three volumes of this book are a great opportunity to see and know many
LY

old and new elementary methods for solving mathematical inequalities: Volume 1 -
Symmetric polynomial inequalities (in real variables and nonnegative real variables),
O

Volume 2 - Symmetric rational and nonrational inequalities, Volume 3 - Cyclic and non-
.M

cyclic inequalities. As a rule, the inequalities from each section of these volumes are in-
creasingly ordered by the number of variables: two, three, four, five, six and n-variables.
The last volume (Volume 4) contains new beautiful and efficient original methods for
W

creating and solving inequalities: half or partially convex function method - for Jensen’s
type inequalities, equal variable method - for nonnegative or real variables, arithmetic
W

compensation method - for symmetric inequalities, the highest coefficient cancellation


method - for symmetric homogeneous polynomial inequalities of degree six, seven and
W

eight in nonnegative or real variables, methods involving either strong or necessary and
sufficient conditions - for cyclic homogeneous inequalities of degree four in real or non-
negative variables and so on.
Many problems, the majority I would say, are made up by the author himself. The
chapters and volumes are relatively independent, and you can open the book somewhere
to solve an inequality or only read its solution. If you carefully make a thorough study
of the book, then you will find that your skills in solving inequalities are considerably
improved.
The book is a rich and meaningful collection of more than 1000 beautiful inequalities,
hints, solutions and methods, some of them being posted in the last ten years by the
author and other inventive mathematicians on Art of Problem Solving website (Vo Quoc

iii
FOREWORD

Ba Can, Pham Kim Hung, Michael Rozenberg, Nguyen Van Quy, Gabriel Dospinescu,
Darij Grinberg, Pham Huu Duc, Tran Quoc Anh, Le Huu Dien Khue, Marius Stanean,
Cezar Lupu, Nguyen Anh Tuan, Pham Van Thuan, Bin Zhao, Ji Chen etc.)
Most inequalities and methods are old and recent own creations of the author. Among
these, I would like to point out the following inequalities:

(a2 + b2 + c 2 )2 ≥ 3(a3 b + b3 c + c 3 a), a, b, c ∈ R;


X
(a − k b)(a − kc)(a − b)(a − c) ≥ 0, a, b, c, k ∈ R;
 a 2  b ‹2  c 2  d ‹2
+ + + ≥ 1, a, b, c, d ≥ 0;
a+b b+c c+d d+a

L
.M
4
X 1
≥ 1, a1 , a2 , a3 , a4 > 0, a1 a2 a3 a4 = 1;
i=1
1 + ai + ai2 + ai3

D
a1 a2 an
+ + ··· + ≥ 1, a1 , a2 , · · · , an ≥ 0;

A
a1 + (n − 1)a2 a2 + (n − 1)a3 an + (n − 1)a1
a ea + b e b ≥ a e b + b ea , a, b > 0,PI e ≈ 2.7182818.
M
a3b + b3a ≤ 2, a, b ≥ 0, a + b = 2.
LY

The book represents a rich source of beautiful, serious and profound mathematics,
dealing with classical and new approaches and techniques which help the reader to de-
velop his inequality-solving skills, intuition and creativity. As a result, it is suitable for
O

a wide variety of audiences; high school students and teachers, college and university
.M

students, mathematics educators and mathematicians will find something of interest


here. Each problem has a hint, and many problems have multiple solutions, almost all
of which are, not surprisingly, quite ingenious. Almost all inequalities require careful
W

thought and analysis, making the book a rich and rewarding source for anyone inter-
ested in Olympiad-type problems and in the development of the inequality field. Many
W

problems and methods can be used in a classroom for advanced high school students as
group projects.
W

What makes this book so attractive? The answer is simple: the great number of in-
equalities, their quality and freshness, as well as the new approaches and methods for
solving mathematical inequalities. Nevertheless, you will find this book to be delight-
ful, inspired, original and enjoyable. Certainly, any interested reader will remark the
tenacity, enthusiasm and ability of the author in creating and solving nice and difficult
inequalities. And this book is neither more, nor less than a Work of a Master.
The author must be congratulated for publishing such interesting and original in-
equality book. I highly recommend it.

Marian Tetiva
Contents

L
About the author i

.M
Foreword iii

D
1 Some Classic and New Inequalities and Methods 1

A
2 Symmetric Polynomial Inequalities in Real Variables 19
PI
2.1 Applications . . . . . . . . . . . . . . . . . . . . . . . . . . . . . . . . . . . . . . 19
2.2 Solutions . . . . . . . . . . . . . . . . . . . . . . . . . . . . . . . . . . . . . . . . 35
M
3 Symmetric Polynomial Inequalities in Nonnegative Variables 149
LY

3.1 Applications . . . . . . . . . . . . . . . . . . . . . . . . . . . . . . . . . . . . . . 149


3.2 Solutions . . . . . . . . . . . . . . . . . . . . . . . . . . . . . . . . . . . . . . . . 169
O

4 Bibliography 309
.M
W
W
W

v
vi Vasile Cîrtoaje

L
D
.M
A
PI
M
LY
O
.M
W
W
W
Chapter 1

Some Classic and New Inequalities

L
and Methods

D
.M
A
1. AM-GM (ARITHMETIC MEAN-GEOMETRIC MEAN) INEQUALITY
If a1 , a2 , · · · , an are nonnegative real numbers, then PI
M
a1 + a2 + · · · + an ≥ n n a1 a2 · · · an ,
p
LY

with equality if and only if a1 = a2 = · · · = an .


O

2. WEIGHTED AM-GM INEQUALITY


.M

Let p1 , p2 , · · · , pn be positive real numbers satisfying

p1 + p2 + · · · + pn = 1.
W
W

If a1 , a2 , · · · , an are nonnegative real numbers, then


p p
W

p1 a1 + p2 a2 + · · · + pn an ≥ a1 1 a2 2 · · · anpn ,

with equality if and only if a1 = a2 = · · · = an .

3. AM-HM (ARITHMETIC MEAN-HARMONIC MEAN) INEQUALITY


If a1 , a2 , · · · , an are positive real numbers, then

1 1 1
 ‹
(a1 + a2 + · · · + an ) + + ··· + ≥ n2 ,
a1 a2 an

with equality if and only if a1 = a2 = · · · = an .

1
2 Vasile Cîrtoaje

4. POWER MEAN INEQUALITY


The power mean of order k of positive real numbers a1 , a2 , · · · , an , that is

1
a1 +a2 +···+ank k
 k k
, k 6= 0


n

Mk = p ,
 a1 a2 · · · an ,

 n
k=0

is an increasing function with respect to k ∈ R. For instant, M2 ≥ M1 ≥ M0 ≥ M−1 is


equivalent to

L
v

.M
t a1 + a22 + · · · + an2
u 2
a1 + a2 + · · · + an p n
≥ ≥ n a1 a2 · · · an ≥ .
n n 1 1 1
+ + ··· +

D
a1 a2 an

A
5. BERNOULLI’S INEQUALITY PI
For any real number x ≥ −1, we have
M
a) (1 + x) r ≥ 1 + r x for r ≥ 1 and r ≤ 0;
b) (1 + x) r ≤ 1 + r x for 0 ≤ r ≤ 1.
LY

In addition, if a1 , a2 , · · · , an are real numbers such that either a1 , a2 , · · · , an ≥ 0 or −1 ≤


a1 , a2 , · · · , an ≤ 0, then
O

(1 + a1 )(1 + a2 ) · · · (1 + an ) ≥ 1 + a1 + a2 + · · · + an .
.M
W

6. SCHUR’S INEQUALITY
W

For any nonnegative real numbers a, b, c and any positive number k, the inequality
holds
W

a k (a − b)(a − c) + b k (b − c)(b − a) + c k (c − a)(c − b) ≥ 0,

with equality for a = b = c, and for a = 0 and b = c (or any cyclic permutation).
For k = 1, we get the third degree Schur’s inequality, which can be rewritten as follows

a3 + b3 + c 3 + 3a bc ≥ a b(a + b) + bc(b + c) + ca(c + a),

(a + b + c)3 + 9a bc ≥ 4(a + b + c)(a b + bc + ca),


9a bc
a2 + b2 + c 2 + ≥ 2(a b + bc + ca),
a+b+c
(b − c)2 (b + c − a) + (c − a)2 (c + a − b) + (a − b)2 (a + b − c) ≥ 0.
Some Classic and New Inequalities and Methods 3

For k = 2, we get the fourth degree Schur’s inequality, which holds for any real numbers
a, b, c, and can be rewritten as follows

a4 + b4 + c 4 + a bc(a + b + c) ≥ a b(a2 + b2 ) + bc(b2 + c 2 ) + ca(c 2 + a2 ),

(b − c)2 (b + c − a)2 + (c − a)2 (c + a − b)2 + (a − b)2 (a + b − c)2 ≥ 0,


6a bc p ≥ (p2 − q)(4q − p2 ),
where p = a + b + c, q = a b + bc + ca.
A generalization of the fourth degree Schur’s inequality, which holds for any real
numbers a, b, c and any real number m, is the following (Vasile Cirtoaje, 2004)

L
X

.M
(a − mb)(a − mc)(a − b)(a − c) ≥ 0,

where the equality holds for a = b = c, and for a/m = b = c (or any cyclic permutation).

D
This inequality is equivalent to

A
X X X X
a4 + m(m + 2) a2 b2 + (1 − m2 )a bc a ≥ (m + 1) a b(a2 + b2 )
PI
and
M
X
(b − c)2 (b + c − a − ma)2 ≥ 0.
LY

A more general result is given by the following theorem (Vasile Cirtoaje, 2004).
Theorem. Let
O

X X X X
f4 (a, b, c) = a4 + α a2 b2 + β a bc a−γ a b(a2 + b2 ),
.M

where α, β, γ are real constants such that


W

1 + α + β = 2γ.
W

(a) f4 (a, b, c) ≥ 0 for all real numbers a, b, c if and only if


W

1 + α ≥ γ2 ;

(b) f4 (a, b, c) ≥ 0 for all a, b, c ≥ 0 if and only if

α ≥ (γ − 1) max{2, γ + 1}.

7. CAUCHY-SCHWARZ INEQUALITY

For any real numbers a1 , a2 , · · · , an and b1 , b2 , · · · , bn we have

(a12 + a22 + · · · + an2 )(b12 + b22 + · · · + bn2 ) ≥ (a1 b1 + a2 b2 + · · · + an bn )2 ,


4 Vasile Cîrtoaje

with equality if and only if ai and bi are proportional for all i.

8. HÖLDER’S INEQUALITY
If x i j (i = 1, 2, · · · , m; j = 1, 2, · · · n) are nonnegative real numbers, then
! v !m
m
Y n
X n uY
X m
m
xi j ≥ .
t
xi j
i=1 j=1 j=1 i=1

9. CHEBYSHEV’S INEQUALITY

L
Let a1 ≥ a2 ≥ · · · ≥ an be real numbers.

.M
a) If b1 ≥ b2 ≥ · · · bn , then

D
n n n
‚ Œ‚ Œ

A
X X X
n ai bi ≥ ai bi ;

b) If b1 ≤ b2 ≤ · · · ≤ bn , then
i=1 i=1
PI
i=1
M
n n n
‚ Œ‚ Œ
LY

X X X
n ai bi ≤ ai bi .
i=1 i=1 i=1
O

10. MINKOWSKI’S INEQUALITY


.M

For any real number k ≥ 1 and any positive real numbers a1 , a2 , · · · , an and b1 , b2 , · · · , bn ,
the following inequalities hold:
W

 1
Œk Œk k
n n n
‚ ‚
W

X 1 X X
aik + bi k k
≥ ai + bi  ;
W

i=1 i=1 i=1

 1
Œk Œk Œk k
n n n n
‚ ‚ ‚
X  1 X X X
aik + bik + ci
k k
≥ ai + bi + ci  .
i=1 i=1 i=1 i=1

11. REARRANGEMENT INEQUALITY


(1) If a1 , a2 , · · · , an and b1 , b2 , · · · , bn are two increasing (or decreasing) real se-
quences, and (i1 , i2 , · · · , in ) is an arbitrary permutation of (1, 2, · · · , n), then

a1 b1 + a2 b2 + · · · + an bn ≥ a1 bi1 + a2 bi2 + · · · + an bin


Some Classic and New Inequalities and Methods 5

and
n(a1 b1 + a2 b2 + · · · + an bn ) ≥ (a1 + a2 + · · · + an )(b1 + b2 + · · · + bn ).
(2) If a1 , a2 , · · · , an is decreasing and b1 , b2 , · · · , bn is increasing, then

a1 b1 + a2 b2 + · · · + an bn ≤ a1 bi1 + a2 bi2 + · · · + an bin

and
n(a1 b1 + a2 b2 + · · · + an bn ) ≤ (a1 + a2 + · · · + an )(b1 + b2 + · · · + bn ).
(3) Let b1 , b2 , · · · , bn and c1 , c2 , · · · , cn be two real sequences such that

b1 + · · · + bk ≥ c1 + · · · + ck , k = 1, 2, · · · , n.

L
.M
If a1 ≥ a2 ≥ · · · ≥ an ≥ 0, then

a1 b1 + a2 b2 + · · · + an bn ≥ a1 c1 + a2 c2 + · · · + an cn .

D
A
Notice that all these inequalities follow immediately from the identity

X n n
X i
X i
X
!PI
ai (bi − ci ) = (ai − ai+1 ) bj − cj ,
M
i=1 i=1 j=1 j=1
LY

where an+1 = 0.
O

12. MACLAURIN’S INEQUALITY and NEWTON’S INEQUALITY


.M

If a1 , a2 , . . . , an are nonnegative real numbers, then

S1 ≥ S2 ≥ · · · ≥ S n (M acl aur in)


W

and
W

Sk2 ≥ Sk−1 Sk+1 , (N ewt on)


where
W

v X
u
u
u a i1 a i2 · · · a i k
k 1≤i1 <···<ik ≤n
Sk = u
u
 ‹ .
t n
k

13. CONVEX FUNCTIONS


A function f defined on a real interval I is said to be convex if

f (αx + β y) ≤ α f (x) + β f ( y)
6 Vasile Cîrtoaje

for all x, y ∈ I and any α, β ≥ 0 with α + β = 1. If the inequality is reversed, then f is


said to be concave.
If f is differentiable on I, then f is (strictly) convex if and only if the derivative f 0 is
(strictly) increasing. If f 00 ≥ 0 on I, then f is convex on I. Also, if f 00 ≥ 0 on (a, b) and
f is continuous on [a, b], then f is convex on [a, b].
A function f : I → R is half convex on a real interval I if there exists a point s ∈ I such
that f is convex on Iu≤s or Iu≥s .
A function f : I → R is right partially convex related to a a point s ∈ I if there exists a
number s0 ∈ I, s0 > s, such that f is convex on Iu∈[s,s0 ] . Also, a function f : I → R is left
partially convex related to a point s ∈ I if there exists a point s0 ∈ I, s0 < s, such that f
is convex on Iu∈[s0 ,s] .

L
.M
Jensen’s inequality. Let p1 , p2 , . . . , pn be positive real numbers. If f is a convex
function on a real interval I, then for any a1 , a2 , . . . , an ∈ I, the inequality holds

D
p1 f (a1 ) + p2 f (a2 ) + · · · + pn f (an ) p1 a1 + p2 a2 + · · · + pn an
 ‹
≥f .

A
p1 + p2 + · · · + p n p1 + p2 + · · · + p n
For p1 = p2 = · · · = pn , Jensen’s inequality becomes PI
a + a + ··· + a 
M
1 2 n
f (a1 ) + f (a2 ) + · · · + f (an ) ≥ n f .
n
LY

Based on the following three theorems, we can extend this form of Jensen’s in-
equality to half or partially convex functions.
O

Half Convex Function-Theorem (Vasile Cirtoaje, 2004). Let f (u) be a function


.M

defined on a real interval I and convex on Iu≥s or Iu≤s , where s ∈ I. The inequality
a + a + ··· + a 
1 2 n
f (a1 ) + f (a2 ) + · · · + f (an ) ≥ n f
W

n
holds for all a1 , a2 , . . . , an ∈ I satisfying a1 + a2 + . . . + an = ns if and only if
W

f (x) + (n − 1) f ( y) ≥ n f (s)
W

for all x, y ∈ I such that x + (n − 1) y = ns.

Half Convex Function Theorem for Ordered Variables (Vasile Cirtoaje, 2007). Let
f (u) be a function defined on a real interval I and convex on Iu≥s / Iu≤s , where s ∈ I. The
inequality a + a + ··· + a 
1 2 n
f (a1 ) + f (a2 ) + · · · + f (an ) ≥ n f
n
holds for all a1 , a2 , . . . , an ∈ I such that a1 + a2 + · · · + an = ns and at least n − m of
a1 , a2 , . . . , an are smaller/greater than or equal to s if and only if

f (x) + m f ( y) ≥ (1 + m) f (s)
Some Classic and New Inequalities and Methods 7

for all x, y ∈ I such that x + m y = (1 + m)s.

Right Partially Convex Function-Theorem (Vasile Cirtoaje, 2012). Let f be a func-


tion defined on a real interval I and convex on [s, s0 ], where s, s0 ∈ I, s < s0 . In addition,
f is decreasing on Iu≤s0 and
min f (u) = f (s0 ).
u∈I

The inequality
a + a + ··· + a 
1 2 n
f (a1 ) + f (a2 ) + · · · + f (an ) ≥ n f
n

L
holds for all a1 , a2 , · · · , an ∈ I satisfying a1 + a2 + · · · + an = ns if and only if

.M
f (x) + (n − 1) f ( y) ≥ n f (s)

D
for all x, y ∈ I such that x ≤ s ≤ y and x + (n − 1) y = ns.

A
PI
Left Partially Convex Function-Theorem (Vasile Cirtoaje, 2012). Let f be a func-
tion defined on a real interval I and convex on [s0 , s], where s0 , s ∈ I, s0 < s. In addition,
M
f is increasing on Iu≥s0 and satisfies
LY

min f (u) = f (s0 ).


u∈I
O

The inequality
.M

a + a + ··· + a 
1 2 n
f (a1 ) + f (a2 ) + · · · + f (an ) ≥ n f
n
W

holds for all x 1 , x 2 , · · · , x n ∈ I satisfying a1 + a2 + · · · + an = ns if and only if


W

f (x) + (n − 1) f ( y) ≥ n f (s)
W

for all x, y ∈ I such that x ≥ s ≥ y and x + (n − 1) y = ns.

Right Partially Convex Function Theorem for Ordered Variables (Vasile Cirtoaje,
2012). Let f be a function defined on a real interval I and convex on [s, s0 ], where s, s0 ∈ I,
s < s0 . In addition, f is decreasing on Iu≤s0 and

min f (u) = f (s0 ).


u∈I

The inequality
a + a + ··· + a 
1 2 n
f (a1 ) + f (a2 ) + · · · + f (an ) ≥ n f
n
8 Vasile Cîrtoaje

holds for all a1 , a2 , · · · , an ∈ I such that a1 + a2 + · · · + an = ns and at least n − m of


a1 , a2 , . . . , an are smaller than or equal to s if and only if

f (x) + m f ( y) ≥ (1 + m) f (s)

for all x, y ∈ I such that x ≤ s ≤ y and x + m y = (1 + m)s.

Left Partially Convex Function Theorem for Ordered Variables (Vasile Cirtoaje,
2012). Let f be a function defined on a real interval I and convex on [s0 , s], where s0 , s ∈ I,
s0 < s. In addition, f is increasing on Iu≥s0 and satisfies

min f (u) = f (s0 ).

L
u∈I

.M
The inequality
a + a + ··· + a 

D
1 2 n
f (a1 ) + f (a2 ) + · · · + f (an ) ≥ n f
n

A
holds for all x 1 , x 2 , · · · , x n ∈ I such that a1 + a2 + · · · + an = ns and at least n − m of
PI
a1 , a2 , . . . , an are greater than or equal to s if and only if if and only if
M
f (x) + m f ( y) ≥ (1 + m)n f (s)
LY

for all x, y ∈ I such that x ≥ s ≥ y and x + m y = (1 + m)s.


O

In all these theorems, we may replace the hypothesis condition


.M

f (x) + m f ( y) ≥ (1 + m) f (s),

by the equivalent condition


W

h(x, y) ≥ 0 for all x, y ∈ I such that x + m y = (1 + m),


W

where
g(x) − g( y) f (u) − f (s)
h(x, y) = g(u) =
W

, .
x−y u−s
The following theorem is also useful to prove some symmetric inequalities.
Left Convex-Right Concave Function Theorem (Vasile Cirtoaje, 2004). Let a < c
be real numbers, let f be a continuous function on I = [a, ∞), strictly convex on [a, c]
and strictly concave on [c, ∞), and let

E(a1 , a2 , . . . , an ) = f (a1 ) + f (a2 ) + · · · + f (an ).

If a1 , a2 , . . . , an ∈ I such that

a1 + a2 + · · · + an = S = const ant,
Some Classic and New Inequalities and Methods 9

then
(a) E is minimum for a1 = a2 = · · · = an−1 ≤ an ;
(b) E is maximum for either a1 = a or a < a1 ≤ a2 = · · · = an .
On the other hand, it is known the following result concerning the best upper bound
of Jensen’s difference.
Best Upper Bound of Jensen’s Difference-Theorem (Vasile Cirtoaje, 1989). Let
p1 , p2 , . . . , pn be fixed positive real numbers, and let f be a convex function on a closed
interval I = [a, b]. If a1 , a2 , · · · , an ∈ I, then Jensen’s difference
p1 f (a1 ) + p2 f (a2 ) + · · · + pn f (an ) p1 a1 + p2 a2 + · · · + pn an
 ‹
D= −f
p1 + p2 + · · · + p n p1 + p2 + · · · + pn

L
.M
is maximum when some of ai are equal to a, and the others ai are equal to b; that is, when
all ai ∈ {a, b}.

D
14. KARAMATA’S MAJORIZATION INEQUALITY

A


We say that a vector A = (a1 , a2 , . . . , an ) with a1 ≥ a2 ≥ · · · ≥ an majorizes a vector

→ PI
B = (b1 , b2 , . . . , bn ) with b1 ≥ b2 ≥ · · · ≥ bn , and write it as
M

→ − →
A ≥ B,
LY

if
a 1 ≥ b1 ,
O

a1 + a2 ≥ b1 + b2 ,
·····················
.M

a1 + a2 + · · · + an−1 ≥ b1 + b2 + · · · + bn−1 ,
a1 + a2 + · · · + an = b1 + b2 + · · · + bn .
W

Let f be a convex function on a real interval I. If a decreasingly ordered vector




W

A = (a1 , a2 , . . . , an ), ai ∈ I,
W

majorizes a decreasingly ordered vector




B = (b1 , b2 , . . . , bn ), bi ∈ I,

then
f (a1 ) + f (a2 ) + · · · + f (an ) ≥ f (b1 ) + f (b2 ) + · · · + f (bn ).

15. POPOVICIU’S INEQUALITY


If f is a convex function on a real interval I and a1 , a2 , . . . , an ∈ I, then
a + a + ··· + a 
1 2 n
f (a1 ) + f (a2 ) + · · · + f (an ) + n(n − 2) f ≥
n
10 Vasile Cîrtoaje

≥ (n − 1)[ f (b1 ) + f (b2 ) + · · · + f (bn )],


where
1 X
bi = aj, i = 1, 2, · · · , n.
n − 1 j6=i

16. SQUARE PRODUCT INEQUALITY


Let a, b, c be real numbers, and let

p = a + b + c, q = a b + bc + ca, r = a bc,

L
Æ p
s = p2 − 3q = a2 + b2 + c 2 − a b − bc − ca.

.M
From the identity

D
27(a − b)2 (b − c)2 (c − a)2 = 4(p2 − 3q)3 − (2p3 − 9pq + 27r)2 ,

A
it follows that

−2p3 + 9pq − 2(p2 − 3q) p2 − 3q


p PI
−2p3 + 9pq + 2(p2 − 3q) p2 − 3q
p
≤r≤
M
,
27 27
LY

which is equivalent to

p3 − 3ps2 − 2s3 p3 − 3ps2 + 2s3


O

≤r≤ .
27 27
.M

Therefore, for constant p and q, the product r is minimal and maximal when two of
a, b, c are equal.
W

17. SYMMETRIC INEQUALITIES OF DEGREE THREE, FOUR OR FIVE


W

Let f n (a, b, c) be a symmetric homogeneous polynomial of degree n = 3, n = 4 or n = 5.


W

Theorem.
(a) The inequality f4 (a, b, c) ≥ 0 holds for all real numbers a, b, c if and only if
f4 (a, 1, 1) ≥ 0 for all real a;
(b) The inequality f n (a, b, c) ≥ 0 holds for all a, b, c ≥ 0 if and only if f n (a, 1, 1) ≥ 0
and f n (0, b, c) ≥ 0 for all a, b, c ≥ 0.

18. SYMMETRIC INEQUALITIES OF DEGREE SIX

Any sixth degree symmetric homogeneous polynomial f6 (a, b, c) can be written in the
form
f6 (a, b, c) = Ar 2 + B(p, q)r + C(p, q),
Some Classic and New Inequalities and Methods 11

where A is called the highest coefficient of f6 , and

p = a + b + c, q = a b + bc + ca, r = a bc.

Theorem (Vasile Cirtoaje, 2008). Let A ≤ 0.


(a) The inequality f6 (a, b, c) ≥ 0 holds for all real numbers a, b, c if and only if
f6 (a, 1, 1) ≥ 0 for all real a;
(b) The inequality f6 (a, b, c) ≥ 0 holds for all a, b, c ≥ 0 if and only if f6 (a, 1, 1) ≥ 0
and f6 (0, b, c) ≥ 0 for all a, b, c ≥ 0.
For A > 0, we can use the highest coefficient cancellation method (Vasile Cirtoaje,

L
2008). This method consists in finding some suitable real numbers B, C and D such
that the following sharper inequality holds

.M
2
q2

3

D
f6 (a, b, c) ≥ A r + Bp + C pq + D .
p

A
Because the function g6 defined by
PI 2
q2

M
3
g6 (a, b, c) = f6 (a, b, c) − A r + Bp + C pq + D
p
LY

has the highest coefficient A1 = 0, we can prove the inequality g6 (a, b, c) ≥ 0 using
O

Theorem above.
Notice that sometimes it is useful to break the problem into two parts, p2 ≤ ξq and
.M

p2 > ξq, where ξ is a suitable real number.


W

19. EQUAL VARIABLE METHOD


W

The Equal Variable Theorem (EV-Theorem) for nonnegative real variables has the fol-
lowing statement (Vasile Cirtoaje, 2005).
W

EV-Theorem (for nonnegative variables). Let a1 , a2 , · · · , an (n ≥ 3) be fixed non-


negative real numbers, and let x 1 ≤ x 2 ≤ · · · ≤ x n be nonnegative real variables such
that
x 1 + x 2 + · · · + x n = a1 + a2 + · · · + an ,

x 1k + x 2k + · · · + x nk = a1k + a2k + · · · + ank ,

where k is a real number; for k = 0, assume that x 1 x 2 · · · x n = a1 a2 · · · an > 0. Let


f : (0, ∞) → R be a differentiable function such that g : (0, ∞) → R defined by
€ 1 Š
g(x) = f 0 x k−1
12 Vasile Cîrtoaje

is strictly convex, and let

Sn = f (x 1 ) + f (x 2 ) + · · · + f (x n ).

(1) If k ≤ 0, then Sn is maximum for

x 1 = x 2 = · · · = x n−1 ≤ x n ,

and is minimum for


0 < x1 ≤ x2 = x3 = · · · = x n;
(2) If k > 0 and either f is continuous at x = 0 or f (0+ ) = −∞, then Sn is maximum

L
for
x 1 = x 2 = · · · = x n−1 ≤ x n ;

.M
(3) If k > 0 and either f is continuous at x = 0 or f (0+ ) = ∞, then Sn is minimum

D
for
x 1 = · · · = x j−1 = 0, x j+1 = · · · = x n , j ∈ {1, 2, . . . , n}.

A
For f (x) = x m , we get the following corollary.
PI
M
EV-COROLLARY (for nonnegative variables). Let a1 , a2 , · · · , an (n ≥ 3) be fixed
LY

nonnegative real numbers, let x 1 ≤ x 2 ≤ · · · ≤ x n be nonnegative real variables such that

x 1 + x 2 + · · · + x n = a1 + a2 + · · · + an ,
O

x 1k + x 2k + · · · + x nk = a1k + a2k + · · · + ank ,


.M

and let
Sn = x 1m + x 2m + · · · + x nm .
W

Case 1 : k ≤ 0 (for k = 0, assume that x 1 x 2 · · · x n = a1 a2 · · · an > 0 ).


W

(a) If m ∈ (k, 0) ∪ (1, ∞), then Sn is maximum for


W

x 1 = x 2 = · · · = x n−1 ≤ x n ,

and is minimum for


x1 ≤ x2 = x3 = · · · = x n;
(b) If m ∈ (−∞, k) ∪ (0, 1), then Sn is minimum for

x 1 = x 2 = · · · = x n−1 ≤ x n ,

and is maximum for


x1 ≤ x2 = x3 = · · · = x n.
Case 2 : 0 < k < 1.
Some Classic and New Inequalities and Methods 13

(a) If m ∈ (0, k) ∪ (1, ∞), then Sn is maximum for

x 1 = x 2 = · · · = x n−1 ≤ x n ,

and is minimum for

x 1 = · · · = x j−1 = 0, x j+1 = · · · = x n , j ∈ {1, 2, . . . , n};

(b) If m ∈ (−∞, 0) ∪ (k, 1), then Sn is minimum for

x 1 = x 2 = · · · = x n−1 ≤ x n ;

L
(c) If m ∈ (k, 1), then Sn is maximum for

.M
x 1 = · · · = x j−1 = 0, x j+1 = · · · = x n , j ∈ {1, 2, . . . , n}.

D
Case 3 : k > 1.

A
(a) If m ∈ (0, 1) ∪ (k, ∞), then Sn is maximum for
PI
x 1 = x 2 = · · · = x n−1 ≤ x n ,
M
and is minimum for
LY

x 1 = · · · = x j−1 = 0, x j+1 = · · · = x n , j ∈ {1, 2, . . . , n};


O

(b) If m ∈ (−∞, 0) ∪ (1, k), then Sn is minimum for


.M

0 ≤ x 1 = x 2 = · · · = x n−1 ≤ x n ;

(c) If m ∈ (1, k), then Sn is maximum for


W

x 1 = · · · = x j−1 = 0, x j+1 = · · · = x n ,
W

j ∈ {1, 2, . . . , n}.
W

The Equal Variable Theorem (EV-Theorem) for real variables has the following
statement (Vasile Cirtoaje, 2012).
EV-Theorem (for real variables). Let a1 , a2 , · · · , an (n ≥ 3) be fixed real numbers,
let x 1 ≤ x 2 ≤ · · · ≤ x n be real variables such that

x 1 + x 2 + · · · + x n = a1 + a2 + · · · + an ,

x 1k + x 2k + · · · + x nk = a1k + a2k + · · · + ank ,


where k is an even positive integer, and let f be a differentiable function on R such that the
associated function g : R → R defined by
14 Vasile Cîrtoaje

p 
g(x) = f 0
k−1
x
is strictly convex on R. Then, the sum

Sn = f (x 1 ) + f (x 2 ) + · · · + f (x n )

is minimum for x 2 = x 3 = · · · = x n , and is maximum for x 1 = x 2 = · · · = x n−1 .

20. ARITHMETIC COMPENSATION METHOD


The Arithmetic Compensation Theorem (AC-Theorem) has the following statement (Vasile

L
Cirtoaje, 2002).

.M
AC-THEOREM. Let s > 0 and let F be a symmetric continuous function on the compact
set in Rn

D
A
S = {(x 1 , x 2 , · · · , x n ) : x 1 + x 2 + · · · + x n = s, x i ≥ 0, i = 1, 2, · · · , n}.

If PI
F (x 1 , x 2 , x 3 , · · · , x n ) ≥
M
n x + x x + x  o
1 2 1 2
≥ min F , , x 3 , · · · , x n , F (0, x 1 + x 2 , x 3 , · · · , x n )
LY

2 2
for all (x 1 , x 2 , · · · , x n ) ∈ S, then F (x 1 , x 2 , x 3 , · · · , x n ) is minimal when
O

s
x1 = x2 = · · · = x k = , x k+1 = · · · = x n = 0;
.M

k
that is, s s 
F (x 1 , x 2 , x 3 , · · · , x n ) ≥ min F
W

, · · · , , 0, · · · , 0
1≤k≤n k k
for all (x 1 , x 2 , · · · , x n ) ∈ S.
W
W

Notice that if
x + x x + x 
1 2 1 2
F (x 1 , x 2 , x 3 , · · · , x n ) < F , , x3, · · · , x n
2 2
involves
F (x 1 , x 2 , x 3 , · · · , x n ) ≥ F (0, x 1 + x 2 , x 3 , · · · , x n ),
then the hypothesis
F (x 1 , x 2 , x 3 , · · · , x n ) ≥
n x + x x + x  o
1 2 1 2
≥ min F , , x 3 , · · · , x n , F (0, x 1 + x 2 , x 3 , · · · , x n )
2 2
is satisfied.
Some Classic and New Inequalities and Methods 15

21. VASC’S INEQUALITY


If a, b, c are real numbers, then

(a2 + b2 + c 2 )2 ≥ 3(a3 b + b3 c + c 3 a),

with equality for a = b = c, and also for


a b c
= = π
sin2 4π
7 sin2 2π
7
sin2 7

(or any cyclic permutation) - Vasile Cirtoaje, 1991.


The following theorem gives a generalization of Vasc’s inequality.

L
.M
Theorem 0 (Vasile Cirtoaje, 2007). Let
X X X X X
f4 (a, b, c) = a4 + A a2 b2 + Ba bc a+C a3 b + D a b3 ,

D
where A, B, C, D are real constants such that 1 + A + B + C + D = 0. The inequality

A
f4 (a, b, c) ≥ 0 holds for all real numbers a, b, c if and only if

3(1 + A) ≥ C 2 + C D + D2 .
PI
M
Notice that
LY

4 C−D 2 4
 ‹
f4 (a, b, c) = (U + V + C + D)2 + 3 U − V + + (3 + 3A − C 2 − C D − D2 ),
O

S 3 3
where
.M

a3 b − a2 bc a b3 − a2 bc
P P P P
X X
2 2 2
S= a b − a bc, U= , V= .
S S
W

For A = B = 0, C = −2 and D = 1, we get the following inequality


W

a4 + b4 + c 4 + a b3 + bc 3 + ca3 ≥ 2(a3 b + b3 c + c 3 a),


W

with equality for a = b = c, and also for


a b c
= =
sin π9 7π
sin 9 sin 13π
9

(or any cyclic permutation) - Vasile Cirtoaje, 1991.

22. CYCLIC INEQUALITIES OF DEGREE THREE AND FOUR


Consider the third degree cyclic homogeneous polynomial
X X X
f3 (a, b, c) = a3 + Ba bc + C a2 b + D a b2 ,
16 Vasile Cîrtoaje

where B, C, D are real constants. The following Theorem 1 holds.


Theorem 1 (Pham Kim Hung, 2007). The cyclic inequality f3 (a, b, c) ≥ 0 holds for
all a, b, c ≥ 0 if and only if
f3 (1, 1, 1) ≥ 0
and
f3 (a, 1, 0) ≥ 0
for all a ≥ 0.
Consider now the fourth degree cyclic homogeneous polynomial

L
X X X X X
f4 (a, b, c) = a4 + A a2 b2 + Ba bc a+C a3 b + D a b3 ,

.M
where A, B, C, D are real constants.
The following Theorem 2 states the necessary and sufficient conditions that f4 (a, b, c) ≥

D
0 for all real numbers a, b, c.

A
Theorem 2 (Vasile Cirtoaje and Yuanzhe Zhou, 2011). The inequality f4 (a, b, c) ≥ 0
PI
holds for all real numbers a, b, c if and only if g4 (t) ≥ 0 for all t ≥ 0, where
M
g4 (t) = 3(2 + A − C − D)t 4 − F t 3 + 3(4 − B + C + D)t 2 + 1 + A + B + C + D,
LY

Æ
F= 27(C − D)2 + E 2 , E = 8 − 4A + 2B − C − D.
Note that in the special case f4 (1, 1, 1) = 0 (when 1 + A + B + C + D = 0), Theorem 1
O

yields Theorem 0 from the preceding section 21.


.M

The following Theorem 3 states some strong sufficient conditions that f4 (a, b, c) ≥ 0
for all real numbers a, b, c.
Theorem 3 (Vasile Cirtoaje and Yuanzhe Zhou, 2012). The inequality f4 (a, b, c) ≥ 0
W

holds for all real numbers a, b, c if the following two conditions are satisfied:
W

(a) 1 + A + B + C + D ≥ 0; p p
(b) there exists a real number t ∈ (− 3, 3) such that f (t) ≥ 0, where
W

f (t) = 2G t 3 − (6 + 2A + B + 3C + 3D)t 2 + 2(1 + C + D)G t + H,


p
G = 1 + A + B + C + D, H = 2 + 2A − B − C − D − C 2 − C D − D2 .
The following Theorem 4 states the necessary and sufficient conditions that f4 (a, b, c) ≥
0 for all a, b, c ≥ 0.
Theorem 4 (Vasile Cirtoaje, 2013). Let
Æ
E = 8 − 4A + 2B − C − D, F= 27(C − D)2 + E 2 ,

g4 (t) = 3(2 + A − C − D)t 4 − F t 3 + 3(4 − B + C + D)t 2 + 1 + A + B + C + D,


Some Classic and New Inequalities and Methods 17

2E 3
g3 (t) = t + 3t 2 − 1.
F
For F = 0, the inequality f4 (a, b, c) ≥ 0 holds for all a, b, c ≥ 0 if and only if g4 (t) ≥ 0
for all t ∈ [0, 1].
For F 6= 0, the inequality f4 (a, b, c) ≥ 0 holds for all a, b, c ≥ 0 if and only if the
following two conditions are satisfied:
(a) g4 (t) ≥ 0 for all t ∈ [0, t 1 ], where t 1 ∈ [1/2, 1] such that g3 (t 1 ) = 0;
(b) f4 (a, 1, 0) ≥ 0 for all a ≥ 0.
The following Theorem 5 states some strong sufficient conditions that f4 (a, b, c) ≥ 0
for all a, b, c ≥ 0.

L
Theorem 5 (Vasile Cirtoaje and Yuanzhe Zhou, 2013). The inequality f4 (a, b, c) ≥ 0

.M
holds for all a, b, c ≥ 0 if
1 + A+ B + C + D ≥ 0

D
and one of the following two conditions is satisfied:

A
(a) 3(1 + A) ≥ C 2 + C D + D2 ;
(b) 3(1 + A) < C 2 + C D + D2 , and there exists t ≥ 0 such that
PI
Æ
(C + 2D)t 2 + 6t + 2C + D ≥ 2 (t 4 + t 2 + 1)(C 2 + C D + D2 − 3 − 3A).
M
LY
O
.M
W
W
W
18 Vasile Cîrtoaje

L
D
.M
A
PI
M
LY
O
.M
W
W
W
Chapter 2

Symmetric Polynomial Inequalities

L
in Real Variables

D
.M
A
2.1 Applications
PI
2.1. Let a, b, c, d be real numbers such that a2 + b2 + c 2 + d 2 = 9. Prove that
M
a3 + b3 + c 3 + d 3 ≤ 27.
LY
O

2.2. If a, b, c are real numbers such that a + b + c = 0, then


.M

(2a2 + bc)(2b2 + ca)(2c 2 + a b) ≤ 0.


W

2.3. Let a, b, c be real numbers such that a + b ≥ 0, b + c ≥ 0, c + a ≥ 0. Prove that


W

9(a + b)(b + c)(c + a) ≥ 8(a + b + c)(a b + bc + ca).


W

2.4. Let a, b, c be real numbers such that a b + bc + ca = 3. Prove that

(3a2 + 1)(3b2 + 1)(3c 2 + 1) ≥ 64.

When does equality hold?

2.5. If a and b are real numbers, then

3(1 − a + a2 )(1 − b + b2 ) ≥ 2(1 − a b + a2 b2 ).

19
20 Vasile Cîrtoaje

2.6. If a, b, c are real numbers, then

3(1 − a + a2 )(1 − b + b2 )(1 − c + c 2 ) ≥ 1 + a bc + a2 b2 c 2 .

2.7. If a, b, c are real numbers, then

(a2 + b2 + c 2 )3 ≥ (a + b + c)(a b + bc + ca)(a3 + b3 + c 3 ).

2.8. If a, b, c are real numbers, then

L
.M
2(a2 + b2 )(b2 + c 2 )(c 2 + a2 ) ≥ [a b(a + b) + bc(b + c) + ca(c + a) − 2a bc]2 .

D
2.9. If a, b, c are real numbers, then

A
PI
(a2 + 1)(b2 + 1)(c 2 + 1) ≥ 2(a b + bc + ca).
M
2.10. If a, b, c are real numbers, then
LY

5
(a2 + 1)(b2 + 1)(c 2 + 1) ≥ (a + b + c + 1)2 .
O

16
.M

2.11. If a, b, c are real numbers, then


W

(a) a6 + b6 + c 6 − 3a2 b2 c 2 + 2(a2 + bc)(b2 + ca)(c 2 + a b) ≥ 0;


W

(b) a6 + b6 + c 6 − 3a2 b2 c 2 ≥ (a2 − 2bc)(b2 − 2ca)(c 2 − 2a b).


W

2.12. If a, b, c are real numbers, then


2 6
(a + b6 + c 6 ) + a3 b3 + b3 c 3 + c 3 a3 + a bc(a3 + b3 + c 3 ) ≥ 0.
3

2.13. If a, b, c are real numbers, then

4(a2 + a b + b2 )(b2 + bc + c 2 )(c 2 + ca + a2 ) ≥ (a − b)2 (b − c)2 (c − a)2 .


Symmetric Polynomial Inequalities in Real Variables 21

2.14. If a, b, c are real numbers, then

(a2 + a b + b2 )(b2 + bc + c 2 )(c 2 + ca + a2 ) ≥ 3(a2 b + b2 c + c 2 a)(a b2 + bc 2 + ca2 ).

2.15. If a, b, c are real numbers such that a bc > 0, then

1 1 1
 ‹ ‹ ‹
4 a+ b+ c+ ≥ 9(a + b + c).
a b c

L
2.16. If a, b, c are real numbers, then

.M
(a) (a2 + 2bc)(b2 + 2ca)(c 2 + 2a b) ≤ (a2 + b2 + c 2 )(a b + bc + ca)2 ;

D
(b) (2a2 + bc)(2b2 + ca)(2c 2 + a b) ≤ (a + b + c)2 (a2 b2 + b2 c 2 + c 2 a2 ).

A
PI
2.17. If a, b, c are real numbers such that a b + bc + ca ≥ 0, then
M
27(a2 + 2bc)(b2 + 2ca)(c 2 + 2a b) ≤ (a + b + c)6 .
LY
O

2.18. If a, b, c are real numbers such that a2 + b2 + c 2 = 2, then


.M

(a2 + 2bc)(b2 + 2ca)(c 2 + 2a b) + 2 ≥ 0.


W

2.19. If a, b, c are real numbers such that a + b + c = 3, then


W

3(a4 + b4 + c 4 ) + a2 + b2 + c 2 + 6 ≥ 6(a3 + b3 + c 3 ).
W

2.20. If a, b, c are real numbers such that a bc = 1, then

3(a2 + b2 + c 2 ) + 2(a + b + c) ≥ 5(a b + bc + ca).

2.21. If a, b, c are real numbers such that a bc = 1, then

3 1 1 1
 ‹
2 2 2
a + b +c +6≥ a+b+c+ + + .
2 a b c
22 Vasile Cîrtoaje

2.22. If a, b, c are real numbers, then


1
(1 + a2 )(1 + b2 )(1 + c 2 ) + 8a bc ≥ (1 + a)2 (1 + b)2 (1 + c)2 .
4

2.23. Let a, b, c be real numbers such that a + b + c = 0. Prove that


2049 4 4 4
a12 + b12 + c 12 ≥ a b c .
8

L
2.24. If a, b, c are real numbers such that a bc ≥ 0, then

.M
a2 + b2 + c 2 + 2a bc + 4 ≥ 2(a + b + c) + a b + bc + ca.

D
A
2.25. Let a, b, c be real numbers such that a + b + c = 3.
(a) If a, b, c ≥ −3, then
PI
M
1 1 1 1 1 1
2
+ 2+ 2≥ + + .
a b c a b c
LY

(b) If a, b, c ≥ −7, then


O

1−a 1− b 1−c
+ + ≥ 0.
(1 + a)2 (1 + b)2 (1 + c)2
.M
W

2.26. If a, b, c are real numbers, then


1
W

a6 + b6 + c 6 − 3a2 b2 c 2 ≥ (a − b)2 (b − c)2 (c − a)2 .


2
W

2.27. If a, b, c are real numbers, then


3
a + b2 + c 2
 2
1
≥ a2 b2 c 2 + (a − b)2 (b − c)2 (c − a)2 .
3 16

2.28. If a, b, c are real numbers, then


108 2 2 2
(a2 + b2 + c 2 )3 ≥ a b c + 2(a − b)2 (b − c)2 (c − a)2 .
5
Symmetric Polynomial Inequalities in Real Variables 23

2.29. If a, b, c are real numbers, then

2(a2 + b2 )(b2 + c 2 )(c 2 + a2 ) ≥ (a − b)2 (b − c)2 (c − a)2 .

2.30. If a, b, c are real numbers, then

32(a2 + bc)(b2 + ca)(c 2 + a b) + 9(a − b)2 (b − c)2 (c − a)2 ≥ 0.

2.31. If a, b, c are real numbers, then

L
.M
1
a4 (b − c)2 + b4 (c − a)2 + c 4 (a − b)2 ≥ (a − b)2 (b − c)2 (c − a)2 .
2

D
A
2.32. If a, b, c are real numbers, then

a2 (b − c)4 + b2 (c − a)4 + c 2 (a − b)4 ≥


1PI
(a − b)2 (b − c)2 (c − a)2 .
2
M
LY

2.33. If a, b, c are real numbers, then


O

3
a2 (b2 − c 2 )2 + b2 (c 2 − a2 )2 + c 2 (a2 − b2 )2 ≥ (a − b)2 (b − c)2 (c − a)2 .
8
.M

2.34. If a, b, c are real numbers such that a b + bc + ca = 3, then


W

(a2 + a b + b2 )(b2 + bc + c 2 )(c 2 + ca + a2 ) ≥ 3(a + b + c)2 ;


W

(a)

3 2
W

(b) (a2 + a b + b2 )(b2 + bc + c 2 )(c 2 + ca + a2 ) ≥ (a + b2 + c 2 ).


2

2.35. If a, b, c are real numbers, then

(a2 + a b + b2 )(b2 + bc + c 2 )(c 2 + ca + a2 ) ≥ 3(a b + bc + ca)(a2 b2 + b2 c 2 + c 2 a2 ).

2.36. If a, b, c are real numbers, not all of the same sign, then

(a2 + a b + b2 )(b2 + bc + c 2 )(c 2 + ca + a2 ) ≥ 3(a b + bc + ca)3 .


24 Vasile Cîrtoaje

2.37. If a, b, c are real numbers, then


3 2
(a2 + a b + b2 )(b2 + bc + c 2 )(c 2 + ca + a2 ) ≥ (a + b2 )(b2 + c 2 )(c 2 + a2 ).
8

2.38. If a, b, c are real numbers, then

2(a2 + b2 )(b2 + c 2 )(c 2 + a2 ) ≥ (a2 − a b + b2 )(b2 − bc + c 2 )(c 2 − ca + a2 ).

L
2.39. If a, b, c are real numbers, then

.M
9(1 + a4 )(1 + b4 )(1 + c 4 ) ≥ 8(1 + a bc + a2 b2 c 2 )2 .

D
A
2.40. If a, b, c are real numbers, then
PI
2(1 + a2 )(1 + b2 )(1 + c 2 ) ≥ (1 + a)(1 + b)(1 + c)(1 + a bc).
M
LY

2.41. If a, b, c are real numbers, then


O

3(a2 − a b + b2 )(b2 − bc + c 2 )(c 2 − ca + a2 ) ≥ a3 b3 + b3 c 3 + c 3 a3 .


.M

2.42. If a, b, c are nonzero real numbers, then


W

X b2 − bc + c 2 X a2 €X Š X 1 ‹
W

+ 2 ≥ a .
a2 bc a
W

2.43. Let a, b, c be real numbers. Prove that


(a) if a, b, c ∈ [0, 1], then

a bc − (b + c − a)(c + a − b)(a + b − c) ≤ 1;

(b) if a, b, c ∈ [−1, 1], then

a bc − (b + c − a)(c + a − b)(a + b − c) ≤ 4.
Symmetric Polynomial Inequalities in Real Variables 25

2.44. Let a, b, c be real numbers. Prove that


(a) if a, b, c ∈ [0, 1], then
X
a2 (a − b)(a − c) ≤ 1;

(b) if a, b, c ∈ [−1, 1], then


X
a2 (a − b)(a − c) ≤ 4.

L
2.45. Let a, b, c be real numbers such that

.M
a b + bc + ca = a bc + 2.

D
Prove that p
a2 + b2 + c 2 − 3 ≥ (2 + 3)(a + b + c − 3).

A
PI
M
2.46. Let a, b, c be real numbers such that
LY

(a + b)(b + c)(c + a) = 10.

Prove that
O

(a2 + b2 )(b2 + c 2 )(c 2 + a2 ) + 12a2 b2 c 2 ≥ 30.


.M

2.47. Let a, b, c be real numbers such that


W

(a + b)(b + c)(c + a) = 5.
W

Prove that
W

(a2 + a b + b2 )(b2 + bc + c 2 )(c 2 + ca + a2 ) + 12a2 b2 c 2 ≥ 15.

2.48. Let a, b, c be real numbers such that a + b + c = 1 and a3 + b3 + c 3 = k. Prove


that
(a) if k = 25, then |a| ≤ 1 or |b| ≤ 1 or |c| ≤ 1;
(b) if k = −11, then 1 < a ≤ 2 or 1 < b ≤ 2 or 1 < c ≤ 2.
26 Vasile Cîrtoaje

2.49. Let a, b, c be real numbers such that

a + b + c = a3 + b3 + c 3 = 2.
5
• ˜
/
Prove that a, b, c ∈ ,2 .
4

2.50. If a, b, c and k are real numbers, then


X
(a − b)(a − c)(a − k b)(a − kc) ≥ 0.

L
.M
2.51. If a, b, c are real numbers, then

(b + c − a)2 (c + a − b)2 (a + b − c)2 ≥ (b2 + c 2 − a2 )(c 2 + a2 − b2 )(a2 + b2 − c 2 ).

D
A
2.52. If a, b, c are real numbers, then PI
M
X (a − b)2 (b − c)2 (c − a)2
a2 (a − b)(a − c) ≥ .
a2 + b2 + c 2 + a b + bc + ca
LY
O

2.53. Let a ≤ b ≤ c be real numbers such that


.M

a + b + c = p, a b + bc + ca = q,

where p and q are fixed real numbers satisfying p2 ≥ 3q. Prove that the product
W

r = a bc
W

is minimal when b = c, and is maximal when a = b.


W

2.54. Let a, b, c be real numbers such that a + b + c = 3. Prove that

(a b + bc + ca − 3)2 ≥ 27(a bc − 1).

2.55. Let a, b, c be real numbers such that a + b + c = 3. Prove that

(a b + bc + ca)2 + 9 ≥ 18a bc.


Symmetric Polynomial Inequalities in Real Variables 27

2.56. If a, b, c are real numbers such that a2 + b2 + c 2 = 9, then

a bc + 10 ≥ 2(a + b + c).

2.57. If a, b, c are real numbers such that

a + b + c + a bc = 4,

then
a2 + b2 + c 2 + 3 ≥ 2(a b + bc + ca).

L
.M
2.58. If a, b, c are real numbers such that

D
a b + bc + ca = 3a bc,

A
then PI
4(a2 + b2 + c 2 ) + 9 ≥ 7(a b + bc + ca).
M
LY

2.59. If a, b, c are real numbers such that a + b + c = 3, then


O

(a2 + 1)(b2 + 1)(c 2 + 1) ≥ (a + 1)(b + 1)(c + 1).


.M

2.60. Let f4 (a, b, c) be a symmetric homogeneous polynomial of degree four. Prove that
W

the inequality f4 (a, b, c) ≥ 0 holds for all real numbers a, b, c if and only if f4 (a, 1, 1) ≥ 0
W

for all real a.


W

2.61. If a, b, c are real numbers, then


X
10(a4 + b4 + c 4 ) + 64(a2 b2 + b2 c 2 + c 2 a2 ) ≥ 33 a b(a2 + b2 ).

2.62. If a, b, c are real numbers such that a + b + c = 3, then

3(a4 + b4 + c 4 ) + 33 ≥ 14(a2 + b2 + c 2 ).
28 Vasile Cîrtoaje

2.63. If a, b, c are real numbers such that a2 + b2 + c 2 = 3, then

a4 + b4 + c 4 + 3(a b + bc + ca) ≤ 12.

2.64. Let α, β, γ be real numbers such that

1 + α + β = 2γ.

The inequality
X X X X

L
a4 + α a2 b2 + β a bc a≥γ a b(a2 + b2 )

.M
holds for any real numbers a, b, c if and only if

D
1 + α ≥ γ2 .

A
PI
2.65. If a, b, c are real numbers such that a2 + b2 + c 2 = 2, then
M
a b(a2 − a b + b2 − c 2 ) + bc(b2 − bc + c 2 − a2 ) + ca(c 2 − ca + a2 − b2 ) ≤ 1.
LY
O

2.66. If a, b, c are real numbers, then


.M

4 4
(a + b)4 + (b + c)4 + (c + a)4 ≥ (a + b4 + c 4 ).
7
W

2.67. Let a, b, c be real numbers, and let


W

p = a + b + c, q = a b + bc + ca, r = a bc.
W

Prove that
p2 + q2 − pq
(3 − p)r + ≥ q.
3

2.68. If a, b, c are real numbers, then

a b(a + b) + bc(b + c) + ca(c + a) 3


≤ .
(a2 + 1)(b2 + 1)(c 2 + 1) 4
Symmetric Polynomial Inequalities in Real Variables 29

2.69. If a, b, c are real numbers such that a bc > 0, then

1 1 1 1 1 1 1
 ‹ ‹ ‹  ‹
a + − 1 b + − 1 c + − 1 + 2 ≥ (a + b + c) + + .
a b c 3 a b c

2.70. If a, b, c are real numbers, then

1 1 1 1 1 1
 ‹ ‹ ‹  ‹ ‹ ‹
2 2 2
a + b + c + ≥ a+b− b+c− c+a− .
2 2 2 2 2 2

L
2.71. If a, b, c are real numbers such that a + b + c = 3, then

.M
a(a − 1) b(b − 1) c(c − 1)
+ + 2 ≥ 0.

D
8a2 + 9 8b2 + 9 8c + 9

A
PI
2.72. If a, b, c are real numbers such that a + b + c = 3, then
M
(a − 11)(a − 1) (b − 11)(b − 1) (c − 11)(c − 1)
+ + ≥ 0.
2a2 + 1 2b2 + 1 2c 2 + 1
LY
O

2.73. If a, b, c are real numbers, then


.M

(a2 + 2)(b2 + 2)(c 2 + 2) ≥ 9(a b + bc + ca).


W

2.74. If a, b, c are real numbers such that a b + bc + ca = 3, then


W

4(a4 + b4 + c 4 ) + 11a bc(a + b + c) ≥ 45.


W

2.75. Any sixth degree symmetric homogeneous polynomial f6 (a, b, c) can be written
in the form
f6 (a, b, c) = Ar 2 + B(p, q)r + C(p, q),
where A is called the highest coefficient of f6 , and

p = a + b + c, q = a b + bc + ca, r = a bc.

In the case A ≤ 0, prove that the inequality f6 (a, b, c) ≥ 0 holds for all real numbers
a, b, c if and only if f6 (a, 1, 1) ≥ 0 for all real a.
30 Vasile Cîrtoaje

2.76. If a, b, c are real numbers such that a b + bc + ca = −1, then

(a) 5(a2 + b2 )(b2 + c 2 )(c 2 + a2 ) ≥ 8;

(b) (a2 + a b + b2 )(b2 + bc + c 2 )(c 2 + ca + a2 ) ≥ 1.

2.77. If a, b, c are real numbers, then

a2 (a − b)(a − c)(a + 2b)(a + 2c) + (a − b)2 (b − c)2 (c − a)2 ≥ 0;


P
(a)

L
a2 (a − b)(a − c)(a − 4b)(a − 4c) + 7(a − b)2 (b − c)2 (c − a)2 ≥ 0.
P
(b)

.M
D
2.78. If a, b, c are real numbers, then

A
(a2 + 2bc)(b2 + 2ca)(c 2 + 2a b) + (a − b)2 (b − c)2 (c − a)2 ≥ 0.
PI
M
2.79. If a, b, c are real numbers, then
LY

(2a2 + 5a b + 2b2 )(2b2 + 5bc + 2c 2 )(2c 2 + 5ca + 2a2 ) + (a − b)2 (b − c)2 (c − a)2 ≥ 0.
O
.M

2.80. If a, b, c are real numbers, then

2 2 2 64 2
 ‹ ‹ ‹
2 2 2 2 2 2
a + ab + b b + bc + c c + ca + a ≥ (a + bc)(b2 + ca)(c 2 + a b).
W

3 3 3 27
W
W

2.81. If a, b, c are real numbers, then


X 2(a − b)2 (b − c)2 (c − a)2
a2 (a − b)(a − c) ≥ .
a2 + b2 + c 2

2.82. If a, b, c are real numbers, then


X 8(a − b)2 (b − c)2 (c − a)2
(a − b)(a − c)(a − 2b)(a − 2c) ≥ .
a2 + b2 + c 2
Symmetric Polynomial Inequalities in Real Variables 31

2.83. If a, b, c are real numbers, no two of which are zero, then

a2 + 3bc b2 + 3ca c 2 + 3a b
+ 2 + 2 ≥ 0.
b2 + c 2 c + a2 a + b2

2.84. If a, b, c are real numbers, no two of which are zero, then

a2 + 6bc b2 + 6ca c 2 + 6a b
+ + ≥ 0.
b2 − bc + c 2 c 2 − ca + a2 a2 − a b + b2

L
2.85. If a, b, c are real numbers such that a b + bc + ca ≥ 0, then

.M
4a2 + 23bc 4b2 + 23ca 4c 2 + 23a b
+ + ≥ 0.

D
b2 + c 2 c 2 + a2 a2 + b2

A
PI
2.86. If a, b, c are real numbers such that a b + bc + ca = 3, then
M
20(a6 + b6 + c 6 ) + 43a bc(a3 + b3 + c 3 ) ≥ 189.
LY

2.87. If a, b, c are real numbers, then


O

X
4 (a2 + bc)(a − b)(a − c)(a − 3b)(a − 3c) ≥ 7(a − b)2 (b − c)2 (c − a)2 .
.M

2.88. Let a, b, c be real numbers such that a b + bc + ca ≥ 0. For any real k, prove that
W

X
4bc(a − b)(a − c)(a − k b)(a − kc) + (a − b)2 (b − c)2 (c − a)2 ≥ 0.
W
W

2.89. If a, b, c are real numbers, then


 2 2
(a b + b2 c + c 2 a) + (a b2 + bc 2 + ca2 ) ≥ 4(a b + bc + ca)(a2 b2 + b2 c 2 + c 2 a2 ).

2.90. If a, b, c are real numbers such that a + b + c = 3, then


(a − 1)(a − 25) (b − 1)(b − 25) (c − 1)(c − 25)
+ + ≥ 0.
a2 + 23 b2 + 23 c 2 + 23
32 Vasile Cîrtoaje

2.91. If a, b, c are real numbers such that a bc 6= 0, then


‹2 ‹2
b+c  c + a 2 a+b
 
+ + > 2.
a b c

2.92. If a, b, c are real numbers, then

8
(a) (a2 + 1)(b2 + 1)(c 2 + 1) ≥ p |(a − b)(b − c)(c − a)|;
3 3
(b) (a2 − a + 1)(b2 − b + 1)(c 2 − c + 1) ≥ |(a − b)(b − c)(c − a)|.

L
.M
2.93. If a, b, c are real numbers such that a + b + c = 3, then

D
(1 − a + a2 )(1 − b + b2 )(1 − c + c 2 ) ≥ 1.

A
2.94. If a, b, c are real numbers such that a + b + c = 0, then
PI
M
a(a − 4) b(b − 4) c(c − 4)
+ 2 + 2
LY

≥ 0.
a2 + 2 b +2 c +2
O

2.95. If a, b, c, d are real numbers, then


.M

‹2
1 + a bcd

2 2 2 2
(1 − a + a )(1 − b + b )(1 − c + c )(1 − d + d ) ≥ .
2
W
W

2.96. Let a, b, c, d be real numbers such that a bcd > 0. Prove that
W

1 1 1 1 1 1 1 1
 ‹ ‹ ‹ ‹  ‹
a+ b+ c+ d+ ≥ (a + b + c + d) + + + .
a b c d a b c d

2.97. Let a, b, c, d be real numbers such that

a + b + c + d = 4, a2 + b2 + c 2 + d 2 = 7.

Prove that
a3 + b3 + c 3 + d 3 ≤ 16.
Symmetric Polynomial Inequalities in Real Variables 33

2.98. Let a, b, c, d be real numbers such that a + b + c + d = 0. Prove that

12(a4 + b4 + c 4 + d 4 ) ≤ 7(a2 + b2 + c 2 + d 2 )2 .

2.99. Let a, b, c, d be real numbers such that a + b + c + d = 0. Prove that

(a2 + b2 + c 2 + d 2 )3 ≥ 3(a3 + b3 + c 3 + d 3 )2 .

2.100. If a, b, c, d are real numbers such that a bcd = 1. Prove that

L
.M
(1 + a2 )(1 + b2 )(1 + c 2 )(1 + d 2 ) ≥ (a + b + c + d)2 .

D
2.101. Let a, b, c, d be real numbers such that

A
a2 + b2 + c 2 + d 2 = 4.
PI
M
Prove that
(a bc)3 + (bcd)3 + (cd a)3 + (d a b)3 ≤ 4.
LY
O

2.102. Let a, b, c, d be real numbers such that a2 + b2 + c 2 + d 2 = 1. Prove that


.M

(1 − a)4 + (1 − b)4 + (1 − c)4 + (1 − d)4 ≥ a4 + b4 + c 4 + d 4 .


W

−1
2.103. If a, b, c, d ≥ such that a + b + c + d = 4, then
2
W

1−a 1− b 1−c 1−d


+ + + ≥ 0.
W

1−a+a 2 1− b+ b 2 1−c+c 2 1 − d + d2

2.104. If a, b, c, d, e ≥ −3 such that a + b + c + d + e = 5, then


1−a 1− b 1−c 1−d 1−e
+ + + + ≥ 0.
1+a+a 2 1+ b+ b 2 1+c+c 2 1+d +d 2 1 + e + e2

2.105. Let a, b, c, d, e be real numbers such that a + b + c + d + e = 0. Prove that

30(a4 + b4 + c 4 + d 4 + e4 ) ≥ 7(a2 + b2 + c 2 + d 2 + e2 )2 .
34 Vasile Cîrtoaje

2.106. Let a1 , a2 , . . . , an ≥ −1 such that a1 + a2 + · · · + an = 0. Prove that

(n − 2)(a12 + a22 + · · · + an2 ) ≥ a13 + a23 + · · · + an3 .

2.107. Let a1 , a2 , . . . , an ≥ −1 such that a1 + a2 + · · · + an = 0. Prove that

(n − 2)(a12 + a22 + · · · + an2 ) + (n − 1(a13 + a23 + · · · + an3 ) ≥ 0.

p
2.108. Let a1 , a2 , . . . , an ≥ n − 1 − n2 − n + 1 be nonzero real numbers such that a1 +

L
a2 + · · · + an = n. Prove that

.M
1 1 1
+ + ··· + ≥ n.
a12 a22 an2

D
A
2.109. Let a1 , a2 , . . . , an ≤
n
n−2
be real numbers such that PI
M
a1 + a2 + · · · + an = n.
LY

If k is a positive integer, k ≥ 2, then


O

a1k + a2k + · · · + ank ≥ n.


.M

2.110. If a1 , a2 , . . . , an (n ≥ 3) are real numbers such that


W

−(3n − 2)
a1 , a2 , . . . , an ≥ , a1 + a2 + · · · + an = n,
n−2
W

then
W

1 − a1 1 − a2 1 − an
+ + ··· + ≥ 0.
(1 + a1 )2 (1 + a2 )2 (1 + an )2

2.111. Let a1 , a2 , . . . , an be real numbers. Prove that

(a1 + a2 + · · · + an )2 (n − 1)n−1
(a) ≤ ;
(a12 + 1)(a22 + 1) · · · (an2 + 1) nn−2
1
a1 + a2 + · · · + an (2n − 1)n− 2
(b) ≤ .
(a12 + 1)(a22 + 1) · · · (an2 + 1) 2n nn−1
Symmetric Polynomial Inequalities in Real Variables 35

2.2 Solutions

P 2.1. Let a, b, c, d be real numbers such that a2 + b2 + c 2 + d 2 = 9. Prove that

a3 + b3 + c 3 + d 3 ≤ 27.

Solution. From a2 + b2 + c 2 + d 2 = 9, we get a2 ≤ 9, a ≤ 3, a2 (a − 3) ≤ 0, a3 ≤ 3a2 .


Similarly, b3 ≤ 3b2 , c 3 ≤ 3c 2 and d 3 ≤ 3d 2 . Therefore,

a3 + b3 + c 3 + d 3 ≤ 3(a2 + b2 + c 2 + d 2 ) = 27.

L
The equality holds for a = 3 and b = c = d = 0 (or any cyclic permutation thereof).

D
.M
P 2.2. If a, b, c are real numbers such that a + b + c = 0, then

A
PI
(2a2 + bc)(2b2 + ca)(2c 2 + a b) ≤ 0.
M
First Solution. Among a, b, c there are two with the same sign. Let bc ≥ 0. We need to
LY

show that
(2b2 + ca)(2c 2 + a b) ≤ 0.
O

This is equivalent to
[2b2 − c(b + c)][2c 2 − (b + c)b] ≤ 0,
.M

(b − c)2 (2b + c)(b + 2c) ≥ 0.


Since
W

(2b + c)(b + 2c) = 2(b2 + c 2 ) + 5bc ≥ 0,


W

−a
the conclusion follows. The equality holds for = b = c (or any cyclic permutation).
2
W

Second Solution. We have

2a2 + bc = (a − b)(a − c) + a(a + b + c) = (a − b)(a − c),

2b2 + ca = (b − c)(b − a) + b(a + b + c) = (b − c)(b − a),


2c 2 + a b = (c − a)(c − b) + c(a + b + c) = (c − a)(c − b).
Therefore,

(2a2 + bc)(2b2 + ca)(2c 2 + a b) = −(a − b)2 (b − c)2 (c − a)2 ≤ 0.


36 Vasile Cîrtoaje

P 2.3. Let a, b, c be real numbers such that a + b ≥ 0, b + c ≥ 0, c + a ≥ 0. Prove that

9(a + b)(b + c)(c + a) ≥ 8(a + b + c)(a b + bc + ca).

(Nguyen Van Huyen, 2014)

Solution. Write the inequality in the form

a(b − c)2 + b(c − a)2 + c(a − b)2 ≥ 0.

For a, b, c ≥ 0, the inequality is clearly true. Otherwise, without loss of generality,


assume that a ≤ b ≤ c. From a + b ≥ 0, b + c ≥ 0, c + a ≥ 0, it follows that a ≤ 0 ≤ b ≤ c

L
and a + b ≥ 0. Replacing a by −a, we need to show that 0 ≤ a ≤ b ≤ c involves

.M
−a(c − b)2 + b(c + a)2 + c(a + b)2 ≥ 0.

D
This is true since

A
PI
−a(c − b)2 + b(c + a)2 ≥ −b(c − b)2 + b(c + a)2 = b(a + b)(a − b + 2c) ≥ 0.

The equality holds for a = b = c ≥ 0.


M
LY
O

P 2.4. Let a, b, c be real numbers such that a b + bc + ca = 3. Prove that


.M

(3a2 + 1)(3b2 + 1)(3c 2 + 1) ≥ 64.

When does equality hold?


W
W

Solution. Using the substitution


W

x y z
a= p , b= p , c=p ,
3 3 3
we need to show that
(x 2 + 1)( y 2 + 1)(z 2 + 1) ≥ 64
for all real x, y, z such that x y + yz + z x = 9.
First Solution. Applying the Cauchy-Schwarz inequality, we have

(x 2 + 1)( y 2 + 1)(z 2 + 1) = (x 2 + 1)[( y + z)2 + ( yz − 1)2 ]


≥ [x( y + z) + ( yz − 1)]2 = 64.
Symmetric Polynomial Inequalities in Real Variables 37

y +z
The equality holds for x y + yz + z x = 9 and = yz − 1; that is, for
x
( yz − 1)(9 − yz)
y + z = ( yz − 1)x = ,
y +z

( y + z)2 + ( yz − 1)( yz − 9) = 0,
( y − z)2 + ( yz − 3)2 = 0,
p
y = z = ± 3.
In addition, from x y + yz + z x = 9, we get

L
p
x = y = z = ± 3.

.M
Therefore, the original inequality becomes an equality for

D
a = b = c = ±1.

A
Second Solution. We have PI X X
(x 2 + 1)( y 2 + 1)(z 2 + 1) − 64 = x 2 y 2 z 2 + x2 y2 + x 2 − 63
M
Š2 €X Š2
LY

€X X X
= x 2 y 2z2 + xy − 2x yz x+ x −2 x y − 63
X €X Š2 € X Š2
O

= x 2 y 2 z 2 − 2x yz x+ x = x yz − x ≥ 0.
.M
W

P 2.5. If a and b are real numbers, then


W

3(1 − a + a2 )(1 − b + b2 ) ≥ 2(1 − a b + a2 b2 ).


W

(Titu Andreescu, 2006)

Solution. Since

(1 − a + a2 )(1 − b + b2 ) = (a + b)2 − (a b + 1)(a + b) + (1 − a b + a2 b2 ),

we can write the inequality as

3(a + b)2 − 3(a b + 1)(a + b) + a2 b2 − a b + 1 ≥ 0.

Clearly, this inequality is true if a ≤ 0 and b ≤ 0. Otherwise, write the inequality in the
form
3(2a + 2b − a b − 1)2 + a2 b2 − 10a b + 1 ≥ 0.
38 Vasile Cîrtoaje

This inequality is obviously true for a b ≤ 0. Therefore, consider further that a > 0
and b > 0. If a2 b2 − 10a b + 1 ≥ 0, then the inequality is true. Assume now that
a2 b2 − 10a b + 1 ≤ 0. Since
p
2a + 2b − a b − 1 ≥ 4 a b − a b − 1

and
p 14a b − a2 b2 − 1 10a b − a2 b2 − 1
4 ab − ab − 1 = p > p ≥ 0,
4 ab + ab + 1 4 ab + ab + 1
it suffices to show that
p

L
3(4 a b − a b − 1)2 + a2 b2 − 10a b + 1 ≥ 0.

.M
This inequality is equivalent to

D
p
(a b − 3 a b + 1)2 ≥ 0,

A
1 p
which is obviously true. The equality holds for a = b = PI (3 ± 5).
2
M
LY

P 2.6. If a, b, c are real numbers, then


O

3(1 − a + a2 )(1 − b + b2 )(1 − c + c 2 ) ≥ 1 + a bc + a2 b2 c 2 .


.M

(Vasile Cîrtoaje and Mircea Lascu, 1989)

First Solution. From the identity


W

2(1 − a + a2 )(1 − b + b2 ) = 1 + a2 b2 + (a − b)2 + (1 − a)2 (1 − b)2 ,


W

it follows that
W

2(1 − a + a2 )(1 − b + b2 ) ≥ 1 + a2 b2 .
Thus, it is enough to prove that

3(1 + a2 b2 )(1 − c + c 2 ) ≥ 2(1 + a bc + a2 b2 c 2 ).

This inequality is equivalent to

(3 + a2 b2 )c 2 − (3 + 2a b + 3a2 b2 )c + 1 + 3a2 b2 ≥ 0,

which is true, since the quadratic in c has the discriminant

D = −3(1 − a b)4 ≤ 0.
Symmetric Polynomial Inequalities in Real Variables 39

This means that we can write the inequality as

[2(3 + a2 b2 )c − 3 − 2a b − 3a2 b2 ]2 + 3(1 − a b)4 ≥ 0.

The equality holds for a = b = c = 1.

Second Solution. Write the required inequality as

3(1 − a + a2 )(1 − b + b2 )(1 − c + c 2 ) − a bc ≥ 1 + a2 b2 c 2 .

Substituting a, b, c by |a|, |b|, |c|, respectively, the left side of this inequality remains

L
unchanged or decreases, while the right side remains unchanged. Therefore, it suffices
to prove the inequality only for a, b, c ≥ 0. For a = b = c, the inequality is true since

.M
3(1 − a + a2 )3 − (1 + a3 + a6 ) = (1 − a)4 (2 − a + 2a2 ) ≥ 0.

D
A
Multiplying the inequalities
p
3 3
p PI
3(1 − a + a2 ) ≥ 1 + a3 + a6 ,
M
p
3 3
p
3(1 − b + b2 ) ≥ 1 + b3 + b6 ,
LY

p3 3
p
3(1 − c + c 2 ) ≥ 1 + c 3 + c 6 ,
O

we get
.M

Æ
3
3(1 − a + a2 )(1 − b + b2 )(1 − c + c 2 ) ≥ (1 + a3 + a6 )(1 + b3 + b6 )(1 + c 3 + c 6 ).
W

Therefore, it suffices to prove that


W

Æ
3
(1 + a3 + a6 )(1 + b3 + b6 )(1 + c 3 + c 6 ) ≥ 1 + a bc + a2 b2 c 2 ,
W

which follows immediately from Hölder’s inequality.

P 2.7. If a, b, c are real numbers, then

(a2 + b2 + c 2 )3 ≥ (a + b + c)(a b + bc + ca)(a3 + b3 + c 3 ).

(Vasile Cîrtoaje, 2007)


40 Vasile Cîrtoaje

Solution. Substituting a, b, c by |a|, |b|, |c|, respectively, the left side of the inequality
remains unchanged, while the right side either remains unchanged or increases. There-
fore, it suffices to prove the inequality only for a, b, c ≥ 0. Let p = a + b + c and
q = a b + bc + ca. Since

a2 (b − c)2 + b2 (c − a)2 + c 2 (a − b)2


q2 − 3a bc p = ≥ 0,
2
we have

(a + b + c)(a3 + b3 + c 3 ) = p(p3 − 3pq + 3a bc) ≤ p4 − 3p2 q + q2 .

L
Thus, it suffices to show that

.M
(p2 − 2q)3 ≥ q(p4 − 3p2 q + q2 ),

D
which is equivalent to the obvious inequality

A
(p2 − 3q)2 (p2 − q) ≥ 0.
PI
The equality holds for a = b = c.
M
LY
O

P 2.8. If a, b, c are real numbers, then


.M

2(a2 + b2 )(b2 + c 2 )(c 2 + a2 ) ≥ [a b(a + b) + bc(b + c) + ca(c + a) − 2a bc]2 .

(Vo Quoc Ba Can, 2009)


W

Solution. Since
W

(a2 + b2 )(a2 + c 2 ) = (a2 + bc)2 + (a b − ac)2


W

and
2(b2 + c 2 ) = (b + c)2 + (b − c)2 ,
the required inequality follows by applying the Cauchy-Schwarz inequality as follows

2(a2 + b2 )(b2 + c 2 )(c 2 + a2 ) ≥ [(a2 + bc)(b + c) + (a b − ac)(b − c)]2


= [a b(a + b) + bc(b + c) + ca(c + a) − 2a bc]2 .

The equality holds when two of a, b, c are equal.


Symmetric Polynomial Inequalities in Real Variables 41

P 2.9. If a, b, c are real numbers, then

(a2 + 1)(b2 + 1)(c 2 + 1) ≥ 2(a b + bc + ca).

First Solution. Substituting a, b, c by |a|, |b|, |c|, respectively, the left side of this in-
equality remains unchanged, while the right side remains unchanged or increases. There-
fore, it suffices to prove the inequality only for a, b, c ≥ 0. Without loss of generality,
assume that a ≥ b ≥ c ≥ 0. Since

3(a2 + 1)(b + c)
2(a b + bc + ca) ≤ 3a(b + c) ≤ ,
2

L
it suffices to prove that

.M
2(b2 + 1)(c 2 + 1) ≥ 3(b + c),
which is equivalent to

D
2(b + c)2 − 3(b + c) + 2(bc − 1)2 ≥ 0.

A
PI
Case 1: 4bc ≤ 1. We have
M
‹2
3 (1 − 4bc)(7 − 4bc)

LY

2 2
2(b + c) − 3(b + c) + 2(bc − 1) = 2 b + c − + ≥ 0.
4 8
O

Case 2: 4bc ≥ 1. We get the required inequality by summing


.M

9(b + c)2
− 3(b + c) + 2 ≥ 0,
8
W

and
W

7(b + c)2
+ 2b2 c 2 − 4bc ≥ 0.
8
W

We have
9(b + c)2 [3(b + c) − 4]2
− 3(b + c) + 2 = ≥0
8 8
and
7(b + c)2 7bc bc(4bc − 1)
+ 2b2 c 2 − 4bc ≥ + 2b2 c 2 − 4bc = .
8 2 2

Second Solution Write the inequality as


˜2
b+c
•
(b2 + 1)(c 2 + 1) a − + A ≥ 0,
(b + 1)(c 2 + 1)
2
42 Vasile Cîrtoaje

where
(b + c)2
A = (b2 + 1)(c 2 + 1) − 2bc − .
(b2 + 1)(c 2 + 1)
We need to show that A ≥ 0. By virtue of the Cauchy-Schwarz inequality,

(b2 + 1)(c 2 + 1) ≥ (b + c)2 .

Then,
A ≥ (b2 + 1)(c 2 + 1) − 2bc − 1 = b2 c 2 + (b − c)2 ≥ 0.

L
.M
P 2.10. If a, b, c are real numbers, then
5

D
(a2 + 1)(b2 + 1)(c 2 + 1) ≥ (a + b + c + 1)2 .
16

A
(Vasile Cîrtoaje, 2006)
PI
First Solution. Replacing a, b, c respectively by a/2, b/2, c/2, the inequality becomes
M
(a2 + 4)(b2 + 4)(c 2 + 4) ≥ 5(a + b + c + 2)2 .
LY

Since the equality in this inequality holds for a = b = c = 1, we apply the Cauchy-
Schwarz inequality in the form
O

b+c+2 2
  ‹ 
.M

2 2
(a + b + c + 2) ≤ (a + 4) 1 + .
2

Thus, it suffices to prove that


W

‹2 
b+c+2
 
W

2 2
(b + 4)(c + 4) ≥ 5 1 + .
2
W

This inequality is equivalent to

11(b + c)2 − 20(b + c) + 4b2 c 2 − 32bc + 24 ≥ 0.

Since 4b2 c 2 − 8bc + 4 = 4(bc − 1)2 ≥ 0, it suffices to show that

11(b + c)2 − 20(b + c) − 24bc + 20 ≥ 0.

Indeed,

11(b + c)2 − 20(b + c) − 24bc + 20 ≥ 11(b + c)2 − 20(b + c) − 6(b + c)2 + 20


= 5(b + c − 2)2 ≥ 0.
Symmetric Polynomial Inequalities in Real Variables 43

1
Equality in the original inequality occurs for a = b = c = .
2
Second Solution. Obviously, among a2 , b2 , c 2 there are two either less than or equal to
1 1
, or greater than or equal to . Let b and c be these numbers; that is,
4 4

(4b2 − 1)(4c 2 − 1) ≥ 0.

Then, we have
 2  2 
16 2 2 4b − 1 4c − 1
(b + 1)(c + 1) = 5 +1 +1
5 5 5

L
.M
4b2 − 1 4c 2 − 1
 
≥5 + + 1 = 4b2 + 4c 2 + 3.
5 5

D
Hence, it suffices to prove that

A
(a2 + 1)(4b2 + 4c 2 + 3) ≥ (a + b + c + 1)2 .
PI
Writing this inequality as
M
1 1 1
 ‹
LY

2
a + + + (1 + 4b2 + 4c 2 + 2) ≥ (a + b + c + 1)2 ,
4 4 2
O

we recognize the Cauchy-Schwarz inequality.


.M
W

P 2.11. If a, b, c are real numbers, then


W

(a) a6 + b6 + c 6 − 3a2 b2 c 2 + 2(a2 + bc)(b2 + ca)(c 2 + a b) ≥ 0;


W

(b) a6 + b6 + c 6 − 3a2 b2 c 2 ≥ (a2 − 2bc)(b2 − 2ca)(c 2 − 2a b).

Solution. (a) Since


X X
(a2 + bc)(b2 + ca)(c 2 + a b) = 2a2 b2 c 2 + a3 b3 + a bc a3 ,

we can write the desired inequality as follows


X X X
a6 + 2 a3 b3 + 2a bc a3 + a2 b2 c 2 ≥ 0,
X X
( a3 )2 + 2a bc a3 + a2 b2 c 2 ≥ 0,
44 Vasile Cîrtoaje

X
( a3 + a bc)2 ≥ 0.

The equality holds for a3 + b3 + c 3 + a bc = 0.


(b) Since
X X
(a2 − 2bc)(b2 − 2ca)(c 2 − 2a b) = −7a2 b2 c 2 − 2 a3 b3 + 4a bc a3 ,

we can write the desired inequality as follows


X X X
a6 + 2 a3 b3 − 4a bc a3 + 4a2 b2 c 2 ≥ 0,

L
X X
( a3 )2 − 4a bc a3 + 4a2 b2 c 2 ≥ 0,

.M
X
( a3 − 2a bc)2 ≥ 0.

D
The equality holds for a3 + b3 + c 3 − 2a bc = 0.

A
PI
M
P 2.12. If a, b, c are real numbers, then

2 6
LY

(a + b6 + c 6 ) + a3 b3 + b3 c 3 + c 3 a3 + a bc(a3 + b3 + c 3 ) ≥ 0.
3
O

Solution. Write the inequality as follows


.M

4 6
(a + b6 + c 6 ) + 2(a3 b3 + b3 c 3 + c 3 a3 ) + 2a bc(a3 + b3 + c 3 ) ≥ 0,
3
W

1 6
(a + b6 + c 6 ) + (a3 + b3 + c 3 )2 + 2a bc(a3 + b3 + c 3 ) ≥ 0.
W

3
By virtue of the AM-GM inequality, we have
W

1 6
(a + b6 + c 6 ) ≥ a2 b2 c 2 .
3
Therefore, it suffices to show that

a2 b2 c 2 + (a3 + b3 + c 3 )2 + 2a bc(a3 + b3 + c 3 ) ≥ 0,

which is equivalent to
(a bc + a3 + b3 + c 3 )2 ≥ 0.
The equality holds for −a = b = c (or any cyclic permutation).
Symmetric Polynomial Inequalities in Real Variables 45

P 2.13. If a, b, c are real numbers, then

4(a2 + a b + b2 )(b2 + bc + c 2 )(c 2 + ca + a2 ) ≥ (a − b)2 (b − c)2 (c − a)2 .

(Vasile Cîrtoaje, 2009)

Solution. Using the identity

4x y = (x + y)2 − (x − y)2 ,

we have

L
4(a2 + a b + b2 )(a2 + ac + c 2 ) = [(2a2 + a b + ac + 2bc) + (b − c)2 ]2 − (a + b + c)2 (b − c)2

.M
= (2a2 + a b + ac + 2bc)2 + 3a2 (b − c)2 .

D
From this result and

A
4(b2 + bc + c 2 ) = (b − c)2 + 3(b + c)2 ,
PI
M
we get
LY

Y
(a2 + a b + b2 ) = (2a2 + a b + ac + 2bc)2 + 3a2 (b − c)2 (b − c)2 + 3(b + c)2 .
  
16
O

So, the Cauchy-Schwarz inequality gives


.M

Y 2
(a2 + a b + b2 ) ≥ (2a2 + a b + ac + 2bc)(b − c) + 3a(b − c)(b + c)

16
= 4(b − c)2 (a − b)2 (a − c)2 .
W

The equality holds for a b(a + b) + bc(b + c) + ca(c + a) = 0.


W

Remark. The inequality is a consequence of the identity


W

Y
4 (a2 + a b + b2 ) = 3[a b(a + b) + bc(b + c) + ca(c + a)]2 + (a − b)2 (b − c)2 (c − a)2 .

P 2.14. If a, b, c are real numbers, then

(a2 + a b + b2 )(b2 + bc + c 2 )(c 2 + ca + a2 ) ≥ 3(a2 b + b2 c + c 2 a)(a b2 + bc 2 + ca2 ).

(Gabriel Dospinescu, 2009)


46 Vasile Cîrtoaje

Solution (by Vo Quoc Ba Can). As we have shown in the proof of the preceding P 2.13,
Y
(a2 + a b + b2 ) = (2a2 + a b + ac + 2bc)2 + 3a2 (b − c)2 3(b + c)2 + (b − c)2 .
  
16

Thus, by the Cauchy-Schwarz inequality, we get


Y
16 (a2 + a b + b2 ) ≥ 3[(b + c)(2a2 + a b + ac + 2bc) + a(b − c)2 ]2
= 12[(a2 b + b2 c + c 2 a) + (a b2 + bc 2 + ca2 )]2 .

To prove the desired inequality, it suffices to show that

[(a2 b + b2 c + c 2 a) + (a b2 + bc 2 + ca2 )]2 ≥ 4(a2 b + b2 c + c 2 a)(a b2 + bc 2 + ca2 ),

L
.M
which follows immediately from the AM-GM inequality. The equality holds when two
of a, b, c are equal.

D
Remark. The inequality is a consequence of the identity

A
Y
(a2 + a b + b2 ) = 3(a2 b + b2 c + c 2 a)(a b2 + bc 2 + ca2 ) + (a − b)2 (b − c)2 (c − a)2 .
PI
M
LY

P 2.15. If a, b, c are real numbers such that a bc > 0, then


O

1 1 1
 ‹ ‹ ‹
4 a+ b+ c+ ≥ 9(a + b + c).
a b c
.M

Solution. Write the inequality as


W

4(a2 + 1)(b2 + 1)(c 2 + 1) ≥ 9a bc(a + b + c).


W

p
First Solution. It is easy to checkpthat the equality
p occurs
p for a = b = c = 2. There-
fore, using the substitution a = x 2, b = y 2, c = z 2, the inequality can be written
W

as
(2x 2 + 1)(2 y 2 + 1)2z 2 + 1) ≥ 9x yz(x + y + z).
Since
1X 2
(x y + yz + z x)2 − 3x yz(x + y + z) = x ( y − z)2 ≥ 0,
2
it suffices to prove the stronger inequality

(2x 2 + 1)(2 y 2 + 1)(2z 2 + 1) ≥ 3(x y + yz + z x)2 .

Let
A = ( y 2 − 1)(z 2 − 1), B = (z 2 − 1)(x 2 − 1), C = (x 2 − 1)( y 2 − 1).
Symmetric Polynomial Inequalities in Real Variables 47

From
ABC = (x 2 − 1)2 ( y 2 − 1)2 (z 2 − 1)2 ≥ 0,

it follows that at least one of A, B, C is nonnegative. Due to symmetry, assume that

A = ( y 2 − 1)(z 2 − 1) ≥ 0.

Applying the Cauchy-Schwarz inequality, we have

(x y + yz + z x)2 ≤ (x 2 + 1 + x 2 )( y 2 + y 2 z 2 + z 2 ).

L
Therefore, it suffices to show that

.M
(2 y 2 + 1)(2z 2 + 1) ≥ 3( y 2 + y 2 z 2 + z 2 ),

D
which reduces to the obvious inequality

A
( y 2 − 1)(z 2 − 1) ≥ 0.
PI
M
Second Solution. Since
LY

4(b2 + 1)(c 2 + 1) − 3[(b + c)2 + b2 c 2 ] = (b − c)2 + (bc − 2)2 ≥ 0,


O

it suffices to show that


.M

(a2 + 1)[(b + c)2 + b2 c 2 ] ≥ 3a bc(a + b + c).

Applying the Cauchy-Schwarz inequality, we get


W

(a2 + 1)[(b + c)2 + b2 c 2 ] ≥ [a(b + c) + bc]2 ≥ 3a bc(a + b + c).


W
W

P 2.16. If a, b, c are real numbers, then

(a) (a2 + 2bc)(b2 + 2ca)(c 2 + 2a b) ≤ (a2 + b2 + c 2 )(a b + bc + ca)2 ;

(b) (2a2 + bc)(2b2 + ca)(2c 2 + a b) ≤ (a + b + c)2 (a2 b2 + b2 c 2 + c 2 a2 ).

(Vasile Cîrtoaje, 2005)


48 Vasile Cîrtoaje

Solution. (a) Let q = a b + bc + ca. Since

a2 + 2bc = q + (a − b)(a − c),

b2 + 2ca = q + (b − c)(b − a),

c 2 + 2a b = q + (c − a)(c − b),

we can write the required inequality as follows

q2 (a2 + b2 + c 2 ) ≥ [q + (a − b)(a − c)][q + (b − c)(b − a)][q + (c − a)(c − b],

L
X
q2 (a2 + b2 + c 2 ) ≥ q3 + q2 (a − b)(a − c) − (a − b)2 (b − c)2 (c − a)2 .

.M
Since X
(a − b)(a − c) = a2 + b2 + c 2 − q,

D
A
the inequality reduces to the obvious form
PI
(a − b)2 (b − c)2 (c − a)2 ≥ 0.
M
The equality holds for a = b, or b = c, or c = a.
LY

(b) Since For a = 0, this inequality reduces to b2 c 2 (b − c)2 ≥ 0. Otherwise, for


a bc 6= 0, the inequality follows from the inequality in (a) by substituting a, b, c with
O

1/a, 1/b, 1/c, respectively. The equality occurs for a = b, or b = c, or c = a.


.M
W

P 2.17. If a, b, c are real numbers such that a b + bc + ca ≥ 0, then


W

27(a2 + 2bc)(b2 + 2ca)(c 2 + 2a b) ≤ (a + b + c)6 .


W

Solution. In virtue of the AM-GM inequality, we have

(a + b + c)6 = [a2 + b2 + c 2 + (a b + bc + ca) + (a b + bc + ca)]3


≥ 27(a2 + b2 + c 2 )(a b + bc + ca)2 .

Thus, the required inequality follows immediately from the inequality (a) in P 2.16. The
equality holds for a = b = c.
Symmetric Polynomial Inequalities in Real Variables 49

P 2.18. If a, b, c are real numbers such that a2 + b2 + c 2 = 2, then

(a2 + 2bc)(b2 + 2ca)(c 2 + 2a b) + 2 ≥ 0.

(Vasile Cîrtoaje, 2011)


Solution (by Vo Quoc Ba Can). If a, b, c have the same sign, then the inequality is triv-
ial. Otherwise, since the inequality is symmetric and does not change by substituting
−a, −b, −c for a, b, c, respectively, it suffices to consider the case where a ≤ 0 and
b, c ≥ 0. Substituting −a for a, we need to prove that

(a2 + 2bc)(b2 − 2ac)(c 2 − 2a b) + 2 ≥ 0

L
for all a, b, c ≥ 0 satisfying a2 + b2 + c 2 = 2. If b2 −2ac and c 2 −2a b have the same sign,

.M
then the inequality is also trivial. Due to symmetry in b and c, we may assume that

b2 − 2ac ≥ 0 ≥ c 2 − 2a b.

D
A
On the other hand, it is easy to check that the desired inequality becomes an equality
for a = b = 1 and c = 0, when a2 + 2bc = b2 − 2ac = a b − c 2 /2. Then, we rewrite the
PI
desired inequality in the form
M
c2
 
2 2
(a + 2bc)(b − 2ac) a b − ≤ 1.
LY

2
Using the AM-GM inequality, we have
O

3
c2 c2
   
.M

2 2 2 2
27(a + 2bc)(b − 2ac) a b − ≤ (a + 2bc) + (b − 2ac) + a b − .
2 2

Thus, it suffices to prove that


W

c2
 
2 2
(a + 2bc) + (b − 2ac) + a b −
W

≤ 3.
2
W

This inequality can be written in the homogeneous form

2(a2 + 2bc) + 2(b2 − 2ac) + (2a b − c 2 ) ≤ 3(a2 + b2 + c 2 ),

which is equivalent to (a − b + 2c)2 ≥ 0. The equality occurs for for a = 0 and b + c = 0


(or any cyclic permutation).
Remark. In the same manner, we can prove the following generalization.
• Let a, b, c be real numbers such that a2 + b2 + c 2 = 2. If 0 < k ≤ 2, then

(a2 + k bc)(b2 + kca)(c 2 + ka b) + k ≥ 0.


50 Vasile Cîrtoaje

P 2.19. If a, b, c are real numbers such that a + b + c = 3, then

3(a4 + b4 + c 4 ) + a2 + b2 + c 2 + 6 ≥ 6(a3 + b3 + c 3 ).

(Vasile Cîrtoaje, 2006)

Solution. Write the inequality as F (a, b, c) ≥ 0, where

F (a, b, c) = 3(a4 + b4 + c 4 ) + (a2 + b2 + c 2 ) − 6(a3 + b3 + c 3 ) + 6.

Due to symmetry, we may assume that a ≤ b ≤ c. To prove the required inequality, we


use the mixing variable method. More precisely, we show that

L
F (a, b, c) ≥ F (a, x, x) ≥ 0,

.M
where x = (b + c)/2, x ≥ 1. We have

D
F (a, b, c) − F (a, x, x) = 3(b4 + b4 − 2x 4 ) + (b2 + c 2 − 2x 2 ) − 6(b3 + c 3 − 2x 3 )

A
= 3[(b2 + c 2 )2 − 4x 4 ] + 6(x 4 − b2 c 2 ) + (b2 + c 2 − 2x 2 ) − 6(b3 + c 3 − 2x 3 )
PI
= (b2 + c 2 − 2x 2 )[3(b2 + c 2 + 2x 2 ) + 1] + 6(x 2 − bc)(x 2 + bc) − 12x(b2 + c 2 − bc − x 2 ).
M
Since
1
b2 + c 2 − 2x 2 = (b − c)2 ,
LY

2
1
x 2 − bc = (b − c)2 ,
O

4
3
.M

b2 + c 2 − bc − x 2 = (b − c)2 ,
4
we have
W

1
F (a, b, c) − F (a, x, x) =(b − c)2 [3(b2 + c 2 + 2x 2 ) + 1 + 3(x 2 + bc) − 18x]
W

2
1
= (b − c)2 [3(x 2 − bc) + 18x(x − 1) + 1] ≥ 0.
W

2
Also,
F (a, x, x) = F (3 − 2x, x, x) = 6(x − 1)2 (3x − 4)2 ≥ 0.
This completes the proof. The equality holds for a = b = c = 1, and for a = 1/3 and
b = c = 4/3 (or any cyclic permutation).

P 2.20. If a, b, c are real numbers such that a bc = 1, then

3(a2 + b2 + c 2 ) + 2(a + b + c) ≥ 5(a b + bc + ca).


Symmetric Polynomial Inequalities in Real Variables 51

Solution. From a bc = 1, it follows that either a, b, c > 0, or one of a, b, c is positive and


the others are negative. In the last case, due to symmetry, we may assume that a > 0
and b, c < 0.
Case 1: a, b, c > 0. Let p = a + b + c and q = a b + bc + ca. By the AM-GM inequality
3
p
a + b + c ≥ 3 a bc,

we get p ≥ 3, while by Schur’s inequality

p3 + 9a bc ≥ 4pq,

we get

L
p3 + 9

.M
q≤ .
4p
Write the required inequality as

D
3(p2 − 2q) + 2p ≥ 5q,

A
3p2 + 2p ≥ 11q. PI
This is true since
M
11(p3 + 9) (p − 3)(p2 + 11p + 33)
LY

3p2 + 2p − 11q ≥ 3p2 + 2p − = ≥ 0.


4p 4p
O

Case 2: a > 0 and b, c < 0. Substituting −b for b and −c for c, we need to prove that
.M

3(a2 + b2 + c 2 ) + 2a + 5a(b + c) ≥ 2(b + c) + 5bc

for a, b, c > 0 satisfying a bc = 1. It suffices to show that


W

3(b2 + c 2 ) − 5bc ≥ (2 − 5a)(b + c).


W

Since p
3(b2 + c 2 ) − 5bc b+c bc 1
W

≥ ≥ = p ,
b+c 4 2 2 a
we need to prove that
1
p ≥ 2 − 5a.
2 a
Indeed, by the AM-GM inequality,
v
1 1 1 t 1 1
5a + p = 5a + p + p ≥ 3 3 5a · p · p > 2.
2 a 4 a 4 a 4 a 4 a
This completes the proof. The equality holds for a = b = c = 1.
52 Vasile Cîrtoaje

P 2.21. If a, b, c are real numbers such that a bc = 1, then

3 1 1 1
 ‹
2 2 2
a + b +c +6≥ a+b+c+ + + .
2 a b c

Solution. Substituting a, b, c by |a|, |b|, |c|, respectively, the left side of the inequality
remains unchanged, while the right side either remains unchanged or increases. There-
fore, it suffices to prove the inequality only for a, b, c > 0.

First Solution. Write the inequality in the form

L
.M
3(6x 2 − 3x + 4) ≥ 7(a b + bc + ca),

a+b+c

D
where x = . By virtue of the AM-GM inequality, we get x ≥ 1. The third degree
3

A
Schur’s inequality states that
PI
(a + b + c)3 + 9a bc ≥ 4(a + b + c)(a b + bc + ca),
M
which is equivalent to
LY

3(3x 3 + 1)
a b + bc + ca ≤ .
4x
O

Therefore, it suffices to show that


.M

21(3x 3 + 1)
3(6x 2 − 3x + 4) ≥ .
4x
W

This inequality reduces to (x − 1)(3x 2 − 9x + 7) ≥ 0, which is true because


W

3 1
3x 2 − 9x + 7 = 3(x − )2 + > 0.
W

2 4

The equality holds for a = b = c = 1.

Second Solution. Due to symmetry, assume that a = min{a, b, c}, and then use the
mixing variable method. Let

3 1 1 1
 ‹
F (a, b, c) = a2 + b2 + c 2 + 6 − a+b+c+ + + .
2 a b c

We will show that


p p
F (a, b, c) ≥ F (a, bc, bc) ≥ 0,
Symmetric Polynomial Inequalities in Real Variables 53

p
where x = bc (x ≥ 1). We have
3 1 1 2
p p  p ‹
2
F (a, b, c) − F (a, bc, bc) = (b − c) − b + c − 2 bc + + − p
2 b c
˜ bc
1 p p 2 p 2 3
• p
= ( b − c) 2( b + c) − 3 −
2 bc
1 p p 2 p 3
 ‹
≥ ( b − c) 8 bc − 3 −
2 bc
1 p p 2
≥ ( b − c) (8 − 3 − 3) ≥ 0
2

L
and

.M
p p 1
F (a, bc, bc) = F ( , x, x)
x2
x 6 − 6x 5 + 12x 4 − 6x 3 − 3x 2 + 2

D
=
2x 4

A
(x − 1) (x − 4x 3 + 3x 2 + 4x + 2
2 4
= PI
2x 4
(x − 1) [(x − 2x − 1)2 + x 2 + 1]
2 2
M
= ≥ 0.
2x 4
LY
O

P 2.22. If a, b, c are real numbers, then


.M

1
(1 + a2 )(1 + b2 )(1 + c 2 ) + 8a bc ≥ (1 + a)2 (1 + b)2 (1 + c)2 .
4
W
W

Solution. It is easy to check that


W

(1 + a2 )(1 + b2 )(1 + c 2 ) + 8a bc = (1 + a bc)2 + (a + bc)2 + (b + ca)2 + (c + a b)2 .

Thus, using the Cauchy-Schwarz inequality, we have

[(1 + a bc) + (a + bc) + (b + ca) + (c + a b)]2


(1 + a2 )(1 + b2 )(1 + c 2 ) + 8a bc ≥
4
1
(1 + a)2 (1 + b)2 (1 + c)2 .
=
4
The equality holds for b = c = 1 (or any cyclic permutation), and also for a = b = c =
−1.
54 Vasile Cîrtoaje

P 2.23. Let a, b, c be real numbers such that a + b + c = 0. Prove that


2049 4 4 4
a12 + b12 + c 12 ≥ a b c .
8

Solution. Consider only the nontrivial case a bc 6= 0, and rewrite the inequality as fol-
lows
2049 4 4
a12 + b12 + (a + b)12 ≥ a b (a + b)4 ,
8
2049 4 4 2
(a6 + b6 )2 − 2a6 b6 + (a2 + b2 + 2a b)6 ≥ a b (a + b2 + 2a b)2 .
8

L
Let us denote

.M
a2 + b2
d= , |d| ≥ 2.
ab

D
Since
a6 + b6 = (a2 + b2 )3 − 3a2 b2 (a2 + b2 ),

A
the inequality can be restated as PI
2049
M
(d 3 − 3d)2 − 2 + (d + 2)6 ≥ (d + 2)2 ,
8
LY

which is equivalent to
O

(d − 2)(2d + 5)2 (4d 3 + 12d 2 + 87d + 154) ≥ 0.


.M

Since this inequality is obvious for d ≥ 2, we only need to show that

4d 3 + 12d 2 + 87d + 154 ≤ 0


W

for d ≤ −2. Indeed,


W

4d 3 + 12d 2 + 87d + 154 < 4d 3 + 12d 2 + 85d + 154


W

= (d + 2)[(2d + 1)2 + 76] ≤ 0.

The equality holds for a = b = −c/2 (or any cyclic permutation).

P 2.24. If a, b, c are real numbers such that a bc ≥ 0, then

a2 + b2 + c 2 + 2a bc + 4 ≥ 2(a + b + c) + a b + bc + ca.

(Vasile Cîrtoaje, 2012)


Symmetric Polynomial Inequalities in Real Variables 55

Solution. Let us denote

x = a(b − 1)(c − 1), y = b(c − 1)(a − 1), z = c(a − 1)(b − 1).

Since
x yz = a bc(a − 1)2 (b − 1)2 (c − 1)2 ≥ 0,
at least one of x, y, z is nonnegative; let a(1 − b)(1 − c) ≥ 0. Thus, we have

a bc ≥ a(b + c − 1),

and it suffices to show that

L
a2 + b2 + c 2 + 2a(b + c − 1) + 4 ≥ 2(a + b + c) + a b + bc + ca,

.M
which is equivalent to

D
a2 − (4 − b − c)a + b2 + c 2 − bc − 2(b + c) + 4 ≥ 0,

A
b+c 2 3
 ‹
a−2+
2 4
PI
+ (b − c)2 ≥ 0.
M
The equality holds for a = b = c = 1, and also for a = 0 and b = c = 2 (or any cyclic
permutation).
LY
O

P 2.25. Let a, b, c be real numbers such that a + b + c = 3.


.M

(a) If a, b, c ≥ −3, then


1 1 1 1 1 1
W

2
+ 2+ 2≥ + + .
a b c a b c
W

(b) If a, b, c ≥ −7, then


W

1−a 1− b 1−c
+ + ≥ 0.
(1 + a)2 (1 + b)2 (1 + c)2
(Vasile Cîrtoaje, 2012)

Solution. Assume that a = min{a, b, c} and denote


b+c
t= , a + 2t = 3.
2
(a) From a, b, c ≥ −3 and a + b + c = 3, it follows that
a+b+c
−3 ≤ a ≤ = 1.
3
56 Vasile Cîrtoaje

We will show that


E(a, b, c) ≥ E(a, t, t) ≥ 0,
where
1 1 1 1 1 1
E(a, b, c) = + + − − − .
a2 b2 c 2 a b c
We have
1 1 2 1 1 2
 ‹
E(a, b, c) − E(a, t, t) = + − − + −
b2 c 2 t 2 b c t
(b − c)2 (b2 + c 2 + 4bc) (b − c)2
= −
b2 c 2 (b + c)2 bc(b + c)

L
(b − c)2 [(b + c)2 − bc(b + c − 2)]

.M
= ≥ 0,
b2 c 2 (b + c)2
since

D
(b + c)2 − bc(b + c − 2) = (b + c)2 − bc(1 − a)

A
(b + c)2 (1 − a) (b + c)2 (3 + a)
≥ (b + c)2 −
4
= PI4
≥ 0.
M
Also,
1 − a 2(1 − t) 3(a − 1)2 (a + 3)
E(a, t, t) = + = ≥ 0.
LY

a2 t2 a2 (3 − a)2
The equality holds for a = b = c = 1, and also for a = −3 and b = c = 3 (or any cyclic
O

permutation).
.M

(b) From
a+b+c 3−a
t≥ = 1, t= ≤ 5,
3 2
W

it follows that
t ∈ [1, 5].
W

We will show that


W

E(a, b, c) ≥ E(a, t, t) ≥ 0,
where
1−a 1− b 1−c
E(a, b, c) = + + .
(1 + a)2 (1 + b)2 (1 + c)2
Write the left inequality as follows:

1− b 1− t 1−c 1− t
• ˜ • ˜
− + − ≥ 0,
(1 + b)2 (1 + t)2 (1 + c)2 (1 + t)2

(b − 1)t − b − 3 (c − 1)t − c − 3
• ˜
(b − c) − ≥ 0,
(1 + b)2 (1 + c)2
Symmetric Polynomial Inequalities in Real Variables 57

(b − c)2 [(3 + b + c − bc)t + 3(b + c) + bc] ≥ 0,


(b − c)2 [2t 2 + 9t + 5 − bc(t − 1)] ≥ 0.
The last inequality is true since

2t 2 + 9t + 5 − bc(t − 1) ≥ 2t 2 + 9t + 5 − t 2 (t − 1) = (5 − t)(1 + t)2 ≥ 0.

Also, we have
1−a 2(1 − t) t −1 2(1 − t)
E(a, t, t) = + = +
(1 + a)2 (1 + t)2 2(2 − t)2 (1 + t)2

L
3(1 − t)2 (5 − t)
= ≥ 0.

.M
2(2 − t)2 (1 + t)2
The proof is completed. The equality occurs for a = b = c = 1, and also for a = −7 and
b = c = 5 (or any cyclic permutation).

D
A
P 2.26. If a, b, c are real numbers, then
PI
M
1
a6 + b6 + c 6 − 3a2 b2 c 2 ≥ (a − b)2 (b − c)2 (c − a)2 .
LY

2
(Sungyoon Kim, 2006)
O

Solution. Applying the Cauchy-Schwarz inequality, we have


.M

1 2
a6 + b6 + c 6 − 3a2 b2 c 2 = (a + b2 + c 2 )[(b2 − c 2 )2 + (c 2 − a2 )2 + (a2 − b2 )2 ]
2
W

1
≥ [a(b2 − c 2 ) + b(c 2 − a2 ) + c(a2 − b2 )]2
2
W

1
= (a − b)2 (b − c)2 (c − a)2 .
2
W

Thus, the proof is completed. The equality holds for a = b = c, and for a = 0 and
b + c = 0 (or any cyclic permutation).

P 2.27. If a, b, c are real numbers, then


3
a + b2 + c 2
 2
1
≥ a2 b2 c 2 + (a − b)2 (b − c)2 (c − a)2 .
3 16

(Vasile Cîrtoaje, 2011)


58 Vasile Cîrtoaje

Solution (by Vo Quoc BapCan). Without loss of generality, assume that b and c have the
same sign. Denote x = (b2 + c 2 )/2. Since
3 3
a2 + b2 + c 2 a2 + 2x 2
 
2 2 2
−a b c = − a2 x 4 + a2 (x 4 − b2 c 2 )
3 3
(a2 − x 2 )2 (a2 + 8x 2 )
= + a2 (x 4 − b2 c 2 )
27
(2a2 − b2 − c 2 )2 (a2 + 4b2 + 4c 2 ) a2 (b2 − c 2 )2
= + ,
108 4
the desired inequality can be rewritten as

L
27

.M
(2a2 − b2 − c 2 )2 (a2 + 4b2 + 4c 2 ) ≥ (b − c)2 [(a − b)2 (a − c)2 − 4a2 (b + c)2 ].
4

D
1
According to the inequalities x 2 − y 2 ≤ 2x(x + y) and 2x y ≤ (x + y)2 , we have
2

A
(a − b)2 (a − c)2 − 4a2 (b + c)2 ≤ 2(a − b)(a − c)[(a − b)(a − c) + 2a(b + c)]
PI
1
= 2(a2 − b2 )(a2 − c 2 ) ≤ (2a2 − b2 − c 2 )2 .
M
2
LY

Therefore, it suffices to show that

8(a2 + 4b2 + 4c 2 ) ≥ 27(b − c)2 ,


O

which is equivalent to the obvious inequality 8a2 + 5b2 + 5c 2 + 54bc ≥ 0. The equality
.M

holds for a = b = c, and for −a = b = c (or any cyclic permutation).


W
W

P 2.28. If a, b, c are real numbers, then


108 2 2 2
W

(a2 + b2 + c 2 )3 ≥ a b c + 2(a − b)2 (b − c)2 (c − a)2 .


5
(Vo Quoc Ba Can and Vasile Cîrtoaje, 2011)

Solution. Write the inequality as f (a, b, c) ≥ 0, where


108 2 2 2
f (a, b, c) = (a2 + b2 + c 2 )3 − a b c − 2(a − b)2 (b − c)2 (c − a)2 .
5
Without loss of generality, assume that b and c have the same sign. Since f (−a, −b, −c) =
f (a, b, c), we may consider b ≥ 0, c ≥ 0. In addition, for a > 0, we have

f (a, b, c) − f (−a, b, c) = 8a(b + c)(a2 + bc)(b − c)2 ≥ 0.


Symmetric Polynomial Inequalities in Real Variables 59

Therefore, it suffices to prove the desired inequality for a ≤ 0, b ≥ 0, c ≥ 0. For


b = c = 0, the inequality is trivial. Otherwise, due to homogeneity, we may assume that
b + c = 1. Denoting x = bc, we can write the desired inequality as follows:

108 2 2
(a2 + 1 − 2x)3 ≥ a x + 2(1 − 2x)(a2 − a + x)2,
5
2
(4a − 5)2 x 2 + 2(a + 1)(a3 − 9a2 + 5a − 3)x + (a + 1)2 (a4 − 2a3 + 4a2 − 2a + 1) ≥ 0.
5
This inequality holds if

L
(4a − 5)2 (a4 − 2a3 + 4a2 − 2a + 1) ≥ (a3 − 9a2 + 5a − 3)2 .
5

.M
Since

D
10(a4 − 2a3 + 4a2 − 2a + 1) = (a + 1)2 + (3a2 − 4a + 3)2 ≥ (3a2 − 4a + 3)2 ,

A
it suffices to prove that PI
(4a − 5)2 (3a2 − 4a + 3)2 ≥ 25(a3 − 9a2 + 5a − 3)2 .
M
LY

This is true for a ≤ 0 if

(5 − 4a)(3a2 − 4a + 3) ≥ 5(−a3 + 9a2 − 5a + 3),


O
.M

which reduces to
a(a + 1)2 ≤ 0.
Thus, the proof is completed. The equality holds for a = 0 and b + c = 0 (or any cyclic
W

permutation).
W
W

P 2.29. If a, b, c are real numbers, then

2(a2 + b2 )(b2 + c 2 )(c 2 + a2 ) ≥ (a − b)2 (b − c)2 (c − a)2 .

(Vasile Cîrtoaje, 2011)

First Solution. Since


2(a2 + b2 ) = (a − b)2 + (a + b)2
and
(b2 + c 2 )(c 2 + a2 ) = (a b + c 2 )2 + (bc − ac)2 ,
60 Vasile Cîrtoaje

by virtue of the Cauchy-Schwarz inequality, we have

2(a2 + b2 )(b2 + c 2 )(c 2 + a2 ) ≥ [(a − b)(a b + c 2 ) + (a + b)(bc − ac)]2


= (a2 b + b2 c + c 2 a − a b2 − bc 2 − ca2 )2
= (a − b)2 (b − c)2 (c − a)2 .

This completes the proof. The equality holds for (a − b)(bc − ac) = (a + b)(a b + c 2 ),
which is equivalent to
(a + b + c)(a b + bc + ca) = 5a bc.

L
Second Solution. Making the substitution

.M
X X
x= a b2 = a b2 + bc 2 + ca2 , y= a2 b = a2 b + b2 c + c 2 a,

D
we have X X X

A
a2 b4 = ( a b2 )2 − 2a bc a2 b = x 2 − 2a bc y,
X
a4 b2 = (
X
a2 b)2 − 2a bc
X PI
a b2 = y 2 − 2a bc x,
M
and hence
LY

Y X X
(a2 + b2 ) = a2 b4 + a4 b2 + 2a2 b2 c 2
O

= x 2 + y 2 − 2a bc(x + y) + 2a2 b2 c 2 .
.M

Then, the desired inequality is equivalent to

2[x 2 + y 2 − 2a bc(x + y) + 2a2 b2 c 2 ] ≥ (x − y)2 ,


W

(x + y)2 − 4a bc(x + y) + 4a2 b2 c 2 ≥ 0,


W

(x + y − 2a bc)2 ≥ 0.
W

The last inequality is obviously true.

P 2.30. If a, b, c are real numbers, then

32(a2 + bc)(b2 + ca)(c 2 + a b) + 9(a − b)2 (b − c)2 (c − a)2 ≥ 0.

(Vasile Cîrtoaje, 2011)


Symmetric Polynomial Inequalities in Real Variables 61

Solution (by Vo Quoc Ba Can). For a, b, c ≥ 0, the inequality is trivial. Otherwise, since
the inequality is symmetric and does not change by substituting −a, −b, −c for a, b, c,
we may assume that a ≤ 0 and b, c ≥ 0. Substituting −a for a, we need to prove that

32(a2 + bc)(b2 − ac)(c 2 − a b) + 9(a + b)2 (a + c)2 (b − c)2 ≥ 0

for all a, b, c ≥ 0. By the AM-GM inequality, we have

(a + b)2 (a + c)2 = [a(b + c) + (a2 + bc)]2 ≥ 4a(b + c)(a2 + bc).

Thus, it suffices to prove that

L
8(b2 − ac)(c 2 − a b) + 9a(b + c)(b − c)2 ≥ 0.

.M
Since

D
(b2 − ac)(c 2 − a b) = bc(bc + a2 ) − a(b3 + c 3 )
p

A
p p
≥ 2a bc bc − a(b3 + c 3 ) = −a(b b − c c)2 ,

it is enough to show that


PI
M
p p
9(b + c)(b − c)2 − 8(b b − c c)2 ≥ 0.
LY

p p
Setting b = x and c = y, the inequality can be rewritten as
O

(x − y)2 [9(x 2 + y 2 )(x + y)2 − 8(x 2 + x y + y 2 )2 ] ≥ 0.


.M

This follows from the Cauchy-Schwarz inequality as follows

9(x 2 + y 2 )(x + y)2 = 9[(x − y)2 + 2x y][(x − y)2 + 4x y]


W

 p 2
2
p 2 2 2 2
p
≥ 9[(x − y) + 2 2x y] ≥ 9 (x − y) + 2 2x y
W

3
= 8(x 2 + x y + y 2 )2 ≥ 0.
W

The equality occurs when two of a, b, c are zero, and when −a = b = c (or any cyclic
permutation).

P 2.31. If a, b, c are real numbers, then


1
a4 (b − c)2 + b4 (c − a)2 + c 4 (a − b)2 ≥ (a − b)2 (b − c)2 (c − a)2 .
2
(Vasile Cîrtoaje, 2011)
62 Vasile Cîrtoaje

Solution. Since
1 2
b4 (c − a)2 + c 4 (a − b)2 ≥ [b (c − a) + c 2 (a − b)]2
2
1
= (b − c)2 (bc − ca − a b)2 ,
2

it suffices to prove that

2a4 + (a b − bc + ca)2 ≥ (a − b)2 (a − c)2 ,

which is equivalent to

L
a2 (a2 − 2bc + 2ca + 2a b) ≥ 0.

.M
Therefore, the desired inequality is true if

D
a2 − 2bc + 2ca + 2a b ≥ 0.

A
Indeed, from X X PI
(a2 − 2bc + 2ca + 2a b) = (a + b + c)2 ≥ 0,
M
due to symmetry, we may assume that a2 − 2bc + 2ca + 2a b ≥ 0. Thus, the proof is
LY

completed. The equality occurs when a = b = c, when two of a, b, c are equal to zero,
and when a = 0 and b + c = 0 (or any cyclic permutation).
O
.M

P 2.32. If a, b, c are real numbers, then


W

1
a2 (b − c)4 + b2 (c − a)4 + c 2 (a − b)4 ≥ (a − b)2 (b − c)2 (c − a)2 .
W

2
W

(Vasile Cîrtoaje, 2011)

Solution. Let us denote


X X
x= a b2 = a b2 + bc 2 + ca2 , y= a2 b = a2 b + b2 c + c 2 a.

Since X X X
a2 b4 = ( a b2 )2 − 2a bc a2 b = x 2 − 2a bc y,
X X X
a4 b2 = ( a2 b)2 − 2a bc a b2 = y 2 − 2a bc x,
X
a2 b2 (a2 + b2 ) = x 2 + y 2 − 2a bc(x + y),
Symmetric Polynomial Inequalities in Real Variables 63

we have
X X
a2 (b − c)4 = a2 (b4 − 4b3 c + 6b2 c 2 − 4bc 3 + c 4 )
X X X
= a2 b2 (a2 + b2 ) − 4a bc( a b2 + a2 b) + 18a2 b2 c 2
= x 2 + y 2 − 6a bc(x + y) + 18a2 b2 c 2 .

Therefore, we can write the desired inequality as

1
x 2 + y 2 − 6a bc(x + y) + 18a2 b2 c 2 ≥ (x − y)2 ,
2

L
which is equivalent to the obvious inequality

.M
(x + y − 6a bc)2 ≥ 0.

D
The equality holds for a(b − c)2 + b(c − a)2 + c(a − b)2 = 0.

A
PI
M
P 2.33. If a, b, c are real numbers, then
LY

3
a2 (b2 − c 2 )2 + b2 (c 2 − a2 )2 + c 2 (a2 − b2 )2 ≥ (a − b)2 (b − c)2 (c − a)2 .
8
O

(Vasile Cîrtoaje, 2011)


.M

Solution. We see that the inequality remains unchanged and the product
W

(a + b)(b + c)(c + a)
W

changes its sign by replacing a, b, c with −a, −b, −c, respectively. Thus, without loss of
generality, we may assume that (a + b)(b + c)(c + a) ≥ 0. According to this condition,
W

at least one of a, b, c is nonnegative. So, we may consider a ≥ 0, and hence

a(a + b)(b + c)(c + a) ≥ 0.

In virtue of the Cauchy-Schwarz inequality, we get

1 2 2 1
b2 (c 2 − a2 )2 + c 2 (a2 − b2 )2 ≥ b(c − a2 ) + c(a2 − b2 ) = (b − c)2 (a2 + bc)2 .
2 2
Thus, it suffices to show that

3
2a2 (b + c)2 + (a2 + bc)2 ≥ (a − b)2 (a − c)2 ,
4
64 Vasile Cîrtoaje

which is equivalent to

(a + b)(a + c)[a2 + 5a(b + c) + bc] ≥ 0,

(a + b)(a + c)[(a + b)(a + c) + 4a(b + c)] ≥ 0,


(a + b)2 (a + c)2 + 4a(a + b)(b + c)(c + a) ≥ 0.
Since the last inequality is clearly true, the proof is completed. The equality holds for
a = b = c, for −a = b = c (or any cyclic permutation), and for b = c = 0 (or any cyclic
permutation).

L
.M
P 2.34. If a, b, c are real numbers such that a b + bc + ca = 3, then

D
(a) (a2 + a b + b2 )(b2 + bc + c 2 )(c 2 + ca + a2 ) ≥ 3(a + b + c)2 ;

A
3 2
(b) (a2 + a b + b2 )(b2 + bc + c 2 )(c 2 + ca + a2 ) ≥ (a + b2 + c 2 ).
PI 2
(Vasile Cîrtoaje, 1995)
M
LY

Solution. Let
p = a + b + c, q = a b + bc + ca, r = a bc.
O

We have
.M

Y Y
(b2 + bc + c 2 ) = [(b + c)2 − bc]
Y X X
= (b + c)2 − bc(a + b)2 (a + c)2 + a bc a(b + c)2 − a2 b2 c 2 .
W

Since
W

Y
(b + c)2 = (pq − r)2 = r 2 − 2pqr + p2 q2 ,
W

X X X
bc(a + b)2 (a + c)2 = bc(a2 + q)2 = r a3 + 2pqr + q2

= r(3r + p3 − 3pq) + 2pqr + q2 = 3r 2 + (p3 − pq)r + q3 ,


X
a bc a(b + c)2 = r(3r + pq) = 3r 2 + pqr,
we get Y
(b2 + bc + c 2 ) = (p2 − q)q2 − p3 r.

(a) Write the inequality as follows


Y
3 (b2 + bc + c 2 ) ≥ (a + b + c)2 (a b + bc + ca)2 ,
Symmetric Polynomial Inequalities in Real Variables 65

(2p2 − 3q)q2 − 3p3 r ≥ 0,


q2 (p2 − 3q) + p2 (q2 − 3pr) ≥ 0,
X X
q2 (b − c)2 + p2 a2 (b − c)2 ≥ 0.
Clearly, the last inequality holds for all real a, b, c. The equality holds when a = b =
c = ±1.
(b) Write the inequality in the homogeneous forms
Y
2 (b2 + bc + c 2 ) ≥ (a2 + b2 + c 2 )(a b + bc + ca)2 ,

L
2(p2 − q)q2 − 2p3 r − (p2 − 2q)q2 ≥ 0,

.M
p2 (q2 − 2pr) ≥ 0,
(a + b + c)2 (a2 b2 + b2 c 2 + c 2 a2 ) ≥ 0.

D
A
The equality holds when a + b + c = 0 and a b + bc + ca = 3.
PI
M
P 2.35. If a, b, c are real numbers, then
LY

(a2 + a b + b2 )(b2 + bc + c 2 )(c 2 + ca + a2 ) ≥ 3(a b + bc + ca)(a2 b2 + b2 c 2 + c 2 a2 ).


O

(Vasile Cîrtoaje, 2011)


.M

Solution. As we have shown in the proof of the preceding P 2.34,


W

Y
(b2 + bc + c 2 ) = (p2 − q)q2 − p3 r,
W

where
W

p = a + b + c, q = a b + bc + ca, r = a bc.
Thus, we can write the desired inequality as

(p2 − q)q2 − p3 r ≥ 3q(q2 − 2pr),

q2 (p2 − 4q) + p(6q − p2 )r ≥ 0.


Consider further two cases: 6q − p2 ≥ 0 and 6q − p2 ≤ 0.

Case 1: 6q − p2 ≥ 0. By Schur’s inequality of degree four, we have

6pr ≥ (p2 − q)(4q − p2 ).


66 Vasile Cîrtoaje

Therefore, it suffices to show that

6q2 (p2 − 4q) + (6q − p2 )(p2 − q)(4q − p2 ) ≥ 0,

which is equivalent to the obvious inequality

(p2 − 4q)2 (p2 − 3q) ≥ 0.

Case 2: 6q − p2 ≤ 0. Since 3pr ≤ q2 , it suffices to show that

3q2 (p2 − 4q) + (6q − p2 )q2 ≥ 0,

which is equivalent to the obvious inequality

L
.M
q2 (p2 − 3q) ≥ 0.

The inequality is an equality when a = b = c, and when a = 0 and b = c (or any cyclic

D
permutation).

A
PI
M
P 2.36. If a, b, c are real numbers, not all of the same sign, then
LY

(a2 + a b + b2 )(b2 + bc + c 2 )(c 2 + ca + a2 ) ≥ 3(a b + bc + ca)3 .

(Vasile Cîrtoaje, 2011)


O

Solution. Since the inequality is symmetric and does not change by substituting −a, −b, −c
.M

for a, b, c, we may assume that a ≤ 0 and b, c ≥ 0. Substituting −a for a, we need to


prove that
W

(a2 − a b + b2 )(b2 + bc + c 2 )(c 2 − ca + a2 ) ≥ 3(bc − a b − ac)3


W

for a, b, c ≥ 0. Since the left hand side of this inequality is nonnegative, consider further
the nontrivial case
W

bc − a b − ac > 0.
Since
b2 + bc + c 2 − 3(bc − a b − ac) = (b − c)2 + 3a(b + c) ≥ 0,
it suffices to show that

(a2 − a b + b2 )(a2 − ac + c 2 ) ≥ (bc − a b − ac)2 .

First Solution. From bc − a b − ac > 0, it follows that a = min{a, b, c}. Since a2 − a b +


b2 ≥ (b − a)2 and a2 − ac + c 2 ≥ (c − a)2 , it suffices to show that

(b − a)2 (c − a)2 ≥ (bc − a b − ac)2 .


Symmetric Polynomial Inequalities in Real Variables 67

This is true if (b − a)(c − a) ≥ bc − a b − ac; indeed,

(b − a)(c − a) − (bc − a b − ac) = a2 ≥ 0.

The original inequality is an equality when two of a, b, c are zero, and when a = 0 and
b = c (or any cyclic permutation).
Second Solution. Since

4(a2 − a b + b2 ) = (a + b)2 + 3(a − b)2 ,

4(a2 − ac + c 2 ) = (a + c)2 + 3(a − c)2 ,

L
by the Cauchy-Schwarz inequality, we get

.M
16(a2 − a b + b2 )(b2 + bc + c 2 ) ≥ [(a + b)(a + c) + 3(a − b)(a − c)]2 .

D
Thus, we only need to show that

A
(a + b)(a + c) + 3(a − b)(a − c) ≥ 4(bc − a b − ac),
PI
which is equivalent to the obvious inequality a(2a + b + c) ≥ 0.
M
LY
O

P 2.37. If a, b, c are real numbers, then


3 2
.M

(a2 + a b + b2 )(b2 + bc + c 2 )(c 2 + ca + a2 ) ≥ (a + b2 )(b2 + c 2 )(c 2 + a2 ).


8
(Vasile Cîrtoaje, 2011)
W

Solution. If a, b, c have the same sign, then the inequality follows from
W

a2 + a b + b2 ≥ a2 + b2 , b2 + bc + c 2 ≥ b2 + c 2 , c 2 + ca + a2 ≥ c 2 + a2 .
W

Consider now that a, b, c have not the same sign. Since the inequality is symmetric and
does not change by substituting −a, −b, −c for a, b, c, we may assume that a ≤ 0 and
b, c ≥ 0. Substituting −a for a, we need to prove that
3 2
(a2 − a b + b2 )(a2 − ac + c 2 )(b2 + bc + c 2 ) ≥ (a + b2 )(b2 + c 2 )(c 2 + a2 )
8
for a, b, c ≥ 0. Write this inequality in the form

[(a2 + b2 ) + (a − b)2 ][(a2 + c 2 ) + (a − c)2 ][2(b2 + c 2 ) + 2bc] ≥

≥ 3(a2 + b2 )(b2 + c 2 )(c 2 + a2 ).


68 Vasile Cîrtoaje

It suffices to show that

2(b2 + c 2 )[(a − b)2 (a2 + c 2 ) + (a − c)2 (a2 + b2 )] + 2bc(a2 + b2 )(a2 + c 2 ) ≥

≥ (a2 + b2 )(b2 + c 2 )(c 2 + a2 ),


which is equivalent to

2(b2 + c 2 )[(a − b)2 (a2 + c 2 ) + (a − c)2 (a2 + b2 )] ≥ (b − c)2 (a2 + b2 )(a2 + c 2 ),

or
(a − b)2 (a − c)2 ) (b − c)2
+

L
≥ .
a2 + b2 a2 + c 2 2(b2 + c 2 )

.M
Consider further two cases.
Case 1: 2a2 ≤ b2 + c 2 . By the Cauchy-Schwarz inequality, we have

D
A
(a − b)2 (a − c)2 ) [(b − a) + (a − c)]2 (b − c)2
+ ≥ = .
a2 + b2 a2 + c 2 PI
(a2 + b2 ) + (a2 + c 2 ) 2a2 + b2 + c 2

Thus, it suffices to show that


M
1 1
LY

≥ ,
2a2 +b +c
2 2 2(b + c 2 )
2
O

which reduces to b2 + c 2 ≥ 2a2 .


.M

Case 2: 2a2 ≥ b2 + c 2 . By the Cauchy-Schwarz inequality, we have

(a − b)2 (a − c)2 [c(b − a) + b(a − c)]2 a2 (b − c)2


+ =
W

≥ .
a2 + b2 a2 + c 2 c 2 (a2 + b2 ) + b2 (a2 + c 2 ) a2 (b2 + c 2 ) + 2b2 c 2
W

Therefore, it suffices to prove that


W

a2 1
≥ .
a (b + c ) + 2b c
2 2 2 2 2 2(b + c 2 )
2

This reduces to a2 (b2 + c 2 ) ≥ 2b2 c 2 , which is true because

2a2 (b2 + c 2 ) − 4b2 c 2 ≥ (b2 + c 2 )2 − 4b2 c 2 = (b2 − c 2 )2 ≥ 0.

Thus, the proof is completed. The equality holds when two of a, b, c are zero, and when
−a = b = c (or any cyclic permutation).
Symmetric Polynomial Inequalities in Real Variables 69

P 2.38. If a, b, c are real numbers, then

2(a2 + b2 )(b2 + c 2 )(c 2 + a2 ) ≥ (a2 − a b + b2 )(b2 − bc + c 2 )(c 2 − ca + a2 ).

(Vasile Cîrtoaje, 2014)

Solution. Since the inequality is symmetric and does not change by substituting −a, −b, −c
for a, b, c, we may assume that a ≤ 0 and b, c ≥ 0. Substituting −a for a, we need to
prove that

2(a2 + b2 )(b2 + c 2 )(c 2 + a2 ) ≥ (b2 − bc + c 2 )(c 2 + ca + a2 )(a2 + a b + b2 )

L
for a, b, c ≥ 0. Using the notation

.M
A = b2 + c 2 , B = c 2 + a2 , C = a2 + b2 ,

D
we can write the inequality as follows:

A
PI
2ABC ≥ (A − bc)(B + ca)(C + a b),
M
ABC + a2 b2 c 2 ≥ a b(AB − c 2 C) + ac(AC − b2 B) − bc(BC − a2 A),

ABC + a2 b2 c 2 ≥ a b(c 4 + a2 b2 ) + ac(b4 + a2 c 2 ) − bc(a4 + b2 c 2 ).


LY

It suffices to show that


O

ABC + a2 b2 c 2 ≥ a b(c 4 + a2 b2 ) + ac(b4 + a2 c 2 ) + bc(a4 + b2 c 2 ).


.M

Moreover, since 2a b ≤ a2 + b2 , 2ac ≤ a2 + c 2 , 2bc ≤ b2 + c 2 , it is enough to prove that


W

2ABC + 2a2 b2 c 2 ≥ (a2 + b2 )(c 4 + a2 b2 ) + (a2 + c 2 )(b4 + a2 c 2 ) + (b2 + c 2 )(a4 + b2 c 2 ).


W

Indeed, this inequality reduces to the obvious inequality


W

6a2 b2 c 2 ≥ 0.

The equality holds when two of a, b, c are zero.

P 2.39. If a, b, c are real numbers, then

9(1 + a4 )(1 + b4 )(1 + c 4 ) ≥ 8(1 + a bc + a2 b2 c 2 )2 .

(Vasile Cîrtoaje, 2004)


70 Vasile Cîrtoaje

Solution. Substituting a, b, c by |a|, |b|, |c|, respectively, the left side of the inequality
remains unchanged, while the right side either remains unchanged or increases. There-
fore, it suffices to prove the inequality only for a, b, c ≥ 0. If a = b = c, then the
inequality reduces to
9(1 + a4 )3 ≥ 8(1 + a3 + a6 )2 ,

1 3 1
9(a2 + 2
) ≥ 8(a3 + 3 + 1)2 .
a a
1
Setting a + = x, this inequality can be written as follows
a

L
9(x 2 − 2)3 ≥ 8(x 3 − 3x + 1)2 ,

.M
x 6 − 6x 4 − 16x 3 + 36x 2 + 48x − 80 ≥ 0,

D
(x − 2)2 [x(x 3 − 8) + 4(x 3 − 5) + 6x 2 ] ≥ 0.

A
PI
Since x ≥ 2, the last inequality is clearly true. Multiplying now the inequalities

9(1 + a4 )3 ≥ 8(1 + a3 + a6 )2 ,
M
LY

9(1 + b4 )3 ≥ 8(1 + b3 + b6 )2 ,
O

9(1 + c 4 )3 ≥ 8(1 + c 3 + c 6 )2 ,
.M

we get

[9(1 + a4 )(1 + b4 )(1 + c 4 )]3 ≥ 83 (1 + a3 + a6 )2 (1 + b3 + b6 )2 (1 + c 3 + c 6 )2 .


W

According to Hölder’s inequality


W

(1 + a3 + a6 )(1 + b3 + b6 )(1 + c 3 + c 6 ) ≥ (1 + a bc + a2 b2 c 2 )3 ,
W

the conclusion follows. The equality holds for a = b = c = 1.

P 2.40. If a, b, c are real numbers, then

2(1 + a2 )(1 + b2 )(1 + c 2 ) ≥ (1 + a)(1 + b)(1 + c)(1 + a bc).

(Vasile Cîrtoaje, 2001)


Symmetric Polynomial Inequalities in Real Variables 71

Solution. Substituting a, b, c by |a|, |b|, |c|, respectively, the left side of the inequality
remains unchanged, while the right side either remains unchanged or increases. There-
fore, it suffices to prove the inequality only for a, b, c ≥ 0.
First Solution. For a = b = c, the inequality reduces to

2(1 + a2 )3 ≥ (1 + a)3 (1 + a3 ).

This is true, since

2(1 + a2 )3 − (1 + a)3 (1 + a3 ) = (1 − a)4 (1 + a + a2 ) ≥ 0.

L
Multiplying the inequalities

.M
2(1 + a2 )3 ≥ (1 + a)3 (1 + a3 ),

D
2(1 + b2 )3 ≥ (1 + b)3 (1 + b3 ),

A
2(1 + c 2 )3 ≥ (1 + c)3 (1 + c 3 ),
we get
PI
M
8(1 + a2 )3 (1 + b2 )3 (1 + c 2 )3 ≥ (1 + a)3 (1 + b)3 (1 + c)3 (1 + a3 )(1 + b3 )(1 + c 3 ).
LY

Using this result, we still have to show that


O

(1 + a3 )(1 + b3 )(1 + c 3 ) ≥ (1 + a bc)3 ,


.M

which is just Hölder’s inequality. We can also prove this inequality using the AM-GM
inequality. To do this, we write the inequality as
W

(a3 b3 + b3 c 3 + c 3 a3 − 3a2 b2 c 2 ) + (a3 + b3 + c 3 − 3a bc) ≥ 0.


W

The equality holds for a = b = c = 1.


W

Second Solution. We use the substitution


1− x 1− y 1−z
a= , b= , c= ,
1+ x 1+ y 1+z

where x, y, z ∈ (−1, 1]. Since

1 + a2 1 + x 2 1 + b2 1 + y 2 1 + c2 1 + z2
= , = , =
1+a 1+ x 1+ b 1+ y 1+c 1+z

and
2(1 + x y + yz + z x)
1 + a bc = ,
(1 + x)(1 + y)(1 + z)
72 Vasile Cîrtoaje

the required inequality becomes

(1 + x 2 )(1 + y 2 )(1 + z 2 ) ≥ 1 + x y + yz + z x,

x 2 y 2 + y 2 z 2 + z 2 x 2 + x 2 + y 2 + z 2 ≥ x y + yz + z x,
1 1 1
x 2 y 2 + y 2 z 2 + z 2 x 2 + (x − y)2 + ( y − z)2 + (z − x)2 ≥ 0.
2 2 2

P 2.41. If a, b, c are real numbers, then

L
.M
3(a2 − a b + b2 )(b2 − bc + c 2 )(c 2 − ca + a2 ) ≥ a3 b3 + b3 c 3 + c 3 a3 .

(Titu Andreescu, 2006)

D
Solution. Substituting a, b, c by |a|, |b|, |c|, respectively, the left side of the inequality

A
remains unchanged or decreases, while the right side remains unchanged or increases.
PI
Therefore, it suffices to prove the inequality for a, b, c ≥ 0. If a = 0, then the inequality
reduces to b2 c 2 (b − c)2 ≥ 0. Consider further then a, b, c > 0. We first show that
M
3(a2 − a b + b2 )3 ≥ a6 + a3 b3 + b6 .
LY

a b
Indeed, setting x = + , x ≥ 2, we can write this inequality as
O

b a
3(x − 1)3 ≥ x 3 − 3x + 1,
.M

(x − 2)2 (2x − 1) ≥ 0.
W

Using this inequality, together with the similar ones, we have


W

27(a2 − a b + b2 )3 (b2 − bc + c 2 )3 (c 2 − ca + a2 )3 ≥
≥ (a6 + a3 b3 + b6 )(b6 + b3 c 3 + c 6 )(c 6 + c 3 a3 + a6 ).
W

Therefore, it suffices to show that

(a6 + a3 b3 + b6 )(b6 + b3 c 3 + c 6 )(c 6 + c 3 a3 + a6 ) ≥ (a3 b3 + b3 c 3 + c 3 a3 )3 .

Writing this inequality in the form

(a3 b3 + b6 + a6 )(b6 + b3 c 3 + c 6 )(a6 + c 6 + c 3 a3 ) ≥ (a3 b3 + b3 c 3 + c 3 a3 )3 ,

we see that it is just Hölder’s inequality. The equality holds when a = b = c, when a = 0
and b = c (or any cyclic permutation), and when two of a, b, c are 0.
Symmetric Polynomial Inequalities in Real Variables 73

P 2.42. If a, b, c are nonzero real numbers, then


X b2 − bc + c 2 X a2 €X Š X 1 ‹
+ 2 ≥ a .
a2 bc a
(Vasile Cîrtoaje, 2010)

Solution. We have
X b2 − bc + c 2 X a2 X  b2 − bc + c 2 b2 c2

+2 = + +
a2 bc a2 ca a b
X (b2 − bc + c 2 )(a b + bc + ca)
=

L
a2 bc

.M
a b + bc + ca X
= bc(b2 − bc + c 2 ).
a2 b2 c 2

D
Then, we can write the inequality as

A
X X
(a b + bc + ca)[ bc(b2 − bc + c 2 ) − a bc a] ≥ 0.
PI
Since
M
X X X X X X
bc(b2 − bc + c 2 ) − a bc a=( bc)( a2 ) − b2 c 2 − 2a bc a
LY

X X X
=( bc)( a2 ) − ( bc)2
O

X X X
=( bc)( a2 − bc),
.M

the inequality is equivalent to

(a b + bc + ca)2 (a2 + b2 + c 2 − a b − bc − ca) ≥ 0,


W

which is true. The equality holds for a = b = c, and for a b + bc + ca = 0.


W
W

P 2.43. Let a, b, c be real numbers. Prove that


(a) if a, b, c ∈ [0, 1], then

a bc − (b + c − a)(c + a − b)(a + b − c) ≤ 1;

(b) if a, b, c ∈ [−1, 1], then

a bc − (b + c − a)(c + a − b)(a + b − c) ≤ 4.

(Vasile Cîrtoaje, 2011)


74 Vasile Cîrtoaje

Solution. We will show that if a, b, c ∈ [m, M ], where M ≥ 0, then


a bc − (b + c − a)(c + a − b)(a + b − c) ≤ M (M − m)2 .
Without loss of generality, assume that
M ≥ a ≥ b ≥ c ≥ m.
We have two cases to consider.
Case 1: a ≤ 0. The required inequality is true, since
a bc − (b + c − a)(c + a − b)(a + b − c) ≤ 0 ≤ M (M − m)2 .

L
Indeed, substituting −a, −b, −c for a, b, c, respectively, the left inequality can be restated

.M
as
a bc ≥ (b + c − a)(c + a − b)(a + b − c),

D
where a, b, c ≥ 0. This is just the well-known Schur’s inequality of degree three.
Case 2: a > 0. Since (M − m)2 ≥ (a − b)2 and M ≥ a, we have

A
PI
M (M − m)2 ≥ a(a − c)2 .
Therefore, it suffices to show that
M
a bc − (b + c − a)(c + a − b)(a + b − c) ≤ a(a − c)2 ,
LY

which is equivalent to
O

(b − c)[a2 + (b − 2c)a − b2 + c 2 ] ≥ 0.
.M

This is true, since


a2 + (b − 2c)a − b2 + c 2 = (a − b)(a + 2b − 2c) + (b − c)2
W

≥ 2(a − b)(b − c) + (b − c)2 ≥ 0.


W

Thus, the proof is completed. The equality holds when two of a, b, c are equal to m, and
the other is equal to M .
W

P 2.44. Let a, b, c be real numbers. Prove that


(a) if a, b, c ∈ [0, 1], then
X
a2 (a − b)(a − c) ≤ 1;
(b) if a, b, c ∈ [−1, 1], then
X
a2 (a − b)(a − c) ≤ 4.
(Vasile Cîrtoaje, 2011)
Symmetric Polynomial Inequalities in Real Variables 75

Solution. We will show that if a, b, c ∈ [m, M ], then


X
a2 (a − b)(a − c) ≤ (M − m)2 · max{m2 , M 2 }.

Without loss of generality, assume that

M ≥ a ≥ b ≥ c ≥ m.

Since b2 (b − c)(b − a) ≤ 0, (a − c)2 ≤ (M − m)2 , and max{a2 , c 2 } ≤ max{m2 , M 2 }, it


suffices to show that

a2 (a − b)(a − c) + c 2 (c − a)(c − b) ≤ (a − c)2 · max{a2 , c 2 }.

L
.M
This is equivalent to

(a − c)2 (a2 + c 2 + ac − a b − bc − max{a2 , c 2 }) ≤ 0.

D
A
Case 1: a2 ≥ c 2 . From a2 − c 2 = (a − c)(a + c) ≥ 0, it follows that a + c ≥ 0. Then,
PI
a2 + c 2 + ac − a b − bc − max{a2 , c 2 } = (a + c)(c − b) ≤ 0.
M
Case 2: a2 ≤ c 2 . From a2 − c 2 = (a − c)(a + c) ≤ 0, it follows that a + c ≤ 0. Then,
LY

a2 + c 2 + ac − a b − bc − max{a2 , c 2 } = (a + c)(a − b) ≤ 0.
O

Thus, the proof is completed. For M 2 ≥ m2 , the equality holds when two of a, b, c are
equal to m, and the other is equal to M . For M 2 ≤ m2 , the equality holds when two of
.M

a, b, c are equal to M , and the other is equal to m.


W
W

P 2.45. Let a, b, c be real numbers such that


W

a b + bc + ca = a bc + 2.

Prove that p
a2 + b2 + c 2 − 3 ≥ (2 + 3)(a + b + c − 3).
(Vasile Cîrtoaje, 2011)

Solution. Substituting a + 1, b + 1, c + 1 for a, b, c, respectively, we need to prove that

a + b + c = a bc

implies p
a2 + b2 + c 2 ≥ 3(a + b + c).
76 Vasile Cîrtoaje

This inequality is true if

(a2 + b2 + c 2 )2 ≥ 3(a + b + c)2 ,

which is equivalent to the homogeneous inequality

(a2 + b2 + c 2 )2 ≥ 3a bc(a + b + c).

Since
1X 2
(a b + bc + ca)2 − 3a bc(a + b + c) = a (b − c)2 ≥ 0,
2
it suffices to prove that

L
(a2 + b2 + c 2 )2 ≥ (a b + bc + ca)2 ,

.M
which is equivalent to

D
(a2 + b2 + c 2 − a b − bc − ca)(a2 + b2 + c 2 + a b + bc + ca) ≥ 0.

A
This inequality is true, since
PI
2(a2 + b2 + c 2 − a b − bc − ca) = (a − b)2 + (b − c)2 + (c − a)2 ≥ 0,
M
2(a2 + b2 + c 2 + a b + bc + ca) = (a + b)2 + (b + c)2 + (c + a)2 ≥ 0.
p
LY

The equality holds for a = b = c = 1, and for a = b = c = 1 + 3.


O
.M

P 2.46. Let a, b, c be real numbers such that

(a + b)(b + c)(c + a) = 10.


W

Prove that
W

(a2 + b2 )(b2 + c 2 )(c 2 + a2 ) + 12a2 b2 c 2 ≥ 30.


(Vasile Cîrtoaje, 2011)
W

Solution. Since
2(b2 + c 2 ) = (b + c)2 + (b − c)2
and
(a2 + b2 )(a2 + c 2 ) = (a2 + bc)2 + a2 (b − c)2 ,
by virtue of the Cauchy-Schwarz inequality, we have

2(a2 + b2 )(b2 + c 2 )(c 2 + a2 ) ≥ [(b + c)(a2 + bc) + a(b − c)2 ]2


= [(a + b)(b + c)(c + a) − 4a bc]2
= 4(5 − 2a bc)2 .
Symmetric Polynomial Inequalities in Real Variables 77

Thus, it suffices to show that

(5 − 2a bc)2 + 6a2 b2 c 2 ≥ 15.

This inequality is equivalent to (a bc − 1)2 = 0. The homogeneous inequality

10(a2 + b2 )(b2 + c 2 )(c 2 + a2 ) + 120a2 b2 c 2 ≥ 3(a + b)2 (b + c)2 (c + a)2


a 1
becomes an equality for = b = c (or any cyclic permutation), where k + = 3.
k k

L
P 2.47. Let a, b, c be real numbers such that

.M
(a + b)(b + c)(c + a) = 5.

D
Prove that

A
(a2 + a b + b2 )(b2 + bc + c 2 )(c 2 + ca + a2 ) + 12a2 b2 c 2 ≥ 15.
PI (Vasile Cîrtoaje, 2011)
M
Solution. Since
3 1
b2 + bc + c 2 = (b + c)2 + (b − c)2
LY

4 4
and
O

‹2
a b + ac 3

2 2 2 2 2
(a + a b + b )(a + ac + c ) = a + + bc + a2 (b − c)2 ,
2 4
.M

by the Cauchy-Schwarz inequality, we have

(a2 + a b + b2 )(b2 + bc + c 2 )(c 2 + ca + a2 ) ≥


W

p ‹ p 2
3 a b + ac 3

2 2
(b + c) a + + bc +
W

≥ a(b − c)
2 2 4
W

3 3
= [(a + b)(b + c)(c + a) − 2a bc]2 = (5 − 2a bc)2 .
4 4
Thus, it suffices to show that
3
(5 − 2a bc)2 + 12a2 b2 c 2 ≥ 15,
4
which is equivalent to (2a bc − 1)2 = 0. The homogeneous inequality

5(a2 + a b + b2 )(b2 + bc + c 2 )(c 2 + ca + a2 ) + 60a2 b2 c 2 ≥ 3(a + b)2 (b + c)2 (c + a)2


a 1
becomes an equality for = b = c (or any cyclic permutation), where k + = 3.
k k
78 Vasile Cîrtoaje

P 2.48. Let a, b, c be real numbers such that a + b + c = 1 and a3 + b3 + c 3 = k. Prove


that
(a) if k = 25, then |a| ≤ 1 or |b| ≤ 1 or |c| ≤ 1;
(b) if k = −11, then 1 < a ≤ 2 or 1 < b ≤ 2 or 1 < c ≤ 2.
(Vasile Cîrtoaje, 2011)

Solution. Without loss of generality, assume that a ≤ b ≤ c. If b = 1, then a + c = 0,


and hence
k = a3 + b3 + c 3 = 1 + a3 + c 3 = 1,
which is false in (a) and (b), too. From (b − a)(b − c) ≤ 0, we get

L
.M
b2 − (a + c)b + ac ≤ 0,

which is equivalent to

D
2b2 − b + ac ≤ 0.

A
(a) It suffices to show that |b| ≤ 1. We have
PI
25 − b3 = a3 + c 3 = (a + c)3 − 3ac(a + c) = (1 − b)3 − 3ac(1 − b),
M
which yields
LY

b2 − b − 8
ac = .
1− b
O

Thus, the inequality 2b2 − b + ac ≤ 0 is equivalent to


.M

(1 + b)(4 − 3b + b2 )
≥ 0,
1− b
W

which involves −1 ≤ b < 1, and hence |b| ≤ 1. The equality |b| = 1 holds for a = b = −1
and c = 3.
W

(b) It suffices to show that 1 < b ≤ 2. We have


W

−11 − b3 = a3 + c 3 = (a + c)3 − 3ac(a + c) = (1 − b)3 − 3ac(1 − b),

which yields
b2 − b + 4
ac = .
1− b
Thus, the inequality 2b2 − b + ac ≤ 0 is equivalent to

(b − 2)(b2 + 1)
≥ 0,
1− b
which involves 1 < b ≤ 2. The equality b = 2 holds for a = −3 and b = c = 2.
Symmetric Polynomial Inequalities in Real Variables 79

P 2.49. Let a, b, c be real numbers such that

a + b + c = a3 + b3 + c 3 = 2.

5
• ˜
/
Prove that a, b, c ∈ ,2 .
4
(Vasile Cîrtoaje, 2011)

Solution. From

2 = a3 + b3 + c 3 = a3 + (b + c)2 − 3bc(b + c) = a3 + (2 − a)2 − 3bc(2 − a),

L
we get

.M
2(1 − a)2
bc = .
2−a

D
Thus,we can write the false inequality 4bc > (b + c)2 as

A
8(1 − a)2 PI
> (2 − a)2 ,
2−a
M
a(a2 + 2a − 4)
> 0,
LY

2−a
p p
a ∈ (−1 − 5, 0) ∪ (−1 + 5, 2).
O

In addition, the case a = 2 is also not possible, because it involves b + c = 0 and


.M

b3 + c 3 = −6, which is false. Therefore,


p p
W

/ (−1 −
a, b, c ∈ 5, 0) ∪ (−1 + 5, 2].
W

Since
5 p p
• ˜
, 2 ⊂ (−1 − 5, 0) ∪ (−1 + 5, 2],
W

4
the conclusion follows.

P 2.50. If a, b, c and k are real numbers, then


X
(a − b)(a − c)(a − k b)(a − kc) ≥ 0.

(Vasile Cîrtoaje, 2005)


80 Vasile Cîrtoaje

Solution. For a = b = c, the equality holds. Otherwise, using the substitution m = k+2,
u = (1 − k)a, b = a + x, c = a + y, the inequality can be written as

Au2 + Bu + c ≥ 0,

where
A = x 2 − x y + y 2,
B = (x + y)(2A − mx y),
C = (x + y)2 (A − mx y) + m2 x 2 y 2 .
The quadratic Au2 + Bu + C has the discriminant

L
.M
D = B 2 − 4AC = −3m2 x 2 y 2 (x − y)2 .

Since A > 0 and D ≤ 0, the conclusion follows. The equality holds for a = b = c, and

D
for a/k = b = c or any cyclic permutation.

A
Remark 1. The inequality is equivalent to
X
a4 + k(k + 2)
X
a2 b2 + (1 − k2 )a bc
X PI
a ≥ (k + 1)
X
a b(a2 + b2 ).
M
For k = 0, we get Schur’s inequality of degree four
LY

X
a4 + b4 + c 4 + a bc(a + b + c) ≥ a b(a2 + b2 ).
O

For k = 1, we get the inequality


.M

X
a4 + b4 + c 4 + 3(a2 b2 + b2 c 2 + c 2 a2 ) ≥ 2 a b(a2 + b2 ),

with equality for a = b = c.


W

For k = 2, we get the inequality


W

a4 + b4 + c 4 + 8(a2 b2 + b2 c 2 + c 2 a2 ) ≥ 3(a b + bc + ca)(a2 + b2 + c 2 ),


W

which can be rewritten as

9(a4 + b4 + c 4 ) + 126(a2 b2 + b2 c 2 + c 2 a2 ) ≥ 5(a + b + c)4 ,

with equality for a = b = c, and for a/2 = b = c (or any cyclic permutation).
Remark 2. The inequality in P 2.50 is equivalent to
X
(a − b)2 (a + b − c − kc)2 ≥ 0.
Symmetric Polynomial Inequalities in Real Variables 81

P 2.51. If a, b, c are real numbers, then

(b + c − a)2 (c + a − b)2 (a + b − c)2 ≥ (b2 + c 2 − a2 )(c 2 + a2 − b2 )(a2 + b2 − c 2 ).

(Poland, 1992)

Solution. Consider the nontrivial case where a ≥ b ≥ c and b2 + c 2 − a2 ≥ 0. We get


the desired inequality by multiplying the inequalities

(c + a − b)2 (a + b − c)2 ≥ (c 2 + a2 − b2 )(a2 + b2 − c 2 ),

(b + c − a)2 (a + b − c)2 ≥ (b2 + c 2 − a2 )(a2 + b2 − c 2 ),

L
(b + c − a)2 (c + a − b)2 ≥ (b2 + c 2 − a2 )(c 2 + a2 − b2 ).

.M
We have
(c + a − b)2 (a + b − c)2 − (c 2 + a2 − b2 )(a2 + b2 − c 2 ) =

D
= [a2 − (b − c)2 ]2 − a4 + (b2 − c 2 )2 = 2(b − c)2 (b2 + c 2 − a2 ) ≥ 0.

A
PI
The equality holds for a = b = c, for a = 0 and b = c (or any cyclic permutation), and
for a = 0 and b + c = 0 (or any cyclic permutation).
M
LY

P 2.52. If a, b, c are real numbers, then


O

X (a − b)2 (b − c)2 (c − a)2


.M

a2 (a − b)(a − c) ≥ .
a2 + b2 + c 2 + a b + bc + ca
W

Solution (by Michael Rozenberg). Since


W

X 1X
a2 (a − b)(a − c) = (b − c)2 (b + c − a)2 ,
2
W

we can write the inequality in the form


”X — ”X —
(b + c)2 (b − c)2 (b + c − a)2 ≥ 4(a − b)2 (b − c)2 (c − a)2 .

Using now the Cauchy-Schwarz inequality, is suffices to show that


”X —2
(b + c)(b − c)(b + c − a) ≥ 4(a − b)2 (b − c)2 (c − a)2 ,

which is an identity. The equality holds for a = b = c, for a = 0 and b = c (or any cyclic
permutation), and for a = 0 and b + c = 0 (or any cyclic permutation).
82 Vasile Cîrtoaje

P 2.53. Let a ≤ b ≤ c be real numbers such that

a + b + c = p, a b + bc + ca = q,

where p and q are fixed real numbers satisfying p2 ≥ 3q. Prove that the product

r = a bc

is minimal when b = c, and is maximal when a = b.

First Solution. We show first that a ∈ [a1 , a2 ], where

L
p p

.M
p − 2 p2 − 3q p − p2 − 3q
a1 = , a2 = .
3 3

D
From

A
(b − c)2 = (b + c)2 − 4bc = (b + c)2 + 4a(b + c) − 4q
PI
= (p − a)2 + 4a(p − a) − 4q = −3a2 + 2pa + p2 − 4q ≥ 0,
M
we get a ≥ a1 , with equality for b = c. Similarly, from
LY

(a − b)(a − c) = a2 − 2a(b + c) + q = a2 − 2a(p − a) + q = 3a2 − 2pa + q ≥ 0,


O

we get a ≤ a2 , with equality for a = b. On the other hand, from


.M

a bc = a[q − a(b + c)] = aq − a2 (p − a) = a3 − pa2 + qa,

we get
W

r(a) = a3 − pa2 + qa.


Since r 0 (a) = 3a2 − 2pa + q = (a − b)(a − c) ≥ 0, r(a) is strictly increasing on [a1 , a2 ],
W

and hence r(a) is minimal for a = a1 , when b = c, and is maximal for a = a2 , when
W

a = b.
Second Solution. From

(a − b)2 (b − c)2 (c − a)2 = −27r 2 + 2(9pq − 2p3 )r + p2 q2 − 4q3 ≥ 0,

we get r1 ≤ r ≤ r2 , where

9pq − 2p3 − 2(p2 − 3q) p2 − 3q


p
r1 = ,
27

9pq − 2p3 + 2(p2 − 3q) p2 − 3q


p
r2 = .
27
Symmetric Polynomial Inequalities in Real Variables 83

Obviously, r attains its minimal and maximal values when two of a, b, c are equal; that
is, when either a = b or b = c. For a = b, from a + b + c = p and a b + bc + ca = q, we
get
p + 2 p2 − 3q
p p
p − p2 − 3q
a=b= , c= ,
3 3

(p − p2 − 3q)2 (p + 2 p2 − 3q)
p p
r= = r2 .
27
Similar, for b = c, we get

p+
p p
p2 − 3q p − 2 p2 − 3q

L
b=c= , a= ,
3 3

.M
(p + p2 − 3q)2 (p − 2
p p
p2 − 3q)
r= = r1 .

D
27

A
Remark. Using this result, we can prove the following generalization:
• Let a1 , a2 , . . . , an be real numbers such that
PI
M
a1 + a2 + · · · + an = p, a12 + a22 + · · · + an2 = p1 ,
LY

where p and p1 are fixed real numbers satisfying p2 ≤ np1 . Then, the product
O

r = a1 a2 · · · an
.M

is minimal and maximal when n − 1 numbers of a1 , a2 , . . . , an are equal.


Assume, by the sake of contradiction, that the product r is minimal/maximal when three
W

of a1 , a2 , . . . , an are distinct, let a1 < a2 < a3 . According to P .53, the product r can be
W

increased/decreased (which is a contradiction) by choosing some suitable non-distinct


numbers b1 , b2 , b3 such that
W

b1 + b2 + b3 = a1 + a2 + a3 , b12 + b22 + b32 = a12 + a22 + a32 .

P 2.54. Let a, b, c be real numbers such that a + b + c = 3. Prove that

(a b + bc + ca − 3)2 ≥ 27(a bc − 1).

(Vasile Cîrtoaje, 2011)


84 Vasile Cîrtoaje

First Solution. Let q = a b + bc + ca. We need to show that (q − 3)2 + 27 ≥ 27a bc.
According to P 2.53, for fixed q, the product a bc is maximal when two of a, b, c are
equal. Therefore, it suffices to prove the desired inequality for b = c; that is, to show
that (2a b + b2 − 3)2 ≥ 27(a b2 − 3) for a + 2b = 3. This inequality is equivalent to

(b − 1)2 (b + 2)2 ≥ 0.

The equality holds for a = b = c = 1, and also for a = 7 and b = c = −2 (or any cyclic
permutation).
Second Solution. Assume that a = max{a, b, c}, a ≥ 1. Since

L
1 1 3
3 − a b − bc − ca ≥ 3 − a(b + c) − (b + c)2 = 3 − a(3 − a) − (3 − a)2 = (a − 1)2

.M
4 4 4
and
1 1 1

D
a bc − 1 ≤ a(b + c)2 − 1 = a(3 − a)2 − 1 = (a − 1)2 (a − 4),
4 4 4

A
it suffices to prove that
9
(a − 1)4 ≥
27
PI
(a − 1)2 (a − 4),
16 4
M
which is equivalent to
LY

(a − 1)2 (a − 7)2 ≥ 0.
O
.M

P 2.55. Let a, b, c be real numbers such that a + b + c = 3. Prove that


W

(a b + bc + ca)2 + 9 ≥ 18a bc.


W

(Vasile Cîrtoaje, 2011)

First Solution. Let q = a b+ bc+ca. We need to show that q2 +9 ≥ 18a bc. According to
W

P 2.53, for fixed q, the product a bc is maximal when two of a, b, c are equal. Therefore,
it suffices to prove the desired inequality for b = c; that is, to show that (2a b+ b2 )2 +9 ≥
18a b2 for a + 2b = 3. This inequality is equivalent to

(b − 1)2 (b + 1)2 ≥ 0.

The equality holds for a = b = c = 1, and also for a = 5 and b = c = −1 (or any cyclic
permutation).
Second Solution. Using the substitution a = x + 1, b = y + 1, c = z + 1, we need to
show that
(x y + yz + z x)2 ≥ 12(x y + yz + z x) + 18x yz,
Symmetric Polynomial Inequalities in Real Variables 85

where x, y, z are real numbers such that x + y + z = 0. Substituting x by − y − z, the


inequality can be restated as

( y 2 + yz + z 2 )2 + 12( y 2 + yz + z 2 ) ≥ −18 yz( y + z).

Since
3
y 2 + yz + z 2 ≥ ( y + z)2 ≥ 3 yz,
4
it suffices to show that

9 y 2 z 2 + 9( y + z)2 ≥ −18 yz( y + z),

L
which is equivalent to

.M
9( yz + y + z)2 ≥ 0.

D
A
P 2.56. If a, b, c are real numbers such that a2 + b2 + c 2 = 9, then
PI
a bc + 10 ≥ 2(a + b + c).
M
LY

(Vietnam TST, 2002)

Solution. Let p = a + b + c, q = a b + bc + ca and r = a bc. We need to show that


O

r ≥ 2p − 10 for p2 − 2q = 9. According to P 2.53, for fixed p and q, r is minimal when


two of a, b, c are equal. Therefore, it suffices to prove the desired inequality for b = c;
.M

that is, to show that a b2 + 10 ≥ 2(a + 2b) for a2 + 2b2 = 9. Since

2(a b2 + 10) − 4(a + 2b) = a(9 − a2 ) + 20 − 4(a + 2b) = 20 + 5a − a3 − 8b,


W

we need to show that


W

Æ
20 + 5a − a3 ≥ 4 2(9 − a2 )
W

for −3 ≤ a ≤ 3. For −3 ≤ a ≤ 0, we have 20+5a − a3 ≥ 5(4+ a) > 0, and for 0 ≤ a ≤ 3,


we have 20 + 5a − a3 = 4(5 − a) + a(9 − a2 ) > 0. Therefore it suffices to prove that

(20 + 5a − a3 )2 ≥ 32(9 − a2 ).

This is true, since it is equivalent to (a + 1)2 f (a) ≥ 0, where

f (a) = a4 − 2a3 − 7a2 − 24a + 112


= 4 + 12(3 − a) + (3 − a)2 (a2 + 4a + 8) > 0.

The equality holds for a = −1 and b = c = 2 (or any cyclic permutation).


86 Vasile Cîrtoaje

P 2.57. If a, b, c are real numbers such that


a + b + c + a bc = 4,
then
a2 + b2 + c 2 + 3 ≥ 2(a b + bc + ca).
(Vasile Cîrtoaje, 2011)
Solution. Without loss of generality, assume that a ≥ b ≥ c. The case a ≤ 0 is not
possible, since it involves a + b + c + a bc ≤ 0 < 4. If a ≥ 0 ≥ b ≥ c, then
a2 + b2 + c 2 + 3 − 2(a b + bc + ca) ≥ a2 + (b − c)2 + 3 > 0.

L
Also, if a ≥ b ≥ 0 ≥ c, then

.M
a2 + b2 + c 2 + 3 − 2(a b + bc + ca) ≥ (a − b)2 + c 2 + 3 > 0.
Consider further that a ≥ b ≥ c ≥ 0. Let p = a + b + c, q = a b + bc + ca and r = a bc.

D
We need to show that p2 + 3 ≥ 4q for p + r = 4.

A
First Solution. By Schur’s inequality of degree three, we have p3 +9r ≥ 4pq. Therefore,
we get
PI
p(p2 + 3 − 4q) ≥ p3 + 3p − (p3 + 9r) = 12(p − 3).
M
To complete the proof, we need to show that p ≥ 3. By virtue of the AM-GM inequality,
LY

we get
p3 ≥ 27r,
O

p3 ≥ 27(4 − p),
.M

(p − 3)(p2 + 3p + 36) ≥ 0,
p ≥ 3.
W

The equality holds for a = b = c = 1.


Second Solution. For the sake of contradiction, assume that p2 + 3 < 4q. Then, it
W

suffices to show that p + r > 4. According to P 2.53, for fixed p and q, r is minimal
when two of a, b, c are equal. Therefore, it suffices to consider that b = c; that is, to
W

prove that a + 2b ≥ 0 and a2 + 3 < 4a b imply a + 2b + a b2 > 4. From a2 + 3 < 4a b,


it follows that a and b have the same sign. In addition, from a + 2b ≥ 0, it follows that
a > 0 and b > 0. Therefore, it suffices to prove that a + 2b + a b2 > 4 if a > 0 and
a2 + 3
b> . Indeed,
4a
a2 + 3 (a2 + 3)2
a + 2b + a b2 − 4 > a + + −4
2a 16a
(a − 1)2 (a2 + 2a + 33)
= ≥ 0.
16a
Symmetric Polynomial Inequalities in Real Variables 87

P 2.58. If a, b, c are real numbers such that

a b + bc + ca = 3a bc,

then
4(a2 + b2 + c 2 ) + 9 ≥ 7(a b + bc + ca).
(Vasile Cîrtoaje, 2011)

Solution. If one of a, b, c is 0, then the inequality is trivial. Otherwise, write the in-
equality in the homogeneous form

L
81a2 b2 c 2
4(a2 + b2 + c 2 ) + ≥ 7(a b + bc + ca),

.M
(a b + bc + ca)2
or

D
81a2 b2 c 2 ≥ (a b + bc + ca)2 A,

A
where
A = 7(a b + bc + ca) − 4(a2 + b2 + c 2 ).
PI
First Solution (by Vo Quoc Ba Can). Consider the nontrivial case A > 0. Substitut-
M
ing a, b, c by |a|, |b|, |c|, respectively, the left side of the inequality remains unchanged,
LY

while the right side remains unchanged or increases. Therefore, it suffices to prove the
inequality only for a, b, c > 0 and A > 0. Assume that a ≥ b ≥ c > 0. There are two
cases to consider.
O

Case 1: 4b2 ≤ 3a b + 3bc + ca. Since


.M

2
(a b + bc + ca)2

2
4(a b + bc + ca) A ≤ + bA ,
b
W

it suffices to show that


W

(a b + bc + ca)2
18a bc ≥ + bA,
b
W

which is equivalent to the obvious inequality

(a − b)(b − c)(3a b + 3bc + ca − 4b2 ) ≥ 0.

Case 2: 4b2 > 3a b + 3bc + ca. Since


2
(a b + bc + ca)2

2
4(a b + bc + ca) A ≤ + aA ,
a

it suffices to show that


(a b + bc + ca)2
18a bc ≥ + aA,
a
88 Vasile Cîrtoaje

which is equivalent to

(a − b)(a − c)(4a2 − 3a b − bc − 3ca) ≥ 0.

This is true, since

4a2 − 3a b − bc − 3ca = (4b2 − 3a b − 3bc − ca) + 2(a − b)(2a + 2b − c) > 0.

The equality holds for a = b = c = 1, and for a = 1/2 and b = c = 2 (or any cyclic
permutation).
Second Solution. Let p = a + b + c, q = a b + bc + ca, r = a bc. Since A = 15q − 4p2 ,we

L
need to show that

.M
81r 2 ≥ q2 (15q − 4p2 )
for all real numbers a, b, c. For fixed p and q, r 2 is minimal when r = 0, or when r is

D
either minimal or maximal. For a = 0, the inequality is true since

A
2(15q − 4p2 ) = −b2 − c 2 − 7(b − c)2 ≤ 0.
PI
According to P 2.53, r is minimal and maximal when two of a, b, c are equal. Therefore,
M
due to symmetry and homogeneity, it suffices to prove the inequality for b = c = 1. In
LY

this case, the inequality can be written as

(a − 1)2 (4a − 1)2 ≥ 0.


O
.M
W

P 2.59. If a, b, c are real numbers such that a + b + c = 3, then


W

(a2 + 1)(b2 + 1)(c 2 + 1) ≥ (a + 1)(b + 1)(c + 1).


W

(Tran Quoc Anh, 2010)

Solution. Write the inequality as

a2 b2 c 2 + a2 b2 + b2 c 2 + c 2 a2 + a2 + b2 + c 2 ≥ a bc + a b + bc + ca + 3.

Since a2 b2 c 2 ≥ 2a bc − 1, it suffices to prove that

a bc + a2 b2 + b2 c 2 + c 2 a2 + a2 + b2 + c 2 ≥ a b + bc + ca + 4,

which is equivalent to
5(1 − a bc) ≥ q(3 − q),
Symmetric Polynomial Inequalities in Real Variables 89

where q = a b + bc + ca. For fixed q, according to P 2.53, the product a bc is maxi-


mal when two of a, b, c are equal. Therefore, due to symmetry, it suffices to prove the
inequality for b = c. We have a = 3 − 2b and

5(1 − a bc) − q(3 − q) = 5(1 − a b2 ) − (2a b + b2 )(3 − 2a b − b2 )


= (b − 1)2 (9b2 − 8b + 5) ≥ 0.

The equality holds for a = b = c = 1.

L
P 2.60. Let f4 (a, b, c) be a symmetric homogeneous polynomial of degree four. Prove that

.M
the inequality f4 (a, b, c) ≥ 0 holds for all real numbers a, b, c if and only if f4 (a, 1, 1) ≥ 0
for all real a.

D
Solution. Let p = a + b + c, q = a b + bc + ca and r = a bc. Any symmetric homogeneous

A
polynomial f4 (a, b, c) can be written as
PI
f4 (a, b, c) = Apr + Bp4 + C p2 q + Dq2 ,
M
where A, B, C, D are real constants. For fixed p and q, the linear function g(r) =
Apr + Bp4 + C p2 q + Dq2 is minimal when r is either minimal or maximal. By P 2.53,
LY

r is minimal and maximal when two of a, b, c are equal. Since f4 (a, b, c) is symmet-
ric, homogeneous and satisfies f4 (−a, −b, −c) = f4 (a, b, c), it follows that the inequal-
O

ity f4 (a, b, c) ≥ 0 holds for all real numbers a, b, c if and only if f4 (a, 1, 1) ≥ 0 and
f4 (a, 0, 0) ≥ 0 for all real a. Notice that the condition " f4 (a, 0, 0) ≥ 0 for all real a"
.M

is not necessary because it follows from the condition " f4 (a, 1, 1) ≥ 0 for all real a" as
follows:
W

f4 (a, 0, 0) = lim f4 (a, t, t) = lim t 4 f4 (a/t, 1, 1) ≥ 0.


t→0 t→0
W

Remark. Similarly, we can prove the following more general statement, where f4 (a, b, c)
W

is only a symmetric polynomial (homogeneous or non-homogeneous).


• Let f4 (a, b, c) be a symmetric polynomial function of degree n = 4. The inequality
f4 (a, b, c) ≥ 0 holds for all real numbers a, b, c if and only if f4 (a, b, b) ≥ 0 for all real
numbers a and b.
Notice that a function f (a, b, c) is symmetric if it is unchanged by any permutation
of its variables. A function f (a, b, c) is a polynomial function if it is a polynomial in one
variable when the other two variables are fixed. In addition, f (a, b, c) is a polynomial
of degree n if f (a, a, a) is a polynomial of degree n.
90 Vasile Cîrtoaje

P 2.61. If a, b, c are real numbers, then


X
10(a4 + b4 + c 4 ) + 64(a2 b2 + b2 c 2 + c 2 a2 ) ≥ 33 a b(a2 + b2 ).

(Vasile Cîrtoaje, 2008)

Solution. According to P 2.60, it suffices to prove the required inequality for b = c = 1,


when it becomes
5a4 − 33a3 + 64a2 − 33a + 9 ≥ 0,

(a − 3)2 (5a2 − 3a + 1) ≥ 0.

L
This is true since

.M
3 2 11
5a2 − 3a + 1 = 5(a − ) + > 0.
10 20
The equality holds for a/3 = b = c (or any cyclic permutation).

D
A
PI
M
P 2.62. If a, b, c are real numbers such that a + b + c = 3, then
LY

3(a4 + b4 + c 4 ) + 33 ≥ 14(a2 + b2 + c 2 ).
O

(Vasile Cîrtoaje, 2009)

First Solution. Write the inequality as F (a, b, c) ≥ 0, where


.M

F (a, b, c) = 3(a4 + b4 + c 4 ) + 33 − 14(a2 + b2 + c 2 ).


W

Due to symmetry, we may assume that a ≤ b ≤ c. Let us denote x = (b + c)/2, x ≥ 1.


W

To prove the desired inequality, we use the mixing variable method. We will show that
W

F (a, b, c) ≥ F (a, x, x) ≥ 0.

We have

F (a, b, c) − F (a, x, x) = 3(b4 + b4 − 2x 4 ) − 14(b2 + c 2 − 2x 2 )

= 3[(b2 + c 2 )2 − 4x 4 ] + 6(x 4 − b2 c 2 ) − 14(b2 + c 2 − 2x 2 )

= (b2 + c 2 − 2x 2 )[3(b2 + c 2 + 2x 2 ) − 14] + 6(x 2 − bc)(x 2 + bc).

Since
1
b2 + c 2 − 2x 2 = 2(x 2 − bc) = (b − c)2 ,
2
Symmetric Polynomial Inequalities in Real Variables 91

we get

1
F (a, b, c) − F (a, x, x) =(b − c)2 [3(b2 + c 2 + 2x 2 ) − 14 + 3(x 2 + bc)]
2
1
= (b − c)2 [3(x 2 − bc) + 18x 2 − 14] ≥ 0.
2
Also,
F (a, x, x) = F (3 − 2x, x, x) = 6(x − 1)2 (3x − 5)2 ≥ 0.

This completes the proof. The equality holds for a = b = c = 1, and for a = −1/3 and
b = c = 5/3 (or any cyclic permutation).

L
.M
Second Solution. Write the inequality in the homogeneous form

81(a4 + b4 + c 4 ) + 11(a + b + c)4 ≥ 42(a2 + b2 + c 2 )(a + b + c)2 .

D
According to P 2.60, it suffices to prove this inequality for b = c = 1, when it becomes

A
PI
25a4 − 40a3 + 6a2 + 8a + 1 ≥ 0,
M
(a − 1)2 (5a + 1)2 ≥ 0.
LY
O
.M

P 2.63. If a, b, c are real numbers such that a2 + b2 + c 2 = 3, then

a4 + b4 + c 4 + 3(a b + bc + ca) ≤ 12.


W

Solution. Write the inequality in the homogeneous form


W

3(a4 + b4 + c 4 ) + 3(a b + bc + ca)(a2 + b2 + c 2 ) ≤ 4(a2 + b2 + c 2 )2 .


W

According to P 2.60, it suffices to prove this inequality for b = c = 1, when it becomes

a4 − 6a3 + 13a2 − 12a + 4 ≥ 0,

(a − 1)2 (a − 2)2 ≥ 0.
p p
= b = c = ±1, forpa = 2 and b = c = 2/2 (or any cyclic
The equality holds for a p
permutation), and a = − 2 and b = c = − 2/2 (or any cyclic permutation).
92 Vasile Cîrtoaje

P 2.64. Let α, β, γ be real numbers such that

1 + α + β = 2γ.

The inequality
X X X X
a4 + α a2 b2 + β a bc a≥γ a b(a2 + b2 )

holds for any real numbers a, b, c if and only if

1 + α ≥ γ2 .

L
(Vasile Cîrtoaje, 2009)

.M
Solution. Let

D
X X X X
f4 (a, b, c) = a4 + α a2 b2 + β a bc a−γ a b(a2 + b2 .

A
According to P 2.60, the inequality f4 (a, b, c) ≥ 0 holds for any real numbers a, b, c if
and only if f4 (a, 1, 1) ≥ 0 for any real a. From PI
M
f4 (a, 1, 1) = (a − 1)2 [(a − γ + 1)2 + 1 + α − γ2 ],
LY

the conclusion follows. The equality holds for a = b = c.


Remark. For γ = k + 1 and α = k(k + 2), we get
O

X X X X
a4 + k(k + 2) a2 b2 + (1 − k2 )a bc a ≥ (k + 1) a b(a2 + b2 ), k ∈ R,
.M

which is equivalent to the elegant inequality from P 2.50, namely


W

X
(a − b)(a − c)(a − k b)(a − kc) ≥ 0,
W

where the equality holds for a = b = c, and also for a/k = b = c (or any cyclic permu-
tation). In addition, for k = 0, we get Schur’s inequality of degree four
W

X
a2 (a − b)(a − c),

with equality for a = b = c, and also for a = 0 and b = c (or any cyclic permutation).

P 2.65. If a, b, c are real numbers such that a2 + b2 + c 2 = 2, then

a b(a2 − a b + b2 − c 2 ) + bc(b2 − bc + c 2 − a2 ) + ca(c 2 − ca + a2 − b2 ) ≤ 1.


Symmetric Polynomial Inequalities in Real Variables 93

Solution. Write the inequality in the homogeneous form


X
(a2 + b2 + c 2 )2 ≥ 4 a b(a2 − a b + b2 − c 2 ).

According to P 2.60, it suffices to prove this inequality for b = c = 1, when it can be


written as
a2 (a − 4)2 ≥ 0.

The equality holds for


a2 + b2 + c 2 = 2(a b + bc + ca).

L
.M
P 2.66. If a, b, c are real numbers, then

D
A
4 4
(a + b)4 + (b + c)4 + (c + a)4 ≥ (a + b4 + c 4 ).
7
PI (Vietnam TST, 1996)
M
Solution. Denote the left side of the inequality by f4 (a, b, c). According to P 2.60, it
LY

suffices to prove that f4 (a, 1, 1) ≥ 0 for all real a. Indeed,


O

2
f4 (a, 1, 1) = (5a4 + 28a3 + 42a2 + 28a + 59) > 0
7
.M

since, for the nontrivial case a < 0, we have


W

23 2 2
5a4 + 28a3 + 42a2 + 28a + 59 = (5a2 − 2a)(a + 3)2 + 9(a + ) + > 0.
9 9
W

The equality holds for a = b = c = 0.


W

P 2.67. Let a, b, c be real numbers, and let

p = a + b + c, q = a b + bc + ca, r = a bc.

Prove that
p2 + q2 − pq
(3 − p)r + ≥ q.
3
(Vasile Cîrtoaje, 2011)
94 Vasile Cîrtoaje

First Solution. Write the inequality as

(p2 − 3q) + (q2 − 3pr) ≥ pq − 9r,

1X 1X 2 X
(b − c)2 + a (b − c)2 ≥ a(b − c)2 .
2 2
According to the AM-GM inequality, it suffices to prove that
r”X — ”X — X
(b − c)2 a2 (b − c)2 ≥ a(b − c)2 .

Clearly, this inequality follows immediately from the Cauchy-Schwarz inequality. The

L
equality holds for a = b = c, and for b = c = 1 (or any cyclic permutation).

.M
Second Solution Write the inequality as f4 (a, b, c) ≥ 0, where

f4 (a, b, c) = 3(3 − p)r + p2 + q2 − pq − 3q.

D
A
is a symmetric polynomial of degree four in a, b, c. According to Remark from the proof
PI
of P 2.60, it suffices to prove that f4 (a, b, b) ≥ 0 for all real numbers a and b. Indeed,
we have
M
f4 (a, b, b) = (a − b)2 (b − 1)2 ≥ 0.
LY
O

P 2.68. If a, b, c are real numbers, then


.M

a b(a + b) + bc(b + c) + ca(c + a) 3


≤ .
(a2 + 1)(b2 + 1)(c 2 + 1) 4
W

(Vasile Cîrtoaje, 2011)


W

First Solution. We try to get a stronger homogeneous inequality of third order. Accord-
W

ing to the AM-GM inequality, we have

(a2 + 1)(b2 + 1)(c 2 + 1) = (a2 b2 c 2 + 1) + (a2 b2 + b2 c 2 + c 2 a2 ) + (a2 + b2 + c 2 )


Æ
≥ 2a bc + 2 (a2 b2 + b2 c 2 + c 2 a2 )(a2 + b2 + c 2 ).
Therefore, it suffices to prove that
Æ X
3a bc + 3 (a2 b2 + b2 c 2 + c 2 a2 )(a2 + b2 + c 2 ) ≥ 2 a b(a + b).

Indeed, using the identity


X
9(a2 + b2 + c 2 ) = (2a + 2b − c)2
Symmetric Polynomial Inequalities in Real Variables 95

and the Cauchy-Schwarz inequality, we get


Æ
3 (a2 b2 + b2 c 2 + c 2 a2 )(a2 + b2 + c 2 ) =
r”X — ”X — X
= a2 b2 (2a + 2b − c)2 ≥ a b(2a + 2b − c)
X
=2 a b(a + b) − 3a bc.
The equality holds for a = b = c = 1.
Second Solution Since the equality holds for a = b = c = 1, we write the inequality as

L
3(a bc − 1)2 + f4 (a, b, c) ≥ 0,

.M
where X X 4X

D
f4 (a, b, c) = a2 b2 + a2 + 2a bc − a b(a + b)
3

A
is a symmetric polynomial of degree four. Clearly, it suffices to prove that f4 (a, b, c) ≥ 0.
PI
According to Remark from P 2.60, it suffices to prove that f4 (a, b, b) ≥ 0 for all real
numbers a and b. Indeed, we have
M
3 f4 (a, b, b) = (6b2 − 8b + 3)a2 − 2b2 a + b2 (3b2 − 8b + 6)
LY

2
b2 18b2 (b − 1)4

O

2
= (6b − 8b + 3) a − 2 + ≥ 0.
6b − 8b + 3 6b2 − 8b + 3
.M

Remark. The inequality is equivalent to


X
3(a bc − 1)2 + (a − 1)2 (b − c)2 + (a b + bc + ca − a − b − c)2 ≥ 0.
W
W
W

P 2.69. If a, b, c are real numbers such that a bc > 0, then

1 1 1 1 1 1 1
 ‹ ‹ ‹  ‹
a + − 1 b + − 1 c + − 1 + 2 ≥ (a + b + c) + + .
a b c 3 a b c

(Vasile Cîrtoaje, 2011)

Solution. Let p = a + b + c, q = a b + bc + ca and r = a bc. Multiplying by a bc, we can


rewrite the inequality as

4pq
r 2 + (4 − p − q)r + p2 + q2 − − p − q + 1 ≥ 0.
3
96 Vasile Cîrtoaje

Since the equality holds for a = b = c = 1, that is, for p = q = 3 and r = 1, we write
the inequality as
 p − q 2
r −1+ + f (p, q, r) ≥ 0,
2
where

12 f (p, q, r) = 24(3 − p)r + 9(p2 + q2 ) − 10pq − 24q


≥ 24(3 − p)r + 8(p2 + q2 ) − 8pq − 24q

Thus, it suffices to prove that f4 (a, b, c) ≥ 0, where

L
f4 (a, b, c) = 3(3 − p)r + p2 + q2 − pq − 3q ≥ 0.

.M
According to Remark from P 2.60, it suffices to prove that f4 (a, b, b) ≥ 0 for all real

D
numbers a and b. Indeed, we have

A
f4 (a, b, b) = (b − 1)2 (a − b)2 ≥ 0.

The equality holds for a = b = c = 1.


PI
M
Remark. The inequalities in P 2.68 and P 2.69 are particular cases of the following
LY

more general statement (Vasile Cîrtoaje, 2011).


• Let a, b, c be real numbers such that a bc > 0. If −2 ≤ k ≤ 1, then
O

1 1 1
 ‹ ‹ ‹
.M

a + + k b + + k c + + k + (1 − k)(2 + k)2 ≥
a b c

1 1 1 1
 ‹
W

2
≥ (2 + k) (a + b + c) + + .
3 a b c
W
W

P 2.70. If a, b, c are real numbers, then

1 1 1 1 1 1
 ‹ ‹ ‹  ‹ ‹ ‹
a2 + b2 + c2 + ≥ a+b− b+c− c+a− .
2 2 2 2 2 2

(Vasile Cîrtoaje, 2011)

Solution. Since the equality holds for a = b = c = 1, we write the inequality as


‹2
1 a+b+c

a bc + − + f4 (a, b, c) ≥ 0,
2 2
Symmetric Polynomial Inequalities in Real Variables 97

where
Y 1 1 1 a+b+c 2
‹ Y ‹  ‹
2
f4 (a, b, c) = a + − a+b− − a bc + −
2 2 2 2
is a symmetric polynomial of degree four. Clearly, it suffices to prove that f4 (a, b, c) ≥ 0.
According to Remark from P 2.60, it suffices to prove that f4 (a, b, b) ≥ 0 for all real
numbers a and b. Indeed, we have

2 f4 (a, b, b) = [(2b − 1)a − b(2 − b)]2 ≥ 0.

Remark. The inequality is equivalent to

L
.M
(2a bc + 1 − a − b − c)2 + 2(a b + bc + ca − a − b − c)2 ≥ 0.

D
A
P 2.71. If a, b, c are real numbers such that a + b + c = 3, then
PI
a(a − 1) b(b − 1) c(c − 1)
M
+ + 2 ≥ 0.
8a2 + 9 8b2 + 9 8c + 9
LY

(Vasile Cîrtoaje, 2013)


O

Solution (by Michael Rozenberg). We see that the equality holds for a = b = c, and for
a = 3/2 and b = c = 3/4 (or any cyclic permutation). Let k be a positive constant,
.M

k > 3. Write the inequality as


X (k2 − 8)a(a − 1)
W

≥ 0,
8a2 + 9
W

X  (k2 − 8)a(a − 1) 
+ 1 ≥ 3.
8a2 + 9
W

Choosing p
k =3+ 17,
the inequality can be written as
X (ka − 3)2
≥ 3.
8a2 + 9
Let m be a real constant. According to the Cauchy-Schwarz inequality, we have

[ (ka − 3)(ma + 3)]2


P
X (ka − 3)2
≥ P ,
8a2 + 9 (ma + 3)2 (8a2 + 9)
98 Vasile Cîrtoaje

with equality for


ka − 3 kb − 3 kc − 3
= = .
(8a2 + 9)(ma + 3) (8b2 + 9)(mb + 3) (8c 2 + 9)(mc + 3)
Choosing m = k, these conditions are satisfied for a = 3/2 and b = c = 3/4. Therefore,
it suffices to show that
X
[k2 (a2 + b2 + c 2 ) − 27]2 ≥ 3 (ka + 3)2 (8a2 + 9).

Write this inequality in the homogeneous form f4 (a, b, c) ≥ 0, where

f4 (a, b, c) = [k2 (a2 + b2 + c 2 ) − 3(a + b + c)2 ]2

L
X

.M
−3 (ka + a + b + c)2 [8a2 + (a + b + c)2 ].
According to P 2.60, it suffices to prove that f4 (a, 1, 1) ≥ 0 for all real a. Indeed, this

D
inequality is equivalent to (a − 1)2 (a − 2)2 ≥ 0.

A
PI
P 2.72. If a, b, c are real numbers such that a + b + c = 3, then
M
(a − 11)(a − 1) (b − 11)(b − 1) (c − 11)(c − 1)
+ + ≥ 0.
LY

2a2 + 1 2b2 + 1 2c 2 + 1
(Vasile Cîrtoaje, 2013)
O

Solution. Write the inequality as


.M

X• (a − 11)(a − 1)
˜
+ 1 ≥ 3,
2a2 + 1
W

X (a − 2)2
≥ 1.
2a2 + 1
W

According to the Cauchy-Schwarz inequality, we have


W

[ (a − 2)2 ]2
P
X (a − 2)2
≥P ,
2a2 + 1 (a − 2)2 (2a2 + 1)
Therefore, it suffices to show that
X X X X
(a2 + b2 + c 2 )2 ≥ 2 a4 − 8 a3 + 9 a2 − 4 a + 12.

Write this inequality in the homogeneous form f4 (a, b, c) ≥ 0, where


X €X Š €X Š €X Š €X Š2
f4 (a, b, c) = 3(a2 + b2 + c 2 )2 − 6 a4 + 8 a3 a −3 a2 a
”X X X —
=2 a4 + a b(a2 + b2 ) − 3a bc a .
Symmetric Polynomial Inequalities in Real Variables 99

According to P 2.60, it suffices to prove that f4 (a, 1, 1) ≥ 0 for all real a. Indeed,

f4 (a, 1, 1) = 2(a − 1)2 (a + 2)2 ≥ 0.

The equality holds for a = b = c = 1.

P 2.73. If a, b, c are real numbers, then

(a2 + 2)(b2 + 2)(c 2 + 2) ≥ 9(a b + bc + ca).

L
.M
(Vasile Cîrtoaje, 1994)

D
Solution. We will prove the sharper inequality f4 (a, b, c) ≥ 0, where

A
‹2
a+b+c

2 2 2
f4 (a, b, c) = (a + 2)(b + 2)(c + 2) − 9(a b + bc + ca) − a bc −
PI .
3
M
Since f4 (a, b, c) is a symmetric polynomial of degree four, according to Remark from P
2.60, it suffices to prove that f4 (a, b, b) ≥ 0 for all real numbers a and b. For fixed b,
LY

this inequality is equivalent to f (a) ≥ 0, where


O

f (a) = 7(6b2 + 5)a2 + 2b(6b2 − 83)a + 18b4 − 13b2 + 72.


.M

Clearly, it is true for all real a if and only if

7(6b2 + 5)(18b4 − 13b2 + 72) ≥ b2 (6b2 − 83)2 .


W
W

Indeed, we have

7(6b2 + 5)(18b4 − 13b2 + 72) − b2 (6b2 − 83)2 = 360(b2 − 1)2 (2b2 + 7) ≥ 0.


W

The equality holds for a = b = c = 1.

P 2.74. If a, b, c are real numbers such that a b + bc + ca = 3, then

4(a4 + b4 + c 4 ) + 11a bc(a + b + c) ≥ 45.

(Vasile Cîrtoaje, 2014)


100 Vasile Cîrtoaje

Solution. Write the inequality in the homogeneous form

4(a4 + b4 + c 4 ) + 11a bc(a + b + c) ≥ 5(a b + bc + ca)2 .

It suffices to prove that there exists a positive number k such that f4 (a, b, c) ≥ 0, where

f4 (a, b, c) =4(a4 + b4 + c 4 ) + 11a bc(a + b + c) − 5(a b + bc + ca)2


− k(a b + bc + ca)(a2 + b2 + c 2 − a b − bc − ca).

According to P 2.60, the inequality f4 (a, b, c) ≥ 0 holds for all real a, b, c if and only if
f4 (a, 1, 1) ≥ 0 for all real a. We have

L
.M
f4 (a, 1, 1) = (a − 1)2 (2a + 1)(2a + 3) − k(2a + 1)(a − 1)2
= (a − 1)2 (2a + 1)(2a + 3 − k).

D
Setting k = 2, we get

A
f4 (a, 1, 1) = (a − 1)2 (2a + 1)2 ≥ 0.
The equality holds for a = b = c = ±1.
PI
M
LY

P 2.75. Any sixth degree symmetric homogeneous polynomial f6 (a, b, c) can be written in
O

the form
f6 (a, b, c) = Ar 2 + B(p, q)r + C(p, q),
.M

where A is called the highest coefficient of f6 , and


W

p = a + b + c, q = a b + bc + ca, r = a bc.
W

In the case A ≤ 0, prove that the inequality f6 (a, b, c) ≥ 0 holds for all real numbers a, b, c
if and only if f6 (a, 1, 1) ≥ 0 for all real a.
W

(Vasile Cîrtoaje, 2006)

Solution. For A ≤ 0 and fixed p and q,

g(r) = Ar 2 + B(p, q)r + C(p, q)

is a concave quadratic function of r. Therefore, g(r) is minimal when r is minimal or


maximal. By P 2.53, r is minimal and maximal when two of a, b, c are equal. Since
f6 (a, b, c) is symmetric, homogeneous and satisfies f6 (−a, −b, −c) = f6 (a, b, c), it fol-
lows that the inequality f6 (a, b, c) ≥ 0 holds for all real numbers a, b, c if and only if
f6 (a, 1, 1) ≥ 0 and f6 (a, 0, 0) ≥ 0 for all real a. Notice that the condition " f6 (a, 0, 0) ≥ 0
Symmetric Polynomial Inequalities in Real Variables 101

for all real a" is not necessary because it follows from the condition " f6 (a, 1, 1) ≥ 0 for
all real a" as follows:

f6 (a, 0, 0) = lim f6 (a, t, t) = lim t 6 f6 (a/t, 1, 1) ≥ 0.


t→0 t→0

Remark 1. In order to write the polynomial f6 (a, b, c) given by (A) as a function of p,


q and r, we can use the following relations:
X
a3 = 3r + p3 − 3pq,
X
a b(a + b) = −3r + pq,

L
X
a3 b3 = 3r 2 − 3pqr + q3 ,

.M
X
a2 b2 (a2 + b2 ) = −3r 2 − 2(p3 − 2pq)r + p2 q2 − 2q3 ,
X
a b(a4 + b4 ) = −3r 2 − 2(p3 − 7pq)r + p4 q − 4p2 q2 + 2q3 ,

D
X

A
a6 = 3r 2 + 6(p3 − 2pq)r + p6 − 6p4 q + 9p2 q2 − 2q3 .
PI
According to these relations, the highest coefficient A of the polynomuial f6 (a, b, c) has
the expression
M
(B) A = 3(A1 − A2 − A3 + A4 + A5 − A6 + A7 ).
LY

Remark 2. The polynomial


O

X
P1 (a, b, c) = (A1 a2 + A2 bc)(B1 a2 + B2 bc)(C1 a2 + C2 bc)
.M

has the highest coefficient


W

A = 3(A1 + A2 )(B1 + B2 )(C1 + C2 ) = P1 (1, 1, 1).


W

Indeed, since
W

X X €X Š X
P1 (a, b, c) =A1 B1 C1 a6 + A2 B2 C2 b3 c 3 + A1 B1 C2 a bc a3
€X Š
+3 A1 B2 C2 a2 b2 c 2 ,

we have
X X
A = 3A1 B1 C1 + 3A2 B2 C2 + 3 A1 B1 C2 + 3 A1 B2 C2
= 3(A1 + A2 )(B1 + B2 )(C1 + C2 ).

Similarly, we can show that the polynomial


X
P2 (a, b, c) = (A1 a2 + A2 bc)(B1 b2 + B2 ca)(C1 c 2 + C2 a b)
102 Vasile Cîrtoaje

has the highest coefficient

A = 3(A1 + A2 )(B1 + B2 )(C1 + C2 ) = P2 (1, 1, 1),

and the polynomial

P3 (a, b, c) = (A1 a2 + A2 bc)(A1 b2 + A2 ca)(A1 c 2 + A2 a b)

has the highest coefficient

A = (A1 + A2 )3 = P3 (1, 1, 1).

With regard to

L
.M
P4 (a, b, c) = (a − b)2 (b − c)2 (c − a)2 ,

from

D
A
P4 (a, b, c) = (p2 − 2q − c 2 − 2a b)(p2 − 2q − a2 − 2bc)(p2 − 2q − b2 − 2ca),
PI
it follows that P4 has the same highest coefficient as (−c 2 −2a b)(−a2 −2bc)(−b2 −2ca);
that is,
M
A = (−1 − 2)3 = −27.
LY

Remark 3. We can extend the statement in P 2.75 as follows:


• Let f6 (a, b, c) be a sixth degree symmetric homogeneous polynomial having the highest
O

coefficient A ≤ 0, and let k1 , k2 be two fixed real numbers. The inequality f6 (a, b, c) ≥ 0
.M

holds for all real numbers a, b, c satisfying

k1 (a + b + c)2 + k2 (a b + bc + ca) ≥ 0,
W

if and only if f6 (a, 1, 1) ≥ 0 for all real a satisfying k1 (a + 2)2 + k2 (2a + 1) ≥ 0.


W

Notice that the condition " f6 (a, 0, 0) ≥ 0 for all real a satisfying k1 a2 ≥ 0" is not
necessary because it follows from the condition " f6 (a, 1, 1) ≥ 0 for all real a satisfying
W

k1 (a + 2)2 + k2 (2a + 1) ≥ 0". Indeed, for the non-trivial case k1 ≥ 0, when the condition
" f6 (a, 0, 0) ≥ 0 for all real a satisfying k1 a2 ≥ 0" becomes " f6 (a, 0, 0) ≥ 0 for all real a",
we have
f6 (a, 0, 0) = lim f6 (a, t, t) = lim t 6 f6 (a/t, 1, 1) ≥ 0.
t→0 t→0

Remark 4. The statement in P 2.75 and its extension in Remark 3 are also valid in the
more general case when f6 (a, b, c) is a symmetric homogeneous function of the form

f6 (a, b, c) = Ar 2 + B(p, q)r + C(p, q),

where B(p, q) and C(p, q) are rational functions.


Symmetric Polynomial Inequalities in Real Variables 103

P 2.76. If a, b, c are real numbers such that a b + bc + ca = −1, then

(a) 5(a2 + b2 )(b2 + c 2 )(c 2 + a2 ) ≥ 8;

(b) (a2 + a b + b2 )(b2 + bc + c 2 )(c 2 + ca + a2 ) ≥ 1.

(Vasile Cîrtoaje, 2011)

Solution. Let p = a + b + c and q = a b + bc + ca.


(a) Write the inequality in the homogeneous form f6 (a, b, c) ≥ 0, where

f6 (a, b, c) = 5(a2 + b2 )(b2 + c 2 )(c 2 + a2 ) + 8(a b + bc + ca)3 .

L
.M
From Y Y
(b2 + c 2 ) = (p2 − 2q − a2 ),

D
it follows that f6 (a, b, c) has the highest coefficient A = −5. Then, by P 2.75, it suffices

A
to prove that f6 (a, 1, 1) ≥ 0 for all real a. Indeed, we have
PI
f6 (a, 1, 1) = 2(a + 3)2 (5a2 + 2a + 1) ≥ 0.
M
The homogeneous inequality f6 (a, b, c) ≥ 0 is an equality for −a/3 = b = c (or any
LY

cyclic permutation), and for b = c = p


0 (or any cyclic permutation).
p The original inequal-
ity becomes anpequality for a = −3/p 5 and b = c = 1/ 5 (or any cyclic permutation),
O

and for a = 3/ 5 and b = c = −1/ 5 (or any cyclic permutation)


.M

(b) Write the inequality in the homogeneous form f6 (a, b, c) ≥ 0, where


Y
f6 (a, b, c) = (b2 + bc + c 2 ) + (a b + bc + ca)3 .
W

First Solution. From


W

Y Y
(b2 + bc + c 2 ) = (p2 − 2q + bc − a2 ),
W

it follows that f6 (a, b, c) has the same highest coefficient as P3 (a, b, c), where
Y
P3 (a, b, c) = (bc − a2 );

that is, according to Remark 2 from P 2.75,

A = P3 (1, 1, 1) = (1 − 1)3 = 0.

Then, by P 2.75, it suffices to prove that f6 (a, 1, 1) ≥ 0 for all real a. Indeed, we have

f6 (a, 1, 1) = (a + 2)2 (3a2 + 2a + 1) ≥ 0.


104 Vasile Cîrtoaje

The homogeneous inequality f6 (a, b, c) ≥ 0 is an equality when a + b + c = 0, and when


b = c = 0 (or any cyclic permutation). The original inequality becomes an equality for
a b + bc + ca = −1 and a + b + c = 0.
Second Solution. As we have shown in the proof of P 2.35,
Y
(b2 + bc + c 2 ) = (p2 − q)q2 − p3 r,

where
p = a + b + c, q = a b + bc + ca, r = a bc.
Therefore,
1 2X 2

L
f6 (a, b, c) = p2 (q2 − pr) = p a (b + c)2 ≥ 0.
2

D
.M
A
P 2.77. If a, b, c are real numbers, then

(a)
P PI
a2 (a − b)(a − c)(a + 2b)(a + 2c) + (a − b)2 (b − c)2 (c − a)2 ≥ 0;
M
a2 (a − b)(a − c)(a − 4b)(a − 4c) + 7(a − b)2 (b − c)2 (c − a)2 ≥ 0.
P
(b)
LY

(Vasile Cîrtoaje, 2008)


O

Solution. Let p = a + b + c and q = a b + bc + ca. Consider the more general inequality


f6 (a, b, c) ≥ 0, where
.M

f6 (a, b, c) = f (a, b, c) + m(a − b)2 (b − c)2 (c − a)2 ≥ 0,


W

X
f (a, b, c) = a2 (a − b)(a − c)(a − k b)(a − kc).
W

Since X
f (a, b, c) = a2 (a2 + 2bc − q)[a2 + (k + k2 )bc − kq],
W

f (a, b, c) has the same highest coefficient as P1 (a, b, c), where


X
P1 (a, b, c) = a2 (a2 + 2bc)[a2 + (k + k2 )bc].

According to Remark 2 from P 2.75, f6 (a, b, c) has the highest coefficient

A = P1 (1, 1, 1) − 27m = 9(k2 + k + 1 − 3m).

(a) For k = −2 and m = 1, we get A = 0. Then, by P 2.75, it suffices to prove the


original inequality for b = c = 1; that is,

a2 (a − 1)2 (a + 2)2 ≥ 0.
Symmetric Polynomial Inequalities in Real Variables 105

The equality holds for a = b = c, for a + b + c = 0, and for a = 0 and b = c (or any
cyclic permutation).
(b) For k = 4 and m = 7, we get A = 0. Then, by P 2.75, it suffices to prove the
original inequality for b = c = 1; that is,

a2 (a − 1)2 (a − 4)2 ≥ 0.

The equality holds for a = b = c, and for a2 + b2 + c 2 = 2(a b + bc + ca).


Remark. The inequalities in P 2.77 are equivalent to
€X Š2 ”X X X —
a a4 + a bc a− a b(a2 + b2 ) ≥ 0

L
.M
and €X X Š €X X Š2
a2 − ab a2 − 2 ab ≥ 0,

D
respectively.

A
PI
P 2.78. If a, b, c are real numbers, then
M
(a2 + 2bc)(b2 + 2ca)(c 2 + 2a b) + (a − b)2 (b − c)2 (c − a)2 ≥ 0.
LY

(Vasile Cîrtoaje, 2011)


O

First Solution. Denote the left side of the inequality by f6 (a, b, c). According to Remark
.M

2 from P 2.75, f6 (a, b, c) has the highest coefficient

A = (1 + 2)3 − 27 = 0.
W

Then, by P 2.75, it suffices to prove that f6 (a, 1, 1) ≥ 0 for all real a. Indeed,
W

f6 (a, 1, 1) = (a2 + 2)(2a + 1)2 ≥ 0.


W

The equality holds for a b + bc + ca = 0.


Second Solution (by Vo Quoc Ba Can). Without loss of generality, assume that b and c
have the same sign. Since
1 2
(a − b)2 (a − c)2 = [(a + 2bc) + (a2 − 2a b − 2ac)]2
4
≥ (a2 + 2bc)(a2 − 2a b − 2ac)

and a2 + 2bc ≥ 0, it suffices to prove that

(b2 + 2ca)(c 2 + 2a b) + (b − c)2 (a2 − 2a b − 2ac) ≥ 0.


106 Vasile Cîrtoaje

This inequality is equivalent to

(b + c)2 a2 + 2bc(b + c)a + b2 c 2 ≥ 0,

or
[(b + c)a + bc]2 ≥ 0,
which is clearly true. The equality holds for a b + bc + ca = 0.
Remark 1. The inequality is equivalent to

(a2 + b2 + c 2 )(a b + bc + ca)2 ≥ 0.

L
Remark 2. The inequality in P 2.78 is a particular case of the following more general

.M
statement.
• If a, b, c are real numbers and

D
9k2 (k2 − k + 1)

A
 , 1≤k≤2
4(k + 1)3


αk = PI,
k2
, k≥2


4

M
then
LY

(a2 + k bc)(b2 + kca)(c 2 + ka b) + αk (a − b)2 (b − c)2 (c − a)2 ≥ 0,


O

with equality for −ka = b = c, and for b = c = 0 (or any cyclic permutation).
.M

P 2.79. If a, b, c are real numbers, then


W

(2a2 + 5a b + 2b2 )(2b2 + 5bc + 2c 2 )(2c 2 + 5ca + 2a2 ) + (a − b)2 (b − c)2 (c − a)2 ≥ 0.
W

(Vasile Cîrtoaje, 2011)


W

Solution. Let p = a + b + c and q = a b + bc + ca. Write the inequality as f6 (a, b, c) ≥ 0,


where
f6 (a, b, c) = f (a, b, c) + (a − b)2 (b − c)2 (c − a)2 ,
Y
f (a, b, c) = (2b2 + 5bc + 2c 2 ).
Since Y
f (a, b, c) = (2p2 − 4q + 5bc − 2a2 ),
f (a, b, c) has the same highest coefficient as P3 (a, b, c), where
Y
P3 (a, b, c) = (5bc − 2a2 ).
Symmetric Polynomial Inequalities in Real Variables 107

Therefore, according to Remark 2 from P 2.75, f6 (a, b, c) has the highest coefficient

A = P3 (1, 1, 1) − 27 = 0.

Then, by P 2.75, it suffices to prove that f6 (a, 1, 1) ≥ 0 for all real a. Indeed,

f6 (a, 1, 1) = 9(2a2 + 5a + 2)2 ≥ 0.

The equality holds for a + b + c = 0, and also for a b + bc + ca = 0.

Remark 1. The inequality in P 2.79 is equivalent to

L
(a + b + c)2 (a b + bc + ca)2 ≥ 0.

.M
Remark 2. The following more general statement holds.

D
• Let a, b, c be real numbers. If k > −2, then

A
Y
4 (b2 + k bc + c 2 ) ≥ (2 − k)(a − b)2 (b − c)2 (c − a)2 .
Notice that this inequality is equivalent to
PI
M
(k + 2)[(a + b + c)(a b + bc + ca) − (5 − 2k)a bc]2 ≥ 0.
LY
O
.M

P 2.80. If a, b, c are real numbers, then

2 2 2 64 2
 ‹ ‹ ‹
W

a2 + a b + b2 b2 + bc + c 2 c 2 + ca + a2 ≥ (a + bc)(b2 + ca)(c 2 + a b).


3 3 3 27
W

Solution. Let p = a + b + c, q = a b + bc + ca and


W

2 2 2
 ‹ ‹ ‹
2 2 2 2 2 2
f (a, b, c) = a + a b + b b + bc + c c + ca + a .
3 3 3

We need to prove that f6 (a, b, c) ≥ 0, where

64 2
f6 (a, b, c) = f (a, b, c) − (a + bc)(b2 + ca)(c 2 + a b).
27
Since
2 2 2
 ‹ ‹ ‹
f (a, b, c) = p2 − 2q + a b − c 2 p2 − 2q + bc − a2 p2 − 2q + ca − b2 ,
3 3 3
108 Vasile Cîrtoaje

f6 (a, b, c) has the same highest coefficient as


2 2 2 64 2
 ‹ ‹ ‹
2 2 2
ab − c bc − a ca − b − (a + bc)(b2 + ca)(c 2 + a b);
3 3 3 27
that is, according to Remark 2 from P 2.75,
‹3
2 64

A= −1 − (1 + 1)3 < 0.
3 27
Then, it suffices to prove that f6 (a, 1, 1) ≥ 0 for all real a (see P 2.75). Indeed,
‹2
8 2 2 64 2 8


L
f6 (a, 1, 1) = a + a+1 − (a + 1)(a + 1)2 = (a − 1)4 ≥ 0.
3 3 27 27

.M
The equality holds for a = b = c.

D
A
P 2.81. If a, b, c are real numbers, then PI
2(a − b)2 (b − c)2 (c − a)2
M
X
a2 (a − b)(a − c) ≥ .
a2 + b2 + c 2
LY

Solution. Let p = a + b + c, q = a b + bc + ca, r = a bc and


O

X
f6 (a, b, c) = (a2 + b2 + c 2 ) a2 (a − b)(a − c) − 2(a − b)2 (b − c)2 (c − a)2 .
.M

Clearly, f6 (a, b, c) has the highest coefficient


W

A = −2(−27) = 54.

Since A > 0, we will use the highest coefficient cancellation method. It is easy to check
W

that
W

f (1, 1, 1) = 0, f (0, 1, 1) = 0.
Therefore, we define the symmetric homogeneous polynomial of degree three

P(a, b, c) = r + Bp3 + C pq

such that P(1, 1, 1) = 0 and P(0, 1, 1) = 0; that is,


1 3 4
P(a, b, c) = r + p − pq.
9 9
We will prove the sharper inequality g6 (a, b, c) ≥ 0, where

g6 (a, b, c) = f6 (a, b, c) − 54P 2 (a, b, c).


Symmetric Polynomial Inequalities in Real Variables 109

Since g6 (a, b, c) has the highest coefficient A1 = 0, it suffices to show that g6 (a, 1, 1) ≥ 0
for all real a (see P 2.75). Indeed, we have

1
f6 (a, 1, 1) = a2 (a2 + 2)(a − 1)2 , P(a, 1, 1) = a(a − 1)2 ,
81
hence
1 2
g6 (a, 1, 1) = f6 (a, 1, 1) − 54P(a, 1, 1) = a (a − 1)2 (a + 2)2 ≥ 0.
3
The equality holds for a = b = c, for a = 0 and b = c (or any cyclic permutation), and
also for a = 0 and b + c = 0 (or any cyclic permutation).

L
Remark. In the same manner, we can prove the following generalization (Vasile Cîrtoaje,

.M
2014).
• Let x, y, z be real numbers. If k ∈ [−1, 2), then

D
A
X (2 − k)(x − y)2 ( y − z)2 (z − x)2
x 2 (x − y)(x − z) ≥ ,
x 2 + y 2 + z 2 + k(x y + yz + z x)
PI
with equality for x = y = z, and for x = 0 and y 2 = z 2 (or any cyclic permutation).
M
LY
O

P 2.82. If a, b, c are real numbers, then


.M

X 8(a − b)2 (b − c)2 (c − a)2


(a − b)(a − c)(a − 2b)(a − 2c) ≥ .
a2 + b2 + c 2
W

Solution. Let
W

X
f6 (a, b, c) = (a2 + b2 + c 2 ) (a − b)(a − c)(a − 2b)(a − 2c) − 8(a − b)2 (b − c)2 (c − a)2 .
W

Clearly, f6 (a, b, c) has the highest coefficient

A = (−8)(−27) = 216.

Since A > 0, we will use the highest coefficient cancellation method. Since

f (1, 1, 1) = 0, f (2, 1, 1) = 0,

we define the symmetric homogeneous polynomial of degree three

P(a, b, c) = a bc + B(a + b + c)3 + C(a + b + c)(a b + bc + ca)


110 Vasile Cîrtoaje

such that P(1, 1, 1) = 0 and P(2, 1, 1) = 0. We get B = 1/18 and C = −5/18, hence
1 5
P(a, b, c) = a bc + (a + b + c)3 − (a + b + c)(a b + bc + ca).
18 18
Consider now the sharper inequality g6 (a, b, c) ≥ 0, where

g6 (a, b, c) = f6 (a, b, c) − 216P 2 (a, b, c).

Clearly, g6 (a, b, c) has the highest coefficient A1 = 0. By P 2.75, it suffices to prove that
g6 (a, 1, 1) ≥ 0 for all real a. We have

L
f6 (a, 1, 1) = (a2 + 2)(a − 1)2 (a − 2)2 , P(a, 1, 1) = (a − 1)2 (a − 2),
18

.M
hence
1
g6 (a, 1, 1) = f6 (a, 1, 1) − 216P 2 (a, 1, 1) =
(a − 1)2 (a2 − 4)2 ≥ 0.

D
3

A
The equality holds for a = b = c, for a = 0 and b + c = 0 (or any cyclic permutation),
and also for a/2 = b = c (or any cyclic permutation). PI
M
Remark. In the same manner, we can prove the following generalization (Vasile Cîrtoaje,
2014).
LY

• Let x, y, z be real numbers. If k ∈ R, then


O

X (k + 2)2 (x − y)2 ( y − z)2 (z − x)2


(x − y)(x − z)(x − k y)(x − kz) ≥ , (2.1)
2(x 2 + y 2 + z 2 )
.M

with equality for x = y = z, for x/k = y = z (or any cyclic permutation) if k 6= 0, and for
x = 0 and y + z = 0 (or any cyclic permutation).
W
W
W

P 2.83. If a, b, c are real numbers, no two of which are zero, then

a2 + 3bc b2 + 3ca c 2 + 3a b
+ 2 + 2 ≥ 0.
b2 + c 2 c + a2 a + b2
(Vasile Cîrtoaje, 2014)

Solution. Write the inequality as f6 (a, b, c) ≥ 0, where


X
f6 (a, b, c) = (a2 + 3bc)(a2 + b2 )(a2 + c 2 ).

Let
p = a + b + c, q = a b + bc + ca, r = a bc.
Symmetric Polynomial Inequalities in Real Variables 111

From X
f6 (a, b, c) = (a2 + 3bc)(p2 − 2q − c 2 )(p2 − 2q − b2 ),
it follows that f6 (a, b, c) has the same highest coefficient A as f (a, b, c), where
X X
f (a, b, c) = (a2 + 3bc)b2 c 2 = 3r 2 + 3 b3 c 3 = 12r 2 − 9pqr + 3q3 ;

that is, A = 12. Since A > 0, we will use the highest coefficient cancellation method. It is
easy to check that
f6 (−1, 1, 1) = 0.
So, we define the homogeneous polynomial

L
.M
P(a, b, c) = r + Bp3 + (B − 1)pq,

which satisfies the property P(−1, 1, 1) = 0. We will show that there is at least a real

D
value of B such that the following sharper inequality holds

A
f6 (a, b, c) ≥ 12P 2 (a, b, c).
PI
M
Let us denote
g6 (a, b, c) = f6 (a, b, c) − 12P 2 (a, b, c).
LY

Clearly, g6 (a, b, c) has the highest coefficient A1 = 0. By P 2.75, it suffices to prove that
g6 (a, 1, 1) ≥ 0 for all real a. We have
O

f6 (a, 1, 1) = (a + 1)2 (a2 + 1)(a2 − 2a + 7)


.M

and
W

P(a, 1, 1) = (a + 1)[B(a + 2)(a + 5) − 2(a + 1)],


hence
W

g6 (a, 1, 1) = f6 (a, 1, 1) − 12P 2 (a, 1, 1) = (a + 1)2 g(a),


W

where
g(a) = (a2 + 1)(a2 − 2a + 7) − 12[B(a + 2)(a + 5) − 2(a + 1)]2 .
Choosing B = 1/4, we get

4g(a) = a2 (a − 1)2 + 4(4a2 + a + 4) > 0,

hence g6 (a, 1, 1) ≥ 0 for all real a. The proof is completed. The equality holds for
−a = b = c (or any cyclic permutation).
112 Vasile Cîrtoaje

P 2.84. If a, b, c are real numbers, no two of which are zero, then

a2 + 6bc b2 + 6ca c 2 + 6a b
+ + ≥ 0.
b2 − bc + c 2 c 2 − ca + a2 a2 − a b + b2
(Vasile Cîrtoaje, 2014)

Solution. Write the inequality as f6 (a, b, c) ≥ 0, where


X
f6 (a, b, c) = (a2 + 6bc)(a2 − a b + b2 )(a2 − ac + c 2 ).

Let
p = a + b + c, q = a b + bc + ca, r = a bc.

L
.M
From X
f6 (a, b, c) = (a2 + 6bc)(p2 − 2q − c 2 − a b)(p2 − 2q − b2 − ac),

D
it follows that f6 (a, b, c) has the same highest coefficient A as f (a, b, c), where

A
X
f (a, b, c) = (a2 + 6bc)(b2 + ca)(c 2 + a b);

that is, according to Remark 2 from P 2.75,


PI
M
A = f (1, 1, 1) = 84.
LY

Since A > 0, we use the highest coefficient cancellation method. We will show that there
O

are two real numbers B and C such that the following sharper inequality holds

f6 (a, b, c) ≥ 84P 2 (a, b, c),


.M

where
P(a, b, c) = r + Bp3 + C pq.
W

Let us denote
W

g6 (a, b, c) = f6 (a, b, c) − 84P 2 (a, b, c).


W

Clearly, g6 (a, b, c) has the highest coefficient equal to zero. Then, it suffices to prove
that g6 (a, 1, 1) ≥ 0 for all real a.
We have
g6 (a, 1, 1) = f6 (a, 1, 1) − 84P 2 (a, 1, 1),
where
f6 (a, 1, 1) = (a2 − a + 1)(a2 + a + 1)(a2 − 2a + 8)
and
P(a, 1, 1) = a + B(a + 2)3 + C(a + 2)(2a + 1).
Let us denote g(a) = g6 (a, 1, 1). Since g(−2) = 0, we can have g(a) ≥ 0 in the vicinity
of a = −2 only if g 0 (−2) = 0, which involves C = −61/168. On the other hand,
Symmetric Polynomial Inequalities in Real Variables 113

from g(1) = 0, we get B = 155/1512. Using these values of B and C, the inequality
g6 (a, 1, 1) ≥ 0 is equivalent to

27216(a2 − a + 1)(a2 + a + 1)(a2 − 2a + 8) ≥


2
≥ 155(a + 2)3 − 549(a + 2)(2a + 1) + 1512a ;


that is,
(a + 2)2 (a − 1)2 (3191a2 − 8734a + 49391) ≥ 0,
which is true for all real a.
The proof is completed. The equality holds for a = b+c = 0 (or any cyclic permutation).

L
.M
P 2.85. If a, b, c are real numbers such that a b + bc + ca ≥ 0, then

D
4a2 + 23bc 4b2 + 23ca 4c 2 + 23a b

A
+ + ≥ 0.
b2 + c 2 c 2 + a2 a2 + b2
PI (Vasile Cîrtoaje, 2014)
M
Solution. Write the inequality as f6 (a, b, c) ≥ 0, where
LY

X
f6 (a, b, c) = (4a2 + 23bc)(a2 + b2 )(a2 + c 2 ).
O

Let
p = a + b + c, q = a b + bc + ca, r = a bc.
.M

From X
f6 (a, b, c) = (4a2 + 23bc)(p2 − 2q − c 2 )(p2 − 2q − b2 ),
W

it follows that f6 (a, b, c) has the same highest coefficient A as f (a, b, c), where
W

X X
f (a, b, c) = (4a2 + 23bc)b2 c 2 = 12r 2 + 23 b3 c 3 = 81r 2 − 69pqr + 23q3 ;
W

that is, A = 81. Since A > 0, we will use the highest coefficient cancellation method. It is
easy to check that
f (−1, 2, 2) = 0.
Therefore, define the homogeneous polynomial
4 3
P(a, b, c) = r + p + C pq,
27
which satisfies the property P(−1, 2, 2) = 0. We will show that there is at least a real C
such that the following sharper inequality holds for a b + bc + ca ≥ 0:

f6 (a, b, c) ≥ 81P 2 (a, b, c).


114 Vasile Cîrtoaje

Let us denote
g6 (a, b, c) = f6 (a, b, c) − 81P 2 (a, b, c).
Clearly, g6 (a, b, c) has the highest coefficient A1 = 0. Then, by Remark 3 from P 2.75, it
suffices to prove that g6 (a, 1, 1) ≥ 0 for all real a such that 2a + 1 ≥ 0.
We have
f6 (a, 1, 1) = (2a + 1)(a2 + 1)(2a3 − a2 + 14a + 39),
1
P(a, 1, 1) = (2a + 1)[2a2 + (27C + 11)a + 54C + 32],
27
g6 (a, 1, 1) = f6 (a, 1, 1) − 81P 2 (a, 1, 1).

L
From the condition g6 (1, 1, 1) = 0, we get C = −1/3. For this value of C, we find

.M
2
P(a, 1, 1) = (2a + 1)(a2 + a + 7),
27

D
then

A
1
g6 (a, 1, 1) =
9
PI
(2a + 1)(10a5 − 29a4 + 16a3 + 170a2 − 322a + 155)
M
1
= (2a + 1)(a − 1)2 (10a3 − 9a2 − 12a + 155).
9
LY

We need to show that 10a3 − 9a2 − 12a + 155 ≥ 0 for a ≥ −1/2. This is clearly true for
−1/2 ≤ a ≤ 0. Also, for a > 0, we have
O

10a3 − 9a2 − 12a + 155 = 10a(a2 − a + 1) + (a − 11)2 + 34 > 0.


.M

The proof is completed. The equality holds for −2a = b = c (or any cyclic permutation).
W
W

P 2.86. If a, b, c are real numbers such that a b + bc + ca = 3, then


W

20(a6 + b6 + c 6 ) + 43a bc(a3 + b3 + c 3 ) ≥ 189.

(Vasile Cîrtoaje, 2014)

Solution. Write the inequality in the homogeneous form f6 (a, b, c) ≥ 0, where

f6 (a, b, c) = 20(a6 + b6 + c 6 ) + 43a bc(a3 + b3 + c 3 ) − 7(a b + bc + ca)3 .

Since the highest coefficient of f6 (a, b, c) is positive, namely

A = 20 · 3 + 43 · 3 = 189,
Symmetric Polynomial Inequalities in Real Variables 115

we will use the highest coefficient cancellation method. From

f6 (a, 1, 1) = (2a + 1)(a − 1)2 (10a3 + 15a2 + 44a + 33),

it follows that
f6 (1, 1, 1) = 0, f6 (−1/2, 1, 1) = 0.
Define the homogeneous function

P(a, b, c) = r + Bp3 + C pq, p = a + b + c, q = a b + bc + ca, r = a bc,

such that P(1, 1, 1) = P(−1/2, 1, 1) = 0; that is,

L
4 3 5
P(a, b, c) = r + p − pq,

.M
27 9
hence

D
27a + 4(a + 2)3 − 15(a + 2)(2a + 1) 2(a − 1)2 (2a + 1)
P(a, 1, 1) = = .

A
27 27
PI
We will show that the following sharper inequality holds for a b + bc + ca ≥ 0:

f6 (a, b, c) ≥ 189P 2 (a, b, c).


M
Let us denote
LY

g6 (a, b, c) = f6 (a, b, c) − 189P 2 (a, b, c).


O

Since the highest coefficient of g6 (a, b, c) is zero, it suffices to prove that g6 (a, 1, 1) ≥ 0
for all real a such that 2a + 1 ≥ 0 (see Remark 3 from P 2.75). We have
.M

g6 (a, 1, 1) = f6 (a, 1, 1) − 189P 2 (a, 1, 1) = (2a + 1)(a − 1)2 g(a),

where
W

28
g(a) = 10a3 + 15a2 + 44a + 33 − (a − 1)2 (2a + 1).
27
W

Since
W

g(a) ≥ 10a3 + 15a2 + 44a + 33 − 5(a − 1)2 (2a + 1) = 22(a + 1)2 + 8a2 + 6 > 0,

we have g6 (a, 1, 1) ≥ 0 for all a ≥ −1/2. Thus, the proof is completed. The equality
holds for a = b = c = 1.

P 2.87. If a, b, c are real numbers, then


X
4 (a2 + bc)(a − b)(a − c)(a − 3b)(a − 3c) ≥ 7(a − b)2 (b − c)2 (c − a)2 .

(Vasile Cîrtoaje, 2014)


116 Vasile Cîrtoaje

Solution. Write the inequality as f6 (a, b, c) ≥ 0, where

f6 (a, b, c) = 4 f (a, b, c) − 7(a − b)2 (b − c)2 (c − a)2 ,


X
f (a, b, c) = (a2 + bc)(a − b)(a − c)(a − 3b)(a − 3c).
We have
f6 (a, 1, 1) = 4(a2 + 1)(a − 1)2 (a − 3)2 .
Let
p = a + b + c, q = a b + bc + ca, r = a bc.
Since (a − b)(a − c) = a2 + 2bc − q and (a − 3b)(a − 3c) = a2 + 12bc − 3q, f (a, b, c) has

L
the same highest coefficient A0 as g(a, b, c), where

.M
X
g(a, b, c) = (a2 + bc)(a2 + 2bc)[a2 + 12bc];

D
that is, according to Remark 2 from P 2.75,

A
PI
A0 = g(1, 1, 1) = 3 · 2 · 3 · 13 = 234.
M
Therefore, f6 (a, b, c) has the highest coefficient
LY

A = 4A0 − 7(−27) = 1125.


O

Since the highest coefficient A is positive, we will use the highest coefficient cancellation
method. There are two cases to consider: q ≥ 0 and q < 0.
.M

Case 1: q ≥ 0. Since
f6 (1, 1, 1) = f6 (3, 1, 1) = 0,
W

define the homogeneous function


W

P(a, b, c) = r + Bp3 + C pq
W

such that P(1, 1, 1) = P(3, 1, 1) = 0; that is,

2 3 11
P(a, b, c) = r + p − pq,
45 45
hence

45a + 2(a + 2)3 − 11(a + 2)(2a + 1) 2(a − 1)2 (a − 3)


P(a, 1, 1) = = .
45 45
We will show that the following sharper inequality holds for q ≥ 0:

f6 (a, b, c) ≥ 1125P 2 (a, b, c).


Symmetric Polynomial Inequalities in Real Variables 117

Let us denote
g6 (a, b, c) = f6 (a, b, c) − 1125P 2 (a, b, c).
Since the highest coefficient of g6 (a, b, c) is zero, it suffices to prove that g6 (a, 1, 1) ≥ 0
for all real a such that 2a + 1 ≥ 0 (see Remark 3 from P 2.75). We have

8(a − 1)2 (a − 3)2 (a + 2)(2a + 1)


g6 (a, 1, 1) = f6 (a, 1, 1) − 1125P 2 (a, 1, 1) = ≥ 0.
9
Case 2: q < 0. Define the homogeneous polynomial

1
 ‹
3
P(a, b, c) = r + Bp − 3B + pq,

L
9

.M
which satisfies P(1, 1, 1) = 0. We will show that there is a real number B such that the
following sharper inequality holds

D
f6 (a, b, c) ≥ 1125P 2 (a, b, c).

A
Let us denote PI
g6 (a, b, c) = f6 (a, b, c) − 1125P 2 (a, b, c).
M
Clearly, g6 (a, b, c) has the highest coefficient equal to zero. Then, by Remark 3 from P
2.75, it suffices to prove that g6 (a, 1, 1) ≥ 0 for 2a + 1 < 0. We have
LY

g6 (a, 1, 1) = f6 (a, 1, 1) − 1125P 2 (a, 1, 1),


O

where
.M

˜2
1
•  ‹
2 3
P (a, 1, 1) = a + B(a + 2) − 3B + (a + 2)(2a + 1) .
9
Let us denote g(a) = g6 (a, 1, 1). Since g(−2) = 0, we can have g(a) ≥ 0 in the vicinity
W

of a = −2 only if g 0 (−2) = 0, which involves B = 8/135. Using this value of B, we get


W

4(a − 1)4 (4a − 7)2


P 2 (a, 1, 1) = ,
W

25 · 729

5
• ˜
2 2 2
g6 (a, 1, 1) = 4(a − 1) (a + 1)(a − 3) − (a − 1)2 (4a − 7)2
81
4
= (a − 1)2 (a + 2)2 (a2 − 50a + 121) ≥ 0.
81
The proof is completed. The equality holds for a = b = c, for a/3 = b = c (or any cyclic
permutation), and for a = 0 and b + c = 0 (or any cyclic permutation).
118 Vasile Cîrtoaje

P 2.88. Let a, b, c be real numbers such that a b + bc + ca ≥ 0. For any real k, prove that
X
4bc(a − b)(a − c)(a − k b)(a − kc) + (a − b)2 (b − c)2 (c − a)2 ≥ 0.

(Vasile Cîrtoaje, 2014)

Solution. Write the inequality as f6 (a, b, c) ≥ 0, where

f6 (a, b, c) = 4 f (a, b, c) + (a − b)2 (b − c)2 (c − a)2 ,


X
f (a, b, c) = bc(a − b)(a − c)(a − k b)(a − kc), k ∈ R.

L
Let

.M
p = a + b + c, q = a b + bc + ca, r = a bc.
Since (a − b)(a − c) = a2 + 2bc − q and (a − k b)(a − kc) = a2 + (k + k2 )bc − kq, f (a, b, c)

D
has the same highest coefficient A0 as P1 (a, b, c), where

A
X
P1 (a, b, c) = bc(a2 + 2bc)[a2 + (k + k2 )bc];
PI
that is, according to Remark 2 from P 2.75,
M
A0 = P1 (1, 1, 1) = 3(1 + 2)(1 + k + k2 ) = 9(1 + k + k2 ).
LY

Therefore, f6 (a, b, c) has the highest coefficient


O

A = 4A0 − 27 = 9(2k + 1)2 .


.M

We have
f6 (a, 1, 1) = 4(a − 1)2 (a − k)2 .
W

Consider first that k = −1/2, when A = 0. By P 2.75, it suffices to prove that f6 (a, 1, 1) ≥
W

0 for all real a. Clearly, these conditions are fulfilled. Consider further that k 6= −1/2,
when the highest coefficient A is positive. We will use the highest coefficient cancellation
W

method. Since
f6 (1, 1, 1) = f6 (k, 1, 1) = 0,
define the homogeneous function

q2
P(a, b, c) = r + C pq + D
p

such that P(1, 1, 1) = P(k, 1, 1) = 0; that is,

pq 2(k + 2)q2
P(a, b, c) = r + − .
3(2k + 1) 3(2k + 1)p
Symmetric Polynomial Inequalities in Real Variables 119

We will show that the following sharper inequality holds for a b + bc + ca ≥ 0:

f6 (a, b, c) ≥ 9(2k + 1)2 P 2 (a, b, c).

Let us denote
g6 (a, b, c) = f6 (a, b, c) − 9(2k + 1)2 P 2 (a, b, c).
Clearly, g6 (a, b, c) has the highest coefficient A1 = 0. Then, by Remark 4 from P 2.75, it
suffices to prove that g6 (a, 1, 1) ≥ 0 for all real a such that 2a + 1 ≥ 0. We have

(a + 2)(2a + 1) 2(k + 2)(2a + 1)2


P(a, 1, 1) = a + −
3(2k + 1) 3(2k + 1)(a + 2)

L
2(a − 1) (a − k)
2

.M
= ,
3(2k + 1)(a + 2)

D
then

A
g6 (a, 1, 1) = f6 (a, 1, 1) − 9(2k + 1)2 P 2 (a, 1, 1)

=
PI
12(a − 1)2 (a − k)2 (2a + 1)
≥ 0.
(a + 2)2
M
The proof is completed. The equality holds for a = b = c, for a/k = b = c (or any cyclic
LY

permutation) - if k 6= 0, and for b = c = 0 (or any cyclic permutation).


O
.M

P 2.89. If a, b, c are real numbers, then


W

2
(a2 b + b2 c + c 2 a) + (a b2 + bc 2 + ca2 ) ≥ 4(a b + bc + ca)(a2 b2 + b2 c 2 + c 2 a2 ).

W

First Solution. Consider the nontrivial case a b + bc + ca ≥ 0, and write the inequality
W

as f6 (a, b, c) ≥ 0, where

f6 (a, b, c) = [(a2 b+ b2 c +c 2 a)+(a b2 + bc 2 +ca2 )]2 −4(a b+ bc +ca)(a2 b2 + b2 c 2 +c 2 a2 ).

Since

(a2 b + b2 c + c 2 a) + (a b2 + bc 2 + ca2 ) = (a + b + c)(a b + bc + ca) − 3a bc,

f6 has the highest coefficient A = (−3)2 = 9. Since A > 0, we will use the highest
coefficient cancellation method. Because

f (1, 1, 1) = f (0, 1, 1) = 0,
120 Vasile Cîrtoaje

define the homogeneous function

(a b + bc + ca)2
P(a, b, c) = a bc + C(a + b + c)(a b + bc + ca) + D
a+b+c
such that P(1, 1, 1) = P(0, 1, 1) = 0; that is,

(a + b + c)(a b + bc + ca) 4(a b + bc + ca)2


P(a, b, c) = a bc + − .
3 3(a + b + c)

We will show that the following sharper inequality holds for a b + bc + ca ≥ 0:

L
f6 (a, b, c) ≥ 9P 2 (a, b, c).

.M
Let us denote
g6 (a, b, c) = f6 (a, b, c) − 9P 2 (a, b, c).

D
Clearly, g6 (a, b, c) has the highest coefficient A1 = 0. Then, by Remark 4 from P 2.75, it

A
suffices to prove that g6 (a, 1, 1) ≥ 0 for all real a such that 2a + 1 ≥ 0. We have
PI
f6 (a, 1, 1) = 4a2 (a − 1)2 ,
M
(a + 2)(2a + 1) 4(2a + 1)2 2a(a − 1)2
P(a, 1, 1) = a + − = ,
LY

3 3(a + 2) 3(a + 2)
hence
O

12a2 (a − 1)2 (2a + 1)


f6 (a, 1, 1) = f6 (a, 1, 1) − 9P 2 (a, 1, 1) = ≥ 0.
(a + 2)2
.M

The proof is completed. The equality holds for a = b = c, for a = 0 and b = c (or any
cyclic permutation), and for b = c = 0 (or any cyclic permutation).
W

Second Solution (by Nguyen Van Quy). Since the inequality remains unchanged by
replacing a, b, c with −a, −b, −c, we may assume that a+ b+c ≥ 0. In addition, consider
W

the non-trivial case a b + bc + ca > 0. Using the Cauchy-Schwarz inequality, we have


W

(a2 b + b2 c + c 2 a) + (a b2 + bc 2 + ca2 ) + (a3 + b3 + c 3 ) = (a2 + b2 + c 2 )(a + b + c)


Ç  
= a4 + b4 + c 4 + 2(a2 b2 + b2 c 2 + c 2 a2 ) a2 + b2 + c 2 + 2(a b + bc + ca)
Æ Æ
≥ (a4 + b4 + c 4 )(a2 + b2 + c 2 ) + 2 (a2 b2 + b2 c 2 + c 2 a2 )(a b + bc + ca).
Thus, it suffices to show that
Æ
(a4 + b4 + c 4 )(a2 + b2 + c 2 ) ≥ a3 + b3 + c 3 ,

which follows also from the Cauchy-Schwarz inequality.


Symmetric Polynomial Inequalities in Real Variables 121

P 2.90. If a, b, c are real numbers such that a + b + c = 3, then

(a − 1)(a − 25) (b − 1)(b − 25) (c − 1)(c − 25)


+ + ≥ 0.
a2 + 23 b2 + 23 c 2 + 23

Solution. Let p = a+b+c. Write the inequality in the homogeneous form f6 (a, b, c) ≥ 0,
where X
f6 (a, b, c) = (3a − p)(3a − 25p)(9b2 + 23p2 )(9c 2 + 23p2 ).

Since the highest coefficient of f6 is positive, namely

L
A = 3 · 93 ,

.M
we use the highest coefficient cancellation method. Thus, we will prove that there exist

D
two real numbers B and C such that g6 (a, b, c) ≥ 0, where

A
g6 (a, b, c) = f6 (a, b, c) − A[a bc + B(a + b + c)3 + C(a + b + c)(a b + bc + ca)]2 .
PI
Since g6 has the highest coefficient equal to zero, it suffices to show that g6 (a, 1, 1) ≥ 0
M
for all real a. Notice that
LY

f6 (a, 1, 1) = 12(a − 1)2 (7a + 11)2 [23(a + 2)2 + 9]


O

and
g6 (a, 1, 1) = f6 (a, 1, 1) − 3 · 93 [a + B(a + 2)3 + C(a + 2)(2a + 1)]2 .
.M

Let us denote g(a) = g6 (a, 1, 1). Since g(−2) = 0, we can have g(a) ≥ 0 in the vicinity
of a = −2 only if g 0 (−2) = 0; this involves C = −13/9, hence
W

g6 (a, 1, 1) = 12(a−1)2 (7a+11)2 [23(a+2)2 +9]−27[9a+9B(a+2)3 −13(a+2)(2a+1)]2 .


W

There are two cases to consider: 5p2 + q ≤ 0 and 5p2 + q ≥ 0.


W

Case 1: 5p2 + q ≤ 0. By Remark 3 from the proof of P 2.75, we only need to show that
there exist a real number B such that g6 (a, 1, 1) ≥ 0 for all real a satisfying 5(a + 2)2 +
2a + 1 ≤ 0; that is, for a ∈ [−3, −7/5]. From g6 (−11/7, 1, 1) = 0, we get B = 28/9,
then

g6 (a, 1, 1) = 12(a − 1)2 (7a + 11)2 [23(a + 2)2 + 9] − 108(7a + 11)2 (2a2 + 7a + 9)2
= −12(a + 2)2 (7a + 11)2 (13a2 + 154a + 157) ≥ 0.

Case 2: 5p2 + q ≥ 0. By Remark 3 from the proof of P 2.75, we only need to show
that there exist a real number B such that g6 (a, 1, 1) ≥ 0 for all real a satisfying 5(a +
122 Vasile Cîrtoaje

2)2 + 2a + 1 ≥ 0; that is, for a ∈ (−∞, −3] ∪ [−7/5, ∞). From g6 (1, 1, 1) = 0, we get
B = 4/9, then

g6 (a, 1, 1) = 12(a − 1)2 (7a + 11)2 [23(a + 2)2 + 9] − 108(a − 1)4 (2a + 3)2
= 12(a + 2)2 (a − 1)2 (1091a2 + 3650a + 3035) ≥ 0.

The proof is completed. The equality holds for a = b = c = 1, and for a = −11 and
b = c = 7 (or any cyclic permutation).

P 2.91. If a, b, c are real numbers such that a bc 6= 0, then

L
.M
b + c 2  c + a 2 a+b 2
 ‹  ‹
+ + > 2.
a b c

D
(Michael Rozenberg, 2014)

A
Solution. Assume that a2 = min{a2 , b2 , c 2 }. By the Cauchy-Schwarz inequality, we
have
 c + a 2  a + b ‹2 [(c + a) + (−a − b)]2
PI (b − c)2
+ ≥ = .
M
b c b2 + c 2 b2 + c 2
On the other hand,
LY

b + c 2 (b + c)2
 ‹
≥ 2 .
a b + c2
O

Therefore,
.M

b + c 2  c + a 2 a + b 2 (b + c)2 (b − c)2
 ‹  ‹
+ + ≥ 2 + 2 = 2.
a b c b + c2 b + c2
W

The equality holds if and only if


 c + a 2 ‹2
a+b (b − c)2
W


+ =
b c b2 + c 2
W

and b + c = 0. Since these relations involves a = 0, we conclude that the inequality is


strict (the equality does not hold).

P 2.92. If a, b, c are real numbers, then


8
(a) (a2 + 1)(b2 + 1)(c 2 + 1) ≥ p |(a − b)(b − c)(c − a)|;
3 3
(b) (a2 − a + 1)(b2 − b + 1)(c 2 − c + 1) ≥ |(a − b)(b − c)(c − a)|.
(Kwon Ji Mun, 2011)
Symmetric Polynomial Inequalities in Real Variables 123

Solution. (a) First Solution. Without loss of generality, assume that a ≤ b ≤ c, when

|(a − b)(b − c)(c − a)| = (a − b)(b − c)(c − a).

Denote
4
k= p
3 3
and write the inequality as
Aa2 + 2Ba + C ≥ 0,
where
A = (b2 + 1)(c 2 + 1) + 2k(b − c),

L
.M
B = −k(b2 − c 2 ),
C = (b2 + 1)(c 2 + 1) + 2k bc(b − c).

D
−x y
Substituting b = p and c = p , by the Cauchy-Schwarz inequality, we get

A
3 3
PI
9A = (x 2 + 1 + 2)(1 + y 2 + 2) − 8(x + y) ≥ (x + y + 2)2 − 8(x + y) = (x + y − 2)2 ≥ 0.
M
p p
We have A = 0 only for b = −1/ 3, c = 1/ 3, when Aa2 + Ba + C = 64/27. Otherwise,
for A > 0, it suffices to prove that AC − B 2 ≥ 0. Let us denote
LY

E = b − c, F = bc + 1.
O

Since
.M

(b2 + 1)(c 2 + 1) = (b − c)2 + (bc + 1)2 = E 2 + F 2 ,


(b + c)2 = (b − c)2 + 4(bc + 1) − 4 = E 2 + 4F − 4,
W

(b2 − c 2 )2 = (b − c)2 (b + c)2 = E 2 (E 2 + 4F − 4),


W

we have
W

A = E 2 + F 2 + 2kE, B 2 = k2 E 2 (E 2 + 4F − 4), C = E 2 + F 2 + 2kE(F − 1),

and hence

AC − B 2 = (E 2 + F 2 + 2kE)(E 2 + F 2 + 2kE F − 2kE) − k2 E 2 (E 2 + 4F − 4)

1 p p
= (E 2 + F 2 )(E 2 + F 2 + 2kE F ) − k2 E 4 = (E + 3F )2 (11E 2 − 2 3E F + 9F 2 ) ≥ 0.
27
The equality holds for
p b+c
b−c+ 3(bc + 1) = 0, a+ =0
1 + 3bc
124 Vasile Cîrtoaje

(or any cyclic permutation).


p p p
Second Solution (by Vo Quoc Ba Can). Substituting a = x 3, b = y 3, c = z 3, the
inequality becomes

(3x 2 + 1)(3 y 2 + 1)(3z 2 + 1) ≥ 8|(x − y)( y − z)(z − x)|.

It suffices to show that

E 2 ≥ 64(x − y)2 ( y − z)2 (z − x)2 ,

where

L
.M
X X
E = (3x 2 + 1)(3 y 2 + 1)(3z 2 + 1) = 27x 2 y 2 z 2 + 9 x2 y2 + 3 x 2 + 1.

It it easy to check that the equality holds for x = −1, y = 0 and z = 1 (or any cyclic

D
permutation), when

A
x + y + z = 0, x y + yz + z x = −1, x yz = 0.
PI
From
M
€X Š2
xy +1 ≥ 0,
LY

we get X X X
1≥− x 2 y 2 − 2x y x x −2 x y,
O

and from
.M

X
(9x yz + x)2 ≥ 0,
we get
W

X X X
81x 2 y 2 z 2 ≥ −18x yz x− x2 − 2 x y.
W

Therefore,
W

€ X X X Š X X
3E ≥ −18x yz x− x2 − 2 x y + 27 x2 y2 + 9 x2
€ X X X Š
+3 − x 2 y 2 − 2x y x x −2 xy
€X X Š €X X Š
= 24 x 2 y 2 − x yz x +8 x2 − xy
X 4X
= 12 x 2 ( y − z)2 + (2x − y − z)2 .
3
By the AM-GM inequality, we have
r”X — ”X —
3E ≥ 8 x 2 ( y − z)2 (2x − y − z)2 .
Symmetric Polynomial Inequalities in Real Variables 125

In addition, by the Cauchy-Schwarz inequality, we get


64 ”X —2
E2 ≥ x( y − z)(2x − y − z)
9
€X X Š2
= 64 x2 y − x y2
= 64(x − y)2 ( y − z)2 (z − x)2 .

(b) Write the inequality as

1 2 3 1 2 3 1 2 3
 ‹   ‹   ‹ 
a− + b− + c− + ≥ (a − b)(b − c)(c − a).

L
2 4 2 4 2 4

.M
Using the substitution
p p p
1 3 1 3 1 3

D
a− = x, b− = y, c− = z,
2 2 2 2 2 2

A
the inequality turns out into the inequality in (a). From the equality conditions in (a),
namely
p
PI
y +z
y − z + 3( yz + 1) = 0, x + = 0,
M
1 + 3 yz
LY

we get the following equality conditions


c−1 1
b= , a=
O

c 1−c
(or any cyclic permutation).
.M
W

P 2.93. If a, b, c are real numbers such that a + b + c = 3, then


W

(1 − a + a2 )(1 − b + b2 )(1 − c + c 2 ) ≥ 1.
W

Solution (by Marian Tetiva). Assume first that a, b, c ≥ 0. Among the numbers a, b, c
always there exist two (let b and c) which are either less than or equal to 1, or greater
than or equal to 1. Then,
bc(b − 1)(c − 1) ≥ 0,
hence

(1 − b + b2 )(1 − c + c 2 ) = 1 + (b2 − b) + (c 2 − c) + (b2 − b)(c 2 − c)


1
≥ 1 − b − c + b2 + c 2 ≥ 1 − (b + c) − (b + c)2
2
1 1
= 1 − (3 − a) − (3 − a) = (5 − 4a + a2 ).
2
2 2
126 Vasile Cîrtoaje

Therefore, it suffices to show that

(1 − a + a2 )(5 − 4a + a2 ) ≥ 2.

Indeed,
(1 − a + a2 )(5 − 4a + a2 ) − 2 = (a − 1)2 (a2 − 3a + 3) ≥ 0.
Assume now that a ≤ b ≤ c and a<0. We have

b+c a+b+c 3
c≥ > = .
2 2 2

L
The desired inequality is true since

.M
1 − a + a2 > 1,

D
‹2
1 3 3

2
1− b+ b = −b + ≥ ,

A
2 4 4

1 − c + c2 > 1 − c +
3c
2
c 3 7
=1+ >1+ = .
2 4 4
PI
M
The proof is completed. The equality holds for a = b = c = 1.
LY
O

P 2.94. If a, b, c are real numbers such that a + b + c = 0, then


.M

a(a − 4) b(b − 4) c(c − 4)


+ 2 + 2 ≥ 0.
a2 + 2 b +2 c +2
W

Solution. Write the inequality as follows


W

X • a(a − 4) ˜
+ 1 ≥ 3,
W

a2 + 2
X (a − 1)2 3
≥ .
a2 +2 2
From
a2 = (b + c)2 ≤ 2(b2 + c 2 ),
we get
3a2 ≤ 2(a2 + b2 + c 2 ).
Similarly,
3b2 ≤ 2(a2 + b2 + c 2 ), 3c 2 ≤ 2(a2 + b2 + c 2 ).
Symmetric Polynomial Inequalities in Real Variables 127

Therefore, we have
X (a − 1)2 X 3(a − 1)2 3(a − 1)2
X
= ≥
a2 + 2 3a2 + 6 2(a2 + b2 + c 2 ) + 6
3 X 3
= (a − 1)2 = .
2(a + b + c + 3)
2 2 2 2
Thus, the proof is completed. The equality holds for a = b = c = 0, and also for a = −2
and b = c = 1 (or any cyclic permutation).

L
P 2.95. If a, b, c, d are real numbers, then

.M
‹2
1 + a bcd

2 2 2 2
(1 − a + a )(1 − b + b )(1 − c + c )(1 − d + d ) ≥ .

D
2

A
(Vasile Cîrtoaje, 1992)
PI
Solution. For a = b = c = d, the inequality can be written as
M
2(1 − a + a2 )2 ≥ 1 + a4 .
LY

It is true, since
2(1 − a + a2 )2 − 1 − a4 = (1 − a)4 ≥ 0.
O

Using this result, we get


.M

4(1 − a + a2 )2 (1 − b + b2 )2 ≥ (1 + a4 )(1 + b4 ) ≥ (1 + a2 b2 )2 .

Then, the desired inequality follows by multiplying the inequalities


W

2(1 − a + a2 )(1 − b + b2 ) ≥ 1 + a2 b2 ,
W

2(1 − c + c 2 )(1 − d + d 2 ) ≥ 1 + c 2 d 2 ,
W

(1 + a2 b2 )(1 + c 2 d 2 ) ≥ (1 + a bcd)2 .
The equality holds for a = b = c = d = 1.

P 2.96. Let a, b, c, d be real numbers such that a bcd > 0. Prove that
1 1 1 1 1 1 1 1
 ‹ ‹ ‹ ‹  ‹
a+ b+ c+ d+ ≥ (a + b + c + d) + + + .
a b c d a b c d
(Vasile Cîrtoaje, 2011)
128 Vasile Cîrtoaje

First Solution. Write the inequality as A ≥ B, where

A = (a2 + 1)(b2 + 1)(c 2 + 1)(d 2 + 1)


X X X
= (1 + a2 c 2 )(1 + b2 d 2 ) + a2 + a2 b2 + a2 b2 c 2 ,
X X X
B=( a)( a bc) = 4a bcd + a2 (bc + cd + bd).

Then,

1X 2 1X 2
A − B =(1 − a bcd)2 + (ac − bd)2 + a (1 − bc)2 + a (1 − cd)2
2 2

L
X X
+ a2 b2 − a2 bd,

.M
and hence
X X 1X 2
a2 b2 − a2 bd = a (b − d)2 ≥ 0.

D
A− B ≥
2

A
The equality holds for a = b = c = d = 1.

Second Solution. Since


PI
M
(a + b)(b + c)(c + d)(d + a) − (a + b + c + d)(bcd + cd a + d a b + a bc) = (ac − bd)2 ≥ 0,
LY

it suffices to show that


O

(a2 + 1)(b2 + 1)(c 2 + 1)(d 2 + 1) ≥ (a + b)(b + c)(c + d)(d + a).


.M

By the Cauchy-Schwarz inequality, we have


W

(a2 + 1)(1 + b2 ) ≥ (a + b)2 ,


W

(b2 + 1)(1 + c 2 ) ≥ (b + c)2 ,


W

(c 2 + 1)(1 + d 2 ) ≥ (c + d)2 ,

(d 2 + 1)(1 + a2 ) ≥ (d + a)2 .

Multiplying these inequalities, we get

(a2 + 1)(b2 + 1)(c 2 + 1)(d 2 + 1) ≥ |(a + b)(b + c)(c + d)(d + a)|


≥ (a + b)(b + c)(c + d)(d + a).
Symmetric Polynomial Inequalities in Real Variables 129

P 2.97. Let a, b, c, d be real numbers such that

a + b + c + d = 4, a2 + b2 + c 2 + d 2 = 7.

Prove that
a3 + b3 + c 3 + d 3 ≤ 16.
(Vasile Cîrtoaje, 2010)

First Solution. Assume that a ≤ b ≤ c ≤ d, and denote s = a + b and p = a b, s ≤ 2,


4p ≤ s2 . Since
2(a3 + b3 ) = 2(s3 − 3ps),

L
.M
c + d = 4 − s, c 2 + d 2 = 7 − (a2 + b2 ) = 7 − s2 + 2p,

2(c 3 + d 3 ) = (c + d)[3(c 2 + d 2 ) − (c + d)2 ] = (4 − s)(−4s2 + 8s + 5 + 6p),

D
we have

A
2(a3 + b3 + c 3 + d 3 − 16) = 12p(2 − s) + 6s3 − 24s2 + 27s − 12
PI
≤ 3s2 (2 − s) + 6s3 − 24s2 + 27s − 12
M
= 3(s − 1)2 (s − 4) ≤ 0.
LY

This completes the proof. The equality holds for a = b = c = 1/2 and d = 5/2 (or any
cyclic permutation).
O

Second Solution (by Vo Quoc Ba Can). From


.M

1 1
7 = a2 + b2 + c 2 + d 2 ≥ a2 + (b + c + d)2 = a2 + (4 − a)2 ,
3 3
W

−1 5 −1 5
it follows that a ∈ [ , ]. Similarly, we have b, c, d ∈ [ , ]. On the other hand,
W

2 2 2 2
W

5X 2 X 3 5 2
a3 + b3 + c 3 + d 3 = a + (a − a )
2 2
35 1 X
2
= − a (5 − 2a)
2 2
and, by virtue of the Cauchy-Schwarz inequality,

[ a(5 − 2a)]2 (5 a − 2 a2 )2
P P P
X
2
a (5 − 2a) ≥ P = = 3.
(5 − 2a)
P
20 − 2 a

Therefore,
35 3
a3 + b3 + c 3 + d 3 ≤ − = 16.
2 2
130 Vasile Cîrtoaje

Remark. In the same manner as in the second solution, we can prove the following
generalization.
• If a1 , a2 , . . . , an are real numbers such that

a1 + a2 + · · · + an = 2n, a12 + a22 + · · · + an2 = n(n + 3),

then
a13 + a23 + · · · + an3 ≤ n(n2 + 3n + 4),
with equality for a1 = ... = an−1 = 1 and an = n + 1 (or any cyclic permutation).

L
.M
P 2.98. Let a, b, c, d be real numbers such that a + b + c + d = 0. Prove that

12(a4 + b4 + c 4 + d 4 ) ≤ 7(a2 + b2 + c 2 + d 2 )2 .

D
A
(Vasile Cîrtoaje, 2010)

Solution. Assume that a2 = max{b2 , c 2 , d 2 } and denote


PI
M
v
t b2 + c 2 + d 2
x= , x 2 ≤ a2 .
LY

3
From
O

‹2
b2 + c 2 + d 2 b+c+d a2

x2 = ≥ = ,
3 3 9
.M

we get 9x 2 ≥ a2 . By the Cauchy-Schwarz inequality, we have


W

b4 + c 4 + d 4 = (b2 + c 2 + d 2 )2 − 2(b2 c 2 + c 2 d 2 + d 2 b2 )
= 9x 4 − 2(b2 c 2 + c 2 d 2 + d 2 b2 )
W

2
≤ 9x 4 − (bc + cd + d b)2
W

3
1
= 9x − [(b + c + d)2 − b2 − c 2 − d 2 ]2
4
6
1 45x 4 + 6a2 x 2 − a4
= 9x 4 − (a2 − 3x 2 )2 = .
6 6
and hence
45x 4 + 6a2 x 2 + 5a4
a4 + b4 + c 4 + d 4 ≤ .
6
Therefore, it suffices to prove that

2(45x 4 + 6a2 x 2 + 5a4 ) ≤ 7(a2 + 3x 2 )2 ,


Symmetric Polynomial Inequalities in Real Variables 131

which is equivalent to the obvious inequality

(x 2 − a2 )(9x 2 − a2 ) ≤ 0.

The equality holds for −a/3 = b = c = d (or any cyclic permutation).

Remark. Similarly, we can prove the following generalization.


• If a1 , a2 , . . . , an are real numbers such that

a1 + a2 + · · · + an = 0,

L
then
(a12 + a22 + · · · + an2 )2 n(n − 1)

.M
≥ ,
a14 + a24 + · · · + an4 n2 − 3n + 3

D
with equality for −a1 /(n − 1) = a2 = · · · = an (or any cyclic permutation).

A
PI
M
P 2.99. Let a, b, c, d be real numbers such that a + b + c + d = 0. Prove that
LY

(a2 + b2 + c 2 + d 2 )3 ≥ 3(a3 + b3 + c 3 + d 3 )2 .

(Vasile Cîrtoaje, 2011)


O

Solution. Applying the AM-GM inequality and the identity


.M

(a + b + c)3 = a3 + b3 + c 3 + 3(a + b)(b + c)(c + a),


W

we have
W

(a2 + b2 + c 2 + d 2 )3 = [a2 + b2 + c 2 + (a + b + c)2 ]3


= [(a + b)2 + (b + c)2 + (c + a)2 ]3
W

≥ 27(a + b)2 (b + c)2 (c + a)2


= 3[(a + b + c)3 − a3 − b3 − c 3 ]2
= 3(a3 + b3 + c 3 + d 3 )2 .

The equality holds for a = b = c = −d/3 (or any cyclic permutation).

Remark. The following generalization holds (Vasile Cirtoaje, 2011).


• If a1 , a2 , . . . , an are real numbers such that

a1 + a2 + · · · + an = 0,
132 Vasile Cîrtoaje

then
n(n − 1) 3
(a12 + a22 + · · · + an2 )3 ≥ (a + a23 + · · · + an3 )2 .
(n − 2)2 1
Moreover,
• If k ≥ 3 is an odd number, and a1 , a2 , . . . , an are real numbers such that

a1 + a2 + · · · + an = 0,

then
nk (n − 1)k−2
(a12 + a22 + · · · + an2 )k ≥ (a k + a2k + · · · + ank )2 ,

L
[(n − 1)k−1 − 1]2 1

.M
with equality for a1 = ... = an−1 = −an /(n − 1) (or any cyclic permutation).

D
A
PI
P 2.100. If a, b, c, d are real numbers such that a bcd = 1. Prove that
M
(1 + a2 )(1 + b2 )(1 + c 2 )(1 + d 2 ) ≥ (a + b + c + d)2 .
LY

(Pham Kim Hung, 2006)


O

Solution. Substituting a, b, c, d by |a|, |b|, |c|, |d|, respectively, the left side of the in-
equality remains unchanged, while the right side either remains unchanged or increases.
.M

Therefore, it suffices to prove the inequality only for a, b, c, d ≥ 0. Among a, b, c, d there


are two numbers less than or equal to 1, or greater than or equal to 1. Let b and d be
these numbers; that is,
W

(1 − b)(1 − d) ≥ 0.
W

By the Cauchy-Schwarz inequality, we have


W

(1 + a2 )(1 + b2 )(1 + c 2 )(1 + d 2 ) = (1 + a2 + b2 + a2 b2 )(c 2 + 1 + d 2 + c 2 d 2 )


≥ (c + a + bd + a bcd)2 = (c + a + bd + 1)2 .

So, it suffices to show that

c + a + bd + 1 ≥ a + b + c + d,

which is equivalent to (1 − b)(1 − d) ≥ 0. The equality holds for a = b = c = d = 1.


Symmetric Polynomial Inequalities in Real Variables 133

P 2.101. Let a, b, c, d be real numbers such that

a2 + b2 + c 2 + d 2 = 4.

Prove that
(a bc)3 + (bcd)3 + (cd a)3 + (d a b)3 ≤ 4.
(Vasile Cîrtoaje, 2004)

Solution. Substituting a, b, c, d by |a|, |b|, |c|, |d|, respectively, the hypothesis and the
right side of the inequality remains unchanged, while the left side either remains un-
changed or decreases. Therefore, it suffices to prove the inequality only for a, b, c, d ≥ 0.

L
Setting x = a2 , y = b2 , z = c 2 and t = d 2 , we need to prove that

.M
(x yz)3/2 + ( yz t)3/2 + (z t x)3/2 + (t x y)3/2 ≤ 4

D
for x + y + z + t = 4. By the AM-GM inequality, we have

A
p
PI
4 4 x yz ≤ 1 + x + y + z = 5 − t,
M
p x yz(5 − t)2
(x yz)3/2 = x yz x yz ≤ .
16
LY

Analogously,
O

yz t(5 − x)2 z t x(5 − y)2 t x y(5 − z)2


( yz t)3/2 ≤ , (z t x)3/2 ≤ , (t x y)3/2 ≤ .
16 16 16
.M

Therefore, it suffices to show that


W

x yz(5 − t)2 + yz t(5 − x)2 + z t x(5 − y)2 + t x y(5 − z)2 ≤ 64,


W

which is equivalent to E(x, y, z, t) ≥ 0, where


W

E(x, y, z, t) = 36x yz t − 25(x yz + yz t + z t x + t x y) + 64.

To prove this inequality, we use the mixing variable method.Without loss of generality,
x + y +z 4
assume that x ≥ y ≥ z ≥ t. Setting u = , we have 3u + t = 4, t ≤ u ≤ and
3 3
u3 ≥ x yz. We will show that

E(x, y, z, t) ≥ E(u, u, u, t) ≥ 0.

The left inequality is equivalent to

25[(u3 − x yz) + t(3u2 − x y − yz − z x)] ≥ 36t(u3 − x yz).


134 Vasile Cîrtoaje

By Schur’s inequality

(x + y + z)3 + 9x yz ≥ 4(x + y + z)(x y + yz + z x),

we get
9u3 + 3x yz ≥ 4u(x y + yz + z x),

and hence
3(u3 − x yz)
3u2 − x y − yz − z x ≥ ≥ 0.
4u
Therefore, it suffices to prove that

L
.M
3t
25(1 + ) ≥ 36t.
4u

D
Write this inequality in the homogeneous form

A
25(3u + t)(4u + 3t) ≥ 576ut,
PI
or, equivalently,
M
75(4u2 + t 2 ) ≥ 251ut.
LY

This inequality is true, since


O

75(4u2 + t 2 ) − 251ut ≥ 75(4u2 + t 2 − 4ut) ≥ 300u(u − t) ≥ 0.


.M

The right inequality E(u, u, u, t) ≥ 0 holds, since

E(u, u, u, t) = (36u3 − 75u2 )t − 25u3 + 64


W

= 4(16 − 75u2 + 86u3 − 27u4 )


W

= 4(1 − u)2 (16 + 32u − 27u2 )


= 4(1 − u)2 [4(4 − u) + 9u(4 − 3u)] ≥ 0.
W

This completes the proof. The equality holds for a = b = c = d = 1.

P 2.102. Let a, b, c, d be real numbers such that a2 + b2 + c 2 + d 2 = 1. Prove that

(1 − a)4 + (1 − b)4 + (1 − c)4 + (1 − d)4 ≥ a4 + b4 + c 4 + d 4 .

(Vasile Cîrtoaje, 2007)


Symmetric Polynomial Inequalities in Real Variables 135

Solution. The desired inequality follows by summing the inequalities

(1 − a)4 + (1 − b)4 ≥ c 4 + d 4 ,

(1 − c)4 + (1 − d)4 ≥ a4 + b4 .
Since
(1 − a)4 + (1 − b)4 ≥ 2(1 − a)2 (1 − b)2
and
1 2
c4 + d 4 ≥ (c + d 2 )2 ,
2
the former inequality holds if

L
.M
2(1 − a)(1 − b) ≥ c 2 + d 2 .

Indeed,

D
2(1 − a)(1 − b) − c 2 − d 2 = 2(1 − a)(1 − b) + a2 + b2 − 1 = (a + b − 1)2 ≥ 0.

A
The equality holds for a = b = c = d =
1
2
. PI
M
LY

−1
P 2.103. If a, b, c, d ≥ such that a + b + c + d = 4, then
2
O

1−a 1− b 1−c 1−d


+ + + ≥ 0.
.M

1−a+a 2 1− b+ b 2 1−c+c 2 1 − d + d2
(Vasile Cîrtoaje, 2014)
W

Solution (by Nguyen Van Quy). Assume that a ≤ b ≤ c ≤ d and consider two cases:
a > 0 and a ≤ 0.
W

Case 1: a > 0. Write the inequality as


W

a2 b2 c2 d2
+ + + ≤ 4.
1 − a + a2 1 − b + b2 1 − c + c 2 1 − d + d 2
We have
a2 b2 c2 d2 a2 b2 c 2 d 2
+ + + ≤ + + + = 4.
1 − a + a2 1 − b + b2 1 − c + c 2 1 − d + d 2 a b c d
Case 2: −1/2 ≤ a ≤ 0. We can check that the equality holds for a = −1/2 and b = c =
d = 3/2 (or any cyclic permutation). Define the function
1− x −1
f (x) = + k1 x + k2 , x≥ ,
1 − x + x2 2
136 Vasile Cîrtoaje

such that
f (3/2) = f 0 (3/2) = 0.
We get
12 −4
k1 = , k2 = ,
49 49
when
1− x 12x − 4 (2x − 3)2 (3x + 5)
f (x) = + = .
1 − x + x2 49 49(1 − x + x 2 )
Since f (x) ≥ 0 for x ≥ −1/2, we have

L
1− x 4 − 12x
≥ .

.M
1 − x + x2 49

Therefore,

D
1− b 1−c 1−d 12 − 12(b + c + d) 12(a − 3)

A
+ + ≥ = .
1− b+ b 2 1−c+c 2 1−d +d 2 49 49

Thus, it suffices to show that


PI
M
1−a 12(a − 3)
+ ≥ 0.
LY

1−a+a 2 49
Indeed,
O

1−a 12(a − 3) (2a + 1)(6a2 − 27a + 13)


+ = ≥ 0.
1 − a + a2 49(1 − a + a2 )
.M

49
The proof is completed. The equality holds for a = b = c = d = 1, and also for a = −1/2
and b = c = d = 3/2 (or any cyclic permutation).
W
W
W

P 2.104. If a, b, c, d, e ≥ −3 such that a + b + c + d + e = 5, then

1−a 1− b 1−c 1−d 1−e


+ + + + ≥ 0.
1+a+a 2 1+ b+ b 2 1+c+c 2 1+d +d 2 1 + e + e2
(Vasile Cîrtoaje, 2014)

Solution. Assume that a ≤ b ≤ c ≤ d ≤ e and consider two cases: a ≥ 0 and a ≤ 0.


Case 1: a ≥ 0. For any x ≥ 0, we have

1− x 1− x (x − 1)2 (x + 2)
− = ≥ 0.
1 + x + x2 3 3(1 + x + x 2 )
Symmetric Polynomial Inequalities in Real Variables 137

Therefore, it suffices to show that

1−a 1− b 1−c 1−d 1−e


+ + + + ≥ 0,
3 3 3 3 3
which is an identity.
Case 2: −3 ≤ a ≤ 0. We can check that the equality holds for a = 3 and b = c = d =
e = 2. Based on this, define the function

1− x
f (x) = + k1 x + k2 , x ≥ −3,
1 + x + x2

L
such that

.M
f (2) = f 0 (2) = 0.
We get

D
2 3
k1 = , k2 = ,

A
49 49
when PI
1− x 2x + 3 (x − 2)2 (2x + 13)
f (x) = + = .
1 + x + x2 49(1 + x + x 2 )
M
49
Since f (x) ≥ 0 for x ≥ −3, we have
LY

1− x −2x − 3
≥ .
O

1 + x + x2 49
.M

Thus, it suffices to show that

1−a 2b + 3 2c + 3 2d + 3 2e + 3
− − − − ≥ 0,
1+a+a 2
W

49 49 49 49
which is equivalent to
W

1−a 2(b + c + d + e) + 12
W

− ≥ 0,
1+a+a 2 49

1−a 2(5 − a) + 12
− ≥ 0,
1 + a + a2 49
(a + 3)(2a2 − 26a + 9)
≥ 0.
49(1 + a + a2 )
Clearly, the last inequality is true for −3 ≤ a ≤ 0. The equality holds for a = b = c =
d = e = 1, and also for a = −3 and b = c = d = e = 2 (or any cyclic permutation).
138 Vasile Cîrtoaje

P 2.105. Let a, b, c, d, e be real numbers such that a + b + c + d + e = 0. Prove that

30(a4 + b4 + c 4 + d 4 + e4 ) ≥ 7(a2 + b2 + c 2 + d 2 + e2 )2 .

(Vasile Cîrtoaje, 2010)

Solution. Write the inequality as E(a, b, c, d, e) ≥ 0, where

E(a, b, c, d, e) = 30(a4 + b4 + c 4 + d 4 + e4 ) − 7(a2 + b2 + c 2 + d 2 + e2 )2 .

Among the numbers a, b, c, d, e there exist three with the same sign. Let a, b, c be these
numbers. We will show that

L
.M
E(a, b, c, d, e) ≥ E(a, b, c, x, x) ≥ 0,

where

D
d+e −(a + b + c)
x= = .

A
2 2
PI
The inequality E(a, b, c, d, e) ≥ E(a, b, c, x, x) is equivalent to
M
30(d 4 + e4 − 2x 4 ) ≥ 7(d 2 + e2 − 2x 2 )(2a2 + 2b2 + 2c 2 + d 2 + e2 + 2x 2 ).
LY

Since
(d − e)2 (7d 2 + 10d e + 7e2 )
d 4 + e4 − 2x 4 =
O

8
and
.M

(d − e)2
d 2 + e2 − 2x 2 = ,
2
W

we need to show that

15(7d 2 + 10d e + 7e2 ) ≥ 14(2a2 + 2b2 + 2c 2 + d 2 + e2 + 2x 2 ),


W
W

which reduces to
21(d 2 + e2 ) + 34d e ≥ 7(a2 + b2 + c 2 ).
Since
a2 + b2 + c 2 ≤ (a + b + c)2 = (d + e)2 ,
it suffices to prove that

21(d 2 + e2 ) + 34d e ≥ 7(d + e)2 ,

which is equivalent to the obvious inequality

4(d 2 + e2 ) + 10(d + e)2 ≥ 0.


Symmetric Polynomial Inequalities in Real Variables 139

−(a + b + c)
The inequality E(a, b, c, x, x) ≥ 0, where x = , can be written as
2
X X X X X
23 a4 + 2( a)4 ≥ 7( a2 )( a)2 + 14 a2 b2 .

Since ( a)2 ≥ 3 a b, it suffices to prove that


P P

X X X X X X
23 a4 + 6( a b)( a)2 ≥ 7( a2 )( a)2 + 14 a2 b2 .

This is equivalent to
X X 1X X
a4 + a bc a b(a2 + b2 ) + a2 b2 ,

L
a≥
2

.M
which follows by summing Schur’s inequality of degree four

D
X X X
a4 + a bc a≥ a b(a2 + b2 )

A
and the obvious inequality

1X X
PI
a b(a2 + b2 ) ≥ a2 b2 .
M
2
LY

This completes the proof. The equality holds for a = b = c = 2 and d = e = −3 (or any
permutation thereof).
O

Remark. Notice that the following generalization holds (Vasile Cîrtoaje, 2010).
.M

• If n is an odd positive integer and a1 , a2 , . . . , an are real numbers such that

a1 + a2 + · · · + an = 0,
W

then
(a12 + a22 + · · · + an2 )2
W

n(n2 − 1)
≤ ,
a14 + a24 + · · · + an4 n2 + 3
W

with equality when (n+1)/2 of a1 , a2 , . . . , an are equal to (n−1)/2 and the other numbers
are equal to −(n + 1)/2.

P 2.106. Let a1 , a2 , . . . , an ≥ −1 such that a1 + a2 + · · · + an = 0. Prove that

(n − 2)(a12 + a22 + · · · + an2 ) ≥ a13 + a23 + · · · + an3 .

(Vasile Cîrtoaje, 2005)


140 Vasile Cîrtoaje

Solution. Without loss of generality, assume that

a1 ≥ a2 ≥ · · · ≥ an .

Write the inequality as


n
X
ai f (ai ) ≥ 0,
i=1

where
f (x) = (n − 2)x − x 2 .
Since

L
f (ai ) − f (ai+1 ) = (ai − ai+1 )[n − 2 − (ai + ai+1 )]

.M
≥ (ai − ai+1 )[n − 2 − (a1 + a2 )] = (ai − ai+1 )(n − 2 + a3 + · · · + an )]

D
= (ai − ai+1 )[(1 + a3 ) + · · · + (1 + an )] ≥ 0,

A
we have a1 ≥ a2 ≥ · · · ≥ an and f (a1 ) ≥ f (a2 ) ≥ · · · ≥ f (an ). Therefore, by Chebyshev’s
inequality, we get
n
PI
M
X
n ai f (ai ) ≥ (a1 + a2 + · · · + an )[ f (a1 ) + f (a2 ) + · · · + f (an )] = 0.
LY

i=1

The equality holds for a1 = a2 = · · · = an = 0, and for a1 = n−1 and a2 = · · · = an = −1


O

(or any cyclic permutation).


.M
W

P 2.107. Let a1 , a2 , . . . , an ≥ −1 such that a1 + a2 + · · · + an = 0. Prove that


W

(n − 2)(a12 + a22 + · · · + an2 ) + (n − 1(a13 + a23 + · · · + an3 ) ≥ 0.


W

(Vasile Cîrtoaje, 2005)

Solution. For the nontrivial case a12 + a22 + · · · + an2 6= 0, write the inequality as

S3
(n − 1) + n − 2 ≥ 0,
S2

where
S2 = a12 + a22 + · · · + an2 , S3 = a13 + a23 + · · · + an3 .
Without loss of generality, assume that

a1 ≤ a2 ≤ · · · ≤ an .
Symmetric Polynomial Inequalities in Real Variables 141

For any p > 0 such that a1 + p ≥ 0, by the Cauchy-Schwarz inequality, we have


n
Pn
X
2
[ i=1 (ai + 1)(ai + p)]2
(ai + 1) (ai + p) ≥ Pn ,
i=1 i=1 (ai + p)
n n
Pn
X
3
X
2
( i=1 ai2 + np)2
ai + (p + 2) ai + np ≥ ,
i=1 i=1
np
S3 S
≥ 2 − p.
S2 np
Thus, it suffices to show that

L
n(n − 2)
S2 + p ≥ np2 .

.M
n−1
n
Case 1: S2 ≥ . Choosing p = 1, we have a1 + p ≥ 0, and

D
n−1

A
n(n − 2) n
S2 + p − np2 = S2 − ≥ 0.

n
n−1 n−1 PI
Case 2: 0 < S2 ≤ . We set
M
n−1 v
tn − 1
LY

p= S2 .
n
Since
O

n−1 n−1 2 1
p2 − a12 = S2 − a12 = (a2 + · · · + an2 ) − a12
n n n
.M

2
(a2 + · · · + an )2 a1
≥ − = 0,
n n
W

we have a1 + p ≥ 0. In addition,
W

s
n(n − 2) 2
p  n p ‹
S2 + p − np = (n − 2) S2 − S2 ≥ 0.
n−1 n−1
W

The equality holds for a1 = a2 = · · · = an = 0, and for a1 = −1 and a2 = · · · = an =


1/(n − 1) (or any cyclic permutation).

p
P 2.108. Let a1 , a2 , . . . , an ≥ n − 1 − n2 − n + 1 be nonzero real numbers such that
a1 + a2 + · · · + an = n. Prove that
1 1 1
+ + ··· + ≥ n.
a12 a22 an2

(Vasile Cîrtoaje, 2010)


142 Vasile Cîrtoaje

Solution. Without loss of generality, assume that a1 ≤ a2 ≤ · · · ≤ an . For a1 > 0, by the


Cauchy-Schwarz inequality, we have

1 1 1 1 1 1 1 2
 ‹
+ + ··· + 2 ≥ + + ··· +
a12 a22 an n a1 a2 an
2
n2
 
1
≥ = n.
n a1 + a2 + · · · an

Further, consider that a1 < 0, when there exists k, 1 ≤ k ≤ n − 1, such that

a1 ≤ · · · ≤ ak < 0 < ak+1 ≤ · · · ≤ an .

L
a1 + · · · + ak ak+1 + · · · + an

.M
Let us denote x = and y = . We have
k n−k
p

D
−1 < n − 1 − n2 − n + 1 ≤ x < 0, y > 1, k x + (n − k) y = n.

A
From k ≥ 1 and k( y − x) = n( y − 1) > 0, we get y − x ≤ n( y − 1), and hence

y≥
n− x
.
PI
n−1
M
p
In addition, from n − 1 − n2 − n + 1 ≤ x, we get
LY

n + 2(n − 1)x − x 2 ≥ 0.
O

By the Cauchy-Schwarz inequality, we have


.M

2
1 2 1 k2

1 1 1 1
 ‹
k
2
+ ··· + 2 ≥ + ··· + ≥ = 2
a1 ak k −a1 −ak k −a1 − · · · − ak x
W

and
W

‹2 2
(n − k)2

1 1 1 1 1 1 n−k

+ ··· + 2 ≥ + ··· + ≥ = .
W

2
ak+1 an n−k ak+1 an n−k ak+1 + · · · + an y2

Then, it suffices to prove that


k n−k
2
+ ≥ n,
x y2
which is equivalent to

k( y − x)( y + x) ≥ nx 2 ( y − 1)( y + 1).

Since k( y − x) = n( y − 1) > 0, we need to show that y + x ≥ x 2 ( y + 1), which is


equivalent to
(1 − x)(x + y + x y) ≥ 0.
Symmetric Polynomial Inequalities in Real Variables 143

This is true, since 1 − x > 0 and

(1 + x)(n − x) n + 2(n − 1)x − x 2


x+ y+xy ≥ x+ = ≥ 0.
n−1 n−1
The equality holds when a1 = a2 = · · · = an p
= 1, as well as when one of a1 , a2 , . . . , an is
p 1 + n2 − n + 1
n − 1 − n2 − n + 1 and the others are .
n−1

n
P 2.109. Let a1 , a2 , . . . , an ≤ be real numbers such that

L
n−2

.M
a1 + a2 + · · · + an = n.

If k is a positive integer, k ≥ 2, then

D
a1k + a2k + · · · + ank ≥ n.

A
PI (Vasile Cîrtoaje, 2012)

Solution. First we show that at most one of ai is negative. Assume, for the sake of
M
contradiction, that an−1 < 0 and an < 0. Then,
LY

n
an−1 + an = n − (a1 + · · · + an−2 ) ≥ n − (n − 2) · = 0,
n−2
O

which is a contradiction. There are two cases to consider.


.M

Case 1: a1 , a2 , . . . , an ≥ 0. The desired inequality is just Jensen’s inequality applied to


the convex function f (x) = x k .
Case 2: a1 , a2 , · · · , an−1 ≥ 0 and an < 0. Let us denote
W

a1 + a2 + · · · + an−1
x= , y = −an ,
W

n−1
where
W

(n − 1)x − y = n, x ≥ y > 0.
The condition x ≥ y follows from

x − y = n − (n − 2)x ≥ 0.

By Jensen’s inequality, we have

a1k + a2k + · · · + an−1


k
≥ (n − 1)x k .

In addition, ank ≥ − y k . Thus, it suffices to show that

(n − 1)x k − y k ≥ n.
144 Vasile Cîrtoaje

We will use the inequality


x k − y k > (x − y)k ,
which is equivalent to
 y k  y k
1− > 1− .
x x
Indeed,
 y k y k y  y
1− − 1− >1− − 1− = 0.
x x x x
Therefore, it suffices to show that

(n − 2)x k + (x − y)k ≥ n.

L
.M
By Jensen’s inequality, we have

D
˜k
(n − 2)x + (x − y)  n k−1
•
k k
(n − 2)x + (x − y) ≥ [(n − 2) + 1] =n > n.
(n − 2) + 1

A
n−1
PI
This completes the proof. The equality holds for a1 = a2 = · · · an = 1.
M
LY

P 2.110. If a1 , a2 , . . . , an (n ≥ 3) are real numbers such that


O

−(3n − 2)
a1 , a2 , . . . , an ≥ , a1 + a2 + · · · + an = n,
n−2
.M

then
1 − a1 1 − a2 1 − an
+ + ··· + ≥ 0.
W

(1 + a1 )2 (1 + a2 )2 (1 + an )2
(Vasile Cîrtoaje, 2014)
W

Solution. Since the inequality holds for n = 3 (see P 2.25), consider further that n ≥ 4.
W

Assume that a1 ≤ a2 ≤ · · · ≤ an and consider two cases: a1 ≥ 0 and a1 ≤ 0.


Case 1: a1 ≥ 0. For any x ≥ 0, we have

1− x x − 1 (x − 1)2 (x + 3)
+ = ≥ 0.
(1 + x)2 4 4(1 + x)2

Therefore, it suffices to show that


1 − a1 1 − a2 1 − an
+ + ··· + ≥ 0,
4 4 4
which is an identity.
Symmetric Polynomial Inequalities in Real Variables 145

Case 2: −(3n − 2)/(n − 2) ≤ a1 ≤ 0. We can check that the equality holds for
−(3n − 2) n+2
a1 = , a2 = a3 = · · · = an = .
n−2 n−2
Based on this, define the function
1− x −(3n − 2)
f (x) = + k1 x + k2 , x≥ ,
(1 + x)2 n−2
such that
n+2 0 n+2
 ‹  ‹
f =f = 0.
n−2 n−2
We get

L
(n − 4)(n − 2)2 (n − 2)(−n2 + 6n + 8)
k1 = =

.M
, k 2 ,
4n3 4n3
[(n − 2)x − n − 2]2 [(n − 4)x + 3n − 4]
f (x) = .

D
4n3 (1 + x)2

A
Since f (x) ≥ 0 for n ≥ 4 and x ≥ −(3n − 2)/(n − 2), we have
1− x
(1 + x)2
PI
≥ −k1 x − k2 .
M
Thus, it suffices to show that
LY

1 − a1
− k1 (a2 + a3 + · · · + an ) − (n − 1)k2 ≥ 0,
(1 + a1 )2
O

which is equivalent to
1 − a1
.M

− k1 (n − a1 ) − (n − 1)k2 ≥ 0,
(1 + a1 )2
[(n − 2)a1 + 3n − 2][(n − 4)(n − 2)a12 − 2(n2 + 4n − 8)a1 + n2 − 2n + 8] ≥ 0.
W

Clearly, the last inequality is true for n ≥ 4 and −(3n − 2)/(n − 2) ≤ a1 ≤ 0. This
W

completes the proof. The equality holds for a1 = a2 = · · · an = 1, and also for x 1 =
−(3n − 2) n+2
and x 2 = · · · = x n = (or any cyclic permutation).
W

n−2 n−2

P 2.111. Let a1 , a2 , . . . , an be real numbers. Prove that

(a1 + a2 + · · · + an )2 (n − 1)n−1
(a) ≤ ;
(a12 + 1)(a22 + 1) · · · (an2 + 1) nn−2
1
a1 + a2 + · · · + an (2n − 1)n− 2
(b) ≤ .
(a12 + 1)(a22 + 1) · · · (an2 + 1) 2n nn−1

(Vasile Cîrtoaje, 1994)


146 Vasile Cîrtoaje

xi
Solution. Let m be a positive integer (m ≥ n), and let ai = p for all i. Assume
m−1
that
x 12 ≤ · · · ≤ x k2 ≤ 1 ≤ x k+1
2
≤ · · · ≤ x n2 ,
where 0 ≤ k ≤ n. By Bernoulli’s inequality and the Cauchy-Schwarz inequality, we have
‹ n ‹ n  2
m−1 nY 2 m−1 nY

xi
 
(ai + 1) = +1
m i=1
m i=1
m−1

n  k   n 
x i2 − 1 x i2 − 1 Y x i2 − 1
Y  Y 
= 1+ = 1+ 1+
m m m

L
i=1 i=1 i=k+1

.M
k n
x i2 − 1 x i2 − 1
‚ Œ‚ Œ
X X
≥ 1+ 1+
i=1
m i=k+1
m

D
1
= [x 2 + · · · + x k2 + (m − n) + (n − k)][k + (m − n) + x k+1
2
+ · · · + x n2 ]

A
m2 1
1 PI
≥ 2 (x 1 + · · · + x k + m − n + x k+1 + · · · + x n )2
m
M
1
= 2 (m − n + x 1 + x 2 + · · · + x n )2 .
LY

m
Therefore,
O

‹n Yn
m−1 1

(ai2 + 1) ≥ 2 (m − n + x 1 + x 2 + · · · + x n )2 ,
.M

m i=1
m

and hence
W

n ‹2
mn−2 m−n
Y 
(ai2 + 1) ≥ p + a1 + a2 + · · · + an .
W

i=1
(m − 1)n−1 m−1
W

The equality occurs in this inequality for


1
a1 = a2 = · · · = an = p .
m−1

(a) Choosing m = n, we get the desired inequality. The equality holds for
1
a1 = a2 = · · · = an = p
n−1
or
−1
a1 = a2 = · · · = an = p .
n−1
Symmetric Polynomial Inequalities in Real Variables 147

(b) Since
‹2
m−n 4(m − n)

p + a1 + a2 + · · · + an ≥ p (a1 + a2 + · · · + an ),
m−1 m−1
we get
n
Y 4mn−2 (m − n)
(ai2 + 1) ≥ 1
(a1 + a2 + · · · + an ).
i=1 (m − 1)n− 2
Choosing m = 2n, we get the desired inequality. The equality holds for

1
a1 = a2 = · · · = an = p .

L
2n − 1

D
.M
A
PI
M
LY
O
.M
W
W
W
148 Vasile Cîrtoaje

L
D
.M
A
PI
M
LY
O
.M
W
W
W
Chapter 3

Symmetric Polynomial Inequalities

L
in Nonnegative Variables

D
.M
A
3.1 Applications
PI
M
3.1. If a, b, c are positive real numbers, then

a2 + b2 + c 2 + 2a bc + 1 ≥ 2(a b + bc + ca).
LY
O

p
3.2. Let a, b, c be nonnegative real numbers. If 0 ≤ k ≤ 2, then
.M

a2 + b2 + c 2 + ka bc + 2k + 3 ≥ (k + 2)(a + b + c).
W

3.3. If a, b, c are positive real numbers, then


W

a bc(a + b + c) + 2(a2 + b2 + c 2 ) + 3 ≥ 4(a b + bc + ca).


W

3.4. If a, b, c are positive real numbers, then

a(b2 + c 2 ) + b(c 2 + a2 ) + c(a2 + b2 ) + 3 ≥ 3(a b + bc + ca).

3.5. If a, b, c are positive real numbers, then


3
a2 + b2 + c 2

≥ a2 b2 c 2 + (a − b)2 (b − c)2 (c − a)2 .
3

149
150 Vasile Cîrtoaje

3.6. If a, b, c are positive real numbers, then

(a + b + c − 3)(a b + bc + ca − 3) ≥ 3(a bc − 1)(a + b + c − a b − bc − ca).

3.7. If a, b, c are positive real numbers, then

(a) a3 + b3 + c 3 + a b + bc + ca + 9 ≥ 5(a + b + c);

(b) a3 + b3 + c 3 + 4(a b + bc + ca) + 18 ≥ 11(a + b + c).

L
.M
3.8. If a, b, c are positive real numbers, then

(a) a3 + b3 + c 3 + a bc + 8 ≥ 4(a + b + c);

D
4(a3 + b3 + c 3 ) + 15a bc + 54 ≥ 27(a + b + c).

A
(b)
PI
M
3.9. Let a, b, c be nonnegative real numbers such that

a + b + c = a2 + b2 + c 2 .
LY

Prove that
O

a b + bc + ca ≥ a2 b2 + b2 c 2 + c 2 a2 .
.M

3.10. If a, b, c are nonnegative real numbers, then


W

(a2 + 2bc)(b2 + 2ca)(c 2 + 2a b) ≥ (a b + bc + ca)3 .


W
W

3.11. If a, b, c are nonnegative real numbers, then

(2a2 + bc)(2b2 + ca)(2c 2 + a b) ≥ (a b + bc + ca)3 .

3.12. Let a, b, c be nonnegative real numbers such that a + b + c = 2. Prove that

(a) (a2 + b2 )(b2 + c 2 )(c 2 + a2 ) ≤ (a + b)(b + c)(c + a);

(b) (a2 + b2 )(b2 + c 2 )(c 2 + a2 ) ≤ 2.


Symmetric Polynomial Inequalities in Nonnegative Variables 151

3.13. Let a, b, c be nonnegative real numbers such that a + b + c = 2. Prove that

(a3 + b3 )(b3 + c 3 )(c 3 + a3 ) ≤ 2.

3.14. Let a, b, c be nonnegative real numbers such that a2 + b2 + c 2 = 2. Prove that

(a3 + b3 )(b3 + c 3 )(c 3 + a3 ) ≤ 2.

3.15. If a, b, c are nonnegative real numbers such that a + b + c = 2, then

L
.M
(3a2 − 2a b + 3b2 )(3b2 − 2bc + 3c 2 )(3c 2 − 2ca + 3a2 ) ≤ 36.

D
A
3.16. Let a, b, c be nonnegative real numbers such that a + b + c = 3. Prove that
PI
(a2 − 4a b + b2 )(b2 − 4bc + c 2 )(c 2 − 4ca + a2 ) ≤ 3.
M
LY

3.17. If a, b, c are positive real numbers such that a + b + c = 3, then


O

12
a bc + ≥ 5.
a b + bc + ca
.M

3.18. If a, b, c are positive real numbers such that a2 + b2 + c 2 = 3, then


W

3
5(a + b + c) +
W

≥ 18.
a bc
W

3.19. If a, b, c are positive real numbers such that a2 + b2 + c 2 = 3, then

12 + 9a bc ≥ 7(a b + bc + ca).

3.20. If a, b, c are positive real numbers such that a2 + b2 + c 2 = 3, then

21 + 18a bc ≥ 13(a b + bc + ca).


152 Vasile Cîrtoaje

3.21. If a, b, c are positive real numbers such that a2 + b2 + c 2 = 3, then

(2 − a b)(2 − bc)(2 − ca) ≥ 1.

3.22. Let a, b, c be positive real numbers such that a bc = 1. Prove that


‹5
a+b+c a2 + b2 + c 2

≥ .
3 3

L
3.23. If a, b, c are positive real numbers such that a bc = 1, then

.M
a3 + b3 + c 3 + a−3 + b−3 + c −3 + 21 ≥ 3(a + b + c)(a−1 + b−1 + c −1 ).

D
A
3.24. If a, b, c are positive real numbers such that a bc = 1, then

a2 + b2 + c 2 − a b − bc − ca ≥
9
PI
(a + b + c − 3).
M
4
LY

3.25. If a, b, c are positive real numbers such that a bc = 1, then


O

a2 + b2 + c 2 + a + b + c ≥ 2(a b + bc + ca).
.M

3.26. If a, b, c are positive real numbers such that a bc = 1, then


W

a2 + b2 + c 2 + 15(a b + bc + ca) ≥ 16(a + b + c).


W
W

3.27. If a, b, c are positive real numbers such that a bc = 1, then


2 1 3
+ ≥ .
a+b+c 3 a b + bc + ca

3.28. If a, b, c are positive real numbers such that a bc = 1, then

6
a b + bc + ca + ≥ 5.
a+b+c
Symmetric Polynomial Inequalities in Nonnegative Variables 153

3.29. If a, b, c are positive real numbers such that a bc = 1, then


Æ
3
Æ
4
(1 + a)(1 + b)(1 + c) ≥ 4(1 + a + b + c).

3.30. If a, b, c are positive real numbers, then

a6 + b6 + c 6 − 3a2 b2 c 2 ≥ 18(a2 − bc)(b2 − ca)(c 2 − a b).

L
3.31. If a, b, c are positive real numbers such that a + b + c = 3, then

.M
1 1 1
2
+ 2 + 2 ≥ a2 + b2 + c 2 .
a b c

D
A
3.32. If a, b, c are positive real numbers such that a b + bc + ca = 3, then
PI
a3 + b3 + c 3 + 7a bc ≥ 10.
M
LY

3.33. If a, b, c are nonnegative real numbers such that a3 + b3 + c 3 = 3, then


O

a4 b4 + b4 c 4 + c 4 a4 ≤ 3.
.M
W

3.34. If a, b, c are nonnegative real numbers, then


W

(a + 1)2 (b + 1)2 (c + 1)2 ≥ 4(a + b + c)(a b + bc + ca) + 28a bc.


W

3.35. If a, b, c are positive real numbers such that a + b + c = 3, then

1 + 8a bc ≥ 9 min{a, b, c}.

3.36. If a, b, c are positive real numbers such that a2 + b2 + c 2 = 3, then

1 + 4a bc ≥ 5 min{a, b, c}.
154 Vasile Cîrtoaje

3.37. If a, b, c are positive real numbers such that a + b + c = a bc, then


p
(1 − a)(1 − b)(1 − c) + ( 3 − 1)3 ≥ 0.

3.38. If a, b, c are nonnegative real numbers such that a + b + c = 2, then

(a2 + bc)(b2 + ca)(c 2 + a b) ≤ 1.

3.39. If a, b, c are nonnegative real numbers, then

L
.M
(8a2 + bc)(8b2 + ca)(8c 2 + a b) ≤ (a + b + c)6 .

D
3.40. If a, b, c are positive real numbers such that a2 b2 + b2 c 2 + c 2 a2 = 3, then

A
PI
a + b + c ≥ a bc + 2.
M
LY

3.41. Let a, b, c be nonnegative real numbers such that a + b + c = 5. Prove that

(a2 + 3)(b2 + 3)(c 2 + 3) ≥ 192.


O
.M

3.42. If a, b, c are nonnegative real numbers, then


W

a2 + b2 + c 2 + a bc + 2 ≥ a + b + c + a b + bc + ca.
W
W

3.43. If a, b, c are nonnegative real numbers, then


X
a3 (b + c)(a − b)(a − c) ≥ 3(a − b)2 (b − c)2 (c − a)2 .

3.44. Find the greatest real number k such that

a + b + c + 4a bc ≥ k(a b + bc + ca)

for all a, b, c ∈ [0, 1].


Symmetric Polynomial Inequalities in Nonnegative Variables 155

2
3.45. If a, b, c ≥ such that a + b + c = 3, then
3

a2 b2 + b2 c 2 + c 2 a2 ≥ a b + bc + ca.

3.46. If a, b, c are positive real numbers such that a ≤ 1 ≤ b ≤ c and

a + b + c = 3,

then
1 1 1

L
+ + ≥ a2 + b2 + c 2 .
a b c

D
.M
3.47. If a, b, c are positive real numbers such that a ≤ 1 ≤ b ≤ c and

A
1 1 1
a+b+c = + + ,
a b c PI
then
M
1 1 1
a2 + b2 + c 2 ≤ 2
+ 2 + 2.
a b c
LY
O

3.48. If a, b, c are positive real numbers such that


.M

1 1 1
a+b+c = + + ,
a b c
W

then
1 1 1
 ‹
(1 − a bc) a n + b n + c n − n − n − n ≥ 0
W

a b c
for any integer n ≥ 2.
W

3.49. Let a, b, c be positive real numbers, and let

E(a, b, c) = a(a − b)(a − c) + b(b − c)(b − a) + c(c − a)(c − b).

Prove that

(a) (a + b + c)E(a, b, c) ≥ a b(a − b)2 + bc(b − c)2 + ca(c − a)2 ;

1 1 1
 ‹
(b) 2 + + E(a, b, c) ≥ (a − b)2 + (b − c)2 + (c − a)2 .
a b c
156 Vasile Cîrtoaje

3.50. Let a ≥ b ≥ c be nonnegative real numbers. Schur’s inequalities of third and


fourth degree state that

(a) a(a − b)(a − c) + b(b − c)(b − a) + c(c − a)(c − b) ≥ 0;

(b) a2 (a − b)(a − c) + b2 (b − c)(b − a) + c 2 (c − a)(c − b) ≥ 0.

Prove that (a) is sharper than (b) if


p p p
b+ c≤ a,

L
and (b) is sharper than (a) if p p p
b+

.M
c≥ a.

D
3.51. If a, b, c are nonnegative real numbers such that

A
(a + b)(b + c)(c + a) = 8, PI
then
M
p p p
a+ b+ c ≥ a b + bc + ca.
LY

p
O

3.52. If a, b, c ∈ [1, 4 + 3 2], then


.M

9(a b + bc + ca)(a2 + b2 + c 2 ) ≥ (a + b + c)4 .


W

3.53. If a, b, c are nonnegative real numbers such that a + b + c + a bc = 4, then


W

(a) a2 + b2 + c 2 + 12 ≥ 5(a b + bc + ca);


W

(b) 3(a2 + b2 + c 2 ) + 13(a b + bc + ca) ≥ 48.

3.54. Let a, b, c be the lengths of the sides of a triangle. If a2 + b2 + c 2 = 3, then

a b + bc + ca ≥ 1 + 2a bc.

3.55. Let a, b, c be the lengths of the sides of a triangle. If a2 + b2 + c 2 = 3, then

a2 b2 + b2 c 2 + c 2 a2 ≥ a b + bc + ca.
Symmetric Polynomial Inequalities in Nonnegative Variables 157

3.56. Let a, b, c be the lengths of the sides of a triangle. If a + b + c = 3, then

1 1 1 41
+ + + ≥ 3(a2 + b2 + c 2 ).
a b c 6

3.57. Let a ≤ b ≤ c such that a + b + c = p and a b + bc + ca = q, where p and q are


fixed real numbers satisfying p2 ≥ 3q.
(a) If a, b, c are nonnegative real numbers, then the product r = a bc is maximal when
a = b, and is minimal when b = c or a = 0;
(b) If a, b, c are the lengths of the sides of a triangle (non-degenerate or degenerate),

L
c
then the product r = a bc is maximal when a = b ≥ or a + b = c, and is minimal

.M
2
when b = c ≥ a.

D
A
3.58. If a, b, c are positive real numbers such that a + b + c = 3, then

9
+ 16 ≥
75
PI.
a b + bc + ca
M
a bc
LY

3.59. If a, b, c are positive real numbers such that a + b + c = 3, then


O

1 1 1
 ‹
8 + + + 9 ≥ 10(a2 + b2 + c 2 ).
.M

a b c
W

3.60. If a, b, c are positive real numbers such that a + b + c = 3, then


W

7(a2 + b2 + c 2 ) + 8(a2 b2 + b2 c 2 + c 2 a2 ) + 4a2 b2 c 2 ≥ 49.


W

3.61. If a, b, c are nonnegative real numbers, then

(a3 + b3 + c 3 + a bc)2 ≥ 2(a2 + b2 )(b2 + c 2 )(c 2 + a2 ).

3.62. If a, b, c are nonnegative real numbers, then

[a b(a + b) + bc(b + c) + ca(c + a)]2 ≥ 4(a b + bc + ca)(a2 b2 + b2 c 2 + c 2 a2 ).


158 Vasile Cîrtoaje

3.63. Let a, b, c be nonnegative real numbers such that a b + bc + ca = 3. Prove that

4(a3 + b3 + c 3 ) + 7a bc + 125 ≥ 48(a + b + c).

3.64. If a, b, c ∈ [0, 1], then


p p p
a a + b b + c c + 4a bc ≥ 2(a b + bc + ca).

L
3.65. If a, b, c ∈ [0, 1], then

.M
p p p 3
a a + b b + c c ≥ (a b + bc + ca − a bc).
2

D
A
3.66. If a, b, c ∈ [0, 1], then PI
p p p 500
M
3(a a + b b + c c) + a bc ≥ 5(a b + bc + ca).
81
LY
O

3.67. Let a, b, c be the lengths of the sides of a triangle. If a2 + b2 + c 2 = 3, then


.M

a + b + c ≥ 2 + a bc.
W

3.68. Let f n (a, b, c) be a symmetric homogeneous polynomial of degree n ≤ 5. Prove


W

that
(a) the inequality f n (a, b, c) ≥ 0 holds for all nonnegative real numbers a, b, c if and
W

only if f n (a, 1, 1) ≥ 0 and f n (0, b, c) ≥ 0 for all nonnegative real numbers a, b, c;


(b) the inequality f n (a, b, c) ≥ 0 holds for all the lengths a, b, c of the sides of a non-
degenerate or degenerate triangle if and only if f n (x, 1, 1) ≥ 0 for 0 ≤ x ≤ 2, and
f n ( y + z, y, z) ≥ 0 for all y, z ≥ 0.

3.69. If a, b, c are nonnegative real numbers such that a + b + c = 3, then

4(a4 + b4 + c 4 ) + 45 ≥ 19(a2 + b2 + c 2 ).
Symmetric Polynomial Inequalities in Nonnegative Variables 159

3.70. Let a, b, c be nonnegative real numbers. If k ≤ 2, then


X
a(a − b)(a − c)(a − k b)(a − kc) ≥ 0.

3.71. Let a, b, c be nonnegative real numbers. If k ∈ R, then


X
(b + c)(a − b)(a − c)(a − k b)(a − kc) ≥ 0.

3.72. If a, b, c are nonnegative real numbers, then

L
X

.M
a(a − 2b)(a − 2c)(a − 5b)(a − 5c) ≥ 0.

D
3.73. If a, b, c are the lengths of the sides of a triangle, then

A
a4 + b4 + c 4 + 9a bc(a + b + c) ≤ 10(a2 b2 + b2 c 2 + c 2 a2 ).
PI
M
3.74. If a, b, c are the lengths of the sides of a triangle, then
LY

X
3(a4 + b4 + c 4 ) + 7a bc(a + b + c) ≤ 5 a b(a2 + b2 ).
O
.M

3.75. If a, b, c are the lengths of the sides of a triangle, then

b2 + c 2 − 6bc c 2 + a2 − 6ca a2 + b2 − 6a b
+ + + 4(a + b + c) ≤ 0.
W

a b c
W

3.76. Let f6 (a, b, c) be a sixth degree symmetric homogeneous polynomial written in


W

the form
f6 (a, b, c) = Ar 2 + B(p, q)r + C(p, q), A ≤ 0,
where
p = a + b + c, q = a b + bc + ca, r = a bc.
Prove that
(a) the inequality f6 (a, b, c) ≥ 0 holds for all nonnegative real numbers a, b, c if and
only if f6 (a, 1, 1) ≥ 0 and f6 (0, b, c) ≥ 0 for all nonnegative real numbers a, b, c;
(b) the inequality f6 (a, b, c) ≥ 0 holds for all lengths a, b, c of the sides of a non-
degenerate or degenerate triangle if and only if f6 (x, 1, 1) ≥ 0 for 0 ≤ x ≤ 2, and
f6 ( y + z, y, z) ≥ 0 for all y, z ≥ 0.
160 Vasile Cîrtoaje

3.77. If a, b, c are nonnegative real numbers, then


X
a(b + c)(a − b)(a − c)(a − 2b)(a − 2c) ≥ (a − b)2 (b − c)2 (c − a)2 .

3.78. Let a, b, c be nonnegative real numbers.

(a) If 2 ≤ k ≤ 6, then
X 4(k − 2)(a − b)2 (b − c)2 (c − a)2
a(a − b)(a − c)(a − k b)(a − kc) + ≥ 0;
a+b+c

L
.M
(b) If k ≥ 6, then
X (k + 2)2 (a − b)2 (b − c)2 (c − a)2
a(a − b)(a − c)(a − k b)(a − kc) + ≥ 0.

D
4(a + b + c)

A
3.79. If a, b, c are nonnegative real numbers, then
PI
M
(3a2 + 2a b + 3b2 )(3b2 + 2bc + 3c 2 )(3c 2 + 2ca + 3a2 ) ≥ 8(a2 + 3bc)(b2 + 3ca)(c 2 + 3a b).
LY
O

3.80. Let a, b, c be nonnegative real numbers such that a + b + c = 2. If


.M

−2 11
≤k≤ ,
3 8
W

then
(a2 + ka b + b2 )(b2 + k bc + c 2 )(c 2 + kca + a2 ) ≤ k + 2.
W
W

3.81. Let a, b, c be nonnegative real numbers such that a + b + c = 2. Prove that

(2a2 + bc)(2b2 + ca)(2c 2 + a b) ≤ 4.

3.82. Let a, b, c be nonnegative real numbers, no two of which are zero. Then,
X 5(a − b)2 (b − c)2 (c − a)2
(a − b)(a − c)(a − 2b)(a − 2c) ≥ .
a b + bc + ca
Symmetric Polynomial Inequalities in Nonnegative Variables 161

3.83. Let a ≤ b ≤ c be positive real numbers such that

a + b + c = p, a bc = r,

where p and r are fixed positive numbers satisfying p3 ≥ 27r. Prove that

q = a b + bc + ca

is maximal when b = c, and is minimal when a = b.

L
3.84. If a, b, c are positive real numbers, then

.M
1 1 1 1
 ‹
(a + b + c − 3) + + − 3 + a bc + ≥ 2.
a b c a bc

D
A
3.85. If a, b, c are positive real numbers such that a bc = 1, then
PI
‹ s
3 2 2

M
(a) a b + bc + ca − ≥ (a + b + c) − 1;
7 3 3
LY

46 p
(b) a b + bc + ca − 3 ≥ ( a + b + c − 2 − 1).
27
O
.M

3.86. If a, b, c are positive real numbers such that a bc = 1, then

50 37
W

a b + bc + ca + ≥ .
a+ b+c+5 4
W
W

3.87. Let a, b, c be positive real numbers.


(a) If a bc = 2, then

a2 + b2 + c 2
(a + b + c − 3)2 + 1 ≥ ;
3
1
(b) If a bc = , then
2

a2 + b2 + c 2 + 3(3 − a − b − c)2 ≥ 3.
162 Vasile Cîrtoaje

3.88. If a, b, c are positive real numbers such that a + b + c = 3, then


 ‹
bc ca a b
4 + + + 9a bc ≥ 21.
a b c

3.89. If a, b, c are nonnegative real numbers such that

a b + bc + ca = a bc + 2,

then
a2 + b2 + c 2 + a bc ≥ 4.

L
.M
3.90. If a, b, c are positive real numbers, then

D
‹2
b+c p c + a p 2 a+b p 2
  ‹
+ 2 +

A
−2− 2 −2− − 2 − 2 ≥ 6.
a b c
PI
M
3.91. If a, b, c are positive real numbers, then
LY

2(a3 + b3 + c 3 ) + 9(a b + bc + ca) + 39 ≥ 24(a + b + c).


O

3.92. If a, b, c are positive real numbers such that a2 + b2 + c 2 = 3, then


.M

a3 + b3 + c 3 − 3 ≥ |(a − b)(b − c)(c − a)|.


W
W

3.93. If a, b, c are nonnegative real numbers, then

a4 + b4 + c 4 − a2 b2 − b2 c 2 − c 2 a2 ≥ 2|a3 b + b3 c + c 3 a − a b3 − bc 3 − ca3 |.
W

3.94. If a, b, c are nonnegative real numbers, then


p
a4 + b4 + c 4 − a bc(a + b + c) ≥ 2 2 |a3 b + b3 c + c 3 a − a b3 − bc 3 − ca3 |.

3.95. If a, b, c ≥ −5 such that a + b + c = 3, then


1−a 1− b 1−c
+ + ≥ 0.
1+a+a 2 1+ b+ b 2 1 + c + c2
Symmetric Polynomial Inequalities in Nonnegative Variables 163

1 4
3.96. Let a, b, c 6= be nonnegative real numbers such that a + b + c = 3. If k ≥ ,
k 3
then
1−a 1− b 1−c
+ + ≥ 0.
(1 − ka)2 (1 − k b)2 (1 − kc)2

3.97. Let a, b, c, d be nonnegative real numbers such that

a2 + b2 + c 2 + d 2 = 1.

Prove that
(1 − a)(1 − b)(1 − c)(1 − d) ≥ a bcd.

L
.M
3.98. Let a, b, c, d and x be positive real numbers such that

D
1 1 1 1 4
+ 2 + 2 + 2 = 2.

A
a 2 b c d x
If x ≥ 2, then PI
(a − 1)(b − 1)(c − 1)(d − 1) ≥ (x − 1)4 .
M
LY

3.99. If a, b, c, d are positive real numbers, then


O

(1 + a3 )(1 + b3 )(1 + c 3 )(1 + d 3 ) 1 + a bcd


≥ .
(1 + a2 )(1 + b2 )(1 + c 2 )(1 + d 2 ) 2
.M

3.100. Let a, b, c, d be positive real numbers such that a + b + c + d = 4. Prove that


W

1 1 1 1 1 1 1 1
 ‹ ‹ ‹ ‹
a+ −1 b+ −1 c+ −1 d + −1 +3≥ + + + .
W

a b c d a b c d
W

3.101. If a, b, c, d are nonnegative real numbers, then

4(a3 + b3 + c 3 + d 3 ) + 15(a bc + bcd + cd a + d a b) ≥ (a + b + c + d)3 .

3.102. Let a, b, c, d be positive real numbers such that

a + b + c + d = 4.

Prove that
1 + 2(a bc + bcd + cd a + d a b) ≥ 9 min{a, b, c, d}.
164 Vasile Cîrtoaje

3.103. Let a, b, c, d be nonnegative real numbers such that

a + b + c + d = 4.

Prove that
5(a2 + b2 + c 2 + d 2 ) ≥ a3 + b3 + c 3 + d 3 + 16.

3.104. Let a, b, c, d be nonnegative real numbers such that

a + b + c + d = 4.

L
.M
Prove that
3(a2 + b2 + c 2 + d 2 ) + 4a bcd ≥ 16.

D
A
3.105. Let a, b, c, d be nonnegative real numbers such that

a + b + c + d = 4.
PI
M
Prove that
LY

27(a bc + cd + cd a + d a b) ≤ 44a bcd + 64.


O

3.106. Let a, b, c, d be positive real numbers such that


.M

1 1 1 1
a+b+c+d = + + + .
W

a b c d
Prove that
W

1 1 1 1
 ‹
(1 − a bcd) a2 + b2 + c 2 + d 2 − 2 − 2 − 2 − 2 ≥ 0.
W

a b c d

3.107. Let a, b, c, d be positive real numbers such that

a + b + c + d = 1.

Prove that
1 1 1 1 81
 ‹
(1 − a)(1 − b)(1 − c)(1 − d) + + + ≥ .
a b c d 16
Symmetric Polynomial Inequalities in Nonnegative Variables 165

3.108. Let a, b, c, d be nonnegative real numbers such that

a + b + c + d = a3 + b3 + c 3 + d 3 = 2.

Prove that
7
a2 + b2 + c 2 + d 2 ≥ .
4

3.109. Let a, b, c, d ∈ (0, 4] such that a bcd = 1. Prove that

(1 + 2a)(1 + 2b)(1 + 2c)(1 + 2d) ≥ (5 − 2a)(5 − 2b)(5 − 2c)(5 − 2d).

L
.M
3.110. Let a, b, c, d and k be positive real numbers such that
1 1 1 1
 ‹

D
(a + b + c + d) + + + = k.
a b c d

A
p
If 16 ≤ k ≤ (1 + 10)2 , then any three of a, b, c, d are the lengths of the sides of a
triangle (non-degenerate or degenerate).
PI
M
LY

3.111. Let a, b, c, d and k be positive real numbers such that


1 1 1 1
 ‹
(a + b + c + d) + + + = k.
O

a b c d
.M

119
If 16 ≤ k ≤ , then there exist three numbers of a, b, c, d which are the lengths of
6
the sides of a triangle (non-degenerate or degenerate).
W
W

3.112. Let a, b, c, d and k be positive real numbers such that

(a + b + c + d)2 = k(a2 + b2 + c 2 + d 2 ).
W

11
If ≤ k ≤ 4, then any three of a, b, c, d are the lengths of the sides of a triangle
3
(non-degenerate or degenerate).

3.113. Let a, b, c, d and k be positive real numbers such that

(a + b + c + d)2 = k(a2 + b2 + c 2 + d 2 ).
49
If ≤ k ≤ 4, then there exist three numbers of a, b, c, d which are the lengths of the
15
sides of a triangle (non-degenerate or degenerate).
166 Vasile Cîrtoaje

3.114. Let a, b, c, d, e be nonnegative real numbers.


(a) If a + b + c = 3(d + e), then

4(a4 + b4 + c 4 + d 4 + e4 ) ≥ (a2 + b2 + c 2 + d 2 + e2 )2 ;

(b) If a + b + c = d + e, then

12(a4 + b4 + c 4 + d 4 + e4 ) ≤ 7(a2 + b2 + c 2 + d 2 + e2 )2 .

3.115. Let a, b, c, d, e be nonnegative real numbers such that

L
.M
a + b + c + d + e = 5.

Prove that

D
a4 + b4 + c 4 + d 4 + e4 + 150 ≤ 31(a2 + b2 + c 2 + d 2 + e2 ).

A
PI
M
3.116. Let a, b, c, d, e be positive real numbers such that
LY

a2 + b2 + c 2 + d 2 + e2 = 5.

Prove that
O

a bcd e(a4 + b4 + c 4 + d 4 + e4 ) ≤ 5.
.M

3.117. Let a, b, c, d, e be positive real numbers such that


W

a + b + c + d + e = 5.
W

Prove that
W

1 1 1 1 1 20
+ + + + + 2 ≥ 9.
a b c d e a + b + c 2 + d 2 + e2
2

3.118. If a, b, c, d, e ≥ 1, then
1 1 1 1 1
 ‹ ‹ ‹ ‹ ‹
a+ b+ c+ d+ e+ + 68 ≥
a b c d e
1 1 1 1 1
 ‹
≥ 4(a + b + c + d + e) + + + + .
a b c d e
Symmetric Polynomial Inequalities in Nonnegative Variables 167

3.119. Let a, b, c and x, y, z be positive real numbers such that

(a + b + c)(x + y + z) = (a2 + b2 + c 2 )(x 2 + y 2 + z 2 ) = 4.

Prove that
1
a bc x yz < .
36

3.120. Let a1 , a2 , · · · , an (n ≥ 3) be positive real numbers such that

a1 + a2 + · · · + an = a12 + a22 + · · · + an2 = n − 1.

L
.M
Prove that
1 1 1 n2 (2n − 3)
+ + ··· + ≥ .
a1 a2 an 2(n − 1)(n − 2)

D
A
3.121. Let a1 , a2 , · · · , an be positive real numbers such that a1 + a2 +· · ·+ an = n. Prove
that 
1 1 1
‹
PI
M
n2 + + ··· + − n ≥ 4(n − 1)(a12 + a22 + · · · + an2 − n).
a1 a2 an
LY

3.122. Let a1 , a2 , · · · , an be nonnegative real numbers such that a1 + a2 + · · · + an = n.


O

Prove that
(n + 1)(a12 + a22 + · · · + an2 ) ≥ n2 + a13 + a23 + · · · + an3 .
.M
W

3.123. Let a1 , a2 , · · · , an be nonnegative real numbers such that a1 + a2 + · · · + an = n.


Prove that
W

(n − 1)(a13 + a23 + · · · + an3 ) + n2 ≥ (2n − 1)(a12 + a22 + · · · + an2 ).


W

3.124. Let a1 , a2 , . . . , an (n ≥ 3) be positive real numbers such that

a1 + a2 + · · · + an = 1.

Prove that
‹n
1 1 1 1
 ‹ ‹ ‹ 
a1 + −2 a2 + −2 an + −2 ≥ n+ −2 .
a1 a2 an n
168 Vasile Cîrtoaje

3.125. Let a1 , a2 , . . . , an (n ≥ 3) be positive real numbers such that

a1 + a2 + · · · + an = n.

Prove that
n
a12 + a22 + · · · + an2 − n ≥ (1 − a1 a2 · · · an ).
n−1

3.126. Let a1 , a2 , . . . , an (n ≥ 3) be positive real numbers such that

a1 ≤ a2 ≤ · · · ≤ an ,

L
.M
1 1 1
 ‹
(a1 + a2 + · · · + an ) + + ··· + = k.
a1 a2 an

D
i(n − i)
(a) If n2 ≤ k ≤ n2 + , i ∈ {2, 3, · · · , n − 1}, then ai−1 , ai and ai+1 are the
2

A
lengths of the sides of a non-degenerate or degenerate triangle;

(b) If n2 ≤ k ≤ αn , where αn =
9n2
for even n, and αn =
PI9n2 − 1
for odd n, then
8 8
M
there exist three numbers ai which are the lengths of the sides of a non-degenerate or
degenerate triangle.
LY
O

3.127. Let a1 , a2 , . . . , an (n ≥ 3) be positive real numbers such that


.M

a1 ≤ a2 ≤ · · · ≤ an ,

(a1 + a2 + · · · + an )2 = k(a12 + a22 + · · · + an2 ).


W

(2n − i)2
W

(a) If ≤ k ≤ n, i ∈ {2, 3, · · · , n − 1}, then ai−1 , ai and ai+1 are the lengths
4n − 3i
of the sides of a non-degenerate or degenerate triangle;
W

8n + 1
(b) If ≤ k ≤ n, then there exist three numbers ai which are the lengths of the
9
sides of a non-degenerate or degenerate triangle.
Symmetric Polynomial Inequalities in Nonnegative Variables 169

3.2 Solutions

P 3.1. If a, b, c are positive real numbers, then

a2 + b2 + c 2 + 2a bc + 1 ≥ 2(a b + bc + ca).

(Darij Grinberg, 2004)

First Solution. Setting a = x 3 , b = y 3 and c = z 3 , where x, y, z > 0, we need to prove


that
x 6 + y 6 + z 6 + 2x 3 y 3 z 3 + 1 ≥ 2(x 3 y 3 + y 3 z 3 + z 3 x 3 ).

L
Using Schur’s inequality and the AM-GM inequality, we have

.M
X X
x 6 + y 6 + z 6 + 3x 2 y 2 z 2 ≥ x 2 y 2 (x 2 + y 2 ) ≥ 2 x 3 y 3.

D
Thus, it suffices to show that

A
2x 3 y 3 z 3 − 3x 2 y 2 z 2 + 1 ≥ 0,
PI
which is equivalent to
M
(x yz − 1)2 (2x yz + 1) ≥ 0.
LY

The equality holds for a = b = c = 1.


O

Second Solution. Among the numbers 1 − a, 1 − b and 1 − c there are always two with
the same sign; let us say (1 − b)(1 − c) ≥ 0. Then, we have
.M

a2 + b2 + c 2 + 2a bc + 1 − 2(a b + bc + ca) =
= (a − 1)2 + (b − c)2 + 2a + 2a bc − 2a(b + c)
W

= (a − 1)2 + (b − c)2 + 2a(1 − b)(1 − c) ≥ 0.


W

Remark. The following generalization holds:


W

• Let a, b, c be positive real numbers. If 0 ≤ k ≤ 1, then

a2 + b2 + c 2 + 2ka bc + k ≥ (k + 1)(a b + bc + ca).

Since the both sides of the inequality are linear of k, it suffices to prove it for only k = 0
and k = 1. For k = 0, the inequality reduces to the the known a2 + b2 +c 2 ≥ a b+ bc +ca.

p
P 3.2. Let a, b, c be nonnegative real numbers. If 0 ≤ k ≤ 2, then

a2 + b2 + c 2 + ka bc + 2k + 3 ≥ (k + 2)(a + b + c).
170 Vasile Cîrtoaje

Solution. Since the both sides


p of the inequality are linear of k, it suffices to prove the
inequality for k = 0 and k = 2. For k = 0, the inequality reduces to

(a − 1)2 + (b − 1)2 + (c − 1)2 ≥ 0.


p
Further, we consider that k = 2.
First Solution. Write the inequality as
p
(a − 1)2 + (b − 1)2 + (c − 1)2 ≥ 2 (a + b + c − 2 − a bc).

Using the substitutions x = a − 1, y = b − 1 and z = c − 1, we need to show that

L
p
x 2 + y 2 + z 2 + 2 (x yz + x y + yz + z x) ≥ 0

.M
for x, y, z ≥ −1. Among the numbers x, y and z there are always two of them with the
same sign; let us say yz ≥ 0. Since

D
A
1
y 2 + z2 ≥ ( y + z)2
2 PI
and
M
x yz + x y + yz + z x = (x + 1) yz + x( y + z) ≥ x( y + z),
LY

it suffices to prove that


1 p
x 2 + ( y + z)2 + 2 x( y + z) ≥ 0,
O

2
.M

which is equivalent to
1
[x + p ( y + z)]2 ≥ 0.
2
W

Thus,pthe proof is completed. The equality holds for a = b = c = p


1. In addition, if
k = 2, then the equality holds also for a = 0 and b = c = 1 + 1/ 2 (or any cyclic
W

permutation).
W

Second Solution. Let a + b + c = p. For the nontrivial case p > 0, we write the original
inequality in the homogeneous form

(a + b + c)(a2 + b2 + c 2 ) (2k + 3)(a + b + c)3 (k + 2)(a + b + c)3


+ ka bc + ≥ ,
p p3 p2
or

[p2 − (k + 2)p + 2k + 3](a + b + c)3 + kp3 a bc − 2p2 (a + b + c)(a b + bc + ca) ≥ 0.


p
Since p2 − (k + 2)p + 2k + 3 > 0 for k = 2, using Schur’s inequality

(a + b + c)3 ≥ −9a bc + 4(a + b + c)(a b + bc + ca),


Symmetric Polynomial Inequalities in Nonnegative Variables 171

it suffices to show that

2[p2 − 2(k + 2)p + 2(2k + 3)](a + b + c)(a b + bc + ca)+

+[kp3 − 9p2 + 9(k + 2)p − 9(2k + 3)]a bc ≥ 0.


Since
p2 − 2(k + 2)p + 2(2k + 3) = (p − k − 2)2 ≥ 0
and
(a + b + c)(a b + bc + ca) ≥ 9a bc,
it is enough to prove that

L
.M
18[p2 − 2(k + 2)p + 2(2k + 3)] + kp3 − 9p2 + 9(k + 2)p − 9(2k + 3) ≥ 0;

that is,

D
kp3 + 9p2 − 27(k + 2)p + 27(2k + 3) ≥ 0,

A
(kp + 6k + 9)(p − 3)2 ≥ 0.
PI
M
LY

P 3.3. If a, b, c are positive real numbers, then


O

a bc(a + b + c) + 2(a2 + b2 + c 2 ) + 3 ≥ 4(a b + bc + ca).


.M

First Solution. Applying the AM-GM inequality two times, we get


W

Æ 18a bc
a bc(a + b + c) + 3 ≥ 2 3a bc(a + b + c) ≥ .
a+b+c
W

Therefore, it suffices to prove that


W

18a bc
a2 + b2 + c 2 + ≥ 2(a b + bc + ca),
a+b+c
which is just Schur’s inequality of third degree. The equality holds for a = b = c = 1.
Second Solution. Applying the AM-GM, we get
p p p
a bc(a + b + c) + 3 = (a2 bc + 1) + (a b2 c + 1) + (a bc 2 + 1) ≥ 2a bc + 2b ca + 2c a b.

Thus, it suffices to prove that


p p p
a2 + b2 + c 2 + a bc + b ca + c a b ≥ 2(a b + bc + ca).
172 Vasile Cîrtoaje

p p p
Substituting x = a, y = b, z = c, where x, y, z ≥ 0, we need to show that
x 4 + y 4 + z 4 + x yz(x + y + z) ≥ 2(x 2 y 2 + y 2 z 2 + z 2 x 2 ).
This inequality can be obtained by summing Schur’s inequality of degree four
x 4 + y 4 + z 4 + x yz(x + y + z) ≥ x y(x 2 + y 2 ) + yz( y 2 + z 2 ) + z x(z 2 + x 2 )
to
x y(x 2 + y 2 ) + yz( y 2 + z 2 ) + z x(z 2 + x 2 ) ≥ 2(x 2 y 2 + y 2 z 2 + z 2 x 2 ),
which is equivalent to
x y(x − y)2 + yz( y − z)2 + z x(z − x)2 ≥ 0.

L
D
.M
P 3.4. If a, b, c are positive real numbers, then

A
a(b2 + c 2 ) + b(c 2 + a2 ) + c(a2 + b2 ) + 3 ≥ 3(a b + bc + ca).

Solution. Write the inequality as follows


PI
M
(a + b + c)(a b + bc + ca) + 3 ≥ 3(a bc + a b + bc + ca).
LY

(a + b + c − 3)(a b + bc + ca) + 3 ≥ 3a bc.


Using the known inequality
O

(a + b + c)(a b + bc + ca) ≥ 9a bc,


.M

it suffices to show that


3(a + b + c − 3)(a b + bc + ca) + 9 ≥ (a + b + c)(a b + bc + ca),
W

which is equivalent to
W

[2(a + b + c) − 9](a b + bc + ca) + 9 ≥ 0.


W

For the nontrivial case 2(a + b + c) − 9 < 0, using the known inequality
(a + b + c)2 ≥ 3(a b + bc + ca),
it is enough to show that
[2(a + b + c) − 9](a + b + c)2 + 27 ≥ 0.
This inequality is equivalent to the obvious inequality
(a + b + c − 3)2 [2(a + b + c) + 3] ≥ 0.
The equality holds for a = b = c = 1.
Symmetric Polynomial Inequalities in Nonnegative Variables 173

P 3.5. If a, b, c are positive real numbers, then


3
a2 + b2 + c 2

≥ a2 b2 c 2 + (a − b)2 (b − c)2 (c − a)2 .
3

(Vasile Cîrtoaje, 2011)

Solution (by Vo Quoc Ba Can). Assume that a = min{a, b, c}. By virtue of the AM-GM
inequality, we have
3 3
a2 + b2 + c 2 (a2 + b2 + c 2 − 2bc) + bc + bc
 
= ≥ (a2 + b2 + c 2 − 2bc)b2 c 2
3 3

L
= a2 b2 c 2 + (b − c)2 b2 c 2 .

.M
Thus, it suffices to prove that

D
(b − c)2 b2 c 2 ≥ (b − c)2 (b − a)2 (c − a)2 .

A
PI
This is obvious, because b2 > (b − a)2 and c 2 > (c − a)2 . The equality occurs for
a = b = c.
M
LY

P 3.6. If a, b, c are nonnegative real numbers, then


O

[a b(a + b) + bc(b + c) + ca(c + a)]2 ≥ 4(a b + bc + ca)(a2 b2 + b2 c 2 + c 2 a2 ).


.M

(Vasile Cîrtoaje and Vo Quoc Ba Can, 2011)

First Solution. Assume that a ≥ b ≥ c. For the nontrivial case b > 0, by the AM-GM
W

inequality, we have
W

2
a2 b2 + b2 c 2 + c 2 a2

2 2 2 2 2 2
4(a b + bc + ca)(a b + b c + c a ) ≤ b(a b + bc + ca) + .
W

Thus, it suffices to prove that

a2 b2 + b2 c 2 + c 2 a2
a b(a + b) + bc(b + c) + ca(c + a) ≥ b(a b + bc + ca) + .
b
This inequality reduces to the obvious form

ac(a − b)(b − c) ≥ 0.

The equality holds for a = b = c, for b = c = 0 (or any cyclic permutation), and for
a = 0 and b = c (or any cyclic permutation).
174 Vasile Cîrtoaje

Second Solution. We well prove the stronger inequality

[a b(a + b) + bc(b + c) + ca(c + a)]2 ≥ 4(a b + bc + ca)(a2 b2 + b2 c 2 + c 2 a2 ) + A,

where
A = (a − b)2 (b − c)2 (c − a)2 .
Let p = a + b + c, q = a b + bc + ca, r = a bc. Since

(a − b)2 (b − c)2 (c − a)2 = −27r 2 + 2(9pq − 2p3 )r + p2 q2 − 4q3 ,

we can write this inequality as

L
.M
(pq − 3r)2 ≥ 4q(q2 − 2pr) − 27r 2 + 2(9pq − 2p3 )r + p2 q2 − 4q3 ,

which reduces to

D
r(p3 + 9r − 4pq) ≥ 0.

A
This is true since p3 + 9r − 4pq ≥ 0 (by the third degree Schur’s inequality).
PI
M
LY

P 3.7. If a, b, c are positive real numbers, then


O

(a) a3 + b3 + c 3 + a b + bc + ca + 9 ≥ 5(a + b + c);


.M

(b) a3 + b3 + c 3 + 4(a b + bc + ca) + 18 ≥ 11(a + b + c).


W

(Vasile Cîrtoaje, 2010)

Solution. Let p = a + b + c and q = a b + bc + ca. From


W

a(a − 1)2 + b(b − 1)2 + c(c − 1)2 ≥ 0,


W

we get
a3 + b3 + c 3 ≥ 2(a2 + b2 + c 2 ) − a − b − c = 2p2 − p − 4q.
(a) Using the result above and the known inequality p2 ≥ 3q, we have

a3 + b3 + c 3 + a b + bc + ca + 9 − 5(a + b + c) ≥
≥ (2p2 − p − 4q) + q + 9 − 5p = 2p2 − 6p + 9 − 3q
≥ 2p2 − 6p + 9 − p2 = (p − 3)2 ≥ 0.

The equality holds for a = b = c = 1.


Symmetric Polynomial Inequalities in Nonnegative Variables 175

(b) Using the result above, we have

a3 + b3 + c 3 + 4(a b + bc + ca) + 18 − 11(a + b + 2 ≥


≥ (2p2 − p − 4q) + 4q + 18 − 11p = 2p2 − 12p + 18
= 2(p − 3)3 ≥ 0.

The equality holds for a = b = c = 1.

L
P 3.8. If a, b, c are positive real numbers, then

.M
(a) a3 + b3 + c 3 + a bc + 8 ≥ 4(a + b + c);

D
(b) 4(a3 + b3 + c 3 ) + 15a bc + 54 ≥ 27(a + b + c).

A
PI
Solution. Let p = a + b + c and q = a b + bc + ca. By Schur’s inequality of third degree,
we have
M
p3 + 9a bc ≥ 4pq,
LY

p(4q − p2 )
a bc ≥ .
9
O

a) We have
.M

a3 + b3 + c 3 + a bc = 4a bc + p(p2 − 3q)
4p(4q − p2 )
≥ + p(p2 − 3q)
W

9
p(5p2 − 11q)
= .
W

9
W

Then, it suffices to prove that

p(5p2 − 11q)
+ 8 ≥ 4p,
9
which is equivalent to
5p3 − 36p + 72 ≥ 11pq.
Since p2 ≥ 3q, we have

3(5p3 − 36p + 72 − 11pq) ≥ 3(5p3 − 36p + 72) − 11p2

= 4(p3 − 27p + 54) = 4(p − 3)2 (p + 6) ≥ 0.


176 Vasile Cîrtoaje

The equality holds for a = b = c = 1.

(b) We have

4(a3 + b3 + c 3 ) + 15a bc = 27a bc + 4p(p2 − 3q)


≥ 3p(4q − p2 ) + 4p(p2 − 3q) = p3 .

Then, it suffices to prove that


p3 + 54 ≥ 27p,
which is equivalent to the obvious inequality

(p − 3)2 (p + 6) ≥ 0.

L
.M
The equality holds for a = b = c = 1, and also for a = 0 and b = c = 3/2 (or any cyclic
permutation).

D
Remark. Similarly, we can prove the following generalization (Vasile Cîrtoaje, 2010):

A
• Let a, b, c be nonnegative real numbers. If 0 ≤ k ≤ 27/4, then
PI
a3 + b3 + c 3 + (k − 3)a bc + 2k ≥ k(a + b + c).
M
LY

P 3.9. Let a, b, c be nonnegative real numbers such that


O

a + b + c = a2 + b2 + c 2 .
.M

Prove that
a b + bc + ca ≥ a2 b2 + b2 c 2 + c 2 a2 .
W

(Vasile Cîrtoaje, 2006)


W

Solution (by Michael Rozenberg). From the hypothesis condition, by squaring, we get

a4 + b4 + c 4 − a2 − b2 − c 2 = 2(a b + bc + ca − a2 b2 − b2 c 2 − c 2 a2 ).
W

Therefore, we can write the required inequality as

a4 + b4 + c 4 ≥ a2 + b2 + c 2 .

This inequality has the homogeneous form

(a + b + c)2 (a4 + b4 + c 4 ) ≥ (a2 + b2 + c 2 )3 ,

which follows immediately from Hölder’s inequality. The equality holds for a = b =
c = 1, for a = b = c = 0, for (a, b, c) = (0, 1, 1) (or any cyclic permutation), and for
(a, b, c) = (1, 0, 0) (or any cyclic permutation).
Symmetric Polynomial Inequalities in Nonnegative Variables 177

P 3.10. If a, b, c are nonnegative real numbers, then

(a2 + 2bc)(b2 + 2ca)(c 2 + 2a b) ≥ (a b + bc + ca)3 .

(Vasile Cîrtoaje, 2006)

Solution. We have
X X
(a2 + 2bc)(b2 + 2ca)(c 2 + 2a b) = 9a2 b2 c 2 + 2 a3 b3 + 4a bc a3

and X X
(a b + bc + ca)3 = 6a2 b2 c 2 + a3 b3 + 3a bc a b(a + b).

L
.M
So, we can rewrite the inequality as
X X X
3a2 b2 c 2 + a3 b3 + 4a bc a3 ≥ 3a bc a b(a + b).

D
A
a3 b3 ≥ 3a2 b2 c 2 (by the AM-GM inequality), it suffices to prove that
P
Since

6a bc + 4
X
a3 ≥ 3
X PI
a b(a + b).
M
We can get this inequality by summing the inequalities
LY

1X 3
a ≥ a bc
O

3
and
.M

X X
3a bc + a3 ≥ a b(a + b).
The first inequality follows from the AM-GM inequality, while the second is just the third
W

degree Schur’s inequality. The equality holds when a = b = c, and also when two of
W

a, b, c are zero.
Remark. Similarly, we can also prove the following inequality
W

(2a2 + 7bc)(2b2 + 7ca)(2c 2 + 7a b) ≥ 27(a b + bc + ca)3 .

P 3.11. If a, b, c are nonnegative real numbers, then

(2a2 + bc)(2b2 + ca)(2c 2 + a b) ≥ (a b + bc + ca)3 .

(Vasile Cîrtoaje, 2006)


178 Vasile Cîrtoaje

First Solution. Since


X X
(2a2 + bc)(2b2 + ca)(2c 2 + a b) = 9a2 b2 c 2 + 4 a3 b3 + 2a bc a3

and X X
(a b + bc + ca)3 = 6a2 b2 c 2 + a3 b3 + 3a bc a b(a + b),

the inequality is equivalent to


X X X
3a2 b2 c 2 + 3 a3 b3 + 2a bc a3 ≥ 3a bc a b(a + b).

L
We can get this inequality by summing

.M
2 X
a bc a3 ≥ 2a2 b2 c 2
3

D
A
and X X
a3 b3 + 3a2 b2 c 2 ≥ a bc a b(a + b).
PI
The first inequality follows from the AM-GM inequality, while the second is just the
M
third degree Schur’s inequality applied to the numbers a b, bc and ca. The equality
holds when a = b = c, and also when two of a, b, c are zero.
LY
O

Second Solution. By Hölder’s inequality, we have


.M

(a2 + bc + a2 )(b2 + b2 + ca)(a b + c 2 + c 2 ) ≥ (a b + bc + ca)3 ,

from which the desired inequality follows.


W

Remark. Using the first method, we can also prove the following inequality
W

(5a2 + bc)(5b2 + ca)(5c 2 + a b) ≥ 8(a b + bc + ca)3 .


W

P 3.12. Let a, b, c be nonnegative real numbers such that a + b + c = 2. Prove that

(a) (a2 + b2 )(b2 + c 2 )(c 2 + a2 ) ≤ (a + b)(b + c)(c + a);

(b) (a2 + b2 )(b2 + c 2 )(c 2 + a2 ) ≤ 2.


Symmetric Polynomial Inequalities in Nonnegative Variables 179

Solution. Assume that a = min{a, b, c}. It is easy to check that the equality holds in
both of the inequalities for a = 0 and b = c = 1.
(a) Since
a2 + b2 ≤ b(a + b)
and
c 2 + a2 ≤ c(c + a),
it suffices to show that
bc(b2 + c 2 ) ≤ b + c.
By virtue of the AM-GM inequality and the hypothesis b + c ≤ 2, we have

L
.M
2
2bc + (b2 + c 2 ) (b + c)4

2 2
2bc(b + c ) ≤ = ≤ 2(b + c).
2 4

D
The equality holds for a = 0 and b = c = 1 (or any cyclic permutation).

A
(b) First Solution. Since PI
a2 + b2 ≤ b(a + b)
M
and
c 2 + a2 ≤ c(a + c),
LY

it suffices to show that


O

bc(a + b)(a + c)(b2 + c 2 ) ≤ 2.


.M

By the AM-GM inequality, we have


3
2b(a + c) + 2c(a + b) + (b2 + c 2 )

2 2
4bc(a + b)(a + c)(b + c ) ≤ .
W

3
W

Therefore, we need to show that


W

b2 + c 2 + 4bc + 2a b + 2ac ≤ 6.

This is true since

12 − 2(b2 + c 2 + 4bc + 2a b + 2ac) = 3(a + b + c)2 − 2(b2 + c 2 + 4bc + 2a b + 2ac)

= 2a2 + b2 + c 2 − 2bc + 2a b + 2ac = 2a(a + b + c) + (b − c)2 ≥ 0.


The equality holds for a = 0 and b = c = 1 (or any cyclic permutation).
Second Solution. Let us denote

F (a, b, c) = (a2 + b2 )(b2 + c 2 )(c 2 + a2 ).


180 Vasile Cîrtoaje

We will show that


F (a, b, c) ≤ F (0, b + a/2, c + a/2) ≤ 2.
The left inequality,

(a2 + b2 )(b2 + c 2 )(c 2 + a2 ) ≤ (b + a/2)2 [(b + a/2)2 + (c + a/2)2 ](c + a/2)2 ,

is true since
a2 + b2 ≤ (b + a/2)2 ,
b2 + c 2 ≤ (b + a/2)2 + (c + a/2)2 ,
c 2 + a2 ≤ (c + a/2)2 .

L
The right inequality holds if the original inequality holds for a = 0; that is,

.M
b2 c 2 (b2 + c 2 ) ≤ 2

D
for b + c = 2. Indeed, by virtue of the AM-GM inequality, we have

A
‹2
b+c PI

bc ≤ = 1,
2
M
2
2bc + (b2 + c 2 )

LY

2 2 2 2 2 2
2b c (b + c ) ≤ 2bc(b + c ) ≤
2
O

(b + c)4
= = 4.
4
.M
W

P 3.13. Let a, b, c be nonnegative real numbers such that a + b + c = 2. Prove that


W

(a3 + b3 )(b3 + c 3 )(c 3 + a3 ) ≤ 2.


W

Solution. Due to symmetry, we may assume that a = min{a, b, c}. It is easy to check
that the equality holds for a = 0 and b = c = 1. Write the inequality as
”Y — ”Y —
(a + b) (a2 − a b + b2 ) ≤ 2.

Since Y
(a + b) ≤ (a + b + c)(a b + bc + ca) = 2(a b + bc + ca),
it suffices to show that
Y
(a b + bc + ca) (a2 − a b + b2 ) ≤ 1.
Symmetric Polynomial Inequalities in Nonnegative Variables 181

Since
a2 − a b + b2 ≤ b2
and
c 2 + ca + a2 ≤ c 2 ,
it suffices to show that

b2 c 2 (a b + bc + ca)(b2 − bc + c 2 ) ≤ 1.

In virtue of the AM-GM inequality, we have


4
bc + bc + (a b + bc + ca) + (b2 − bc + c 2 )


L
2 2 2 2
b c (a b + bc + ca)(b − bc + c ) ≤ .

.M
4

Therefore, it remains to show that

D
b2 + c 2 + 2bc + a b + ca ≤ 4.

A
This is true since PI
4 − (b2 + c 2 + 2bc + a b + ca) = (a + b + c)2 − (b2 + c 2 + 2bc + a b + ca)
M

= a(a + b + c) ≥ 0.
LY

The equality holds for a = 0 and b = c = 1 (or any cyclic permutation).


O
.M

P 3.14. Let a, b, c be nonnegative real numbers such that a2 + b2 + c 2 = 2. Prove that


W

(a3 + b3 )(b3 + c 3 )(c 3 + a3 ) ≤ 2.


W

(Vasile Cîrtoaje, 2011)


W

Solution. Let x = a2 , y = b2 , z = c 2 , where x + y + z = 2. Since

(a3 + b3 )2 ≤ (a2 + b2 )(a4 + b4 ) = (x + y)(x 2 + y 2 ),

it suffices to prove that

(x + y)( y + z)(z + x)(x 2 + y 2 )( y 2 + z 2 )(z 2 + x 2 ) ≤ 4.

Due to symmetry, we may assume that x = min{x, y, z}. It is easy to check that the
equality holds for x = 0 and y = z = 1. Since

(x + y)( y + z)(z + x) ≤ (x + y + z)(x y + yz + z x) = 2(x y + yz + z x)


182 Vasile Cîrtoaje

and
x 2 + y 2 ≤ y(x + y), z 2 + x 2 ≤ z(x + z),
it suffices to show that

yz(x y + yz + z x)(x + y)(x + z)( y 2 + z 2 ) ≤ 2.

Write this inequality as

[2 yz][2(x y + yz + z x)][2(x + y)(x + z)]( y 2 + z 2 ) ≤ 16.

By the AM-GM inequality, it suffices to show that

L
4
2 yz + 2(x y + yz + z x) + 2(x + y)(x + z) + ( y 2 + z 2 )


.M
≤ 16.
4

This inequality is equivalent to

D
A
2x 2 + y 2 + z 2 + 6 yz + 4x y + 4z x ≤ 8,
PI
2x 2 + y 2 + z 2 + 6 yz + 4x y + 4z x ≤ 2(x + y + z)2 ,
M
= ( y − z)2 ≥ 0.
The equality holds for a = 0 and b = c = 1 (or any cyclic permutation).
LY
O
.M

P 3.15. If a, b, c are nonnegative real numbers such that a + b + c = 2, then

(3a2 − 2a b + 3b2 )(3b2 − 2bc + 3c 2 )(3c 2 − 2ca + 3a2 ) ≤ 36.


W

(Vasile Cîrtoaje, 2011)


W

Solution. Due to symmetry, assume that a = min{a, b, c}. Under this assumption, we
can check that the equality holds for (a, b, c) = (0, 1, 1). Since
W

0 ≤ 3a2 − 2a b + 3b2 ≤ b(a + 3b)

and
0 < 3c 2 − 2ca + 3a2 ≤ c(a + 3c),
it suffices to show that

bc(a + 3b)(a + 3c)(3b2 − 2bc + 3c 2 ) ≤ 36.

Write this inequality as

[4b(a + 3c)][4c(a + 3b)][3(3b2 − 2bc + 3c 2 )] ≤ 123 .


Symmetric Polynomial Inequalities in Nonnegative Variables 183

By virtue of the AM-GM inequality, it suffices to show that


3
4b(a + 3c) + 4c(a + 3b) + 3(3b2 − 2bc + 3c 2 )

≤ 123 .
3
This is equivalent to
9(b + c)2 + 4a(b + c) ≤ 36.
We have

36 − 9(b + c)2 − 4a(b + c) = 9(a + b + c)2 − 9(b + c)2 − 4a(b + c)


= a(9a + 14b + 14c) ≥ 0.

L
The equality holds for a = 0 and b = c = 1 (or any cyclic permutation).

.M
Remark. Similarly, we can prove the following more general statement.

D
2
• Let a, b, c be nonnegative real numbers. If ≤ k ≤ 2, then
3

A
4
PI
(a2 − ka b + b2 )(b2 − k bc + c 2 )(c 2 − kca + a2 ) ≤
27(2 + k)2
(a + b + c)6 ,
M
b c 3k
with equality for a = 0 and + =1+ (or any cyclic permutation).
c b 2
LY

2
Actually, this inequality holds for ≤ k ≤ 5.
3
O
.M

P 3.16. Let a, b, c be nonnegative real numbers such that a + b + c = 3. Prove that

(a2 − 4a b + b2 )(b2 − 4bc + c 2 )(c 2 − 4ca + a2 ) ≤ 3.


W

(Vasile Cîrtoaje, 2011)


W

Solution. Assume that a ≤ b ≤ c. It is easy to show that the equality holds for a = 0
and b2 + c 2 = 7bc.pIf c 2 − 4ca + a2 < 2
p0, then a −p4a b + b < 0 and b − 4bc + c < 0,
2 2 2
W

since b ≤ c < (2+ 3)a and c < (2+ 3)a ≤ (2+ 3)b. Therefore, there two non-trivial
cases when the left hand side of the inequality is nonnegative: when all three factors
are nonnegative and when a2 − 4a b + b2 ≤ 0, b2 − 4bc + c 2 ≤ 0, c 2 − 4ca + a2 ≥ 0.
Case 1: a2 − 4a b + b2 ≥ 0, b2 − 4bc + c 2 ≥ 0, c 2 − 4ca + a2 ≥ 0. Since a2 − 4a b + b2 ≤ b2
and c 2 − 4ca + a2 ≤ c 2 , it suffices to prove that

b2 c 2 (b2 − 4bc + c 2 ) ≤ 3.

In virtue of the AM-GM inequality, we have


1
b2 c 2 (b2 − 4bc + c 2 ) = (3bc)(3bc)(b2 − 4bc + c 2 )
9
184 Vasile Cîrtoaje

3
1 3bc + 3bc + (b2 − 4bc + c 2 )


9 3
‹6
b + c) a + b + c) 6
  ‹
=3 ≤3 = 3.
3 3

Case 2: a2 − 4a b + b2 ≤ 0, b2 − 4bc + c 2 ≤ 0, c 2 − 4ca + a2 ≥ 0. We have


p
a ≥ (2 − 3)b.

Since

L
(4a b − a2 − b2 )(4bc − b2 − c 2 )(c 2 − 4ca + a2 ) ≤

.M
3
(4a b − a2 − b2 ) + (4bc − b2 − c 2 ) + (c 2 − 4ca + a2 )


3

D
8

A
= (2a b + 2bc − 2ca − b2 )3 ,
27
it suffices to prove that
PI
M
81 1
(2a b + 2bc − 2ca − b2 )3 ≤ = (a + b + c)6 ,
8 72
LY

which is equivalent to
O

p
3
2 9(2a b + 2bc − 2ca − b2 ) ≤ (a + b + c)2 .
.M

p
3 21
Since 2 9 < , it suffices to show that
5
W

21(2a b + 2bc − 2ca − b2 ) ≤ 5(a + b + c)2 .


W

Write this inequality as f (a) ≥ 0, where


W

f (a) = 5a2 + 4(13c − 8b)a + 26b2 + 5c 2 − 32bc.

p b2
From a ≥ (2 − 3)b, we get 4a > b, 5a2 > , and hence
20

b2 1
f (a) > + (13c − 8b)b + 26b2 + 5c 2 − 32bc = (19b − 10c)2 ≥ 0.
20 20
b c
This completes the proof. The equality holds for a = 0 and + = 7 (or any cyclic
c b
permutation).
Symmetric Polynomial Inequalities in Nonnegative Variables 185

P 3.17. If a, b, c are positive real numbers such that a + b + c = 3, then


12
a bc + ≥ 5.
a b + bc + ca

Solution. By the third degree Schur’s inequality

(a + b + c)3 + 9a bc ≥ 4(a + b + c)(a b + bc + ca),

we get 3a bc ≥ 4(a b + bc + ca) − 9. Thus, it suffices to prove that


36

L
4(a b + bc + ca) − 9 + ≥ 15.
a b + bc + ca

.M
This inequality is equivalent to

D
(a b + bc + ca − 3)2 ≥ 0.

A
The equality holds for a = b = c = 1.
PI
M
P 3.18. If a, b, c are positive real numbers such that a2 + b2 + c 2 = 3, then
LY

3
5(a + b + c) + ≥ 18.
O

a bc
.M

Solution. Let x = (a + b + c)/3. From

2(a b + bc + ca) = (a + b + c)2 − (a2 + b2 + c 2 ) = 3(3x 2 − 1) > 0,


W

p
we get x > 1/ 3. By the known inequality
W

(a b + bc + ca)2 ≥ 3a bc(a + b + c),


W

we get
1 4x
≥ .
a bc (3x − 1)2
2

Then, it suffices to prove that


4x
5x + ≥ 6,
(3x 2− 1)2
which is equivalent to

15x 5 − 18x 4 − 10x 3 + 12x 2 + 3x − 2 ≥ 0,


186 Vasile Cîrtoaje

(x − 1)2 (15x 3 + 12x 2 − x − 2) ≥ 0.


p
We still have to show that 15x 3 + 12x 2 − x − 2 ≥ 0. Since x > 1/ 3, we get

1 2 p
15x 3 + 12x 2 − x − 2 > x 2 (12 − − 2 ) > x 2 (12 − 3 − 6) > 0.
x x
The equality holds for a = b = c = 1.

P 3.19. If a, b, c are positive real numbers such that a2 + b2 + c 2 = 3, then

L
.M
12 + 9a bc ≥ 7(a b + bc + ca).

D
(Vasile Cîrtoaje, 2005)

A
Solution. Let x = (a + b + c)/3. Since
PI
2(a b + bc + ca) = (a + b + c)2 − (a2 + b2 + c 2 ) = 3(3x 2 − 1),
M
we can write the inequality as
LY

5 + 2a bc ≥ 7x 2 .
By Schur’s inequality of degree three
O

(a + b + c)3 + 9a bc ≥ 4(a + b + c)(a b + bc + ca),


.M

we get
W

3x 3 + a bc ≥ 2x(3x 2 − 1),

a bc ≥ 3x 3 − 2x.
W

Then,
W

5 + 2a bc − 7x 2 ≥ 5 + 2(3x 3 − 2x) − 7x 2 = (x − 1)2 (6x + 5) ≥ 0.


The equality holds for a = b = c = 1.

P 3.20. If a, b, c are positive real numbers such that a2 + b2 + c 2 = 3, then

21 + 18a bc ≥ 13(a b + bc + ca).

(Vasile Cîrtoaje, 2005)


Symmetric Polynomial Inequalities in Nonnegative Variables 187

Solution. Let p = a + b + c and q = a b + bc + ca. From

2q = (a + b + c)2 − (a2 + b2 + c 2 ) = p2 − 3 > 0,


p
we get p > 3. In addition, from Schur’s inequality of degree four, written in the form

(p2 − q)(4q − p2 )
a bc ≥ ,
6p
we get
(p2 + 3)(p2 − 6)
a bc ≥ .
12p

L
Therefore,

.M
3(p2 + 3)(p2 − 6) 13(p2 − 3)
21 + 18a bc − 13(a b + bc + ca) ≥ 21 + −
2p 2

D
(p − 3) (3p + 5p − 6)
2 2

A
= ≥ 0.
2p
The equality holds for a = b = c = 1.
PI
M
LY

P 3.21. If a, b, c are positive real numbers such that a2 + b2 + c 2 = 3, then


O

(2 − a b)(2 − bc)(2 − ca) ≥ 1.


.M

(Vasile Cîrtoaje, 2005)

First Solution. Let p = a + b + c. From 3(a2 + b2 + c 2 ) ≥ (a + b + c)2 , we get p ≤ 3.


W

Since
W

(2 − a b)(2 − bc)(2 − ca) = 8 − 4(a b + bc + ca) + 2a bc(a + b + c) − a2 b2 c 2


W

= 8 − 2(p2 − 3) + 2a bc p − a2 b2 c 2
= 14 − p2 − (p − a bc)2 ,

we can write the inequality as

13 − p2 − (p − a bc)2 ≥ 0.

Clearly,
3(p − a bc) = (a2 + b2 + c 2 )(a + b + c) − 3a bc > 0.
By Schur’s inequality

(a + b + c)3 + 9a bc ≥ 4(a + b + c)(a b + bc + ca),


188 Vasile Cîrtoaje

we get
p3 + 9a bc ≥ 2p(p2 − 3),

p(p2 − 6)
a bc ≥ .
9
Since
p(p2 − 6) p(15 − p2 )
0 < p − a bc ≤ p − = ,
9 9
it suffices to prove that
p2 (15 − p2 )2
13 − p2 − ≥ 0.

L
81
p

.M
Setting p = 3 x, 0 < x ≤ 1, this inequality becomes

13 − 34x + 30x 2 − 9x 3 ≥ 0.

D
A
It is true because
PI
13 − 34x + 30x 2 − 9x 3 = (1 − x)(13 − 21x + 9x 2 )
M
= (1 − x)[1 + 3(1 − x)(4 − 3x)] ≥ 0.
LY

The equality holds for a = b = c = 1.


O

Second Solution. We use the mixing variable method. Assume that a ≤ 1 and show
that
.M

(2 − a b)(2 − bc)(2 − ca) ≥ (2 − x 2 )(2 − a x)2 ≥ 1,


v v
t b2 + c 2 t 3 − a2 1 3
W

where x = = . Since 2 − bc ≥ 2 − (b2 + c 2 ) ≥ 2 − > 0 and,


2 2 2 2
similarly, 2 − ca > 0, 2 − a b > 0, we can prove the left inequality by multiplying the
W

inequalities
2 − bc ≥ 2 − x 2
W

and
(2 − ca)(2 − a b) ≥ (2 − a x)2 .

The last inequality is true because

(2 − ca)(2 − a b) − (2 − a x)2 = 2a(2x − b − c) − a2 (x 2 − bc)


2a(b − c)2 a2 (b − c)2
= −
2x + b + c 2
a(b − c)2 [4 − a(2x + b + c)]
=
2(2x + b + c)
Symmetric Polynomial Inequalities in Nonnegative Variables 189

and
p
4 − a(2x + b + c) ≥ 4(1 − a x) = 2(2 − a 6 − 2a2 )
4(1 − a2 )(2 − a2 )
= p ≥ 0.
2 + a 6 − 2a2
The right inequality (2 − x 2 )(2 − a x)2 ≥ 1 is equivalent to

(1 + a2 )(2 − a x)2 ≥ 2.

Since 2(1 + a2 ) ≥ (1 + a)2 , it suffices to show that

L
(1 + a)(2 − a x) ≥ 2.

.M
Indeed,

D
a(a4 + 2a3 − 2a2 − 6a + 5)
(1 + a)(2 − a x) − 2 = a(2 − x − a x) =

A
2(2 + x + a x)
a(a − 1)2 (a2 + 4a + 5)
PI
= ≥ 0.
2(2 + x + a x)
M
LY

P 3.22. Let a, b, c be positive real numbers such that a bc = 1. Prove that


O

a + b + c 5 a2 + b2 + c 2
 ‹
.M

≥ .
3 3
W

First Solution. Write the inequality in the homogeneous form


W

(a + b + c)5 ≥ 81a bc(a2 + b2 + c 2 ).


W

Using to the known inequality

(a b + bc + ca)2 ≥ 3a bc(a + b + c),

it suffices to show that

(a + b + c)6 ≥ 27(a b + bc + ca)2 (a2 + b2 + c 2 ).

Setting p = a + b + c and q = a b + bc + ca, we have

(a + b + c)6 − 27(a b + bc + ca)2 (a2 + b2 + c 2 ) = p6 − 27q2 (p2 − 2q)


= (p2 − 3q)2 (p2 + 6q) ≥ 0.
190 Vasile Cîrtoaje

The equality occurs for a = b = c = 1.

Second Solution. Use the mixing variable method. We show that

b+c b+c
E(a, b, c) ≥ E(a, , ) ≥ 0,
2 2
where
E(a, b, c) = (a + b + c)5 − 81a bc(a2 + b2 + c 2 ).
Indeed, we have

1 b+c b+c

L
[E(a, b, c) − E(a, , )] =
81 2 2

.M
b+c 2 b+c 2
= a3 [( ) − bc] + a[2( ) − bc(b2 + c 2 )]
2 2

D
1 1
= a3 (b − c)2 + a(b − c)4 ≥ 0

A
4 8
and PI
b+c b+c
M
E(a, , )=
2 2
LY

81
= (a + b + c)5 − a(b + c)2 [2a2 + (b + c)2 ]
8
1
O

= (2a − b − c)2 [2a3 + 12a2 (b + c) − 9a(b + c)2 + 8(b + c)3 ] ≥ 0,


8
.M

since

2a3 + 12a2 (b + c) − 9a(b + c)2 + 8(b + c)3 ] >


W

> 6a2 (b + c) − 12a(b + c)2 + 6(b + c)3


W

= 6(b + c)(a − b − c)2 ≥ 0.


W

P 3.23. If a, b, c are positive real numbers such that a bc = 1, then

a3 + b3 + c 3 + a−3 + b−3 + c −3 + 21 ≥ 3(a + b + c)(a−1 + b−1 + c −1 ).

Solution. Since
 ‹ ‹
a b c b c a
a3 + b3 + c 3 + a−3 + b−3 + c −3 + 3 = + + + +
b c a a b c
Symmetric Polynomial Inequalities in Nonnegative Variables 191

and  ‹  ‹
−1 −1 −1 a b c b c a
(a + b + c)(a +b +c )= + + + + + + 3,
b c a a b c
we can write the desired inequality in the homogeneous form
 ‹ ‹  ‹  ‹
a b c b c a a b c b c a
+ + + + +9≥3 + + +3 + + ,
b c a a b c b c a a b c
or  ‹ ‹
a b c b c a
+ + −3 + + − 3 ≥ 0.
b c a a b c
This is true since, by the AM-GM inequality,

L
.M
a b c b c a
+ + ≥ 3, + + ≥ 3.
b c a a b c

D
The equality holds for a = b = c = 1.

A
PI
P 3.24. If a, b, c are positive real numbers such that a bc = 1, then
M
9
LY

a2 + b2 + c 2 − a b − bc − ca ≥ (a + b + c − 3).
4
O

Solution. Write the inequality in the form


.M

3(4x 2 − 3x + 3) ≥ 4(a b + bc + ca),


p
W

3
where x = (a + b + c)/3. By the AM-GM inequality, we have x ≥ a bc = 1. The third
degree Schur’s inequality states that
W

(a + b + c)3 + 9a bc ≥ 4(a + b + c)(a b + bc + ca),


W

which is equivalent to
3(3x 3 + 1)
4(a b + bc + ca) ≤ .
x
Therefore, it suffices to show that

3(3x 3 + 1)
3(4x 2 − 3x + 3) ≥ .
x

This inequality reduces to (x − 1)3 ≥ 0, which is obviously true for x ≥ 1. The equality
holds for a = b = c = 1.
192 Vasile Cîrtoaje

P 3.25. If a, b, c are positive real numbers such that a bc = 1, then

a2 + b2 + c 2 + a + b + c ≥ 2(a b + bc + ca).

p = a + b + c and q = a b + bc + ca. By virtue of the AM-GM inequality,


Solution. Let p
3
we have p ≥ 3 a bc = 3, and by Schur’s inequality

p3 + 9a bc ≥ 4pq,

we get
p3 + 9

L
4q ≤ .
p

.M
Therefore,

D
p3 + 9
a2 + b2 + c 2 + a + b + c − 2(a b + bc + ca) = p2 + p − 4q ≥ p2 + p −

A
p
(p − 3)(p + 3)
PI
= ≥ 0.
p
M
The equality holds for a = b = c = 1.
LY
O

P 3.26. If a, b, c are positive real numbers such that a bc = 1, then


.M

a2 + b2 + c 2 + 15(a b + bc + ca) ≥ 16(a + b + c).


W

Solution. Write the inequality as F (a, b, c) ≥ 0, where


W

1 1 1
 ‹
F (a, b, c) = a2 + b2 + c 2 + 15 + + − 16(a + b + c).
W

a b c

Assume that a ≥ b ≥ c, and denote


p
t = bc, 0 < t ≤ 1, at 2 = 1.

We will show that


F (a, b, c) ≥ F (a, t, t) ≥ 0.
Since
1 1 2
 ‹
F (a, b, c) − F (a, t, t) = b2 + c 2 − 2t 2 + 15 + − − 16(b + c − 2t)
b c t
Symmetric Polynomial Inequalities in Nonnegative Variables 193

1 1 2 p
 ‹ p
= (b − c)2 + 15 p − p − 16( b − c)2
b c
p p 15 p 15
p • p ˜ p  p ‹
= ( b − c)2 ( b + c)2 + − 16 ≥ ( b − c)2 4 bc + − 16 ,
bc bc
it suffices to show that
15
4t + − 16 ≥ 0.
t2
Indeed,
15 15 (1 − t)(15 − t)
4t + − 16 > t + − 16 = ≥ 0.
t2 t t

L
The inequality F (a, t, t) ≥ 0 is equivalent to

.M
(t − 1)2 (17t 4 + 2t 3 − 13t 2 + 2t + 1) ≥ 0.

D
We have

A
17t 4 + 2t 3 − 13t 2 + 2t + 1 = (2t − 1)4 + t(t 3 + 34t 2 − 37t + 10)
t
PI
= (2t − 1)4 + [t(2t − 1)2 + 140t 2 − 149t + 40] > 0
M
4
since D = 1492 − 4 · 140 · 40 = −199. The equality holds for a = b = c = 1.
LY
O

P 3.27. If a, b, c are positive real numbers such that a bc = 1, then


.M

2 1 3
+ ≥ .
a+b+c 3 a b + bc + ca
W
W

Solution. Let x = (a b + bc + ca)/3. By virtue of the AM-GM inequality, we have


W

p
3
x ≥ a b · bc · ca = 1.

The third degree Schur’s inequality applied to a b, bc, ca, states that

(a b + bc + ca)3 + 9a2 b2 c 2 ≥ 4a bc(a + b + c)(a b + bc + ca),

which is equivalent to
3 4x
≥ .
a+b+c 3x 3 + 1
Therefore,
2 1 3 8x 3
 ‹
3 + − ≥ + 1 −
a + b + c 3 a b + bc + ca 3x 3 + 1 x
194 Vasile Cîrtoaje

3x 4 − 9x 3 + 8x 2 + x − 3 (x − 1)(3x 3 − 6x 2 + 2x + 3)
= = .
x(3x 3 + 1) x(3x 3 + 1)
Since x ≥ 1, we need to show that 3x 3 − 6x 2 + 2x + 3 ≥ 0. For x ≥ 2, we have

3x 3 − 6x 2 + 2x + 3 > 3x 3 − 6x 2 = 3x 2 (x − 2) ≥ 0,

and for 1 ≤ x < 2, we have

3x 3 − 6x 2 + 2x + 3 = 3x(x − 1)2 + 3 − x > 0.

The equality holds for a = b = c = 1.

L
D
.M
P 3.28. If a, b, c are positive real numbers such that a bc = 1, then

A
6
a b + bc + ca + ≥ 5.
PI
a+b+c
M
(Vasile Cîrtoaje, 2005)
LY

First Solution. Denoting x = (ab + bc + ca)/3, the inequality can be written as

(a + b + c)(3x − 5) + 6 ≥ 0.
O

In virtue of the AM-GM inequality, we get x ≥ 1. Since the inequality holds for x ≥ 5/3,
.M

consider next that 1 ≤ x < 5/3. Applying the third degree Schur’s inequality to the
numbers a b, bc and ca, we have
W

(a b + bc + ca)3 + 9a2 b2 c 2 ≥ 4a bc(a + b + c)(a b + bc + ca),


W

which is equivalent to
W

3(3x 3 + 1)
a+b+c ≤ .
4x
Having in view that 3x − 5 < 0, it suffices to prove that

3(3x 3 + 1)(3x − 5)
+ 6 ≥ 0.
4x
This inequality is equivalent to

9x 4 − 15x 3 + 11x − 5 ≥ 0,

(x − 1)(9x 3 − 6x 2 − 6x + 5) ≥ 0.
Symmetric Polynomial Inequalities in Nonnegative Variables 195

Since

9x 3 − 6x 2 − 6x + 5 > 9x 3 − 6x 2 − 6x + 3 = 3(x − 1)(3x 2 + x − 1) ≥ 0,

the conclusion follows. The equality holds for a = b = c = 1.

Second Solution (by Vo Quoc Ba Can). Among a − 1, b − 1, c − 1 there are two with
the same sign. Due to symmetry, assume that (b − 1)(c − 1) ≥ 0; that is, b + c ≤ 1 + bc.
Then,
6 6 6a
≥ = 2 .
a+b+c a + 1 + bc a +a+1

L
On the other hand, using the AM-GM inequality yields

.M
p p 1
a b + bc + ca = a(b + c) + bc ≥ 2a bc + bc = 2 a + .
a

D
Therefore, it suffices to prove that

A
p 1
2 a+ + 2
6a
a a +a+1
≥ 5.
PI
M
p
Setting a = x, this inequality becomes as follows
LY

1 6x 2
2x + + ≥ 5,
O

x2 x4 + x2 + 1
.M

1 6x 2
2x + − 3 ≥ 2 − ,
x2 x4 + x2 + 1
W

(x − 1)2 (2x + 1) 2(x 2 − 1)2


≥ ,
x2 x4 + x2 + 1
W

(x − 1)2 (2x 5 − x 4 − 2x 3 − x 2 + 2x + 1) ≥ 0,
W

(x − 1)2 [x(x − 1)2 (2x 2 + 3x + 2) + 1] ≥ 0.

P 3.29. If a, b, c are positive real numbers such that a bc = 1, then


Æ
3
Æ
4
(1 + a)(1 + b)(1 + c) ≥ 4(1 + a + b + c).

(Pham Huu Duc, 2008)


196 Vasile Cîrtoaje

Solution. Since

(1 + a)(1 + b)(1 + c) = (1 + a + b + c) + (1 + a b + bc + ca)


Æ
≥ 2 (1 + a + b + c)(1 + a b + bc + ca),

it suffices to prove that

(1 + a b + bc + ca)2 ≥ 4(1 + a + b + c),

which is equivalent to
(1 + q)2 ≥ 4(1 + p),

L
where p = a + b + c, q = a b + bc + ca. Setting x = bc, y = ca, z = a b in Schur’s

.M
inequality
(x + y + z)3 + 9x yz ≥ 4(x + y + z)(x y + yz + z x),

D
we get

A
q3 + 9 ≥ 4pq.
Since
PI
M
q3 + 9
(1 + q)2 − 4(1 + p) ≥ (1 + q)2 − 4 −
LY

q
(q − 3)(2q + 3)
= ,
O

q
.M

it suffices to show that q ≥ 3. Indeed, by the AM-GM inequality, we have


3
p
q = a b + bc + ca ≥ 3 a2 b2 c 2 = 3.
W

The equality holds for a = b = c = 1


W
W

P 3.30. If a, b, c are positive real numbers, then

a6 + b6 + c 6 − 3a2 b2 c 2 ≥ 18(a2 − bc)(b2 − ca)(c 2 − a b).

(Vasile Cîrtoaje, 2007)

Solution. Due to homogeneity, we may assume that a bc = 1, when the inequality can
be written as

a6 + b6 + c 6 − 3 ≥ 18(a3 + b3 + c 3 − a3 b3 − b3 c 3 − c 3 a3 ).
Symmetric Polynomial Inequalities in Nonnegative Variables 197

Substituting a3 , b3 , c 3 by a, b, c, respectively, we need to show that a bc = 1 implies


F (a, b, c) ≥ 0, where

F (a, b, c) = a2 + b2 + c 2 − 3 − 18(a + b + c − a b − bc − ca).

To do this, we use the mixing variable method. Without loss of generality, assume that
a ≥ 1. We claim that p p
F (a, b, c) ≥ F (a, bc, bc) ≥ 0.
We have
p p p p p p
F (a, b, c) − F (a, bc, bc) = (b − c)2 − 18( b − c)2 + 18a( b − c)2

L
p p

.M
= (b − c)2 + 18(a − 1)( b − c)2 ≥ 0.
p
Also, putting bc = t, we have

D
A
p p 1 1 18 36
F (a, bc, bc) = F ( 2
, t, t) = 4 + 20t 2 − 3 − 2 − 36t +
t t PI t t
(t − 1)2 (2t − 1)2 (t + 1)(5t + 1)
= ≥ 0.
M
t4
The equality holds for a = b = c, and for a/2 = b = c (or any cyclic permutation).
LY
O
.M

P 3.31. If a, b, c are positive real numbers such that a + b + c = 3, then

1 1 1
+ 2 + 2 ≥ a2 + b2 + c 2 .
W

a 2 b c
W

(Vasile Cîrtoaje, 2006)


W

First Solution. Since


1 1 1 1 1 1
2
+ 2+ 2≥ + + ,
a b c a b bc ca
it suffices to prove that
1 1 1
+ + ≥ a2 + b2 + c 2 ,
a b bc ca
which is equivalent to
a bc(a2 + b2 + c 2 ) ≤ 3.
Let x = (a b + bc + ca)/3. From the known inequality

(a b + bc + ca)2 ≥ 3a bc(a + b + c),


198 Vasile Cîrtoaje

we get a bc ≤ x 2 . On the other hand, we have

a2 + b2 + c 2 = (a + b + c)2 − 2(a b + bc + ca) = 9 − 6x.

Then,
a bc(a2 + b2 + c 2 ) − 3 ≤ x 2 (9 − 6x) − 3 = −3(x − 1)2 (2x + 1) ≤ 0.

The equality holds for a = b = c = 1.

Second Solution. Since a + b + c = 3, we can write the inequality as

L
X 1
( 2 − a2 + 4a − 4) ≥ 0,

.M
a

which is equivalent to

D
X (1 − a)2 (1 + 2a − a2 )

A
≥ 0.
a2

p p
PI
Without loss of generality, assume that a = max{a, b, c}. We have two cases to consider.
Case 1: a ≤ 1 + 2. Since a, b, c ≤ 1 + 2, we have
M

1 + 2a − a2 ≥ 0, 1 + 2b − b2 ≥ 0, 1 + 2c − c 2 ≥ 0.
LY
O

Thus, the conclusion follows.


p p 2
Case 2: a > 1 + 2. Since b + c = 3 − a < 2 − 2 < , we have
.M

1 1
bc ≤ (b + c)2 < ,
W

4 9
W

and hence

1 1 1 1 1 2
W

2
+ 2+ 2> 2+ 2≥ > 18 > (a + b + c)2 > a2 + b2 + c 2 .
a b c b c bc

P 3.32. If a, b, c are positive real numbers such that a b + bc + ca = 3, then

a3 + b3 + c 3 + 7a bc ≥ 10.

(Vasile Cîrtoaje, 2005)


Symmetric Polynomial Inequalities in Nonnegative Variables 199

Solution. Let x = (a + b + c)/3. By the well-known inequality

(a + b + c)2 ≥ 3(a b + bc + ca),

we get x ≥ 1. Since

a3 + b3 + c 3 = 3a bc + (a + b + c)3 − 3(a + b + c)(a b + bc + ca)


= 3a bc + 27x 3 − 27x,

we can write the inequality as

10a bc + 27x 3 − 27x − 10 ≥ 0.

L
.M
4
For x ≥ , this inequality is true, since
3

D
16
27x 3 − 27x − 10 = 27x(x 2 − 1) − 10 ≥ 36( − 1) − 10 = 18.

A
9

For 1 ≤ x ≤
4
, we use Schur’s inequality
PI
3
M
(a + b + c)3 + 9a bc ≥ 4(a + b + c)(a b + bc + ca),
LY

which is equivalent to
O

a bc + 3x 3 − 4x ≥ 0.
.M

Therefore,

10a bc + 27x 3 − 27x − 10 ≥ 10(−3x 3 + 4x) + 27x 3 − 27x − 10


W

= (x − 1)[4 − 3x + 3(2 − x 2 )] ≥ 0.
W

The equality holds for a = b = c = 1.


W

P 3.33. If a, b, c are nonnegative real numbers such that a3 + b3 + c 3 = 3, then

a4 b4 + b4 c 4 + c 4 a4 ≤ 3.

(Vasile Cîrtoaje, 2003)

Solution. By virtue of the AM-GM inequality, we have

b3 + c 3 + 1 4 − a3
bc ≤ = .
3 3
200 Vasile Cîrtoaje

Then, we have
4b3 c 3 − a3 b3 c 3
b4 c 4 ≤ .
3
Similarly,
4c 3 a3 − a3 b3 c 3 4a3 b3 − a3 b3 c 3
c 4 a4 ≤ , a4 b4 ≤ .
3 3
Summing these inequalities, we obtain

4(a3 b3 + b3 c 3 + c 3 a3 )
a4 b4 + b4 c 4 + c 4 a4 ≤ − a3 b3 c 3 .
3

L
Thus, using the substitutions x = a3 , y = b3 , z = c 3 , it suffices to prove that

.M
4(x y + yz + z x) ≤ 3x yz + 9,

D
where x, y, z are nonnegative real numbers satisfying x + y + z = 3. This follows imme-

A
diately from Schur’s inequality

PI
4(x + y + z)(x y + yz + z x) ≤ 9x yz + (x + y + z)3 .
M
The equality holds for a = b = c = 1.
LY

Remark 1. Using the contradiction method, it is easy to prove the reverse statement.
• If a, b, c are nonnegative real numbers such that a4 b4 + b4 c 4 + c 4 a4 = 3, then
O

a3 + b3 + c 3 ≥ 3.
.M

Remark 2. The inequality in P 3.33 is a particular case of the following more general
statement (Vasile Cîrtoaje, 2003).
W

• Let a, b, c be nonnegative real numbers such that a + b + c = 3. If 0 < k ≤ k0 , where


W

ln 3
k0 = ≈ 1.355,
W

ln 9 − ln 4
then
a k b k + b k c k + c k a k ≤ 3.

P 3.34. If a, b, c are nonnegative real numbers, then

(a + 1)2 (b + 1)2 (c + 1)2 ≥ 4(a + b + c)(a b + bc + ca) + 28a bc.

(Vasile Cîrtoaje, 2011)


Symmetric Polynomial Inequalities in Nonnegative Variables 201

Solution. By the AM-GM inequality, we have

(a + 1)(b + 1)(c + 1) = (a bc + 1) + (a + b + c) + (a b + bc + ca)


p Æ
≥ 2 a bc + 2 (a + b + c)(a b + bc + ca).
Thus, it suffices to prove that
p Æ
[ a bc + (a + b + c)(a b + bc + ca)]2 ≥ (a + b + c)(a b + bc + ca) + 7a bc,

which can be written as

L
Æ
a bc(a + b + c)(a b + bc + ca) ≥ 3a bc.

.M
This is true since

D
(a + b + c)(a b + bc + ca) − 9a bc = a(b − c)2 + b(c − a)2 + c(a − b)2 ≥ 0.

A
The equality holds for a = b = c = 1.
PI
M
LY

P 3.35. If a, b, c are positive real numbers such that a + b + c = 3, then


O

1 + 8a bc ≥ 9 min{a, b, c}.
.M

(Vasile Cîrtoaje, 2007)

Solution. Without loss of generality, assume that a = min{a, b, c}, a ≤ 1. The inequality
W

becomes
1 + 8a bc ≥ 9a.
W

From (a − b)(a − c) ≥ 0, we get


W

bc ≥ a(b + c) − a2 = a(3 − a) − a2 = a(3 − 2a2 ).

Therefore,

1 + 8a bc − 9a ≥ 1 + 8a2 (3 − 2a2 ) − 9a = (1 − a)(1 − 4a)2 ≥ 0.

1 1 5
 ‹
The equality holds for a = b = c = 1, and also for (a, b, c) = , , or any cyclic
4 4 2
permutation.
202 Vasile Cîrtoaje

P 3.36. If a, b, c are positive real numbers such that a2 + b2 + c 2 = 3, then

1 + 4a bc ≥ 5 min{a, b, c}.

(Vasile Cîrtoaje, 2007)

Solution. Without loss of generality, assume that a = min{a, b, c}, a ≤ 1. The inequality
can be written as
1 + 4a bc ≥ 5a.
From (a2 − b2 )(a2 − c 2 ) ≥ 0, we get

L
p p
bc ≥ a b2 + c 2 − a2 = a 3 − 2a2 .

.M
Therefore, it suffices to prove that

D
p
4a2 3 − 2a2 ≥ 5a − 1.

A
We consider two cases. p p PI
Case 1: 0 < a ≤ 1/2. Since 3 − 2a2 ≥ 5/2 > 25/16, it is enough to show that
M
25 2
a ≥ 5a − 1.
4
LY

This inequality is equivalent to (5a − 2)2 ≥ 0.


O

Case 2: 1/2 < a ≤ 1. By squaring, the inequality can be restated as


.M

32a6 − 48a4 + 25a2 − 10a + 1 ≤ 0,

or
W

(1 − a)(32a5 + 32a4 − 16a3 − 16a2 + 9a − 1) ≥ 0.


W

It is true, since
32a5 + 32a4 − 16a3 − 16a2 + 9a − 1 =
W

= (2a − 1)(16a4 + 24a3 + 4a2 − 6a + 1) + a


> (2a − 1)(8a3 + 4a2 − 6a + 1) = (2a − 1)2 (4a2 + 4a − 1) > 0.
The equality holds for a = b = c = 1.

P 3.37. If a, b, c are positive real numbers such that a + b + c = a bc, then


p
(1 − a)(1 − b)(1 − c) + ( 3 − 1)3 ≥ 0.
Symmetric Polynomial Inequalities in Nonnegative Variables 203

Solution. Without loss of generality, assume that a ≥ b ≥ c. If the product (1 − a)(1 −


b)(1 − c) is nonnegative, the inequality is trivial. Otherwise, this product is positive for
either a > 1 > b ≥ c or a ≥ b ≥ c > 1. Since a > 1 > b ≥ c involves

0 = a + b + c − a bc > a(1 − bc) > 0,

which is a contradiction, it suffices to consider only the case a ≥ b ≥ c > 1. Setting


x = a − 1, y = b − 1, z = c − 1, we need to show that
p
x yz ≤ ( 3 − 1)3 ,
p
where x, y, z are positive real numbers satisfying x y + yz +z x + x yz = 2. Let t = 3
x yz,

L
t > 0. By the AM-GM inequality, we have

.M
Æ3
2 = x y + yz + z x + x yz ≥ 3 x 2 y 2 z 2 + x yz = 3t 2 + t 3 ,

D
and hence

A
t 3 + 3t 2 − 2 ≤ 0,
PI
(t + 1)(t 2 + 2t − 2) ≤ 0,
M
t 2 + 2t − 2 ≤ 0,
p
LY

t ≤ 3 − 1,
p
x yz ≤ ( 3 − 1)3 .
O

p
The equality holds for a = b = c = 3.
.M
W

P 3.38. If a, b, c are nonnegative real numbers such that a + b + c = 2, then


W

(a2 + bc)(b2 + ca)(c 2 + a b) ≤ 1.


W

(Vasile Cîrtoaje, 2005)

Solution. Without loss of generality, assume that a ≥ b ≥ c. Since


c
a2 + bc ≤ (a + )2
2
and
1 2
(b2 + ca)(c 2 + a b) ≤ (b + ca + c 2 + a b)2 ,
4
it suffices to show that
(2a + c)(b2 + c 2 + a b + ac) ≤ 4.
204 Vasile Cîrtoaje

Let
E(a, b, c) = (2a + c)(b2 + c 2 + a b + ac).
We will show that
E(a, b, c) ≤ E(a, b + c, 0) ≤ 4.
Indeed,
E(a, b, c) − E(a, b + c, 0) = c(b2 + c 2 + ac − 3a b) ≤ 0
and

E(a, b + c, 0) − 4 = 2a(a + b + c)(b + c) − 4

L
= 4a(2 − a) − 4 = −4(a − 1)2 ≤ 0.

.M
The equality occurs for (a, b, c) = (1, 1, 0) or any cyclic permutation.

D
A
P 3.39. If a, b, c are nonnegative real numbers, then
PI
M
(8a2 + bc)(8b2 + ca)(8c 2 + a b) ≤ (a + b + c)6 .
LY

Solution. We use the mixing variable method. Without loss of generality, assume that
O

a ≤ b ≤ c. Let x = (b + c)/2, x ≥ a, and


.M

E(a, b, c) = (8a2 + bc)(8b2 + ca)(8c 2 + a b) − (a + b + c)6 .

We will prove that


W

E(a, b, c) ≤ E(a, x, x) ≤ 0.
W

The left inequality is equivalent to


W

(8a2 + x 2 )(8x 2 + a x)2 ≥ (8a2 + bc)(8b2 + ca)(8c 2 + a b),

which follows by multiplying the obvious inequality

8a2 + x 2 ≥ 8a2 + bc

and
(8x 2 + a x)2 ≥ (8b2 + ca)(8c 2 + a b).
Write the last inequality as

64(x 4 − b2 c 2 ) + a2 (x 2 − bc) − 8a(b3 + c 3 − 2x 3 ) ≥ 0.


Symmetric Polynomial Inequalities in Nonnegative Variables 205

Since
3(b + c)(b − c)2
b3 + c 3 − 2x 3 = = 6x(x 2 − bc) ≥ 0,
4
we need to show that
64(x 2 + bc) + a2 − 48a x ≥ 0.
This is true, since

64(x 2 + bc) + a2 − 48a x ≥ 64x 2 − 48a x ≥ 48x(x − a) ≥ 0.

The right inequality E(a, x, x) ≤ 0 is equivalent to

(8a2 + x 2 )(8x 2 + a x)2 − (a + 2x)6 ≤ 0,

L
.M
176x 5 − 273a x 4 + 32a2 x 3 + 52a3 x 2 + 12a4 x + a5 ≥ 0,
(x − a)2 (176x 3 + 79a x 2 + 14a2 x + a3 ) ≥ 0,

D
the last being clearly true. The equality holds for a = b = c, and for a = 0 and b = c

A
(or any cyclic permutation).
PI
M
P 3.40. If a, b, c are positive real numbers such that a2 b2 + b2 c 2 + c 2 a2 = 3, then
LY

a + b + c ≥ a bc + 2.
O

(Vasile Cîrtoaje, 2006)


.M

Solution. Without loss of generality,


p assume that a ≥ b ≥ c. From a2 b2 + b2 c 2 + c 2 a2 =
3, it follows that 1 ≤ a b < 3 and
W

v v
t 3 − a2 b2 t 3 − a2 b2
c= ≤ .
a2 + b2
W

2a b
We have
W

a + b + c − a bc − 2 = a + b − 2 − (a b − 1)c
v
p t 3 − a2 b2
≥ 2 a b − 2 − (a b − 1)
2a b
– v ™
p p t 3 − a2 b2
= ( a b − 1) 2 − ( a b + 1) .
2a b

So, we need to prove that


v
p t 3 − a2 b2
2 ≥ ( a b + 1) .
2a b
206 Vasile Cîrtoaje

This inequality is true, since


v v
p t 3 − a2 b2 p p t 3 − a2 b2
( a b + 1) − 2 ≤ ( a b + a b) −2
2ab 2a b
p
= 6 − 2a2 b2 − 2
2(1 − a2 b2 )
=p ≤ 0.
6 − 2a2 b2 + 2
The equality holds for a = b = c = 1.

L
.M
P 3.41. Let a, b, c be nonnegative real numbers such that a + b + c = 5. Prove that

D
(a2 + 3)(b2 + 3)(c 2 + 3) ≥ 192.

A
PI
First Solution. Without loss of generality, assume that a = min{a, b, c}, a ≤
5
. By
3
M
virtue of the Cauchy-Schwarz inequality, we have
LY

(b2 + 3)(c 2 + 3) = (b2 + 3)(3 + c 2 ) ≥ 3(b + c)2 = 3(5 − a)2 .


O

Therefore, it suffices to sow that


.M

(a2 + 3)(5 − a)2 ≥ 64

5
for 0 ≤ a ≤ . Indeed,
W

3
W

(a2 + 3)(5 − a)2 − 64 = (a − 1)2 (a2 − 8a + 11) ≥ 0,


W

since
5 19 4
 ‹ ‹
2
a − 8a + 11 = −a − a + > 0.
3 3 9
The equality holds for a = 3 and b = c = 1 (or any cyclic permutation).
Second Solution. Without loss of generality, assume that a = ma x{a, b, c}. First, we
show that
(b2 + 3)(c 2 + 3) ≥ (x 2 + 3)2 ,
b+c 5
where x = , 0 ≤ x ≤ . This inequality is equivalent to
2 3

(b − c)2 (6 − bc − x 2 ) ≥ 0,
Symmetric Polynomial Inequalities in Nonnegative Variables 207

which is true because


6 − bc − x 2 ≥ 2(3 − x 2 ) > 0.
Thus, it suffices to prove that

(a2 + 3)(x 2 + 3)2 ≥ 192,

which is equivalent to
[(5 − 2x)2 + 3](x 2 + 3)2 ≥ 192,
(x 2 − 5x + 7)(x 2 + 3)2 ≥ 48,
(x − 1)2 (x 4 − 3x 3 + 6x 2 − 15x + 15) ≥ 0.

L
This inequality is true since

.M
‹2 2
3
 x
4 3 2 2
x − 3x + 6x − 15x + 15 = x x− + 15 −1 > 0.
2 2

D
A
P 3.42. If a, b, c are nonnegative real numbers, then
PI
M
a2 + b2 + c 2 + a bc + 2 ≥ a + b + c + a b + bc + ca.
LY

(Michael Rozenberg, 2012)


O

Solution. Among the numbers 1− a, 1− b and 1− c there are always two with the same
sign; let us say (1 − b)(1 − c) ≥ 0. Thus, it suffices to show that
.M

a2 + b2 + c 2 + a(b + c − 1) + 2 ≥ a + b + c + a b + bc + ca,

which is equivalent to
W

a2 − 2a + b2 + c 2 − bc − (b + c) + 2 ≥ 0.
W

Since
W

1
b2 + c 2 − bc ≥ (b + c)2 ,
4
it suffices to show that
1
a2 − 2a + (b + c)2 − (b + c) + 2 ≥ 0,
4
which can be written in the obvious form
‹2
b+c

(a − 1)2 + − 1 ≥ 0.
2
The equality holds for a = b = c = 1.
208 Vasile Cîrtoaje

P 3.43. If a, b, c are nonnegative real numbers, then


X
a3 (b + c)(a − b)(a − c) ≥ 3(a − b)2 (b − c)2 (c − a)2 .

Solution. Without loss of generality, assume that a = min{a, b, c}. Since

a3 (b + c)(a − b)(a − c) ≥ 0

and
b3 (c + a)(b − c)(b − a) + c 3 (a + b)(c − a)(c − b) =

L
.M
(b − c)[bc(b3 − c 3 ) + (b − c)(b3 + c 3 )a − (b3 − c 3 )a2 ]

= (b − c)2 [(b2 + bc + c 2 )(bc − a2 ) + (b3 + c 3 )a]

D
A
≥ (b − c)2 (b2 + bc + c 2 )(bc − a2 ),

it suffices to show that PI


M
(b2 + bc + c 2 )(bc − a2 ) ≥ 3(a − b)2 (c − a)2 .
LY

Since
bc − a2 = (a − b)(a − c) + a(b + c − 2a) ≥ (a − b)(a − c),
O

it suffices to show that


.M

b2 + bc + c 2 ≥ 3(a − b)(a − c),

which is equivalent to the obvious inequality


W

(b − c)2 + 3a(b + c − a) ≥ 0.
W

The equality holds for a = b = c, for a = 0 and b = c (or any cyclic permutation), and
W

for b = c = 0 (or any cyclic permutation).

P 3.44. Find the greatest real number k such that

a + b + c + 4a bc ≥ k(a b + bc + ca)

for all a, b, c ∈ [0, 1].


Symmetric Polynomial Inequalities in Nonnegative Variables 209

Solution. Setting a = b = c = 1, we get k ≤ 7/3, but setting a = 0 and b = c = 1, we


get k ≤ 2. So, we claim that k = 2 is the greatest real number k. To prove this, we only
need to show that
a + b + c + 4a bc ≥ 2(a b + bc + ca)

for all a, b, c ∈ [0, 1]. Write the inequality as

a(1 + 4bc − 2b − 2c) + b + c − 2bc ≥ 0.

Since b + c − 2bc = b(1 − c) + c(1 − b) ≥ 0, the inequality is clearly true for 1 + 4bc −
2b − 2c ≥ 0. Consider further that 1 + 4bc − 2b − 2c < 0. It suffices to show that

L
(1 + 4bc − 2b − 2c) + b + c − 2bc ≥ 0,

.M
which is equivalent to the obvious inequality

D
bc + (1 − b)(1 − c) ≥ 0.

A
PI
Thus, the proof is completed. For k = 2, the equality holds when a = b = c = 0, and
when one of a, b, c is zero and the others are 1.
M
Remark. From the proof above it follows that the following stronger inequality holds
LY

for all a, b, c ∈ [0, 1]:


O

a + b + c + 3a bc ≥ 2(a b + bc + ca),
.M

with equality when a = b = c = 0, when a = b = c = 1, and when one of a, b, c is zero


and the others are 1.
W
W

2
P 3.45. If a, b, c ≥ such that a + b + c = 3, then
W

a2 b2 + b2 c 2 + c 2 a2 ≥ a b + bc + ca.

Solution. We use the mixing variable method. Assume that a = max{a, b, c} and denote
x = (b + c)/2. From a, b, c ≥ 2/3 and a + b + c = 3, it follows that 2/3 ≤ x ≤ 1. We
will show that
E(a, b, c) ≥ E(a, x, x) ≥ 0,

where
E(a, b, c) = a2 b2 + b2 c 2 + c 2 a2 − a b − bc − ca.
210 Vasile Cîrtoaje

We have

E(a, b, c) − E(a, x, x) = a2 (b2 + c 2 − 2x 2 ) − (x 4 − b2 c 2 ) + (x 2 − bc)


= (x 2 − bc)(2a2 − x 2 − bc + 1)
1
= (b − c)2 [a2 + (a2 − bc) + (1 − x 2 )] ≥ 0
4
and
E(a, x, x) = 2a2 x 2 + x 4 − 2a x − x 2 .
Since a + 2x = 3, we get

L
9E(a, x, x) = 18a2 x 2 + 9x 4 − (2a x + x 2 )(a + 2x)2

.M
= x(5x 3 − 12a x 2 + 9a2 x − 2a3 ) = x(x − a)2 (5x − 2a)
= 3x(x − a)2 (3x − 2) ≥ 0.

D
The equality holds for a = b = c = 1, and also for a = 5/3, b = 2/3, c = 2/3 (or any

A
cyclic permutation).
PI
M
LY

P 3.46. If a, b, c are positive real numbers such that a ≤ 1 ≤ b ≤ c and

a + b + c = 3,
O

then
.M

1 1 1
+ + ≥ a2 + b2 + c 2 .
a b c
W

Solution. Let
1 1 1
W

F (a, b, c) = + + − a2 − b2 − c 2 .
a b c
W

We will show that


F (a, b, c) ≥ F (a, 1, b + c − 1) ≥ 0.
The left inequality is true, since

F (a, b, c) − F (a, 1, b + c − 1) =
1 1 1
 ‹
= + −1− + 1 + (b + c − 1)2 − b2 − c 2
b c b+c−1
1 1
 ‹
= (b + c) − + 2(b − 1)(c − 1)
bc b + c − 1
b+c
• ˜
= (b − 1)(c − 1) 2 −
bc(b + c − 1)
Symmetric Polynomial Inequalities in Nonnegative Variables 211

and

2bc(b + c − 1) − b − c = (2bc − 1)(b + c) − 2bc


p
≥ 2(2bc − 1) bc − 2bc
p p p
= 2 bc( bc − 1)(2 bc + 1) ≥ 0.

The right inequality F (a, 1, b + c − 1) ≥ 0 is equivalent to F (a, 1, x) ≥ 0, where x > 0


and x + a = 2. We have

1 1 (x + a)4 (x − a)4
F (a, 1, x) = + − a2 − x 2 = − a2 − x 2 = ≥ 0.
a x 8a x 8a x

L
.M
The equality holds for a = b = c = 1.

D
A
P 3.47. If a, b, c are positive real numbers such that a ≤ 1 ≤ b ≤ c and

a+b+c =
1 1 1
+ + ,
PI
M
a b c
then
LY

1 1 1
a2 + b2 + c 2 ≤ 2
+ 2 + 2.
a b c
O

(Vasile Cîrtoaje, 2008)


.M

Solution. Write the inequality as

1 1
 ‹
b2 − ≤ (a 2
+ c 2
) − 1 .
W

b2 a2 c 2
W

1 1 1
From a + b + c = + + , we have
a b c
W

1 1
 ‹
b − = (a + c) − 1 ≥ 0.
b ac
Thus, the desired inequality holds if

1 1
(a + c)(b + ) ≤ (a2 + c 2 )( + 1).
b ac
1
On the other hand, from (b − c)(1 − ) ≤ 0, one gets
bc
1 1
b+ ≥c+ .
b c
212 Vasile Cîrtoaje

Then, it suffices to prove that


1 1
(a + c)(c + ) ≤ (a2 + c 2 )( + 1),
c ac
which is equivalent to
c(1 − a2 )(a − c) ≤ 0.
Since this is obvious, the proof is completed. The equality holds for b = 1 and ac = 1.

P 3.48. If a, b, c are positive real numbers such that

L
1 1 1
a+b+c = + + ,

.M
a b c
then
1 1 1
 ‹

D
(1 − a bc) a n + b n + c n − n − n − n ≥ 0
a b c

A
for any integer n ≥ 2.
PI (Vasile Cîrtoaje, 2007)
Solution. Since the statement remains unchanged by substituting a, b, c with 1/a, 1/b,
M
1/c, respectively, it suffices to prove that
LY

1 1 1
an + bn + c n −
n
− n − n ≤0
a b c
O

for a bc ≥ 1 and a + b + c = 1/a + 1/b + 1/c. It is easy to check that a + b + c =


1/a + 1/b + 1/c is equivalent to
.M

(a b − 1)(bc − 1)(ca − 1) = a2 b2 c 2 − 1,
W

and the desired inequality is equivalent to


(a n b n − 1)(b n c n − 1)(c n a n − 1) ≥ a2n b2n c 2n − 1.
W

Setting x = bc, y = ca, z = a b, we need to show that


W

(x − 1)( y − 1)(z − 1) = x yz − 1 ≥ 0
involves
(x n − 1)( y n − 1)(z n − 1) ≥ x n y n z n − 1.
This inequality holds if
(x n−1 + x n−2 + · · · + 1)( y n−1 + y n−2 + · · · + 1)(z n−1 + z n−2 + · · · + 1) ≥
≥ x n−1 y n−1 z n−1 + x n−2 y n−2 z n−2 + · · · + 1.
Since the last inequality is clearly true, the proof is completed. The equality occurs for
a = bc = 1, or b = ca = 1, or c = a b = 1.
Symmetric Polynomial Inequalities in Nonnegative Variables 213

P 3.49. Let a, b, c be positive real numbers, and let


E(a, b, c) = a(a − b)(a − c) + b(b − c)(b − a) + c(c − a)(c − b).
Prove that

(a) (a + b + c)E(a, b, c) ≥ a b(a − b)2 + bc(b − c)2 + ca(c − a)2 ;


1 1 1
 ‹
(b) 2 + + E(a, b, c) ≥ (a − b)2 + (b − c)2 + (c − a)2 .
a b c

Solution. (a) Using Schur’s inequality of degree four

L
X
a2 (a − b)(a − c) ≥ 0,

.M
we have

D
X X
(a + b + c)E(a, b, c) = a2 (a − b)(a − c) + a(b + c)(a − b)(a − c)

A
X
≥ a(b + c)(a − b)(a − c)
=
X
a b(a − b)(a − c) +
X PI
ac(a − b)(a − c)
M
X X
= a b(a − b)(a − c) + ba(b − c)(b − a)
LY

X
= a b(a − b)2 ≥ 0.
O

The equality holds for a = b = c. If a, b, c are nonnegative real numbers, then the
equality also holds for a = 0 and b = c (or any cyclic permutation).
.M

(b) Since
(a b + bc + ca)E(a, b, c) =
W

X X
= a bc (a − b)(a − c) + (a2 b + a2 c)(a − b)(a − c)
W

1 X X
= a bc (a − b)2 + [a2 b(a − b)(a − c) + b2 a(b − c)(b − a)]
2
W

1 X X
= a bc (a − b)2 + a b(a − b)2 (a + b − c),
2
the required inequality is equivalent to
X
a b(a − b)2 (a + b − c) ≥ 0.
Without loss of generality, assume that a ≥ b ≥ c. Then,
X
a b(a − b)2 (a + b − c) ≥ bc(b − c)2 (b + c − a) + ac(a − c)2 (a + c − b)
≥ bc(b − c)2 (b + c − a) + ac(b − c)2 (a + c − b)
= c(b − c)2 [(a − b)2 + c(a + b)] ≥ 0.
214 Vasile Cîrtoaje

The equality holds for a = b = c.

P 3.50. Let a ≥ b ≥ c be nonnegative real numbers. Schur’s inequalities of third and


fourth degree state that

(a) a(a − b)(a − c) + b(b − c)(b − a) + c(c − a)(c − b) ≥ 0;

(b) a2 (a − b)(a − c) + b2 (b − c)(b − a) + c 2 (c − a)(c − b) ≥ 0.

L
Prove that (a) is sharper than (b) if

.M
p p p
b+ c≤ a,

D
and (b) is sharper than (a) if

A
p p p
b+ c≥ a.
PI (Vasile Cîrtoaje, 2005)
M
Solution. Let p = a + b + c and q = a b + bc + ca. If we rewrite Schur’s inequalities
LY

above as
a bc ≥ f (p, q)
O

and
.M

a bc ≥ g(p, q),
respectively, then (a) is sharper than (b) if f (p, q) ≥ g(p, q), while (b) is sharper than
(a) if g(p, q) ≥ f (p, q). Therefore, we need to show that
W

p p p
( b + c − a)[g(p, q) − f (p, q)] ≥ 0.
W
W

From the known relation


p p p p p p p p p p p p
4q − p2 = ( a + b + c)( b + c − a)( c + a − b)( a + b − c),
p p p
it follows that 4q − p2 and b+ c− a has the same sign. Therefore, it suffices to
prove that
(4q − p2 )[g(p, q) − f (p, q)] ≥ 0.
In order to find f (p, q), write the inequality in (a) as follows

a3 + b3 + c 3 + 3a bc ≥ a b(a + b) + bc(b + c) + ca(c + a)


(a + b + c)3 + 9a bc ≥ 4(a + b + c)(a b + bc + ca),
Symmetric Polynomial Inequalities in Nonnegative Variables 215

from which
p(4q − p2 )
f (p, q) = .
9
Analogously, write the inequality in (b) as

a4 + b4 + c 4 + a bc(a + b + c) ≥ a b(a2 + b2 ) + bc(b2 + c 2 ) + ca(c 2 + a2 ),

a4 + b4 + c 4 + 2a bc(a + b + c) ≥ (a b + bc + ca)(a2 + b2 + c 2 ).

Since
a2 + b2 + c 2 = p2 − 2q

L
.M
and

a4 + b4 + c 4 = (a2 + b2 + c 2 )2 − 2(a2 b2 + b2 c 2 + c 2 a2 )

D
= (p2 − 2q)2 − 2q2 + 4a bc p,

A
we get PI
(p2 − q)(4q − p2 )
M
g(p, q) = .
6p
LY

Therefore, we have
(p2 − 3q)(4q − p2 )
g(p, q) − f (p, q) =
O

,
18p
.M

and hence
(p2 − 3q)(4q − p2 )2
(4q − p2 )[g(p, q) − f (p, q)] = ≥ 0.
18p
W

Remark. If a, b, c are the lengths of the sides of a triangle, then Schur’s inequality of
W

degree four is stronger than Schur’s inequality of degree three.


W

P 3.51. If a, b, c are nonnegative real numbers such that

(a + b)(b + c)(c + a) = 8,

then
p p p
a+ b+ c ≥ a b + bc + ca.

(Vasile Cîrtoaje, 2010)


216 Vasile Cîrtoaje

First Solution. Assume that a ≥ b ≥ c, and write the inequality in the equivalent ho-
mogeneous forms
p p p Æ p
( a + b + c) (a + b)(b + c)(c + a) ≥ 2 2(a b + bc + ca),
XÆ Æ Æ
a(b + c)[ (a + b)(a + c) − 2a(b + c)] ≥ 0,
p
X (a − b)(a − c) a(b + c)
p p ≥ 0.
(a + b)(a + c) + 2a(b + c)
Since (c − a)(c − b) ≥ 0, it suffices to prove that

L
p p
(a − b)(a − c) a(b + c) (b − c)(b − a) b(c + a)

.M
p p +p p ≥ 0,
(a + b)(a + c) + 2a(b + c) (b + c)(b + a) + 2b(c + a)

D
which is true if

A
p p
(a − c) a(b + c) (b − c) b(c + a)
p p
(a + b)(a + c) + 2a(b + c)
≥p p PI
(b + c)(b + a) + 2b(c + a)
.
M
p p
Since a≥ b,
LY

Æ Æ
(a + b)(a + c) ≥ 2a(b + c)
and
O

Æ Æ
(b + c)(b + a) ≤ 2b(a + c),
.M

it suffices to show that


p p
(a − c) b + c (b − c) c + a
W

p ≥p .
(a + b)(a + c) (b + c)(b + a)
W

This is equivalent to the obvious inequality


p c(a − b) ≥ 0. The equality holds for a = b =
3
W

c = 1, and for a = 0 and b = c = 4 (or any cyclic permutation).


Second Solution. Let p = a + b + c and q = a b + bc + ca. By squaring, the inequality
becomes p p p
p + 2( a b + bc + ca) ≥ q2 .
Since
p 2a b a b(b + c)(c + a) a b(q + c 2 )
ab ≥ = = ,
a+b 4 4
we have
p p p q(a b + bc + ca) + a bc(a + b + c) q2 + a bc p
2( a b + bc + ca) ≥ = .
2 2
Symmetric Polynomial Inequalities in Nonnegative Variables 217

Using this result, it suffices to prove that

q2 + a bc p
p+ ≥ q2 ,
2
or
p(2 + a bc) ≥ q2 .
According to the hypothesis pq − a bc = 8, we can write this inequality in the homoge-
neous forms
pq − a bc
 ‹
p + a bc ≥ q2 ,
4

L
p2 q + 3a bc p ≥ 4q2 .

.M
Since p2 ≥ 3q and p3 + 9a bc ≥ 4pq (Schur’s inequality), we have

D
p(p2 q + 3a bc p − 4q2 ) ≥ q(p3 + 9a bc − 4pq) ≥ 0.

A
PI
p
M
P 3.52. If a, b, c ∈ [1, 4 + 3 2], then
LY

9(a b + bc + ca)(a2 + b2 + c 2 ) ≥ (a + b + c)4 .


O

(Vasile Cîrtoaje, 2005)


.M

Solution. Let A = a2 + b2 + c 2 and B = a b + bc + ca. Since

9(a b + bc + ca)(a2 + b2 + c 2 ) − (a + b + c)4 = 9AB − (A + 2B)2


W

= (A − B)(4B − A)
W

and
2(A − B) = (a − b)2 + (b − c)2 + (c − a)2 ≥ 0,
W

we need to show that 4B − A ≥ 0; that is, to show that E(a, b, c) ≤ 0, where

E(a, b, c) = a2 + b2 + c 2 − 4(a b + bc + ca).


p
We claim that E(a, b, c) is maximal for a, b, c ∈ {1, w}, where w = 4+3 2. For the sake
of contradiction, assume that there exists a triple (a, b, c) with a ∈ (1, w) such that

E(a, b, c) ≥ max{E(1, b, c), E(w, b, c)}.

From
E(a, b, c) − E(1, b, c) = (a − 1)(a + 1 − 4b − 4c) ≥ 0,
218 Vasile Cîrtoaje

we get
4(b + c) − a ≤ 1,
and from
E(a, b, c) − E(w, b, c) = (a − w)(a + w − 4b − 4c) ≥ 0,
we get
4(b + c) − a ≥ w.
These results involve w ≤ 1, which is false. Therefore, since E(a, b, c) is symmetric, we
have
E(a, b, c) ≤ max{E(1, 1, 1), E(1, 1, w), E(1, w, w), E(w, w, w)}

L
= max{−9, w2 − 8w − 2, 1 − 2w2 − 8w, −9w2 } = w2 − 8w − 2 = 0.

.M
This completes
p the proof. The equality holds for a = b = c, and also for a = b = 1 and
c = 4 + 3 2 (or any cyclic permutation).

D
A
PI
P 3.53. If a, b, c are nonnegative real numbers such that a + b + c + a bc = 4, then
M
(a) a2 + b2 + c 2 + 12 ≥ 5(a b + bc + ca);
LY

(b) 3(a2 + b2 + c 2 ) + 13(a b + bc + ca) ≥ 48.


O
.M

Solution. Let p = a + b + c, q = a b + bc + ca, r = a bc.


(a) We need to show that p2 + 12 ≥ 7q for p + r = 4. By Schur’s inequality of degree
W

three, we have p3 + 9r ≥ 4pq. Therefore, we get


W

4p(p2 + 12 − 7q) ≥ 4p3 + 48p − 7(p3 + 9r) = −3(p3 − 37p + 84)


= 3(p − 3)(4 − p)(7 + p).
W

Since 4 − p = r ≥ 0, to complete the proof, we need to show that p ≥ 3. By virtue of


the AM-GM inequality, we get
p3 ≥ 27r,

p3 ≥ 27(4 − p),

(p − 3)(p2 + 3p + 36) ≥ 0,

p ≥ 3.
The equality holds for a = b = c = 1, and for a = 0 and b = c = 2 (or any cyclic
permutation).
Symmetric Polynomial Inequalities in Nonnegative Variables 219

(b) We need to show that 3p2 + 7q ≥ 48 for p + r = 4. Using the known inequality
pq ≥ 9r, we get

p(3p2 + 7q − 48) ≥ 3(p3 + 21r − 16p) = 3(p3 − 37p + 84)


= 3(p − 3)(4 − p)(7 + p) ≥ 0.

The equality holds for a = b = c = 1.

P 3.54. Let a, b, c be the lengths of the sides of a triangle. If a2 + b2 + c 2 = 3, then

L
.M
a b + bc + ca ≥ 1 + 2a bc.

(Vasile Cîrtoaje, 2005)

D
A
Solution. Write the inequality such that the left-hand side and right-hand side are ho-
mogeneous expressions
PI
3(a b + bc + ca) − a2 − b2 − c 2 ≥ 6a bc.
M
From
LY

3(a2 + b2 + c 2 ) ≥ (a + b + c)2 ,
we get a + b + c ≤ 3. Therefore, it suffices to prove the homogeneous inequality
O

(a + b + c)[3(a b + bc + ca) − a2 − b2 − c 2 ] ≥ 18a bc.


.M

This is equivalent to
W

2a b(a + b) + 2bc(b + c) + 2ca(c + a) ≥ a3 + b3 + c 3 + 9a bc.


W

Using the known substitutions a = y +z, b = z + x, c = x + y (x, y, z ≥ 0), the inequality


W

can be written as

x 3 + y 3 + z 3 + 3x yz ≥ x y(x + y) + yz( y + z) + z x(z + x),

which is just the third degree Schur’s inequality. The equality holds for an equilateral
triangle.

P 3.55. Let a, b, c be the lengths of the sides of a triangle. If a2 + b2 + c 2 = 3, then

a2 b2 + b2 c 2 + c 2 a2 ≥ a b + bc + ca.
220 Vasile Cîrtoaje

Solution. Write the inequality as follows:

9(a2 b2 + b2 c 2 + c 2 a2 ) ≥ (a b + bc + ca)(a + b + c)2 ;

3[3(a2 b2 + b2 c 2 + c 2 a2 ) − (a b + bc + ca)2 ] ≥

≥ (a b + bc + ca)[(a + b + c)2 − 3(a b + bc + ca)];

6[a2 (b − c)2 + b2 (c − a)2 + c 2 (a − b)2 ] ≥

≥ (a b + bc + ca)[(b − c)2 + (c − a)2 + (a − b)2 ] ≥ 0;

L
X
Sa (b − c)2 ≥ 0,

.M
where
Sa = 6a2 − a b − bc − ca.

D
A
Without loss of generality, assume that a ≥ b ≥ c. It suffices to show that
PI
S b (a − c)2 + Sc (a − b)2 ≥ 0.
M
Since
LY

(a − c)2 ≥ (a − b)2 ,

S b = 6b2 − bc − a(b + c) ≥ 6b2 − bc − (b + c)2 > 0


O

and
.M

S b + Sc = 6(b2 + c 2 ) − 2bc − 2a(b + c) ≥ 6(b2 + c 2 ) − 2bc − 2(b + c)2


W

= 4(b − c)2 + 2bc > 0,


W

we get
S b (a − c)2 + Sc (a − b)2 ≥ (S b + Sc )(a − b)2 ≥ 0.
W

The equality holds for an equilateral triangle.

P 3.56. Let a, b, c be the lengths of the sides of a triangle. If a + b + c = 3, then

1 1 1 41
+ + + ≥ 3(a2 + b2 + c 2 ).
a b c 6

(Vasile Cîrtoaje, 2010)


Symmetric Polynomial Inequalities in Nonnegative Variables 221

Solution (by Vo Quoc Ba Can). Using the substitutions


y +z z+x x+y
a= , b= , c= ,
2 2 2
where x, y, z ≥ 0 such that x + y + z = 3, the inequality becomes as follow

1 1 1 41 3
+ + + ≥ [( y + z)2 + (z + x)2 + (x + y)2 ],
y + z z + x x + y 12 8
X x + y + z 41 9 €X X Š
+ ≥ x2 + xy ,
y +z 4 4

L
X x 41 9 € X Š
+3+ ≥ 9− xy ,

.M
y +z 4 4
X x 9X
≥7− x y.

D
y +z 4

A
Let us denote t = x y + yz + z x. Since
X x
= ≥
PI
1 X x(x y + yz + z x) 1 X x(x y + z x)
y +z t y +z t y +z
M
1 X 2 9 − 2t
= x = ,
LY

t t
it suffices to show that
O

9 − 2t 9
≥ 7 − t,
t 4
.M

which is equivalent to
(t − 2)2 ≥ 0.
W

The equality holds for a degenerate triangle having a = 3/2, b = 1, c = 1/2 (or any
permutation thereof).
W
W

P 3.57. Let a ≤ b ≤ c such that a + b + c = p and a b + bc + ca = q, where p and q are


fixed nonnegative numbers satisfying p2 ≥ 3q.
(a) If a, b, c are nonnegative real numbers, then the product r = a bc is maximal when
a = b, and is minimal when b = c or a = 0;
(b) If a, b, c are the lengths of the sides of a triangle (non-degenerate or degenerate),
c
then the product r = a bc is maximal when a = b ≥ or a + b = c, and is minimal when
2
b = c ≥ a.

(Vasile Cîrtoaje, 2005)


222 Vasile Cîrtoaje

Solution. (a) The proof is similar to the first proof of problem P 2.53. We have here
 p
p − 2 p2 − 3q
, 3q ≤ p2 ≤ 4q



a1 = 3 .
2


 0, p ≥ 4q

Therefore, a attains its minimum value a1 when b = c (if p2 ≤ 4q) or a = 0 (if p2 ≥ 4q).
This means that r is minimal when b = c or a = 0.
(b) Using the known substitutions a = y+z, b = z+x, c = x + y, where x ≥ y ≥ z ≥ 0,

L
from

.M
a + b + c = 2(x + y + z),
a b + bc + ca = (x + y + z)2 + x y + yz + z x,

D
a bc = (x + y + z)(x y + yz + z x) − x yz,

A
it follows that
x + y +z =
p
,
PI
x y + yz + z x = q −
p2
,
M
2 4
and hence
LY

p2
 
p
a bc = q− − x yz.
2 4
O

Therefore, the product a bc is maximal when x yz is minimal; that is, according to (a),
.M

when x = y or z = 0, which is equivalent to a = b ≥ c/2 or a + b = c. Also, the product


a bc is minimal when x yz is maximal; that is, according to (a), when y = z, which is
equivalent to b = c ≥ a.
W

Remark. Using the result in (a), we can prove by the contradiction method (as in
W

Remark of P 2.53) the following generalization:


• Let a1 , a2 , . . . , an be nonnegative numbers such that
W

a1 + a2 + · · · + an = p, a12 + a22 + · · · + an2 = p1 ,

where p and p1 are fixed real numbers satisfying p2 ≤ np1 . Then, the product

r = a1 a2 · · · an

is maximal when n − 1 numbers of a1 , a2 , . . . , an are equal, and is minimal when one of


a1 , a2 , . . . , an is zero or n − 1 numbers of a1 , a2 , . . . , an are equal.
Symmetric Polynomial Inequalities in Nonnegative Variables 223

P 3.58. If a, b, c are positive real numbers such that a + b + c = 3, then


9 75
+ 16 ≥ .
a bc a b + bc + ca
(Vasile Cîrtoaje, 2005)
Solution. Let q = a b + bc + ca. For fixed q, the product a bc is maximal when two of
a, b, c are equal - see P 3.57-(a). Therefore, it suffices to prove the inequality for b = a
and c = 3 − 2a, a < 3/2. We have
9 75 9 75
+ 16 − = 2 + 16 −
a bc a b + bc + ca a c a(a + 2c)

L
9 25
= 2 + 16 −

.M
a (3 − 2a) a(2 − a)
2(16a − 56a + 73a2 − 42a + 9)
4 3
=

D
a2 (3 − 2a)(2 − a)
2(a − 1)2 (4a − 3)2

A
= 2 ≥ 0,
a (3 − 2a)(2 − a)
PI 3 3 3
 ‹
as desired. The equality holds for (a, b, c) = (1, 1, 1), and also for (a, b, c) =
M
, ,
4 4 2
or any cyclic permutation.
LY
O

P 3.59. If a, b, c are positive real numbers such that a + b + c = 3, then


.M

1 1 1
 ‹
8 + + + 9 ≥ 10(a2 + b2 + c 2 ).
a b c
W

(Vasile Cîrtoaje, 2006)


W

First Solution. Putting q = a b + bc + ca, we can write the inequality as


W

8q
+ 20q ≥ 81.
a bc
By P 3.57-(a), the product a bc is maximal for fixed q when two of a, b, c are equal.
Therefore, it suffices to prove the inequality for b = a and c = 3 − 2a, a < 3/2. We have
8q 24a(2 − a)
+ 20q − 81 = 2 + 60a(2 − a) − 81
a bc a (3 − 2a)
3a(40a4 − 140a3 + 174a2 − 89a + 16)
=
a2 (3 − 2a)
3a(2a − 1)2 (10a2 − 25a + 16)
= .
a2 (3 − 2a)
224 Vasile Cîrtoaje

Since
5 2 3
 ‹
10a2 − 25a + 16 = 10 a − + > 0,
4 8
1 1
 ‹
the proof is completed. The equality holds for (a, b, c) = , , 2 or any cyclic permu-
2 2
tation.
Second Solution (by Vo Quoc Ba Can). It is easy to check that the equality holds when
two of a, b, c are 1/2. Then, let us define
8
f (x) = − 10x 2 − αx − β,
x

L
such that (2x − 1)2 divides f (x). From f (1/2) = 0, we get α + 2β = 27. Therefore,

.M
8 (1 − 2x)h(x)
f (x) = − 10x 2 − (27 − 2β)x − β = ,

D
x x

A
where h(x) = 5x 2 − (β − 16)x + 8. From h(1/2) = 0, we get β = 69/2, and hence

h(x) =
PI
(1 − 2x)(16 − 5x)
,
2
M
(1 − 2x)2 (16 − 5x)
LY

f (x) = .
2x
Thus, we can write the inequality in the form
O

f (a) + f (b) + f (c) ≥ 27.


.M

Assume now that a = max{a, b, c}, a ≥ 1, and rewrite the inequality as


W

(1 − 2b)2 (16 − 5b) (1 − 2c)2 (16 − 5c) 4(a − 2)2 (5a − 1)


+ ≥ .
b c a
W

Since 16 − 5b > 0 and 16 − 5c > 0, the Cauchy-Schwarz inequality yields


W

(1 − 2b)2 (16 − 5b) (1 − 2c)2 (16 − 5c) (1 − 2b + 1 − 2c)2


+ ≥
b c b c
+
16 − 5b 16 − 5c
4(a − 2)2
= .
b c
+
16 − 5b 16 − 5c
Therefore, it suffices to prove that
1 5a − 1
≥ ,
b c a
+
16 − 5b 16 − 5c
Symmetric Polynomial Inequalities in Nonnegative Variables 225

which is equivalent to
a b c
≥ + .
5a − 1 16 − 5b 16 − 5c
Indeed,

a b c a b+c
− − ≥ −
5a − 1 16 − 5b 16 − 5c 5a − 1 16 − 5a
3
= > 0.
(5a − 1)(16 − 5a)

L
Remark. Using the second method, we can prove the following more general inequality.

.M
• If x 1 , x 2 , · · · , x n are positive real numbers such that

D
x 1 + x 2 + · · · + x n = n,

A
then PI
1 1
M
 ‹
(n + 1)2 + ··· + ≥ n(n2 − 3n − 6) + 4(n + 2)(x 12 + x 22 + · · · + x n2 ),
x1 xn
LY

with equality for x 1 = (n + 1)/2 and x 2 = · · · = x n = 1/2 (or any cyclic permutation).
O
.M

P 3.60. If a, b, c are positive real numbers such that a + b + c = 3, then


W

7(a2 + b2 + c 2 ) + 8(a2 b2 + b2 c 2 + c 2 a2 ) + 4a2 b2 c 2 ≥ 49.


W

Solution. Let q = a b + bc + ca. Since


W

a2 + b2 + c 2 = 9 − 2q,

a2 b2 + b2 c 2 + c 2 a2 = q2 − 6a bc,
we can rewrite the inequality as

2(6 − a bc)2 + 4q2 − 7q − 65 ≥ 0.

Since ‹3
a+b+c

a bc ≤ = 1,
3
226 Vasile Cîrtoaje

we have 6 − a bc > 0. By P 3.57-(a), the product a bc is maximal for fixed q when


two of a, b, c are equal. Therefore, it suffices to prove the inequality for a = b. Since
q = a2 + 2ac = 3a(2 − a) and a bc = a2 (3 − 2a), we have

2(6 − a bc)2 + 4q2 − 7q − 65 = 8a6 − 24a5 + 54a4 − 96a3 + 83a2 − 42a + 7)


= (a − 1)2 (2a − 1)2 (2a2 + 7 ≥ 0.

1 1
‹ 
The equality holds for a = b = c = 1, and for (a, b, c) = , , 2 or any cyclic permu-
2 2
tation.

L
.M
P 3.61. If a, b, c are nonnegative real numbers, then

D
(a3 + b3 + c 3 + a bc)2 ≥ 2(a2 + b2 )(b2 + c 2 )(c 2 + a2 ).

A
PI (Aleksandar Bulj, 2011)

First Solution. Let p = a + b + c, q = a b + bc + ca, r = a bc. Using the identities


M
a3 + b3 + c 3 = 3r + p3 − 3pq
LY

and
O

(a2 + b2 )(b2 + c 2 )(c 2 + a2 ) = (a2 + b2 + c 2 )(a2 b2 + b2 c 2 + c 2 a2 ) − a2 b2 c 2


.M

= (p2 − 2q)(q2 − 2pr) − r 2 ,


W

we can write the required inequality as f6 (a, b, c) ≥ 0, where


W

f6 (a, b, c) = 18r 2 + 4p(3p2 − 8q)r + p6 − 6p4 q + 7p2 q2 + 4q3 .


W

Since
3p2 − 8q = 3(p2 − 3q) + q ≥ 0,
for fixed p and q, f6 is an increasing function of r. Therefore, it suffices to prove the
inequality f6 (a, b, c) ≥ 0 for the case when r is minimal; that is, when a = 0 or b = c
(see P 3.57). In this cases, the original inequality becomes respectively

(b − c)2 (b4 + 2b3 c + b2 c 2 + 2bc 3 + c 4 ) ≥ 0

and
a(a − b)2 (a3 + 2a2 b + a b2 + 4b3 ) ≥ 0.
The equality holds for a = b = c, and for a = 0 and b = c (or any cyclic permutation).
Symmetric Polynomial Inequalities in Nonnegative Variables 227

Second Solution. Without loss of generality, assume that a = min{a, b, c}. From

a(a − b)(a − c) ≥ 0,

we get
a3 + a bc ≥ a2 (b + c),
and hence

a3 + b3 + c 3 + a bc ≥ a2 (b + c) + b3 + c 3 = (b + c)(a2 + b2 + c 2 − bc).

On the other hand,

L
1
2(a2 + b2 )(c 2 + a2 ) ≤ (2a2 + b2 + c 2 )2 .

.M
2
Therefore, it suffices to prove that

D
2(b + c)2 (a2 + b2 + c 2 − bc)2 ≥ (b2 + c 2 )(2a2 + b2 + c 2 )2 .

A
PI
We can obtain this inequality by multiplying the obvious inequality

2(a2 + b2 + c 2 − bc) ≥ 2a2 + b2 + c 2


M
LY

and
(b + c)2 (a2 + b2 + c 2 − bc) ≥ (b2 + c 2 )(2a2 + b2 + c 2 ).
O

The last inequality is equivalent to


.M

(b − c)2 (bc − a2 ) ≥ 0,

which is also true.


W

Remark. Using the first method, we can prove the following stronger inequality (Vasile
W

Cîrtoaje, 2011).
• If a, b, c are nonnegative real numbers then
W

(a3 + b3 + c 3 + a bc)2 ≥ 2(a2 + b2 )(b2 + c 2 )(c 2 + a2 ) + 7(a − b)2 (b − c)2 (c − a)2 .

Since
(a − b)2 (b − c)2 (c − a)2 = −27r 2 + 2(9pq − 2p3 )r + p2 q2 − 4q3 ,
we can write this inequality as f6 (a, b, c) ≥ 0, where

f6 (a, b, c) = 207r 2 + 2(20p3 − 79pq)r + p6 − 6p4 q + 32q3 .

We will show that for fixed p and q, f6 is an increasing function of r. From

f6 = 27r 2 + 2pqr + 20g(r) + p6 − 6p4 q + 32q3 ,


228 Vasile Cîrtoaje

where
g(r) = 9r 2 + 2(p3 − 4pq)r,
it suffices to show that g(r) is increasing. Indeed, for r1 ≥ r2 ≥ 0, by the third degree
Schur’s inequality, we have

g(r1 ) − g(r2 ) = (r1 − r2 )[9(r1 + r2 ) + 2p3 − 8pq]


≥ 2(r1 − r2 )(9r2 + p3 − 4pq) ≥ 0.

Therefore, it suffices to prove the inequality f6 (a, b, c) ≥ 0 for the case when r is min-
imal; that is, when a = 0 or b = c (see P 3.57). In this cases, the original inequality

L
becomes respectively

.M
(b − c)4 (b2 + 4bc + c 2 ) ≥ 0
and

D
a(a − b)2 (a3 + 2a2 b + a b2 + 4b3 ) ≥ 0.

A
PI
M
P 3.62. If a, b, c are positive real numbers, then
LY

(a + b + c − 3)(a b + bc + ca − 3) ≥ 3(a bc − 1)(a + b + c − a b − bc − ca).


O

(Vasile Cîrtoaje, 2011)


.M

Solution. Setting

p = a + b + c, q = a b + bc + ca, r = a bc,
W

the inequality becomes


W

(p − 3)(q − 3) ≥ 3(r − 1)(p − q),


W

or
3r(q − p) + pq − 6q + 9 ≥ 0.

First Solution. For fixed p and q, the linear function f (r) = 3r(q − p) + pq − 6q + 9 is
minimal when r is either minimal or maximal. Thus, according to P 3.57-(a), we need
only to prove that f (r) ≥ 0 for a = 0 and for b = c.
For a = 0, we need to show that

(b + c)bc − 6bc + 9 ≥ 0.
Symmetric Polynomial Inequalities in Nonnegative Variables 229

p
Indeed, putting x = bc, we have

(b + c)bc − 6bc + 9 ≥ 2x 3 − 6x 2 + 9 = 2(x + 1)(x − 2)2 + 1 > 0.

For b = c, since p = a + 2b, q = 2a b + b2 and r = a b2 , we need to show that

3a b2 (2a b + b2 − a − 2b) + (a + 2b − 6)(2a b + b2 ) + 9 ≥ 0;

that is,
Aa2 + Ba + C ≥ 0,
where

L
A = b(6b2 − 3b + 2), B = b(3b3 − 6b2 + 5b − 12), C = 2b3 − 6b2 + 9.

.M
Consider two cases.

D
Case 1: b ≥ 12/5. Since A > 0, B = 3b2 (b − 2) + b(5b − 12) > 0, C > 0, we have

A
Aa2 + Ba + C > 0.
Case 2: 0 < b < 12/5. Since PI
p
M
Aa2 + Ba + C ≥ (Aa2 + C) + Ba ≥ a(2 AC + B),
LY

we need to show that 4AC ≥ B 2 , which is equivalent to each of the inequalities


O

4b(6b2 − 3b + 2)(2b3 − 6b2 + 9) ≥ b2 (3b3 − 6b2 + 5b − 12)2 ,


.M

b(b − 1)4 (8 + 4b − b3 ) ≥ 0.
This inequality is true since
W

8 + 4b − b3 = 8 + 4b − 3b2 + b2 (3 − b) > 8 + 4b − 3b2 > 0.


W

The equality holds for a = b = c = 1.


W

Second Solution. Consider the following two cases.


Case 1: p ≥ q. We have

3r(q − p) + pq − 6q + 9 = (q − 3r)(p − q) + (q − 3)2 ≥ 0,

since
q2
q − 3r ≥ q − ≥ 0.
p

Case 2: p ≤ q. For p ≥ 6, we have

3r(q − p) + pq − 6q + 9 = 3r(q − p) + q(p − 6) + 9 > 0.


230 Vasile Cîrtoaje

Consider further that p < 6. From p2 ≥ 3q ≥ 3p, we get p ≥ 3, and from Schur’s
inequality
p3 + 9r ≥ 4pq,
we get
p3 + p2 r ≥ 4pq,

p2 + pr ≥ 4q.
Using this result, we have

p[3r(q − p) + pq − 6q + 9)] ≥ 3(4q − p2 )(q − p) + p(pq − 6q + 9)

L
= 12q2 − 2p(p + 9)q + 3p(p2 + 3)

.M
2
p2 + 9p p(12 − p)(p − 3)2

= 12 q − + ≥ 0.
12 12

D
A
PI
P 3.63. Let a, b, c be nonnegative real numbers such that a b + bc + ca = 3. Prove that
M
LY

4(a3 + b3 + c 3 ) + 7a bc + 125 ≥ 48(a + b + c).


O

(Vasile Cîrtoaje, 2011)

Solution. Since
.M

a3 + b3 + c 3 = 3a bc + (a + b + c)3 − 9(a + b + c),


W

we can write the inequality as


W

19a bc + 4(a + b + c)3 − 84(a + b + c) + 125 ≥ 0.


W

As it is known, for fixed a + b + c, the product a bc is minimal when a = 0 or b = c (see


P 3.57). Therefore it suffices to consider these cases.
Case 1: a = 0. We need to show that bc = 3 yields

4(b3 + c 3 ) + 125 ≥ 48(b + c).

Since
b3 + c 3 = (b + c)3 − 3bc(b + c) = (b + c)3 − 9(b + c)
and p p
b+c ≥2 bc = 2 3,
Symmetric Polynomial Inequalities in Nonnegative Variables 231

we have

4(b3 + c 3 ) + 125 − 48(b + c) = 4(b + c)3 − 84(b + c) + 125 > 0.

Case 2: b = c. We need to show that 2a b + b2 = 3 yields

4(a3 + 2b3 ) + 7a b2 + 125 ≥ 48(a + 2b).

This inequality is equivalent to

8b6 − 114b4 + 250b3 − 171b2 + 27 ≥ 0,

L
(b − 1)2 (2b − 3)2 (2b2 + 10b + 3) ≥ 0.

.M
Since the last inequality is true, the proof is completed.
The equality holds for a = b = c, and also for a = 1/4 and b = c = 3/2 (or any cyclic

D
permutation).

A
PI
M
P 3.64. If a, b, c ∈ [0, 1], then
LY

p p p
a a + b b + c c + 4a bc ≥ 2(a b + bc + ca).
O

(Vasile Cîrtoaje, 2012)


.M

Solution. This inequality is equivalent to

x 3 + y 3 + z 3 + 4x 2 y 2 z 2 ≥ 2(x 2 y 2 + y 2 z 2 + z 2 x 2 ),
W

where x, y, z ∈ [0, 1]. In addition, using the substitutions p = x + y + z and q =


W

x y + yz + z x, we can rewrite this inequality as


W

4x 2 y 2 z 2 + (3 + 4p)x yz + p3 − 3pq − 2q2 ≥ 0.

As it is known, for fixed p and q, the product x yz is minimal when x = 0 or y = z (see


P 3.57). Therefore it suffices to consider these cases.
Case 1: x = 0. We need to show that y 3 + z 3 ≥ 2 y 2 z 2 . Indeed, we have

y 3 + z 3 − 2 y 2 z 2 ≥ y 4 + z 4 − 2 y 2 z 2 = ( y 2 − z 2 )2 ≥ 0.

Case 2: y = z. We need to show that f (x) ≥ 0 for x ∈ [0, 1], where

f (x) = x 3 − 4x 2 y 2 (1 − y 2 ) + 2 y 3 (1 − y), y ∈ [0, 1].


232 Vasile Cîrtoaje

For y = 0 and y = 1, we have f (x) = x 3 ≥ 0. Consider further that y ∈ (0, 1). From

f 0 (x) = x[3x − 8 y 2 (1 − y 2 )],

it follows that f (x)is decreasing on [0, x 1 ] and increasing on [x 1 , 1], where

8 y 2 (1 − y 2 ) 2
 ˜
x1 = , x 1 ∈ 0, .
3 3
Therefore, it remains to show that f (x 1 ) ≥ 0, which is equivalent to

128 y 3 (1 − y)2 (1 + y)3 ≤ 27.

L
Since

.M
1
y 2 (1 − y 2 ) ≤ ,
4
it suffices to show that

D
32 y(1 − y)(1 + y)2 ≤ 27.

A
Using the AM-GM inequality, we get

y

1+ y 2
‹
PI
2
32 y(1 − y)(1 + y) = 1024 · (1 − y)
M
2 4
1+ y 4
LY

y
 
+ (1 − y) + 2 ·
2 4  81
≤ 1024   = < 27.
O

4 4
.M

The equality holds for a = 0 and b = c = 1 (or any cyclic permutation), and also for
a = b = c = 0.
W
W

P 3.65. If a, b, c ∈ [0, 1], then


W

p p p 3
a a + b b + c c ≥ (a b + bc + ca − a bc).
2
(Vasile Cîrtoaje, 2012)

Solution. This inequality is equivalent to

2(x 3 + y 3 + z 3 ≥ 3(x 2 y 2 + y 2 z 2 + z 2 x 2 − x 2 y 2 z 2 ),

where x, y, z ∈ [0, 1]. In addition, using the substitutions p = x + y + z and q =


x y + yz + z x, we can rewrite this inequality as

3x 2 y 2 z 2 + 3(2 + 3p)x yz + 2p3 − 6pq − 3q2 ≥ 0.


Symmetric Polynomial Inequalities in Nonnegative Variables 233

As it is known, for fixed p and q, the product x yz is minimal when x = 0 or y = z (see


P 3.57). Therefore it suffices to consider these cases.
Case 1: x = 0. We need to show that 2( y 3 + z 3 ) ≥ 3 y 2 z 2 . Indeed, we have

2( y 3 + z 3 ) − 3 y 2 z 2 ≥ 2( y 4 + z 4 ) − 4 y 2 z 2 = 2( y 2 − z 2 )2 ≥ 0.

Case 2: y = z. We need to show that f (x) ≥ 0 for x ∈ [0, 1], where

f (x) = 2x 3 − 3x 2 y 2 (2 − y 2 ) + y 3 (4 − 3 y), y ∈ [0, 1].

For y = 0, we have f (x) = x 3 ≥ 0. Consider further that y ∈ (0, 1]. From

L
f 0 (x) = 6x[x − y 2 (2 − y 2 )],

.M
it follows that f (x)is decreasing on [0, x 1 ] and increasing on [x 1 , 1], where

D
x 1 = y 2 (2 − y 2 ), x 1 ∈ (0, 1].

A
PI
Therefore, we only need to show that f (x 1 ) ≥ 0, which is equivalent to

y 3 (2 − y 2 )3 ≤ 4 − 3 y.
M
LY

Indeed,

y 3 (2 − y 2 )3 − (4 − 3 y) ≤ y(2 − y 2 )2 − (4 − 3 y) = ( y − 1)2 ( y 3 + 2 y 2 − y − 4) ≤ 0.
O

The equality holds for a = b = c = 1, and also for a = b = c = 0.


.M
W

P 3.66. If a, b, c ∈ [0, 1], then


W

p p p 500
3(a a + b b + c c) + a bc ≥ 5(a b + bc + ca).
W

81
(Vasile Cîrtoaje, 2012)

Solution. This inequality is equivalent to

500 2 2 2
3(x 3 + y 3 + z 3 ) + x y z ≥ 5(x 2 y 2 + y 2 z 2 + z 2 x 2 ),
81
where x, y, z ∈ [0, 1]. In addition, using the substitutions p = x + y + z and q =
x y + yz + z x, we can rewrite this inequality as

500 2 2 2
x y z + 3(3 + 5p)x yz + 3p3 − 9pq − 5q2 ≥ 0.
81
234 Vasile Cîrtoaje

As it is known, for fixed p and q, the product x yz is minimal when x = 0 or y = z (see


P 3.57). Therefore it suffices to consider these cases.
Case 1: x = 0. We need to show that 3( y 3 + z 3 ) ≥ 5 y 2 z 2 . Indeed, we have

3( y 3 + z 3 ) − 5 y 2 z 2 ≥ 3( y 4 + z 4 ) − 6 y 2 z 2 = 3( y 2 − z 2 )2 ≥ 0.

Case 2: y = z. We need to show that f (x) ≥ 0 for x ∈ [0, 1], where

50 2
 ‹
3 2 2
f (x) = 3x − 10x y 1 − y + y 3 (6 − 5 y), y ∈ [0, 1].
81

For y = 0, we have f (x) = 3x 3 ≥ 0. Consider further that y ∈ (0, 1]. From

L
.M
50 2
•  ‹˜
0 2
f (x) = x 9x − 20 y 1 − y ,
81

D
it follows that f (x)is decreasing on [0, x 1 ] and increasing on [x 1 , 1], where

A
20 2 50 2
 ‹
x1 =
9
y 1−
81
PI
y , x 1 ∈ (0, 1).
M
Therefore, it remains to show that f (x 1 ) ≥ 0, which is equivalent to
LY

4000 3 50 2 3
 ‹
y 1− y ≤ 6 − 5 y.
243 81
O

Substituting
9t 10
.M

y= , 0<t ≤ ,
10 9
this inequality can be written as
W

t 3 (2 − t 2 )3 ≤ 4 − 3t,
W

Indeed,
W

t 3 (2 − t 2 )3 − (4 − 3t) ≤ t(2 − t 2 )2 − (4 − 3t) = (t − 1)2 (t 3 + 2t 2 − t − 4) ≤ 0.

The equality holds for a = b = c = 0, and also for a = b = c = 81/100.

P 3.67. Let a, b, c be the lengths of the sides of a triangle. If a2 + b2 + c 2 = 3, then

a + b + c ≥ 2 + a bc.

(Vasile Cîrtoaje, 2005)


Symmetric Polynomial Inequalities in Nonnegative Variables 235

First Solution. Let p = a + b + c and q = a b + bc + ca. We need to show that p2 −2q = 3


involves p ≥ 2 + a bc. According to P 3.57-(b), for fixed p and q, the product a bc is
maximal when c/2 ≤ a = b ≤ c or a + b = c. Therefore, it suffices to consider only
these two cases.
Case 1: c/2 ≤ a = b ≤ c. From

3 = 2b2 + c 2 ≥ 3b2 ,

we get b ≤ 1. In addition, from

3 = 2b2 + c 2 ≤ 2b2 + 4b2 = 6b2 ,

L
it follows that 2b2 ≥ 1. Therefore, we need to prove that 2b2 + c 2 = 3, b ≤ 1, 2b2 ≥ 1

.M
involve 2b + c ≥ 2 + b2 c. Since

2b + c − 2 − b2 c = (1 − b)(c + bc − 2),

D
A
it suffices to show that c(1 + b) ≥ 2. This is true, since
PI
c 2 (1 + b)2 − 4 = (3 − 2b2 )(1 + b)2 − 4
M
= −1 + 6b + b2 − 4b3 − 2b4 = (1 − b)(−1 + 5b + 6b2 + 2b3 ) ≥ 0.
LY

Case 2: a + b = c. From a2 + b2 + c 2 = 3, we get 2a b = 2c 2 − 3, c 2 ≥ 3/2. In addition,


from 4a b ≤ c 2 , we get c 2 ≤ 2, and hence 3/2 ≤ c 2 ≤ 2. Since
O

3 −2c 3 + 7c − 4
a + b + c − 2 − a bc = 2c − 2 − c(c 2 − ) = ,
.M

2 2
we need to show that
2c 3 − 7c + 4 ≤ 0.
W

From
W

(c 2 − 2)(2c 2 − 3) ≤ 0,
W

we get 2c 4 − 7c 2 ≤ −6. Therefore,

c(2c 3 − 7c + 4) ≤ −6 + 4c < 0.

This completes the proof. The equality holds for a = b = c = 1.


Second Solution. Assume that a ≥ b ≥ c. From
1 3
3 = a2 + b2 + c 2 ≥ a2 + (b + c)2 ≥ a2 ,
2 2
p
it follows that a ≤ 2. Let

E(a, b, c) = a + b + c − 2 − a bc
236 Vasile Cîrtoaje

and v
t b2 + c 2
t= , t ≤ 1 ≤ a.
2
We will show that
E(a, b, c) ≥ E(a, t, t) ≥ 0.

We have

a(b − c)2 (b − c)2


E(a, b, c) − E(a, t, t) = a(t 2 − bc) − (2t − b − c) = −
2 2t + b + c
(b − c) 2 
3a 2
‹

L
= −
2 a2 + 2t 2 2t + b + c

.M
(b − c)2 [2t(3a − 2t) + a(3b + 3c − 2a)]
= ≥ 0,
2(a2 + 2t 2 )(2t + b + c)

D
because 3a − 2t > 2(a − t) ≥ 0 and 3(b + c) − 2a > 2(b + c − a) ≥ 0. On the other hand,

A
from
PI
E(a, t, t) = a + 2t − 2 − at 2 = (1 − t)(a + at − 2),
M
it follows that E(a, t, t) ≥ 0 if at ≥ 2 − a; that is,
LY

v
t 3 − a2
a ≥ 2 − a.
2
O

By squaring, the inequality can be restated as


.M

(a − 1)(8 − a2 − a3 ) ≥ 0.
W

p p
This is true, since 1 ≤ a ≤ 2 implies a − 1 ≥ 0 and 8 − a2 − a3 ≥ 8 − 2 − 2 2 > 0.
W
W

P 3.68. Let f n (a, b, c) be a symmetric homogeneous polynomial of degree n ≤ 5. Prove


that
(a) the inequality f n (a, b, c) ≥ 0 holds for all nonnegative real numbers a, b, c if and
only if f n (a, 1, 1) ≥ 0 and f n (0, b, c) ≥ 0 for all a, b, c ≥ 0;
(b) the inequality f n (a, b, c) ≥ 0 holds for all lengths a, b, c of the sides of a non-
degenerate or degenerate triangle if and only if f n (x, 1, 1) ≥ 0 for 0 ≤ x ≤ 2, and
f n ( y + z, y, z) ≥ 0 for all y, z ≥ 0.

(Vasile Cîrtoaje, 2005)


Symmetric Polynomial Inequalities in Nonnegative Variables 237

Solution. Let p = a + b + c, q = a b + bc + ca, r = a bc. Any symmetric homogeneous


polynomial f n (a, b, c) of degree n ≤ 5 can be written as

f n (a, b, c) = An (p, q)r + Bn (p, q),

where An (p, q) and Bn (p, q) are polynomial functions. For fixed p and q, the linear
function g n (r) = An (p, q)r + Bn (p, q) is minimal when r is either minimal or maximal.
(a) By P 3.57-(a), for fixed p and q, the product r is minimal and maximal when
two of a, b, c are equal or one of a, b, c is 0. Due to symmetry and homogeneity, the
conclusion follows.

L
(b) By P 3.57-(b), for fixed p and q, the product r is minimal and maximal when

.M
two of a, b, c are equal or one of a,b,c is the sum of the others. Due to symmetry and
homogeneity, the conclusion follows.

D
Remark. Similarly, we can prove the following statement, which does not involve the

A
homogeneity property.
PI
• Let f n (a, b, c) be a symmetric polynomial function of degree n ≤ 5. The inequality
f n (a, b, c) ≥ 0 holds for all nonnegative real numbers a, b, c if and only if it holds for a = 0
M
and for b = c.
LY
O

P 3.69. If a, b, c are nonnegative real numbers such that a + b + c = 3, then


.M

4(a4 + b4 + c 4 ) + 45 ≥ 19(a2 + b2 + c 2 ).
W

(Vasile Cîrtoaje, 2009)

First Solution. We use the mixing variable method. Write the inequality as F (a, b, c) ≥
W

0, where
W

F (a, b, c) = 4(a4 + b4 + c 4 ) + 45 − 19(a2 + b2 + c 2 ).


Due to symmetry, we may assume that a ≤ b ≤ c. Let us denote x = (b + c)/2, 1 ≤ x ≤
3/2. We will show that
F (a, b, c) ≥ F (a, x, x) ≥ 0.
We have

F (a, b, c) − F (a, x, x) = 4(b4 + b4 − 2x 4 ) − 19(b2 + c 2 − 2x 2 )

= 4[(b2 + c 2 )2 − 4x 4 ] + 8(x 4 − b2 c 2 ) − 19(b2 + c 2 − 2x 2 )

= (b2 + c 2 − 2x 2 )[4(b2 + c 2 + 2x 2 ) − 19] + 8(x 2 − bc)(x 2 + bc).


238 Vasile Cîrtoaje

Since
1
b2 + c 2 − 2x 2 = 2(x 2 − bc) = (b − c)2 ,
2
we get
1
F (a, b, c) − F (a, x, x) =(b − c)2 [4(b2 + c 2 + 2x 2 ) − 19 + 4(x 2 + bc)]
2
1
= (b − c)2 [4(x 2 − bc) + 24x 2 − 19] ≥ 0.
2
Also,
F (a, x, x) = F (3 − 2x, x, x) = 6(x − 1)2 (3 − 2x)(11 − 6x) ≥ 0.

L
This completes the proof. The equality holds for a = b = c = 1, and for a = 0 and

.M
b = c = 3/2 (or any cyclic permutation).
Second Solution. Write the inequality in the homogeneous form f4 (a, b, c) ≥ 0, where

D
f4 (a, b, c) = 36(a4 + b4 + c 4 ) + 5(a + b + c)4 − 19(a2 + b2 + c 2 )(a + b + c)2 .

A
According to P 3.68-(a), it suffices to prove that f4 (a, 1, 1) ≥ 0 and f4 (0, b, c) ≥ 0 for all
PI
a, b, c ≥ 0. We have
M
f4 (a, 1, 1) = 2a(11a3 − 18a2 + 3a + 4) = 2a(a − 1)2 (11a + 4) ≥ 0,
LY

f4 (0, b, c) = 2(b − c)2 (11b2 + 11c 2 + 13bc).


O

Remark. Similarly, we can prove the following more general statement (Vasile Cîrtoaje
and Le Huu Dien Khue, 2008).
.M

• Let α, β, γ be real numbers such that

1 + α + β = 2γ.
W

The inequality
W

X X X X
a4 + α a2 b2 + β a bc a≥γ a b(a2 + b2 )
W

holds for all a, b, c ≥ 0 if and only if

α ≥ (γ − 1) max{2, γ + 1}.

P 3.70. Let a, b, c be nonnegative real numbers. If k ≤ 2, then


X
a(a − b)(a − c)(a − k b)(a − kc) ≥ 0.

(Vasile Cîrtoaje, 2008)


Symmetric Polynomial Inequalities in Nonnegative Variables 239

Solution. Let us denote by f5 (a, b, c) the left hand side. By P 3.68-(a), it suffices to
show that f5 (a, 1, 1) ≥ 0 and f5 (0, b, c) ≥ 0 for all a, b, c ≥ 0. Indeed, we have

f5 (a, 1, 1) = a(a − 1)2 (a − k)2 ≥ 0

and
f5 (0, b, c) = (b + c)(b − c)2 (b2 − k bc + c 2 ) ≥ 0.
The equality holds for a = b = c, for a = 0 and b = c (or any cyclic permutation), and
for a/k = b = c, k > 0 (or any cyclic permutation).

L
.M
P 3.71. Let a, b, c be nonnegative real numbers. If k ∈ R, then

D
X
(b + c)(a − b)(a − c)(a − k b)(a − kc) ≥ 0.

A
(Vasile Cîrtoaje, 2008)
PI
Solution. Let us denote by f5 (a, b, c) the left hand side. By P 3.68-(a), it suffices to
M
show that f5 (a, 1, 1) ≥ 0 and f5 (0, b, c) ≥ 0 for all a, b, c ≥ 0. Indeed, we have
LY

f5 (a, 1, 1) = 2(a − 1)2 (a − k)2 ≥ 0


O

and
f5 (0, b, c) = k2 (b + c)b2 c 2 + bc(b + c)(b − c)2 ≥ 0.
.M

The equality holds for a = b = c, for b = c = 0 (or any cyclic permutation), and for
a/k = b = c, k > 0 (or any cyclic permutation).
W

Remark. Similarly, we can prove the following generalization:


W

• Let a, b, c be nonnegative real numbers. If m ≥ 0 and m(k − 2) ≤ 1, then


W

X
(ma + b + c)(a − b)(a − c)(a − k b)(a − kc) ≥ 0.

P 3.72. If a, b, c are nonnegative real numbers, then


X
a(a − 2b)(a − 2c)(a − 5b)(a − 5c) ≥ 0.

(Vasile Cîrtoaje, 2008)


240 Vasile Cîrtoaje

Solution. Let f5 (a, b, c) = a(a −2b)(a −2c)(a −5b)(a −5c). By P 3.68-(a), it suffices
P

to show that f5 (a, 1, 1) ≥ 0 and f5 (0, b, c) ≥ 0 for all a, b, c ≥ 0. Indeed, we have

f5 (a, 1, 1) = a3 (a − 7)2 + 20a3 − 60a2 + 44a + 8


≥ 20a3 − 60a2 + 44a + 8,

since
20a3 − 60a2 + 44a + 8 > 20a2 (a − 3) ≥ 0
for a ≥ 3, and

20a3 − 60a2 + 44a + 8 = 5(2a − 3)2 + 8 − a ≥ 8 − a ≥ 0

L
.M
for a ≤ 8. Also,
f5 (0, b, c) = (b + c)(b2 − 4bc + c 2 )2 ≥ 0.

D
The equality holds for a = 0 and b2 − 4bc + c 2 = 0 (or any cyclic permutation).

A
PI
M
P 3.73. If a, b, c are the lengths of the side of a triangle, then
LY

a4 + b4 + c 4 + 9a bc(a + b + c) ≤ 10(a2 b2 + b2 c 2 + c 2 a2 ).
O

First Solution. Let


.M

f4 (a, b, c) = 10(a2 b2 + b2 c 2 + c 2 a2 ) − a4 − b4 − c 4 − 9a bc(a + b + c).


W

By P 3.68-(b), it suffices to show that f4 (x, 1, 1) ≥ 0 for 0 ≤ x ≤ 2 and f4 ( y +z, y, z) ≥ 0


for y, z ≥ 0. Since
W

f4 (x, 1, 1) = 8 − 18x + 11x 2 − x 4 = (2 − x)(4 + x)(1 − x)2 ≥ 0


W

and

f4 ( y + z, y, z) = 8( y 2 + z 2 )2 − 2 yz( y 2 + z 2 ) − 28 y 2 z 2
= 2( y − z)2 (4 y 2 + 4z 2 + 7 yz) ≥ 0,

the proof is completed. The equality holds for an equilateral triangle and for a degen-
erate triangle with a/2 = b = c (or any cyclic permutation).
Second Solution. We use the sum-of-squares method (SOS method). Write the inequal-
ity as follows X X X X
9( b2 c 2 − a bc a) − ( a4 − b2 c 2 ) ≥ 0,
Symmetric Polynomial Inequalities in Nonnegative Variables 241

X X
9 a2 (b − c)2 − (b2 − c 2 )2 ≥ 0,
X
(b − c)2 (3a − b − c)(3a + b + c) ≥ 0.
Without loss of generality, assume that a ≥ b ≥ c. Since

(b − c)2 (3a − b − c)(3a + b + c) ≥ 0,

it suffices to show that

(c − a)2 (3b − c − a)(3b + c + a) + (a − b)2 (3c − a − b)(3c + a + b) ≥ 0.

L
Since

.M
3b − c − a ≥ 2b − a ≥ b + c − a ≥ 0
and (c − a)2 ≥ (a − b)2 ), it is enough to prove that

D
A
(3b − c − a)(3b + c + a) + (3c − a − b)(3c + a + b) ≥ 0.

We have
PI
(3b + c + a) − (3c + a + b) = 2(b − c) ≥ 0,
M
and hence
LY

(3b − c − a)(3b + c + a) + (3c − a − b)(3c + a + b) ≥


O

≥ (3b − c − a)(3c + a + b) + (3c − a − b)(3c + a + b)


.M

= 2(b + c − a)(3c + a + b) ≥ 0.
W
W

P 3.74. If a, b, c are the lengths of the sides of a triangle, then


W

X
3(a4 + b4 + c 4 ) + 7a bc(a + b + c) ≤ 5 a b(a2 + b2 ).

Solution. Let
X
f4 (a, b, c) = 5 a b(a2 + b2 ) − 3(a4 + b4 + c 4 ) − 7a bc(a + b + c).

By P 3.68-(b), it suffices to show that f4 (x, 1, 1) ≥ 0 for 0 ≤ x ≤ 2 and f4 ( y +z, y, z) ≥ 0


for y, z ≥ 0. Since

f4 (x, 1, 1) = 4 − 4x − 7x 2 + 10x 3 − 3x 4 = (2 − x)(2 + 3x)(1 − x)2 ≥ 0


242 Vasile Cîrtoaje

and

f4 ( y + z, y, z) = 4( y 2 + z 2 )2 + 4 yz( y 2 + z 2 ) − 24 y 2 z 2
= 4( y − z)2 ( y 2 + z 2 + 3 yz) ≥ 0,

the proof is completed. The equality holds for an equilateral triangle and for a degen-
erate triangle with a/2 = b = c (or any cyclic permutation).

L
P 3.75. If a, b, c are the lengths of the sides of a triangle, then

.M
b2 + c 2 − 6bc c 2 + a2 − 6ca a2 + b2 − 6a b
+ + + 4(a + b + c) ≤ 0.
a b c

D
(Vasile Cîrtoaje, 2005)

A
PI
First Solution. Write the inequality as f4 (a, b, c) ≥ 0, where
M
X
f4 (a, b, c) = bc(6bc − b2 − c 2 ) − 4a bc(a + b + c).
LY

By P 3.68-(b), it suffices to show that f4 (x, 1, 1) ≥ 0 for 0 ≤ x ≤ 2 and f4 ( y +z, y, z) ≥ 0


for y, z ≥ 0. Since
O

f4 (x, 1, 1) = 2(2 − 5x + 4x 2 − x 3 ) = 2(1 − x)2 (2 − x) ≥ 0


.M

and
W

f4 ( y + z, y, z) = 4( y 2 + z 2 )2 − 2 yz( y 2 + z 2 ) − 12 y 2 z 2
= 2( y − z)2 (2 y 2 + 3 yz + 2z 2 ) ≥ 0,
W
W

the proof is completed. The equality holds for an equilateral triangle and for a degen-
erate triangle with a/2 = b = c (or any cyclic permutation).
Second Solution. We use the SOS method. Write the inequality as follows:
X X
bc(b2 + c 2 − 6bc) + 4a bc a ≤ 0,
X X X
bc(b2 + c 2 − 2bc) − 4( b2 c 2 − a bc a) ≤ 0,
X X
bc(b − c)2 − 2 a2 (b − c)2 ≤ 0,
X
(b − c)2 (2a2 − bc) ≥ 0.
Symmetric Polynomial Inequalities in Nonnegative Variables 243

Without loss of generality, assume that a ≥ b ≥ c. Since (b − c)2 (2a2 − bc) ≥ 0, it


suffices to prove that

(c − a)2 (2b2 − ca) + (a − b)2 (2c 2 − a b) ≥ 0.

Since
2b2 − ca ≥ 2b2 − c(b + c) = (b − c)(2b + c) ≥ 0
and (c − a)2 ≥ (a − b)2 , it is enough to show that

(2b2 − ca) + (2c 2 − a b) ≥ 0.

L
Indeed,

.M
(2b2 − ca) + (2c 2 − a b) = (b − c)2 + (b + c)(b + c − a) ≥ 0.

D
A
P 3.76. Let f6 (a, b, c) be a sixth degree symmetric homogeneous polynomial written in the
PI
form
f6 (a, b, c) = Ar 2 + B(p, q)r + C(p, q), A ≤ 0,
M
where
LY

p = a + b + c, q = a b + bc + ca, r = a bc.
O

Prove that
(a) the inequality f6 (a, b, c) ≥ 0 holds for all nonnegative real numbers a, b, c if and
.M

only if f6 (a, 1, 1) ≥ 0 and f6 (0, b, c) ≥ 0 for all a, b, c ≥ 0;


(b) the inequality f6 (a, b, c) ≥ 0 holds for all for all lengths a, b, c of the sides of a
W

non-degenerate or degenerate triangle if and only if f6 (x, 1, 1) ≥ 0 for 0 ≤ x ≤ 2, and


f6 ( y + z, y, z) ≥ 0 for all y, z ≥ 0.
W

(Vasile Cîrtoaje, 2006)


W

Solution. For fixed p and q, the function f defined by

f (r) = Ar 2 + B(p, q)r + C(p, q)

is a concave quadratic function of r. Therefore, f (r) is minimal when r is minimal or


maximal. According to P 3.57, the conclusion follows. As we have shown in the proof
of P 2.75, A is called the highest coefficient of f6 (a, b, c).
Remark 1. We can extend the part (a) of P 3.76 as follows:
(a1) For A ≤ 0, the inequality f6 (a, b, c) ≥ 0 holds for all nonnegative real numbers
a, b, c satisfying p2 ≤ 4q if and only if f6 (a, 1, 1) ≥ 0 for all 0 ≤ a ≤ 4;.
244 Vasile Cîrtoaje

(a2) For A ≤ 0, the inequality f6 (a, b, c) ≥ 0 holds for all nonnegative real numbers
a, b, c satisfying p2 > 4q if and only if f6 (a, 1, 1) ≥ 0 for all a > 4 and f6 (0, b, c) ≥ 0 for
all b, c ≥ 0.
Notice that the restriction 0 ≤ a ≤ 4 in (a1) follows by setting b = c = 1 in p2 ≤ 4q.
In addition, since a = 0 and p2 ≤ 4q involve b = c, the condition f6 (0, b, c) ≥ 0 is
not necessary in (a1) because it is equivalent to f6 (0, 1, 1) ≥ 0, which follows from
f6 (a, 1, 1) ≥ 0 for all 0 ≤ a ≤ 4. Also, the restriction a > 4 in (a2) follows by setting
b = c = 1 in p2 > 4q.
Remark 2. The statement in P 3.76 and its extension in Remark 1 are also valid in the
more general case when f6 (a, b, c) is a symmetric homogeneous function of the form

L
.M
f6 (a, b, c) = Ar 2 + B(p, q)r + C(p, q),

where B(p, q) and C(p, q) are rational functions.

D
A
PI
P 3.77. If a, b, c are nonnegative real numbers, then
M
X
a(b + c)(a − b)(a − c)(a − 2b)(a − 2c) ≥ (a − b)2 (b − c)2 (c − a)2 .
LY

(Vasile Cîrtoaje, 2008)


O

Solution. Let p = a + b + c, q = a b + bc + ca, r = a bc, and


.M

f6 (a, b, c) = f (a, b, c) − (a − b)2 (b − c)2 (c − a)2 ,


W

where X
f (a, b, c) = a(b + c)(a − b)(a − c)(a − 2b)(a − 2c).
W

Since
W

X
a(b + c)(a − b)(a − c)(a − 2b)(a − 2c) =
X
= a(p − a)(a2 + 2bc − q)(a2 + 6bc − 2q),
f (a, b, c) has the same highest coefficient A0 as
X
P1 (a, b, c) = − a2 (a2 + 2bc)(a2 + 6bc);

that is, according to Remark 2 from P 2.75, A0 = P1 (1, 1, 1) = −3(1 + 2)(1 + 6) = −63.
Then, f6 (a, b, c) has the highest coefficient

A = A0 + 27 = −36.
Symmetric Polynomial Inequalities in Nonnegative Variables 245

Since A < 0, according to P 3.76-(a), it suffices to prove that f6 (a, 1, 1) ≥ 0 and


f6 (0, b, c) ≥ 0 for all a, b, c ≥. Indeed, we have

f6 (a, 1, 1) = 2a(a − 1)2 (a − 2)2 ≥ 0

and
f6 (0, b, c) = bc(b − c)4 ≥ 0.
The equality holds for a = b = c, for a = 0 and b = c (or any cyclic permutation), and
for a/2 = b = c (or any cyclic permutation).

L
.M
P 3.78. Let a, b, c be nonnegative real numbers.

D
(a) If 2 ≤ k ≤ 6, then

A
X 4(k − 2)(a − b)2 (b − c)2 (c − a)2
PI
a(a − b)(a − c)(a − k b)(a − kc) + ≥ 0;
a+b+c
M
(b) If k ≥ 6, then
LY

X (k + 2)2 (a − b)2 (b − c)2 (c − a)2


a(a − b)(a − c)(a − k b)(a − kc) + ≥ 0.
4(a + b + c)
O

(Vasile Cîrtoaje, 2009)


.M

Solution. a) We need to prove that f6 (a, b, c) ≥ 0, where


W

X
f6 (a, b, c) = (a + b + c) a(a − b)(a − c)(a − k b)(a − kc)
W

+4(k − 2)(a − b)2 (b − c)2 (c − a)2 .


W

Since f6 (a, b, c) has the same highest coefficient as

4(k − 2)(a − b)2 (b − c)2 (c − a)2

and (a − b)2 (b − c)2 (c − a)2 has the highest coefficient −27, it follows that f6 (a, b, c)
has the highest coefficient
A = −108(k − 2).
Since A ≤ 0, according to P 3.76-(a), it suffices to prove that f6 (a, 1, 1) ≥ 0 and
f6 (0, b, c) ≥ 0 for all a, b, c ≥ 0. Indeed, we have

f6 (a, 1, 1) = a(a + 2)(a − 1)2 (a − k)2 ≥ 0


246 Vasile Cîrtoaje

and
f6 (0, b, c) = (b − c)6 + (6 − k)bc(b − c)4 ≥ 0.
The equality holds for a = b = c, for a = 0 and b = c (or any cyclic permutation, and
for and for a/k = b = c (or any cyclic permutation).
(b) We need to prove that f6 (a, b, c) ≥ 0, where
X
f6 (a, b, c) = 4(a + b + c) a(a − b)(a − c)(a − k b)(a − kc)

+(k + 2)2 (a − b)2 (b − c)2 (c − a)2 .


Since f6 (a, b, c) has the same highest coefficient as

L
.M
(k + 2)2 (a − b)2 (b − c)2 (c − a)2

and (a − b)2 (b − c)2 (c − a)2 has the highest coefficient −27, it follows that f6 (a, b, c)

D
has the highest coefficient

A
A = −27(k + 2)2 < 0.
PI
According to P 3.76-(a), it suffices to prove that f6 (a, 1, 1) ≥ 0 and f6 (0, b, c) ≥ 0 for all
a, b, c ≥ 0. Indeed, we have
M

f6 (a, 1, 1) = 4a(a + 2)(a − 1)2 (a − k)2 ≥ 0


LY

and
O

f6 (0, b, c) = (b − c)2 [2(b2 + c 2 ) − (k − 2)bc]2 ≥ 0.


.M

The equality holds for a = b = c, for a = 0 and b = c (or any cyclic permutation, and
b c k−2
for and for a/k = b = c (or any cyclic permutation), and for a = 0 and + =
c b 2
W

or any cyclic permutation.


W
W

P 3.79. If a, b, c are nonnegative real numbers, then

(3a2 + 2a b + 3b2 )(3b2 + 2bc + 3c 2 )(3c 2 + 2ca + 3a2 ) ≥ 8(a2 + 3bc)(b2 + 3ca)(c 2 + 3a b).

Solution. Let p = a + b + c, q = a b + bc + ca and

f (a, b, c) = (3a2 + 2a b + 3b2 )(3b2 + 2bc + 3c 2 )(3c 2 + 2ca + 3a2 ).

We need to prove that f6 (a, b, c) ≥ 0, where

f6 (a, b, c) = f (a, b, c) − 8(a2 + 3bc)(b2 + 3ca)(c 2 + 3a b).


Symmetric Polynomial Inequalities in Nonnegative Variables 247

Since

f (a, b, c) = (3p2 − 6q + 2a b − 3c 2 )(3p2 − 6q + 2bc − 3a2 )(3p2 − 6q + 2ca − 3b2 ),

f6 (a, b, c) has the same highest coefficient A as P2 (a, b, c), where

P2 (a, b, c) = (2a b − 3c 2 )(2bc − 3a2 )(2ca − 3b2 ) − 8(a2 + 3bc)(b2 + 3ca)(c 2 + 3a b);

that is, according to Remark 2 from P 2.75,

A = P2 (1, 1, 1) = (2 − 3)3 − 8(1 + 3)3 < 0.

Then, by P 3.76-(a), it suffices to prove that f6 (a, 1, 1) ≥ 0 and f6 (0, b, c) ≥ 0 for all

L
nonnegative real a, b, c. Indeed,

.M
f6 (a, 1, 1) = 8(3a2 + 2a + 3)2 − 8(a2 + 3)(3a + 1)2 = 48(a + 1)(a − 1)2 ≥ 0,

D
A
f6 (0, b, c) = 3b2 c 2 (9b2 − 2bc + 9c 2 ) ≥ 0.
The equality holds for a = b = c. PI
M
LY

P 3.80. Let a, b, c be nonnegative real numbers such that a + b + c = 2. If


O

−2 11
≤k≤ ,
3 8
.M

then
(a2 + ka b + b2 )(b2 + k bc + c 2 )(c 2 + kca + a2 ) ≤ k + 2.
W

(Vasile Cîrtoaje, 2011)


W

Solution. We need to prove that f6 (a, b, c) ≥ 0, where


W

f6 (a, b, c) = (k + 2)(a + b + c)6 − 64(a2 + ka b + b2 )(b2 + k bc + c 2 )(c 2 + kca + a2 ).

Since f6 (a, b, c) has the same highest coefficient A as P2 (a, b, c), where

P2 (a, b, c) = −64(ka b − c 2 )(k bc − a2 )(kca − b2 ),

according to Remark 2 from P 2.75, we have

A = P2 (1, 1, 1) = 64(1 − k)3 .

Also,

f6 (a, 1, 1) = (k + 2)a[(a − 1)2 + 11 − 8k][a3 + 14a2 + (8k + 12)a + 16]


248 Vasile Cîrtoaje

and

f6 (0, b, c) = (k + 2)(b + c)6 − 64b2 c 2 (b2 + k bc + c 2 )


= (b − c)2 (k + 2)(b2 + c 2 )2 + 8(k + 2)bc(b2 + c 2 ) + 4(7k − 2)b2 c 2 .
 

11
Case 1: 1 ≤ k ≤ . Since A ≤ 0, according to P 3.76-(a), it suffices to prove that
8
f6 (a, 1, 1) ≥ 0 and f6 (0, b, c) ≥ 0 for all a, b, c ≥ 0. Clearly, these conditions are satisfied.
The equality holds for a = 0 and b = c = 1 (or any cyclic permutation). If k = 11/8,
then the equality holds also for a = b = c = 2/3.
−2
Case 2: ≤ k < 1. Since A > 0, we will use the highest coefficient cancellation method.

L
3
We will prove the sharper inequality g6 (a, b, c) ≥ 0, where

.M
g6 (a, b, c) = f6 (a, b, c) − 64(1 − k)3 a2 b2 c 2 .

D
Since g6 (a, b, c) has the highest coefficient A1 = 0, it suffices to show that g6 (a, 1, 1) ≥ 0

A
and g6 (0, b, c) ≥ 0 for all a, b, c ≥ 0 (see P 3.76). The inequality g6 (a, 1, 1) ≥ 0 is true
if PI
(k + 2)[(a − 1)2 + 11 − 8k][a3 + 14a2 + (8k + 12)a + 16] ≥ 64(1 − k)3 a.
M
It suffices to show that
LY

(k + 2)(11 − 8k)(6a2 + (8k + 12)a + 16] ≥ 64(1 − k)3 a.


O

In addition, since 11 − 8k > 8(1 − k), we only need to show that

(k + 2)[3a2 + (4k + 6)a + 8] ≥ 4(1 − k)2 a.


.M

Since
W

3a2 + (4k + 6)a + 8 ≥ 3(2a − 1) + (4k + 6)a + 8 = (4k + 12)a + 5 > 4(k + 3)a,
W

it is enough to show that


(k + 2)(k + 3) ≥ (1 − k)2 .
W

Indeed,
2 1
‹
2
(k + 2)(k + 3) − (1 − k) = 7k + 5 = 7 k + + > 0.
3 3
The inequality g6 (0, b, c) ≥ 0 is also true since

g6 (0, b, c) ≥ (b − c)2 [4(k + 2)b2 c 2 + 16(k + 2)b2 c 2 + 4(7k − 2)b2 c 2 ]


= 16(3k + 2)b2 c 2 ≥ 0.

Thus, the proof is completed. The equality holds for a = 0 and b = c = 1 (or any cyclic
permutation). If k = 11/8, then the equality holds also for a = b = c = 2/3.
Symmetric Polynomial Inequalities in Nonnegative Variables 249

P 3.81. Let a, b, c be nonnegative real numbers such that a + b + c = 2. Prove that

(2a2 + bc)(2b2 + ca)(2c 2 + a b) ≤ 4.

Solution. Write the inequality in the homogeneous form f6 (a, b, c) ≥ 0, where

f6 (a, b, c) = (a + b + c)6 − 16(2a2 + bc)(2b2 + ca)(2c 2 + a b).

Since f6 (a, b, c) has the same highest coefficient A as P2 (a, b, c), where

P2 (a, b, c) = −16(2a2 + bc)(2b2 + ca)(2c 2 + a b),

L
.M
according to Remark 2 from P 2.75, we have

D
A = P2 (1, 1, 1) = −432.

A
Since A < 0, according to P 3.76-(a), it suffices to prove that f6 (a, 1, 1) ≥ 0 and
f6 (0, b, c) ≥ 0 for all a, b, c ≥ 0. We have PI
M
f6 (a, 1, 1) = a(a + 2)2 (a3 + 8a2 − 8a + 32) = a(a + 2)2 [a3 + 4a2 + 28 + 4(a − 1)2 ] ≥ 0,
LY

f6 (0, b, c) = (b + c)6 − 64b3 c 3 ≥ 0.


The equality holds for a = 0 and b = c = 1 (or any cyclic permutation).
O
.M

P 3.82. Let a, b, c be nonnegative real numbers, no two of which are zero. Then,
W

5(a − b)2 (b − c)2 (c − a)2


W

X
(a − b)(a − c)(a − 2b)(a − 2c) ≥ .
a b + bc + ca
W

(Vasile Cîrtoaje, 2010)

Solution. Denote
p = a + b + c, q = a b + bc + ca,
and write the inequality as f6 (a, b, c) ≥ 0, where
X
f6 (a, b, c) = q (a − b)(a − c)(a − 2b)(a − 2c) − 5(a − b)2 (b − c)2 (c − a)2 .

Clearly, f6 (a, b, c) has the highest coefficient

A = (−5)(−27) = 135.
250 Vasile Cîrtoaje

Since A > 0, we will use the highest coefficient cancellation method. We have

f6 (a, 1, 1) = (2a + 1)(a − 1)2 (a − 2)2 , f6 (0, b, c) = bc[(b + c)2 − 6bc]2 .

Consider two cases: p2 ≤ 4q and p2 > 4q.


Case 1: p2 ≤ 4q. Since
f6 (1, 1, 1) = 0, f6 (2, 1, 1) = 0,
we define the symmetric homogeneous polynomial of degree three

P(a, b, c) = a bc + B(a + b + c)3 + C(a + b + c)(a b + bc + ca)

L
such that P(1, 1, 1) = 0 and P(2, 1, 1) = 0. We get B = 1/18 and C = −5/18, hence

.M
1 5
P(a, b, c) = a bc + (a + b + c)3 − (a + b + c)(a b + bc + ca).
18 18

D
Consider now the sharper inequality g6 (a, b, c) ≥ 0, where

A
PI
g6 (a, b, c) = f6 (a, b, c) − 135P 2 (a, b, c).
M
Clearly, g6 (a, b, c) has the highest coefficient A = 0. Then, according to Remark 1 from
the proof of P 3.76, it suffices to prove that g6 (a, 1, 1) ≥ 0 for 0 ≤ a ≤ 4. We have
LY

1
P(a, 1, 1) = (a − 1)2 (a − 2),
O

18
hence
.M

1
g6 (a, 1, 1) = f6 (a, 1, 1) − 135P 2 (a, 1, 1) = (a − 1)2 (a − 2)2 (7 + 34a − 5a2 ) ≥ 0.
12
W

Case 2: p2 > 4q. Define the symmetric homogeneous function


W

(a b + bc + ca)2
W

R(a, b, c) = a bc + C(a + b + c)(a b + bc + ca) − (9C + 1) ,


3(a + b + c)

which satisfies R(1, 1, 1) = 0, and consider the sharper inequality g6 (a, b, c) ≥ 0, where

g6 (a, b, c) = f6 (a, b, c) − 135R2 (a, b, c).

Clearly, g6 (a, b, c) has the highest coefficient A = 0. Then, according to Remark 2 from
the proof of P 3.76, it suffices to prove that g6 (a, 1, 1) ≥ 0 for a > 4, and g6 (0, b, c) ≥ 0
for all b, c ≥ 0. We have

(a − 1)2 [3C(2a + 1) − 1] bc[3C(b + c)2 − (9C + 1)bc]


R(a, 1, 1) = , R(0, b, c) = .
3(a + 2) 3(b + c)
Symmetric Polynomial Inequalities in Nonnegative Variables 251

The inequality g6 (0, b, c) ≥ 0 holds for all b, c ≥ 0 only if

9C + 1
= 6;
3C
that is, C = 1/9. For this value of C, we have

2(a − 1)3 bc[(b + c)2 − 6bc]


R(a, 1, 1) = , R(0, b, c) = ,
9(a + 2) 9(b + c)

hence
(a − 1)2
g6 (a, 1, 1) = f6 (a, 1, 1) − 135R2 (a, 1, 1) = g(a),

L
3(a + 2)2

.M
where
g(a) = 3(2a + 1)(a2 − 4)2 − 20(a − 1)4 ,

D
and

A
bc(3b2 + bc + 3c 2 )[(b + c)2 − 6bc]2
g6 (0, b, c) = f6 (0, b, c) − 135R2 (0, b, c) = PI 3(b + c)2
≥ 0.
M
To complete the proof, we need to show that g(a) ≥ 0 for a > 4. This is true since
a2 − 4 > (a − 1)2 and 3(2a + 1) > 20.
LY

The equality holds for a = b = c, for a/2 = b = c (or any cyclic permutation), and
for a = 0 and b/c + c/b = 4 (or any cyclic permutation).
O
.M

P 3.83. Let a ≤ b ≤ c be positive real numbers such that


W

a + b + c = p, a bc = r,
W

where p and r are fixed positive numbers satisfying p3 ≥ 27r. Prove that
W

q = a b + bc + ca

is maximal when b = c, and is minimal when a = b.

Solution. First, we show that b ∈ [b1 , b2 ], where b1 and b2 are the positive roots of
the equation
2x 3 − p x 2 + r = 0.
Let f (x) = 2x 3 − p x 2 + r, x ≥ 0. From f 0 (x) = 2x(3x − p), it follows that f (x)
is decreasing for x ≤ p/3 and increasing for x ≥ p/3. Since f (0) > 0, f (p/3) =
252 Vasile Cîrtoaje

(27r − p3 )/27 ≤ 0 and f (p) = p3 + r > 0, there exists two positive numbers b1 ≤ b2
such that f (b1 ) = f (b2 ) = 0 and f (x) ≤ 0 for x ∈ [b1 , b2 ]. From

f (b) r
= b2 − b(p − b) + = b2 − b(a + c) + bc = (b − a)(b − c) ≤ 0,
b b

it follows that b ∈ [b1 , b2 ]. In addition, we have b = b1 when b = a, and b = b2 when


b = c. On the other hand, from
r
q = b(a + c) + ac = b(p − b) + ,
b

L
we get
r

.M
q(b) = p b − b2 + .
b
Since

D
r −(b − a)(b − c)
q0 (b) = p − 2b − = ≥ 0,

A
b2 b
q(b) is increasing on [b1 , b2 ], and hence q(b) is maximal for b = b2 , when b = c, and
PI
is minimal for b = b1 , when b = a.
M
Remark 1. Substituting 1/a, 1/b, 1/c for a, b, c, respectively, we get the following
LY

similar statement.
• Let a ≤ b ≤ c be positive real numbers such that
O

a b + bc + ca = q, a bc = r,
.M

where q and r are fixed positive numbers satisfying q3 ≥ 27r 2 . The sum
W

p=a+b+c
W

is minimal when b = c, and is maximal when a = b.


Remark 2. We can prove by the contradiction method (as in Remark of P 2.53) the
W

following generalizations:
• Let a1 , a2 , . . . , an be positive real numbers such that

a1 + a2 + · · · + an = np, a1 a2 · · · an = p1 ,

where p and p1 are fixed positive numbers satisfying p n ≥ p1 . Then, the sum

1 1 1
+ + ··· +
a1 a2 an

is maximal and minimal when n − 1 numbers of a1 , a2 , . . . , an are equal.


Symmetric Polynomial Inequalities in Nonnegative Variables 253

• Let a1 , a2 , . . . , an be positive real numbers such that

1 1 1 n
+ + ··· + = , a1 a2 · · · an = p1 ,
a1 a2 an p

where p and p1 are fixed positive numbers satisfying p n ≤ p1 . Then, the sum

a1 + a2 + · · · + an

is maximal and minimal when n − 1 numbers of a1 , a2 , . . . , an are equal.

L
.M
P 3.84. If a, b, c are positive real numbers, then

D
A
1 1 1 1
 ‹
(a + b + c − 3) + + − 3 + a bc + ≥ 2.
a b c a bc
PI
(Vasile Cîrtoaje, 2004)
M
1 1 1
LY

Solution. Since the inequality does not exchange by substituting a, b, c with , , ,


a b c
respectively, we may consider only the case a bc ≥ 1. Using the notation p = a + b + c
O

and r = a bc, r ≥ 1, we can write the inequality as


.M

a b + bc + ca 1
 ‹
(p − 3) − 3 + r + ≥ 2.
r r
W

For fixed p and r, by P 3.83, the expression q = a b + bc + ca is minimal when two of


p
a, b, c are equal. Since p ≥ 3 3 r ≥ 3 (by the AM-GM inequality), it suffices to prove the
W

desired inequality for b = c, when it becomes as follows


W

2 1 1 1
 ‹  ‹  ‹
a b2 + − 3 + + 2b − 3 ≥ 6 b + − 2 ,
b a b2 b

1 1
• ‹  ‹˜
(b − 1)2 ab + − 2 + 2 a + − 2 ≥ 0.
ab a
1 1
Since a b + ≥ 2 and a + ≥ 2, the conclusion follows. The equality holds for
ab a
a = b = 1, or b = c = 1, or c = a = 1.
254 Vasile Cîrtoaje

P 3.85. If a, b, c are positive real numbers such that a bc = 1, then


‹ s
3 2 2

(a) a b + bc + ca − ≥ (a + b + c) − 1;
7 3 3

46 p
(b) a b + bc + ca − 3 ≥ ( a + b + c − 2 − 1).
27
(Vasile Cîrtoaje, 2009)

Solution. Let us denote p = a + b + c.


(a) Write the inequality as

L
.M
‹ vt 2p
3 2

a b + bc + ca − ≥ − 1.
7 3 3

D
For fixed p, the expression q = a b + bc + ca is minimal when two of a, b, c are equal

A
(see P 3.83). Thus, it suffices to consider the case a = b, when the inequality becomes
v PI
t 4a3 − 3a2 + 2
3a3 − 2a + 6 ≥ 7 .
M
3
LY

By squaring, we get
(a − 1)2 (3a − 1)2 (3a2 + 8a + 10) ≥ 0,
O

1 1
 ‹
which is true. The equality holds for a = b = c = 1, and also for (a, b, c) = , , 9 or
3 3
.M

any cyclic permutation.

(b) Write the inequality as


W

46 p
a b + bc + ca − 3 ≥ ( p − 2 − 1).
W

27
For fixed p, the expression q = a b + bc + ca is minimal when two of a, b, c are equal
W

(see P 3.83). Thus, it suffices to consider the case a = b, when the inequality becomes
p
27a3 − 35a + 54 ≥ 46 2a3 − 2a2 + 1.

By squaring, we get

(a − 1)2 (9a − 5)2 (9a2 + 28a + 32) ≥ 0,


5 5 81
 ‹
which is true. The equality holds for a = b = c = 1, and also for (a, b, c) = , ,
9 9 25
or any cyclic permutation.
Symmetric Polynomial Inequalities in Nonnegative Variables 255

P 3.86. If a, b, c are positive real numbers such that a bc = 1, then

50 37
a b + bc + ca + ≥ .
a+ b+c+5 4

(Michael Rozenberg, 2013)

Solution. Using the notation p = a + b + c, we can write the inequality as

50 37
a b + bc + ca + ≥ .
p+5 4

L
For fixed p, the expression q = a b + bc + ca is minimal when two of a, b, c are equal

.M
(see P 3.83). Thus,it suffices to prove the desired inequality for a = b; that is, to show
that a2 c = 1 involves

D
50 37
a2 + 2ac + ≥ .
2a + c + 5 4

A
This is equivalent to

a2 +
2
+ 3
50a2

37
PI
,
a 2a + 5a + 1
M
2 4
which can be written in the obvious form
LY

(a − 1)2 (2a − 1)2 (2a2 + 11a + 8) ≥ 0.


O

The equality holds for a = b = c = 1, and for a = b = 1/2 and c = 4 (or any cyclic
.M

permutation).
W
W

P 3.87. Let a, b, c be positive real numbers.


W

(a) If a bc = 2, then

a2 + b2 + c 2
(a + b + c − 3)2 + 1 ≥ ;
3

1
(b) If a bc = , then
2

a2 + b2 + c 2 + 3(3 − a − b − c)2 ≥ 3.

(Vasile Cîrtoaje, 2007)


256 Vasile Cîrtoaje

Solution. Let us denote p = a + b + c.


(a) Write the inequality as

p2 − 2(a b + bc + ca)
(p − 3)2 + 1 ≥ .
3
For fixed p, the expression q = a b + bc + ca is minimal when two of a, b, c are equal
(see P 3.83). Thus, it suffices to consider the case a = b, when the inequality becomes
in succession ‹2
2 2a2 4

2a + 2 − 3 + 1 ≥ + 4,
a 3 3a

L
5a6 − 18a5 + 15a4 + 12a3 − 18a2 + 4 ≥ 0,

.M
(a − 1)2 (5a4 − 8a3 − 6a2 + 8a + 4) ≥ 0.

D
Since
5a4 − 8a3 − 6a2 + 8a + 4 = 4(a − 1)4 + a(a3 + 8a3 − 30a + 24),

A
PI
it suffices to prove that a3 + 8a3 − 30a + 24 ≥ 0. Indeed, for a ≥ 1, we have
M
a3 + 8a3 − 30a + 24 = (a − 1)3 + 11a2 − 33a + 25
3 1
LY

= (a − 1)3 + 11(a − )2 + > 0,


2 4
O

and for a < 1, we have


.M

a3 + 8a3 − 30a + 24 = a(1 − a)2 + 2(2 − a)(6 − 5a) > 0.

The equality holds for (a, b, c) = (1, 1, 2) or any cyclic permutation.


W

(b) Write the inequality as


W

p2 − 2(a b + bc + ca) + 3(3 − p)2 ≥ 3.


W

For fixed p, the expression q = a b + bc + ca is minimal when two of a, b, c are equal


(see P 3.83). Thus, it suffices to consider the case a = b, when the inequality becomes
in succession
1 1 2
 ‹
2
2a + 2 + 3 3 − 2a − 2 ≥ 3,
4a 2a
14a6 − 36a5 + 24a4 + 6a3 − 9a2 + 1 ≥ 0,

(a − 1)2 (14a4 − 8a3 − 6a2 + 2a + 1) ≥ 0.


Since
14a4 − 8a3 − 6a2 + 2a + 1 = (a − 1)4 + a(13a3 − 4a2 − 12a + 6),
Symmetric Polynomial Inequalities in Nonnegative Variables 257

it suffices to prove that 13a3 − 4a2 − 12a + 6 ≥ 0. Indeed,

9(13a3 − 4a2 − 12a + 6) = 13(a + 1)(3a − 2)2 + 2(a − 1)2 + a2 > 0.

1
 ‹
The equality holds for (a, b, c) = 1, 1, or any cyclic permutation.
2

P 3.88. If a, b, c are positive real numbers such that a + b + c = 3, then

L
 ‹
bc ca a b

.M
4 + + + 9a bc ≥ 21.
a b c

D
Solution. Let p = a + b + c, q = a b + bc + ca, r = a bc. We write the required inequality

A
in the homogeneous form
4p2 q2
+ 9r ≥
5p3
PI
.
M
9r 3
For fixed p and r, it suffices to prove this inequality for the case when q is minimal; that
LY

is, when two of a, b, c are equal (see P 3.83). Due to symmetry and homogeneity, we
can set b = c = 1. Since p = a + 2, q = 2a + 1, r = a, the inequality becomes
O

4(a + 2)2 (2a + 1)2 + 81a2 ≥ 15(a + 2)3 ,


.M

which is equivalent to
(a − 1)2 (a − 4)2 ≥ 0.
W

The equality holds for a = b = c = 1, and for a = 2 and b = c = 1/2 (or any cyclic
W

permutation).
W

P 3.89. If a, b, c are nonnegative real numbers such that

a b + bc + ca = a bc + 2,

then
a2 + b2 + c 2 + a bc ≥ 4.

(Vasile Cîrtoaje, 2011)


258 Vasile Cîrtoaje

First Solution. Among the numbers 1 − a, 1 − b and 1 − c there are always two with
the same sign; let us say (1 − b)(1 − c) ≥ 0. Thus, we have

a(1 − b)(1 − c) ≥ 0,

a + a bc ≥ a b + ac,
a + (a b + bc + ca − 2) ≥ a b + ac,
a + bc ≥ 2,
and hence

L
a2 + b2 + c 2 + a bc − 4 ≥ a2 + 2bc + a bc − 4

.M
= (a + 2)(a + bc − 2) ≥ 0.
p
The equality holds for a = b = c = 1, and for a = 0 and b = c = 2 (or any cyclic

D
permutation).

A
Second Solution For a = 0, we need to show that bc = 2 involves b2 + c 2 ≥ 0. This is
true since PI
b2 + c 2 ≥ 2bc = 4.
M
Consider further that a, b, c are positive, and write the required inequality as
LY

a2 + b2 + c 2 + a bc ≥ 2(a b + bc + ca − a bc),
O

or
3a bc ≥ 2(a b + bc + ca) − a2 − b2 − c 2 .
.M

Let p = a + b + c, q = a b + bc + ca, r = a bc. We need to show that q = r + 2 implies


3r ≥ 4q − p2 . For fixed q and r, the sum p = a + b + c is minimal when two of a, b, c
W

are equal (see Remark 1 from P 3.83). Thus, it suffices to consider the case b = c, when
p = a + 2b, q = 2a b + b2 , r = a b2 . We need to prove that
W

2a b + b2 = a b2 + 2
W

implies
3a b2 ≥ 4(2a b + b2 ) − (a + 2b)2 ,
which reduces to
a(a + 3b2 − 4b) ≥ 0.
This is true since, for the nontrivial case b < 4/3, we have

2 − b2 (1 − b)2 (2 + 4b − 3b2 )
a + 3b2 − 4b = + 3b2 − 4b = ≥ 0.
b(2 − b) b(2 − b)
Symmetric Polynomial Inequalities in Nonnegative Variables 259

P 3.90. If a, b, c are positive real numbers, then


‹2
b+c p c + a p 2 a+b p 2
  ‹
−2− 2 + −2− 2 + − 2 − 2 ≥ 6.
a b c

(Vasile Cîrtoaje, 2012)


p
Solution. Let us denote m = 2 + 2. Without loss of generality, we can assume that
a = max{a, b, c}. We will prove first that
c + a 2  a + b ‹2 ‹2
2a

−m + −m ≥2 −m+1 . (*)
b c b+c

L
.M
According to the identity 2p2 + 2q2 = (p − q)2 + (p + q)2 , we have
‹2 
c+a a+b 2
c + a ‹2
2 a+b c+a a+b
 ‹ 

D
2 −m +2 −m = − + + − 2m .
b c b c b c

A
Thus, we can rewrite the inequality (*) as PI
‹2 ‹2 ‹2
c+a a+b 2a c+a a+b
  
M
− ≥4 −m+1 − + − 2m ,
b c b+c b c
LY

(a + b + c)2 (b − c)2 4a c+a a+b (a + b + c)(b − c)2


 ‹
+ + + − 4m + 2 ≥ 0.
b2 c 2 b+c bc(b + c)
O

b c
This is true if f (a) ≥ 0, where
.M

(a + b + c)(b + c) 4a c+a a+b


f (a) = + + + − 4m + 2.
bc b+c b c
W

b+c
 ‹
Since f (a) is increasing and a = max{a, b, c}, it suffices to show that f ≥ 0.
W

2
Indeed,
W

b+c 3(b − c)2 p p


 ‹
f = + 6 − 4 2 ≥ 6 − 4 2 > 0.
2 bc
Using (*), we only need to show that
‹2 ‹2
b+c 2a
 
−m +2 − m + 1 ≥ 6.
a b+c

b+c
Setting = t, this inequality becomes
a
‹2
2

(t − m)2 + 2 − m + 1 ≥ 6,
t
260 Vasile Cîrtoaje

p 2
(t − 2)2 (t − 2)
≥ 0.
t2
a
The proof is completed. The equality holds for a = b = c, and for p = b = c (or any
2
cyclic permutation).

P 3.91. If a, b, c are positive real numbers, then

2(a3 + b3 + c 3 ) + 9(a b + bc + ca) + 39 ≥ 24(a + b + c).

L
.M
(Vasile Cîrtoaje, 2010)

Solution. Let p = a + b + c and q = a b + bc + ca. Since a3 + b3 + c 3 = 3a bc + p3 − 3pq,

D
we can write the inequality as

A
6a bc + 2p3 + 3(3 − 2p)q + 39 ≥ 24p.
PI
By Schur’s inequality of degree three, we have
M

9a bc ≥ 4pq − p3 .
LY

Therefore, it suffices to show that


O

2
(4pq − p3 ) + 2p3 + 3(3 − 2p)q + 39 ≥ 24p,
.M

3
which is equivalent to
W

4p3 + 117 ≥ 72p + (10p − 27)q.


W

Case 1: 10p − 27 ≥ 0. Since 3q ≤ p2 , we have


W

(10p − 27)p2
4p3 + 117 − 72p − (10p − 27)q ≥ 4p3 + 117 − 72p −
3
1
= (p − 3)2 (2p + 39) ≥ 0.
3

Case 2: 10p − 27 < 0. From (3p − 8)2 ≥ 0, we get

9p2 − 48p + 64 ≥ 0,
X
18q ≥ −9 a2 + 48p − 64.
Symmetric Polynomial Inequalities in Nonnegative Variables 261

Using this inequality and


X
(10a − 9) = 10p − 27 < 0,

we get
” X — X € X Š
2 2 a3 + 9q + 39 − 24p ≥ 4 a3 + −9 a2 + 48p − 64 + 78 − 48p
X 14 14 14 X
‹ X ‹
= 4a3 − 9a2 + < 4a3 − 9a2 + + (10a − 9)
3 3 27
X  140 81 9 2
‹ X  ‹ X  ‹
= a 4a2 − 9a + > a 4a2 − 9a + = a 2a − ≥ 0.
27 16 4

L
The equality holds for a = b = c = 1.

D
.M
P 3.92. If a, b, c are positive real numbers such that a2 + b2 + c 2 = 3, then

A
PI
a3 + b3 + c 3 − 3 ≥ |(a − b)(b − c)(c − a)|.
M
Solution. Assume that a ≤ b ≤ c and write the inequality in the homogeneous form
LY

3/2
a + b2 + c 2
 2
3 3 3
a + b +c −3 ≥ |(a − b)(b − c)(c − a)|.
O

3
The left hand side is nonnegative because, by the Cauchy-Schwarz inequality, we have
.M

3(a3 + b3 + c 3 )2 = (1 + 1 + 1)(a3 + b3 + c 3 )(a3 + b3 + c 3 ) ≥ (a2 + b2 + c 2 )3 .


W

Thus, it suffices to consider that a < b < c. Using the substitution b = a + p and
c = a + q, where 0 < p < q, we need to show that
W

f (a) ≥ pq(q − p),


W

where
3/2
a2 + (a + p)2 + (a + q)2

3 3 3
f (a) = a + (a + p) + (a + q) − 3 .
3
From
v
t a2 + b2 + c 2
f 0 (a) = 3(a2 + b2 + c 2 ) − 3(a + b + c)
3
v ‚v Œ
t a2 + b2 + c 2 t a2 + b2 + c 2 a + b + c
=9 − ≥ 0,
3 3 3
262 Vasile Cîrtoaje

it follows that f is increasing, hence f (a) ≥ f (0). Therefore, it suffices to show that
f (0) ≥ pq(q − p); that is,
3/2
p2 + q2

3 3
p +q −3 ≥ pq(q − p).
3

Due to homogeneity, we may assume that p = 1 and q > 1, when the inequality becomes
as follows: 3/2
1 + q2

3 2
q −q +q+1≥3 ,
3

L
3(q3 − q2 + q + 1)2 ≥ (q2 + 1)3 ,

.M
q6 − 3q5 + 3q4 − 3q2 + 3q + 1 ≥ 0,
q3 (q − 1)3 + q3 − 3q2 + 3q + 1 ≥ 0,

D
(q3 + 1)(q − 1)3 + 2 ≥ 0.

A
The last inequality is clearly true. The equality holds for a = b = c = 1.
PI
M
LY

P 3.93. If a, b, c are nonnegative real numbers, then


O

a4 + b4 + c 4 − a2 b2 − b2 c 2 − c 2 a2 ≥ 2|a3 b + b3 c + c 3 a − a b3 − bc 3 − ca3 |.
.M

Solution. Assume that a ≤ b ≤ c and write the inequality as


W

(a2 − b2 )2 + (b2 − c 2 )2 + (c 2 − a2 )2 ≥ 4(a + b + c)(a − b)(b − c)(c − a).


W

Using the substitution b = a + p and c = a + q, where q ≥ p ≥ 0, the inequality can be


restated as
W

4Aa2 + 4Ba + C ≥ 0,
where
A = p2 − pq + q2 , B = p3 + q(p − q)2 ,
C = p4 + 2p3 q − p2 q2 − 2pq3 + q4 = (p2 + pq − q2 )2 .
Since A ≥ 0, B ≥ 0 and C ≥ 0, the inequality
p is obviously true. The equality occurs for
c 1+ 5
a = b = c, and also for a = 0 and = (or any cyclic permutation).
b 2
Symmetric Polynomial Inequalities in Nonnegative Variables 263

P 3.94. If a, b, c are nonnegative real numbers, then


p
a4 + b4 + c 4 − a bc(a + b + c) ≥ 2 2 |a3 b + b3 c + c 3 a − a b3 − bc 3 − ca3 |.

(Pham Kim Hung, 2006)

Solution. Assume that a ≤ b ≤ c and write the inequality as


p
a2 (a2 − bc) + b2 (b2 − ca) + c 2 (c 2 − a b) ≥ 2 2(a + b + c)(a − b)(b − c)(c − a).

Using the substitution b = a + p and c = a + q, where q ≥ p ≥ 0, the inequality becomes

Aa2 + Ba + C ≥ 0,

L
.M
where
p p
A = 5(p2 − pq + q2 ), B = 4p3 + (6 2 − 1)p2 q − (6 2 + 1)pq2 + 4q3 ,

D
p
C = p4 + q4 + 2 2pq(p2 − q2 ).

A
Since
A ≥ 0,
PI
M
‹2
25 2 5p

2 3
B≥ p q − 10pq + 4q = q − 2q ≥ 0
LY

4 2
and p
O

C = (p2 + 2pq − q2 )2 ≥ 0,

p follows. The equality occurs for a = b = c, and also for a = 0 and


the conclusion
.M

p
c 2+ 6
= (or any cyclic permutation).
b 2
W
W

P 3.95. If a, b, c ≥ −5 such that a + b + c = 3, then


W

1−a 1− b 1−c
+ + ≥ 0.
1 + a + a2 1 + b + b2 1 + c + c 2
(Vasile Cîrtoaje, 2014)

First Solution. Using the substitution

a = x − 5, b = y − 5, c = z − 5,

we need to prove that if x, y, z ≥ 0 such that x + y + z = 18, then


6− x 6− y 6−z
+ 2 + 2 ≥ 0.
x2 − 9x + 21 y − 9 y + 21 z − 9z + 21
264 Vasile Cîrtoaje

Denoting
x + y +z
p= ,
18
we can write this inequality as f5 (x, y, z) ≥ 0, where
X
f5 (x, y, z) = (6p − x)( y 2 − 9 y p + 21p2 )(z 2 − 9z p + 21p2 )

is a symmetric homogeneous polynomial of degree 5. According to P 3.68, it suffices


to prove this inequality for y = z and for x = 0. Therefore, we only need to prove the
original inequality for b = c and for a = −5.
3−a
Case 1: b = c =

L
. Since
2

.M
1− b 1−c 2(a − 1)
= = 2 ,
1+ b+ b 2 1+c+c 2 a − 8a + 19

D
we need to show that

A
1−a 4(a − 1)
+ 2 ≥ 0,
1+a+a 2 a − 8a + 19 PI
which is equivalent to
M
(a − 1)2 (a + 5) ≥ 0.
LY

Case 2: a = −5, b + c = 8. We can write the desired inequality as follows:


O

1 1− b 1 1−c
 ‹  ‹
+ + + ≥ 0,
7 1 + b + b2 7 1 + c + c2
.M

(b − 4)(b − 2) (c − 4)(c − 2)
+ ≥ 0,
1 + b + b2 1 + c + c2
W

b−c b−2 c−2


 ‹
− ≥ 0,
2 1 + b + b2 1 + c + c 2
W

(b − c)2 [3 + 2(b + c) − bc]


≥ 0.
W

2(1 + b + b2 )(1 + c + c 2 )
The last inequality is true since
‹2 ‹2
b+c b−c
 
2(b + c) − bc = − bc = ≥ 0.
2 2

The proof is completed. The equality occurs for a = b = c = 1, and also for a = −5 and
b = c = 4 (or any cyclic permutation).
Second Solution. Assume that a ≤ b ≤ c and denote
b+c 1−a 1− b 1−c
t= , E(a, b, c) = + + .
2 1+a+a 2 1+ b+ b 2 1 + c + c2
Symmetric Polynomial Inequalities in Nonnegative Variables 265

From
a+b+c 3−a
t≥ = 1, t= ≤ 4,
3 2
it follows that
t ∈ [1, 4].

We will show that


E(a, b, c) ≥ E(a, t, t) ≥ 0.

Write the left inequality as follows:

L
1− b 1− t 1−c 1− t
 ‹  ‹
− + − ≥ 0,
1 + b + b2 1 + t + t 2 1 + c + c2 1 + t + t 2

.M
(b − 1)t − b − 2 (c − 1)t − c − 2
• ˜
(b − c)

D
− ≥ 0,
1 + b + b2 1 + c + c2

A
(b − c)2 [(2 + b + c − bc)t + 1 + 2(b + c) + bc] ≥ 0,
PI
(b − c)2 [2t 2 + 6t + 1 − bc(t − 1)] ≥ 0.
M
The last inequality is true since
LY

2t 2 + 6t + 1 − bc(t − 1) ≥ 2t 2 + 6t + 1 − t 2 (t − 1)
O

= t(4 − t)(1 + t) + 2t + 1 > 0.


.M

Also, we have
W

1−a 2(1 − t) 2(t − 1) 2(1 − t)


E(a, t, t) = + = 2 +
1+a+a 2 1+ t + t 2 4t − 14t + 13 1 + t + t 2
W

6(1 − t)2 (4 − t)
=
W

≥ 0.
(4t 2 − 14t + 13)(1 + t + t 2 )

1 4
P 3.96. Let a, b, c 6= be nonnegative real numbers such that a + b + c = 3. If k ≥ ,
k 3
then
1−a 1− b 1−c
+ + ≥ 0.
(1 − ka)2 (1 − k b)2 (1 − kc)2
(Vasile Cîrtoaje, 2012)
266 Vasile Cîrtoaje

First Solution. Denoting p = (a + b + c)/3, we may write the inequality as f5 (x, y, z) ≥


0, where X
f5 (x, y, z) = (p − a)(p − k b)2 (p − kc)2
is a symmetric homogeneous polynomial of degree 5. According to P 3.68, it suffices to
prove this inequality for b = c and for a = 0.
Case 1: b = c. Since a = 3 − 2b, the original inequality is equivalent to the following
sequence of inequalities:
1−a 2(1 − b)
+ ≥ 0,
(1 − ka)2 (1 − k b)2
2(b − 1) 2(1 − b)
+

L
≥ 0,
(1 − 3k + 2k b)2 (1 − k b)2

.M
k(b − 1)2 [k(3 − b) − 2] ≥ 0.
The last inequality holds since

D
4 2(3 − 2b) 2a

A
k(3 − b) − 2 ≥ (3 − b) − 2 = = ≥ 0.
3 3 3
PI
Case 2: a = 0. Since b + c = 3, the original inequality becomes as follows:
M
b−1 c−1
1≥ + ,
(1 − k b)2 (1 − kc)2
LY

(1 − k b)2 (1 − kc)2 ≥ (b − 1)(1 − kc)2 + (c − 1)(1 − k b)2 ,


O

(k2 bc − 3k + 1)2 ≥ 1 + 6k − 9k2 + (5k2 − 4k)bc,


.M

k3 b2 c 2 + 18k − 12 ≥ (6k2 + 3k − 4)bc.


Since Æ
k3 b2 c 2 + 18k − 12 ≥ 2 k3 (18k − 12) bc,
W

it suffices to show that


W

4k3 (18k − 12) ≥ (6k2 + 3k − 4)2 ,


W

which is equivalent to

36k4 − 84k3 + 39k2 + 24k − 16 ≥ 0,

(3k − 4)(12k3 − 12k2 − 3k + 4) ≥ 0.


The last inequality holds since

12k3 − 12k2 − 3k + 4 > 12k3 − 12k2 − 4k + 4 = 4(k − 1)(3k2 − 1) > 0.

The proof is completed. The equality occurs for a = b = c = 1. If k = 4/3, then the
equality holds also for a = 0 and b = c = 3/2 (or any cyclic permutation).
Symmetric Polynomial Inequalities in Nonnegative Variables 267

P 3.97. Let a, b, c, d be nonnegative real numbers such that

a2 + b2 + c 2 + d 2 = 1.

Prove that
(1 − a)(1 − b)(1 − c)(1 − d) ≥ a bcd.
(Vasile Cîrtoaje, 2001)

Solution. The desired inequality follows by multiplying the inequalities

(1 − a)(1 − b) ≥ cd,

L
.M
(1 − c)(1 − d) ≥ a b.
With regard to the first inequality, we have

D
2cd ≤ c 2 + d 2 = 1 − a2 − b2 ,

A
and hence PI
M
2(1 − a)(1 − b) − 2cd ≥ 2(1 − a)(1 − b) − 1 + a2 + b2
= (1 − a − b)2 ≥ 0.
LY

The second inequality can be proved similarly. The equality holds for a = b = c = d =
O

1/2, and also for a, b, c, d) = (1, 0, 0, 0) or any cyclic permutation.


.M
W

P 3.98. Let a, b, c, d and x be positive real numbers such that


W

1 1 1 1 4
2
+ 2 + 2 + 2 = 2.
a b c d x
W

If x ≥ 2, then
(a − 1)(b − 1)(c − 1)(d − 1) ≥ (x − 1)4 .
(Vasile Cîrtoaje, 2001)

Solution. The desired inequality follows by multiplying the inequalities


 ‹
ab
2(a − 1)(b − 1) ≥ (x − 1) x + x −2 ,
cd
 ‹
cd
2(c − 1)(d − 1) ≥ (x − 1) x + x −2 ,
ab
268 Vasile Cîrtoaje

 ‹ ‹
ab cd
x + x −2 x + x − 2 ≥ 4(x − 1)2 .
cd ab
With regard to the first inequality, we write it as

ab
2a b − 2(a + b) + x(3 − x) ≥ x(x − 1) .
cd
Since
2 1 1 4 1 1
≤ 2 + 2 = 2 − 2 − 2,
cd c d x a b
it suffices to show that

L
4 1 1
 ‹
4a b − 4(a + b) + 2x(3 − x) ≥ x(x − 1)a b

.M
− − .
x 2 a2 b2

This is equivalent to

D
A
4a2 b2 − 4a b(a + b)x + 2x 2 (3 − x)a b + x 2 (x − 1)(a2 + b2 ) ≥ 0,

which can be written in the obvious form


PI
M
[2a b − (a + b)x]2 + x 2 (x − 2)(a − b)2 ≥ 0.
LY

The second inequality can be proved similarly. With regard to the third inequality, we
have
O

 ‹ ‹
ab cd
x + x −2 x + x −2 =
.M

cd ab
 ‹
2 a b cd
= 2x − 4x + 4 + + x(x − 2)
cd a b
W

≥ 2x 2 − 4x + 4 + 2x(x − 2) = 4(x − 1)2 .


W

The equality holds for a = b = c = d = x.


W

Remark. Setting x = 2 and substituting a, b, c, d by 1/a, 1/b, 1/c, 1/d, respectively, we


get the inequality from P 3.97.

P 3.99. If a, b, c, d are positive real numbers, then

(1 + a3 )(1 + b3 )(1 + c 3 )(1 + d 3 ) 1 + a bcd


≥ .
(1 + a2 )(1 + b2 )(1 + c 2 )(1 + d 2 ) 2

(Vasile Cîrtoaje, 1992)


Symmetric Polynomial Inequalities in Nonnegative Variables 269

Solution. For a = b = c = d, the inequality can be written as


4
1 + a3 1 + a4

≥ .
1 + a2 2

We will show that 4 2


1 + a3 1 + a3 1 + a4
 
≥ ≥ .
1 + a2 1+a 2
The left side inequality is equivalent to

(1 + a3 )(1 + a) ≥ (1 + a2 )2 ,

L
.M
which reduces to a(1 − a)2 ≥ 0, while the right side inequality is equivalent to

D
2(1 − a + a2 )2 ≥ 1 + a4 ,

A
which reduces to (1 − a)4 ≥ 0. Multiplying the inequalities
4
PI 4
1 + a3 1 + a4 1 + b3 1 + b4
 
M
≥ , ≥ ,
1 + a2 2 1 + b2 2
LY

4 4
1 + c3 1 + c4 1 + d3 1 + d4
 
≥ , ≥ ,
O

1 + c2 2 1 + d2 2
.M

yields

(1 + a3 )(1 + b3 )(1 + c 3 )(1 + d 3 ) 1 Æ


4
≥ (1 + a4 )(1 + b4 )(1 + c 4 )(1 + d 4 ).
W

(1 + a2 )(1 + b2 )(1 + c 2 )(1 + d 2 ) 2


W

Applying the Cauchy-Schwarz inequality produces


W

(1 + a4 )(1 + b4 )(1 + c 4 )(1 + d 4 ) ≥ (1 + a2 b2 )2 (1 + c 2 d 2 )2 ≥ (1 + a bcd)4 ,

from which the desired inequality follows. The equality holds for a = b = c = d = 1.

P 3.100. Let a, b, c, d be positive real numbers such that a + b + c + d = 4. Prove that

1 1 1 1 1 1 1 1
 ‹ ‹ ‹ ‹
a+ −1 b+ −1 c+ −1 d + −1 +3≥ + + + .
a b c d a b c d
270 Vasile Cîrtoaje

Solution. Write the inequality as


Y• 1 1
 ‹˜ X
1+ a+ −2 ≥ − 3.
a a
Since
1 1
a+ − 2 ≥ 0, b + − 2 ≥ 0,
a b
1 1
c + − 2 ≥ 0, d + − 2 ≥ 0,
c d
applying Bernoulli’s inequality, it suffices to show that
X 1 1
‹ X

L
1+ a+ −2 ≥ − 3.
a a

.M
This is an identity, and then the proof is completed. The equality holds for a = b = c =
d = 1.

D
A
P 3.101. If a, b, c, d are nonnegative real numbers, then
PI
M
4(a3 + b3 + c 3 + d 3 ) + 15(a bc + bcd + cd a + d a b) ≥ (a + b + c + d)3 .
LY

Solution. Let
O

E(a, b, c, d) = 4(a3 + b3 + c 3 + d 3 ) + 15(a bc + bcd + cd a + d a b) − (a + b + c + d)3 .


.M

Without loss of generality, assume that a ≤ b ≤ c ≤ d. We will show that

E(a, b, c, d) ≥ E(0, a + b, c, d) ≥ 0.
W

We have
W

E(a, b, c, d) − E(0, a + b, c, d) = 4[a3 + b3 − (a + b)3 ] + 15a b(c + d)


W

= 3a b[5(c + d) − 4(a + b)] ≥ 0.

Now, putting x = a + b, we need to show that E(0, x, c, d) ≥ 0, where

E(0, x, c, d) = 4(x 3 + c 3 + d 3 ) + 15x cd − (x + c + x)3 .

This is equivalent to Schur’s inequality

x 3 + c 3 + d 3 + 3x cd ≥ x c(x + c) + cd(c + d) + d x(d + x).

The equality holds for a = 0 and b = c = d (or any cyclic permutation), and also for
a = b = 0 and c = d (or any permutation thereof).
Symmetric Polynomial Inequalities in Nonnegative Variables 271

P 3.102. Let a, b, c, d be positive real numbers such that

a + b + c + d = 4.

Prove that
1 + 2(a bc + bcd + cd a + d a b) ≥ 9 min{a, b, c, d}.
(Vasile Cîrtoaje, 2008)

Solution. Assume that a = min{a, b, c, d} and use the substitutions b = a+ x, c = a+ y,


d = a + z and t = x + y + z, where x, y, z, t ≥ 0. First, write the inequality in the
homogeneous forms

L
(a + b + c + d)3 + 128bcd + 128a(bc + cd + d b) ≥ 36a(a + b + c + d)2 ,

.M
(4a + t)3 + 128bcd + 128a(bc + cd + d b) ≥ 36a(4a + t)2 .

D
Since

A
bcd = (a + x)(a + y)(a + z) ≥ a3 + a2 t
and PI
M
bc + cd + d b = (a + x)(a + y) + (a + y)(a + z) + (a + z)(a + x)
= 3a2 + 2at + x y + yz + z x ≥ 3a2 + 2at,
LY

it suffices to prove that


O

(4a + t)3 + 128(a3 + a2 t) + 128a(3a2 + 2at) ≥ 36a(4a + t)2 .


.M

This inequality is equivalent to


W

t(t − 12a)2 ≥ 0.
W

1 1 1 13
 ‹
The equality holds for a = b = c = d = 1, and also for (a, b, c, d) = , , , or
4 4 4 4
W

any cyclic permutation.

P 3.103. Let a, b, c, d be nonnegative real numbers such that

a + b + c + d = 4.

Prove that
5(a2 + b2 + c 2 + d 2 ) ≥ a3 + b3 + c 3 + d 3 + 16.
(Vasile Cîrtoaje, 2005)
272 Vasile Cîrtoaje

Solution. Assume that a ≥ b ≥ c ≥ d.


First Solution. We use the mixing variable method. Setting x = (b + c + d)/3, we have
a + 3x = 4 and x ≤ 1. We will show that

E(a, b, c, d) ≥ E(a, x, x, x) ≥ 0,

where
E(a, b, c, d) = 5(a2 + b2 + c 2 + d 2 ) − a3 − b3 − c 3 − d 3 − 16.
The left side inequality is equivalent to

5(b2 + c 2 + d 2 − 3x 2 ) − (b3 + c 3 + d 3 − 3x 3 ) ≥ 0.

L
.M
Since b2 + c 2 + d 2 − 3x 2 ≥ 0 and x ≤ 1, it suffices to prove the homogeneous inequality

5x(b2 + c 2 + d 2 − 3x 2 ) − (b3 + c 3 + d 3 − 3x 3 ) ≥ 0,

D
which is equivalent to

A
PI
2(b3 + c 3 + d 3 ) + 3[b(c 2 + d 2 ) + c(d 2 + b2 ) + d(b2 + c 2 )] ≥ 24bcd.

This is true, since b3 + c 3 + d 3 ≥ 3bcd and


M

b(c 2 + d 2 ) + c(d 2 + b2 ) + d(b2 + c 2 ≥ 2bcd + 2cd b + 2d bc = 6bcd.


LY

The right side inequality is also true, since


O

E(a, x, x, x) = 5(a2 + 3x 2 ) − a3 − 3x 3 − 16
.M

= 5(4 − 3x)2 + 15x 2 − (4 − 3x)3 − 3x 3 − 16


= 24x(x − 1)2 ≥ 0.
W

This completes the proof. The equality holds for a = b = c = d = 1, and also for
W

(a, b, c, d) = (0, 0, 0, 4) or any cyclic permutation.


W

Second Solution. Write the inequality as


X
(5a2 − a3 − 7a + 3) ≥ 0,
X
(1 − a)2 (3 − a) ≥ 0.
For a ≤ 3, the inequality is clearly true. Otherwise, for 3 < a ≤ 4, which involves b ≤ 1,
c ≤ 1, d ≤ 1, we get the required inequality by summing the inequalities

5a2 ≥ a3 + 16

and
5(b2 + c 2 + d 2 ) ≥ b3 + c 3 + d 3 .
Symmetric Polynomial Inequalities in Nonnegative Variables 273

We have

5a2 − a3 − 16 = (4 − a)(a2 − a − 4) = (4 − a)[a(a − 3) + 2(a − 2)] ≥ 0,

and
5(b2 + c 2 + d 2 ) ≥ b2 + c 2 + d 2 ≥ b3 + c 3 + d 3 .

Third Solution. Write the inequality as


X
(a3 − 5a2 + 4a) ≤ 0,

L
or

.M
X
(1 − a) f (a) ≤ 0,
where

D
f (a) = a2 − 4a.

A
Since a + b ≤ 4, we have
PI
f (a) − f (b) = (a − b)(a + b − 4) ≤ 0,
M
and, similarly,
LY

f (b) − f (c) ≤ 0, f (c) − f (d) ≤ 0.


Since
O

a−1≥ b−1≥ c−1≥ d −1


.M

and
f (a) ≤ f (b) ≤ f (c) ≤ f (d),
W

by Chebyshev’s inequality, we get


W

X ”X — ”X —
4 (a − 1) f (a) ≤ (a − 1) f (a) = 0.
W

Remark. Similarly, we can prove the following generalization.


• If a1 , a2 , . . . , an are nonnegative real numbers such that

a1 + a2 + · · · + an = n,

then
(n + 1)(a12 + a22 + · · · + an2 ) ≥ a13 + a23 + · · · + an3 + n2 .
274 Vasile Cîrtoaje

P 3.104. Let a, b, c, d be nonnegative real numbers such that

a + b + c + d = 4.

Prove that
3(a2 + b2 + c 2 + d 2 ) + 4a bcd ≥ 16.
(Vasile Cîrtoaje, 2004)
Solution. We use the mixing variable method. Assume that a = min{a, b, c, d}, a ≤ 1.
Setting x = (b + c + d)/3, we have a + 3x = 4 and x ≤ 4/3. We will show that

E(a, b, c, d) ≥ E(a, x, x, x) ≥ 0,

L
.M
where
E(a, b, c, d) = 3(a2 + b2 + c 2 + d 2 ) + 4a bcd − 16.

D
The left side inequality is equivalent to

A
3(3x 2 − bc − cd − d b) ≥ 2a(x 3 − bcd).

By Schur’s inequality PI
M
(b + c + d)3 + 9bcd ≥ 4(b + c + d)(bc + cd + d b),
LY

we get
4x
x 3 − bcd ≤ (3x 2 − bc − cd − d b).
O

3
Therefore, it suffices to prove that
.M

8a x
3(3x 2 − bc − cd − d b) ≥ (3x 2 − bc − cd − d b);
3
W

that is,
(3x 2 − bc − cd − d b)(9 − 8a x) ≥ 0.
W

This is true, since


W

6(3x 2 − bc − cd − d b) = (b − c)2 + (c − d)2 + (d − b)2 ≥ 0,

and
3(9 − 8a x) = 27 − 8a(4 − a) = 8(1 − a)2 + 16(1 − a) + 3 > 0.
The right side inequality is also true, since

E(a, x, x, x) = 3a2 + 9x 2 + 4a x 3 − 16
= 3(4 − 3x)2 + 9x 2 + 4(4 − 3x)x 3 − 16
= 4(8 − 18x + 9x 2 + 4x 3 − 3x 4 )
= 4(1 − x)2 (2 + x)(4 − 3x) ≥ 0.
Symmetric Polynomial Inequalities in Nonnegative Variables 275

This completes ‹ The equality holds for a = b = c = d = 1, and also for


 the proof.
4 4 4
(a, b, c, d) = 0, , , or any cyclic permutation.
3 3 3
Remark. The following generalization holds (Vasile Cîrtoaje, 2005).
• Let a1 , a2 , . . . , an (n ≥ 3) be nonnegative real numbers such that

a1 + a2 + · · · + an = n.

If k is a positive integer satisfying 2 ≤ k ≤ n + 2, and


 n k−1
r= − 1,

L
n−1

.M
then
a1k + a2k + · · · + ank − n ≥ nr(1 − a1 a2 · · · an ).

D
A
PI
P 3.105. Let a, b, c, d be nonnegative real numbers such that
M
a + b + c + d = 4.
LY

Prove that
O

27(a bc + cd + cd a + d a b) ≤ 44a bcd + 64.


.M

Solution. Use the mixing variable method. Without loss of generality, assume that a ≥
b ≥ c ≥ d. Setting x = (a + b + c)/3, we have 3x + d = 4, d ≤ x ≤ 4/3 and x 3 ≥ a bc.
W

We will show that


W

E(a, b, c, d) ≥ E(x, x, x, d) ≥ 0.
The left inequality is equivalent to
W

27[(x 3 − a bc) + d(3x 2 − a b − bc − ca)] ≥ 44d(x 3 − a bc).

By Schur’s inequality

(a + b + c)3 + 9a bc ≥ 4(a + b + c)(a b + bc + ca),

we get
9x 3 + 3a bc ≥ 4x(a b + bc + ca),
and hence
3(x 3 − a bc)
3x 2 − a b − bc − ca ≥ ≥ 0.
4x
276 Vasile Cîrtoaje

Therefore, it suffices to prove that

3d
27(1 + ) ≥ 44d.
4x
Write this inequality in the homogeneous form

27(3x + d)(4x + 3d) ≥ 704x d,

or, equivalently,
81(4x 2 + d 2 ) ≥ 353x d.
This inequality is true, since

L
.M
81(4x 2 + d 2 ) − 353x d ≥ 81(4x 2 + d 2 − 5ut) = 81(x − d)(4x − d) ≥ 0.

D
The right inequality E(x, x, x, t) ≥ 0 is also true, since

A
E(x, x, x, d) = (44x 3 − 81x 2 )d − 27x 3 + 64
PI
= 4(16 − 81x 2 + 98x 3 − 33x 4 )
= 4(1 − x)2 (16 + 32x − 33x 2 )
M
= 4(1 − x)2 (4 − 3x)(4 + 11x) ≥ 0.
LY

This completes ‹ The equality holds for a = b = c = d = 1, and also for


 the proof.
O

4 4 4
(a, b, c, d) = 0, , , or any cyclic permutation.
3 3 3
.M
W

P 3.106. Let a, b, c, d be positive real numbers such that


W

1 1 1 1
a+b+c+d = + + + .
a b c d
W

Prove that
1 1 1 1
 ‹
2 2 2 2
(1 − a bcd) a + b + c + d − 2 − 2 − 2 − 2 ≥ 0.
a b c d
(Vasile Cîrtoaje, 2007)

Solution. From ‹2
1 1 1 1

2
(a + b + c + d) = + + + ,
a b c d
we get
X X 1 X 1 X
a2 − = 2 − 2 ab
a2 s ym
ab s ym
Symmetric Polynomial Inequalities in Nonnegative Variables 277

X 1 ‹ X 1
=2 − cd = 2(1 − a bcd) .
s ym
ab s ym
ab
Thus, the inequality can be restated as
X 1
2(1 − a bcd)2 ≥ 0,
s ym
ab

which is obviously true. The equality holds for a b = cd = 1, or ac = bd = 1, or


ad = bc = 1.
Conjecture. If a, b, c, d are positive real numbers such that

L
1 1 1 1
a+b+c+d = + + + ,

.M
a b c d
then

D
1 1 1 1
 ‹
n n n n
(1 − a bcd) a + b + c + d − n − n − n − n ≥0
a b c d

A
for any integer n ≥ 2.
PI
M
LY

P 3.107. Let a, b, c, d be positive real numbers such that

a + b + c + d = 1.
O

Prove that
.M

1 1 1 1 81
 ‹
(1 − a)(1 − b)(1 − c)(1 − d) + + + ≥ .
a b c d 16
W

(Keira, 2007)

Solution. Write the inequality as


W

81
W

E(a, b, c, d) ≥ ,
16
where
1 1 1 1
 ‹
E(a, b, c, d) = (1 − a)(1 − b)(1 − c)(1 − d) + + + .
a b c d
Without loss of generality, assume that a ≤ b ≤ c ≤ d. First, we show that for a ≤ b ≤
c ≤ d and a + b + c + d = 1, F (a, b, c, d) is minimal when a = c. This is true if
a+c a+c
E(a, b, c, d) ≥ E( , b, , d).
2 2
Since
(1 − a)(1 − c) = 1 − a − c + ac = b + d + ac
278 Vasile Cîrtoaje

and
(1 − b)(1 − d) = 1 − b − d + bd = a + c + bd,

we have
a+c b+d
 ‹
E(a, b, c, d) = (b + d + ac)(a + c + bd) + ,
ac bd
and the inequality is equivalent to

a+c b+d  a + c 2 ˜  4 b+d


 ‹ • ‹
(b + d + ac) + ≥ b+d+ + ,
ac bd 2 a+c bd

L
or
4bd
 ‹

.M
(a − c)2 − a − c ≥ 0.
ac
Since

D
4bd
− a − c ≥ 4 − a − c = 3 + b + d > 0,

A
ac
PI
the last inequality is clearly true. Since E(a, b, c, d) is minimal when a = c, from a ≤
b ≤ c ≤ d it follows that E(a, b, c, d) is minimal when a = b = c. Therefore, it suffices
M
to prove that 3a + d = 4 involves
LY

81
E(a, a, a, d) ≥ .
16
O

This is equivalent to
.M

21d 4 + 61d 3 − 57d 2 − 153d + 128 ≥ 0,


W

(d − 1)2 (21d 2 + 103d + 128) ≥ 0.


W

The equality holds for a = b = c = d = 1/4.


W

P 3.108. Let a, b, c, d be nonnegative real numbers such that

a + b + c + d = a3 + b3 + c 3 + d 3 = 2.

Prove that
7
a2 + b2 + c 2 + d 2 ≥ .
4
(Vasile Cîrtoaje, 2010)
Symmetric Polynomial Inequalities in Nonnegative Variables 279

Solution. Let us denote x = a2 + b2 + c 2 + d 2 . From


1 1
2 = a3 + b3 + c 3 + d 3 ≥ a3 + (b + c + d)3 = a3 + (2 − a)3 ,
9 9
5 5
it follows that (4a − 5)(a + 1)2 ≤ 0, and hence a ≤ . Similarly, we have b, c, d ≤ .
4 4
On the other hand,
X X X X
5x = 5 a2 = 4 a3 + (5a2 − 4a3 ) = 8 + a2 (5 − 4a)

and, by the Cauchy-Schwarz inequality,

L
[ a(5 − 4a)]2 (5 a − 4 a2 )2
P P P
X (5 − 2x)2

.M
2
a (5 − 4a) ≥ P = = .
(5 − 4a)
P
20 − 4 a 3

D
Therefore, we have
(5 − 2x)2
5x ≥ 8 +

A
.
3
PI
7
This is equivalent to (4x − 7)(x − 7) ≤ 0, and involves x ≥ . The equality holds for
4
M
1 1 1 7
 ‹
(a, b, c, d) = , , , or any cyclic permutation.
12 12 12 4
LY
O

P 3.109. Let a, b, c, d ∈ (0, 4] such that a bcd = 1. Prove that


.M

(1 + 2a)(1 + 2b)(1 + 2c)(1 + 2d) ≥ (5 − 2a)(5 − 2b)(5 − 2c)(5 − 2d).


W

(Vasile Cîrtoaje, 2011)


W

Solution. Assume that a ≥ b ≥ c ≥ d. For the nontrivial case where the right side of
the inequality is positive, there are two cases to consider.
W

Case 1: a < 5/2. In virtue of the AM-GM inequality, we have


3 3 3 3
p p p p
(1 + 2a)(1 + 2b)(1 + 2c)(1 + 2d) ≥ (3 a2 )(3 b2 )(3 c 2 )(3 d 2 ) = 81,

˜4
(5 − 2a) + (5 − 2b) + (5 − 2c) + (5 − 2d)
•
(5 − 2a)(5 − 2b)(5 − 2c)(5 − 2d) ≤
4
˜4  p
4 4
10 − (a + b + c + d) 10 − 4 a bcd
•
= ≤ = 81,
2 2

from which the conclusion follows.


280 Vasile Cîrtoaje

Case 2: a ≥ b > 5/2 > c ≥ d. Write the inequality as

(1 + 2a)(1 + 2b) (5 − 2c)(5 − 2d)


≥ ,
(2a − 5)(2b − 5) (1 + 2c)(1 + 2d)

1 + 4a b + 2(a + b) 25 + 4cd − 10(c + d)


≥ .
25 + 4a b − 10(a + b) 1 + 4cd + 2(c + d)
According to the AM-GM inequality, it suffices to prove that
p p
1 + 4a b + 4 a b 25 + 4cd − 20 cd
p ≥ p .
25 + 4a b − 20 a b 1 + 4cd + 4 cd

L
.M
This is equivalent to
p p
2 a b + 1 5 − 2 cd
p ≥ p ,

D
2 a b − 5 1 + 2 cd
p p

A
2 ab + 1 5 ab − 2
p ≥ p ,
2 ab − 5
p p
ab + 2PI
M
(4 a b − 1)(4 − a b) ≥ 0.
p
The last inequality is true, since a, b ∈ (0, 4] involves 4 − a b ≥ 0.
LY

The equality holds for a = b = c = d = 1, and for (a, b, c, d) = (4, 4, 1/4, 1/4) or any
O

permutation thereof.
.M
W

P 3.110. Let a, b, c, d and k be positive real numbers such that


W

1 1 1 1
 ‹
(a + b + c + d) + + + = k.
a b c d
W

p
If 16 ≤ k ≤ (1 + 10)2 , then any three of a, b, c, d are the lengths of the sides of a triangle
(non-degenerate or degenerate).

Solution. The condition k ≥ 16 follows from the known inequality

1 1 1 1
 ‹
(a + b + c + d) + + + ≥ 16.
a b c d

Without loss of generality, assume that a ≥ b ≥ c ≥ d. Clearly, any three of a, b, c, d


are the lengths of the sides of a triangle if and only if a ≤ c + d. By virtue of the
Symmetric Polynomial Inequalities in Nonnegative Variables 281

Cauchy-Schwarz inequality, we have


p 1 1 1 1
 ‹
2
(1 + 10) ≥ (b + a + c + d) + + +
b a c d
– v ‹™2
1 1 1
t 
≥ 1 + (a + c + d) + +
a c d
– v ‹™2
1 4
t 
≥ 1 + (a + c + d) + ,
a c+d

and hence

L
1 4 p
 ‹ Æ
(a + c + d) + ≤ ( 11 + 2 10 − 1)2 = 10.

.M
a c+d
Writing this inequality as
(a − c − d)(4a − c − d) ≤ 0,

D
A
we get a ≤ c + d. Thus, the proof is completed.
p
PI
Remark 1. The interval [16, 11 + 2 10] is the largest possible range of k such that any
three of a, b, c, d are the lengths of the sides ofpa triangle. In order to prove this, for the
M
sake of contradiction, assume that k > 11 + 2 10. The given relation
LY

1 1 1 1
 ‹
(a + b + c + d) + + + =k
a b c d
O

v
t a(a + 2)
is satisfied for a = p + p2 − 1, b = and c = d = 1, where
p
.M

2a + 1
p
( k − 1)2 − 5
W

p= .
4
p
W

If k > 11 + 2 10, then we get p > 5/4 and a > 2. Clearly, the numbers a, c and d are
not the lengths of the sides of a triangle.
W

Remark 2. In the same manner, we can prove the following generalization.


• Let a1 , a2 , . . . , an be positive real numbers such that

1 1 1
 ‹
(a1 + a2 + · · · + an ) + + ··· + = k.
a1 a2 an
p
If n ≥ 3 and n2 ≤ k ≤ (n + 10 − 3)2 , then any three of a1 , a2 , . . . , an are the lengths of
the sides of a triangle.
282 Vasile Cîrtoaje

P 3.111. Let a, b, c, d and k be positive real numbers such that

1 1 1 1
 ‹
(a + b + c + d) + + + = k.
a b c d
119
If 16 ≤ k ≤ , then there exist three numbers of a, b, c, d which are the lengths of the
6
sides of a triangle (non-degenerate or degenerate).

(Vasile Cîrtoaje, 2010)

Solution. The condition k ≥ 16 follows from the known inequality

L
1 1 1 1
 ‹

.M
(a + b + c + d) + + + ≥ 16.
a b c d
Without loss of generality, assume that a ≥ b ≥ c ≥ d. We need to show that either

D
a ≤ b + c or b ≤ c + d. For the sake of contradiction, consider that a > b + c and

A
b > c + d. To complete the proof, it suffices to show that k > 119/6; that is,

(a + b + c + d)

1 1 1 1
+ + +
PI
‹
>
119
.
M
a b c d 6
Notice that for a = 3, b = 2 and c = d = 1, we have a = b + c, b = c + d and
LY

1 1 1 1 119
 ‹
(a + b + c + d) + + + = .
O

a b c d 6
.M

Then, we apply the Cauchy-Schwarz inequality in the following manner

9 16
 ‹
[a + (b + c + d)] + ≥ (3 + 4)2 = 49.
a b+c+d
W

Thus, it suffices to show that


W

1 1 1 1 17 9 16
 ‹
+ + + > +
W

.
a b c d 42 a b+c+d
Since
1 1 1 1 4
 ‹  ‹
(b + c + d) + + − 10 ≥ (b + c + d) + − 10
b c d b c+d
(b − c − d)(4b − c − d)
= > 0,
b(c + d)
we need to prove that

1 10 17 9 16
 ‹
+ ≥ + .
a b+c+d 42 a b+c+d
Symmetric Polynomial Inequalities in Nonnegative Variables 283

This is equivalent to
4a ≥ 3(b + c + d),
which is true, since

4a − 3(b + c + d) = 4(a − b − c) + (b − c − d) + 2(c − d) > 0.

Remark. The interval [16, 119/6] is the largest domain of k such that among a, b, c, d
there exist three numbers as lengths of the sides of a non-degenerate or degenerate tri-
angle. In the case of the non-degenerate triangles, the largest domain of k is [16, 119/6).
To prove this, for the sake of contradiction, assume that k ≥ 119/6. The hypothesis

L
1 1 1 1
 ‹
(a + b + c + d) + + + =k

.M
a b c d
p
is satisfied for a = (k − 11 + k2 − 22k + 81 )/5, b = 2 and c = d = 1. Since a ≥ 3 for

D
k ≥ 119/6, there exist not three numbers of a, b, c, d as lengths of the sides of a triangle.

A
PI
M
P 3.112. Let a, b, c, d and k be positive real numbers such that
LY

(a + b + c + d)2 = k(a2 + b2 + c 2 + d 2 ).
O

11
If ≤ k ≤ 4, then any three of a, b, c, d are the lengths of the sides of a triangle (non-
3
.M

degenerate or degenerate).

(Vasile Cîrtoaje, 2010)


W

Solution. The condition k ≤ 4 follows from the known inequality


W

(a + b + c + d)2 ≤ 4(a2 + b2 + c 2 + d 2 ).
W

Without loss of generality, assume that a ≥ b ≥ c ≥ d. Then, any three of a, b, c, d are


the lengths of the sides of a triangle if and only if a ≤ c + d. Let us denote x = (c + d)/2,
x ≤ b. Since c 2 + d 2 ≥ 2x 2 , from the hypothesis, we have

(a + b + 2x)2 ≥ k(a2 + b2 + 2x 2 ),

which can be written as

(k − 1)b2 − 2(a + 2x)b + k(a2 + 2x 2 ) − (a + 2x)2 ≤ 0,

a + 2x 2
 ‹
k
(k − 1) b − + k(a2 + 2x 2 ) − (a + 2x)2 ≤ 0.
k−1 k−1
284 Vasile Cîrtoaje

This involves
1
a2 + 2x 2 − (a + 2x)2 ≤ 0.
k−1
11
Since k ≥ , we get
3
3
(a2 + 2x 2 ) − (a + 2x)2 ≤ 0,
8
(a − 2x)(5a − 2x) ≤ 0,
a − b − c ≤ 0.
Thus, the proof is completed.
11

L
Remark 1. The interval [ , 4] is the largest possible. The obvious inequality

.M
3
(a + b + c + d)2 > a2 + b2 + c 2 + d 2

D
involves k > 1. For the sake of contradiction, assume that 1 < k < 11/3. It is easy to

A
check that the hypothesis
PI
(a + b + c + d)2 = k(a2 + b2 + c 2 + d 2 )
p
7 + k(66 − 17k)
M
is satisfied for a = > 4, b = 3 and c = d = 2. Since a > 4 for
k−1
1 < k < 11/3, a, c and d are not the lengths of the sides of a triangle.
LY

Remark 2. In the same manner, we can prove the following generalization.


O

• Let a1 , a2 , . . . , an be positive real numbers such that


.M

(a1 + a2 + · · · + an )2 = k(a12 + a22 + · · · + an2 ).


1
If n ≥ 3 and n − ≤ k ≤ n, then any three of a1 , a2 , . . . , an are the lengths of the sides of
W

3
a triangle.
Notice that the interval [n−1/3, n] is the largest possible of k. From the known inequal-
W

ities
W

(a1 + a2 + · · · + an )2 > a12 + a22 + · · · + an2


and
(a1 + a2 + · · · + an )2 ≤ n(a12 + a22 + · · · + an2 ),
it follows that 1 < k ≤ n. For the sake of contradiction, assume that 1 < k < n − 1/3.
The hypothesis (a1 + a2 + · · · + an )2 = k(a12 + a22 + · · · + an2 ) is satisfied for
p
3n − 5 + k[3(n − 2)(3n − 1) − k(9n − 19)]
a1 = ,
k−1
a2 = · · · = an−2 = 3 and an−1 = an = 2. Since a1 > 4, the numbers a1 , an−1 and an are
not the lengths of the sides of a triangle.
Symmetric Polynomial Inequalities in Nonnegative Variables 285

P 3.113. Let a, b, c, d and k be positive real numbers such that

(a + b + c + d)2 = k(a2 + b2 + c 2 + d 2 ).

If 49/15 ≤ k ≤ 4, then there exist three numbers of a, b, c, d which are the lengths of the
sides of a triangle (non-degenerate or degenerate).

(Vasile Cîrtoaje, 2010)

Solution. The condition k ≤ 4 follows from the known inequality

(a + b + c + d)2 ≤ 4(a2 + b2 + c 2 + d 2 ).

L
.M
Without loss of generality, assume that a ≥ b ≥ c ≥ d. We need to show that either
a ≤ b + c or b ≤ c + d. For the sake of contradiction, consider that a > b + c and

D
b > c + d. To complete the proof, it suffices to show that k < 49/15; that is,

A
49 2
(a + b2 + c 2 + d 2 ) > (a + b + c + d)2 .
PI
15
M
Notice that for a = 3, b = 2 and c = d = 1, we have a = b + c, b = c + d and
LY

49 2
(a + b2 + c 2 + d 2 ) = (a + b + c + d)2 .
15
O

Then, we apply the Cauchy-Schwarz inequality in the following manner


.M

a2 (b + c + d)2
 
(3 + 4) + ≥ (a + b + c + d)2 .
3 4
W

Thus, it suffices to show that


W

7 2 a2 (b + c + d)2
(a + b2 + c 2 + d 2 ) > + .
W

15 3 4
This is equivalent to

15
2a2 + 7(b2 + c 2 + d 2 ) > (b + c + d)2 .
4
Since

8(b2 + c 2 + d 2 ) − 3(b + c + d)2 = 5b2 − 6b(c + d) + 5(c 2 + d 2 ) − 6cd


≥ 5b2 − 6b(c + d) + (c + d)2
= (b − c − d)(5b − c − d) > 0,
286 Vasile Cîrtoaje

it is enough to prove that

21 15
2a2 + (b + c + d)2 ≥ (b + c + d)2 .
8 4

This reduces to
4a ≥ 3(b + c + d),

which is true, since

4a − 3(b + c + d) = 4(a − b − c) + (b − c − d) + 2(c − d) > 0.

L
Remark. The interval [49/15, 4] is the largest possible domain of k. In the case of the

.M
non-degenerate triangles, the interval (49/15, 4] of k is the largest possible domain. To
prove this, we see that the known inequalities

D
(a + b + c + d)2 > a2 + b2 + c 2 + d 2

A
and PI
(a + b + c + d)2 ≤ 4(a2 + b2 + c 2 + d 2 )
M
involves 1 < k ≤ 4. For the sake of contradiction, assume that 1 < k ≤ 49/15. The
LY

hypothesis
(a + b + c + d)2 = k(a2 + b2 + c 2 + d 2 )
O

p
4 + 2k(11 − 3k)
.M

is satisfied for a = , b = 2 and c = d = 1. Since a ≥ 3 for 1 <


k−1
k ≤ 49/15, there exist not three numbers of a, b, c, d as lengths of the sides of a non-
degenerate triangle.
W
W
W

P 3.114. Let a, b, c, d, e be nonnegative real numbers.

(a) If a + b + c = 3(d + e), then

4(a4 + b4 + c 4 + d 4 + e4 ) ≥ (a2 + b2 + c 2 + d 2 + e2 )2 ;

(b) If a + b + c = d + e, then

12(a4 + b4 + c 4 + d 4 + e4 ) ≤ 7(a2 + b2 + c 2 + d 2 + e2 )2 .

(Vasile Cîrtoaje, 2010)


Symmetric Polynomial Inequalities in Nonnegative Variables 287

Solution. (a) Let


E(a, b, c, d, e) = 4(a4 + b4 + c 4 + d 4 + e4 ) − (a2 + b2 + c 2 + d 2 + e2 )2 .
We will show that
E(a, b, c, d, e) ≥ E(a, b, c, d + e, 0) ≥ 0.
The left side inequality is equivalent to
d e(a2 + b2 + c 2 − 3d 2 − 3e2 − 5d e) ≥ 0.
This is true, since
1
a2 + b2 + c 2 − 3d 2 − 3e2 − 5d e ≥ (a + b + c)2 − 3d 2 − 3e2 − 5d e

L
3

.M
= 3(d + e)2 − 3d 2 − 3e2 − 5d e = d e ≥ 0.
Also, in virtue of the Cauchy-Schwarz inequality, we have

D
E(a, b, c, d + e, 0) = 4[a4 + b4 + c 4 + (d + e)4 ] − [a2 + b2 + c 2 + (d + e)2 ]2 ≥ 0.

A
The equality holds for a = b = c = d and e = 0, or for a = b = c = e and d = 0.
PI
(b) Let
M
E(a, b, c, d, e) = 7(a2 + b2 + c 2 + d 2 + e2 )2 − 12(a4 + b4 + c 4 + d 4 + e4 ).
LY

We will show that


E(a, b, c, d, e) ≥ E(a, b, c, d + e, 0) ≥ 0.
O

The left side inequality is equivalent to


.M

d e[12(d 2 + e2 ) + 11d e − 7(a2 + b2 + c 2 )] ≥ 0.


This is true, since
W

12(d 2 + e2 ) + 11d e − 7(a2 + b2 + c 2 ) ≥ 12(d 2 + e2 ) + 11d e − 7(a + b + c)2


W

= 12(d 2 + e2 ) + 11d e − 7(d + e)2


= 5(d 2 + e2 ) − 3d e ≥ 0.
W

Also, we have
1 1
E(a, b, c, d + e, 0) = E(a, b, c, a + b + c, 0)
4 4
X X X X
= a4 + 2 a b(a2 + b2 ) + 3 a2 b2 − 8a bc a
X X X X
≥ a2 b2 + 4 a2 b2 + 3 a2 b2 − 8a bc a
X X X
= 8( a2 b2 − a bc a) = 4 a2 (b − c)2 ≥ 0.
The equality holds for a = b = c = d/3 and e = 0, or for a = b = c = e/3 and d = 0.
288 Vasile Cîrtoaje

P 3.115. Let a, b, c, d, e be nonnegative real numbers such that

a + b + c + d + e = 5.

Prove that
a4 + b4 + c 4 + d 4 + e4 + 150 ≤ 31(a2 + b2 + c 2 + d 2 + e2 ).
(Vasile Cîrtoaje, 2007)
Solution. Write the inequality as
X
(a4 − 31a2 + 30a) ≤ 0,

L
or X
(1 − a) f (a) ≤ 0,

.M
where
f (a) = a3 + a2 − 30a.

D
Without loss of generality, assume that a ≥ b ≥ c ≥ d ≥ e. Since a + b ≤ 5, we have

A
PI
f (a) − f (b) = (a − b)(a2 + a b + b2 + a + b − 30)
≤ (a − b)[(a + b)2 + a + b − 30]
M
= (a − b)(a + b − 5)(a + b + 6) ≤ 0.
LY

Similarly,
f (b) − f (c) ≤ 0, f (c) − f (d) ≤ 0, f (d) − f (e) ≤ 0.
O

Since
.M

a−1≥ b−1≥ c−1≥ d −1≥ e−1


and
W

f (a) ≤ f (b) ≤ f (c) ≤ f (d) ≤ f (e),


by Chebyshev’s inequality, we get
W

X ”X — ”X —
5 (a − 1) f (a) ≤ (a − 1) f (a) = 0.
W

The equality holds for a = b = c = d = e = 1, and for (a, b, c, d, e) = (5, 0, 0, 0, 0) or


any cyclic permutation.
Remark. Similarly, we can prove the following generalization.
• If a1 , a2 , . . . , an are nonnegative real numbers such that

a1 + a2 + · · · + an = n,

then
a14 + a24 + · · · + an4 + n2 (n + 1) ≤ (n2 + n + 1)(a12 + a22 + · · · + an2 ).
Symmetric Polynomial Inequalities in Nonnegative Variables 289

P 3.116. Let a, b, c, d, e be positive real numbers such that

a2 + b2 + c 2 + d 2 + e2 = 5.

Prove that
a bcd e(a4 + b4 + c 4 + d 4 + e4 ) ≤ 5.
(Vasile Cîrtoaje, 2006)

First Solution. Without loss of generality, assume that a ≤ b ≤ c ≤ d ≤ e. First, we


prove that the expression

L
E(a, b, c, d, e) = a bcd e(a4 + b4 + c 4 + d 4 + e4 )

.M
is maximal for a = d. We need to show that

D
‚v v Œ
t a2 + d 2 t a2 + d 2
E(a, b, c, d, e) ≤ E , b, c, ,e .

A
2 2

This inequality is true if


PI
M
4ad(a4 + b4 + c 4 + d 4 + e4 ) ≤ (a2 + d 2 )[(a2 + d 2 )2 + 2b4 + 2c 4 + 2e4 ],
LY

which is equivalent to
O

(a2 + d 2 )(a − d)4 − 4a2 d 2 (a − d)2 + 2(b4 + c 4 + e4 )(a − d)2 ≥ 0.


.M

To prove this inequality, it suffices to show that

b4 + c 4 + e4 ≥ 2a2 d 2 .
W

Indeed, we have
W

b4 + c 4 + e4 − 2a2 d 2 ≥ b4 + a4 + d 4 − 2a2 d 2 = b4 + (a2 − d 2 )2 > 0.


W

Since E(a, b, c, d, e) is maximal for a = d, from a ≤ b ≤ c ≤ d it follows that E(a, b, c, d, e)


is maximal for a = b = c = d. Then, it suffices to show that the desired homogeneous
inequality
9 2
a2 + b2 + c 2 + d 2 + e2 a4 + b4 + c 4 + d 4 + e4
 
2
≥ (a bcd e)
5 5

holds for a = b = c = d = 1. Setting x = e2 , we need to show that f (x) ≥ 0 for x > 0,


where
4+ x 4 + x2
f (x) = 9 ln − ln x − 2 ln .
5 5
290 Vasile Cîrtoaje

From
9 1 4x 4(x − 1)(x − 2)2
f 0 (x) = − − = ,
4 + x x 4 + x2 x(4 + x)(4 + x 2 )
it follows that f (x) is decreasing for 0 < x ≤ 1 and increasing for x ≥ 1. Therefore,
f (x) ≥ f (1) = 0. This completes the proof. The equality holds if and only if a = b =
c = d = e = 1.
p p p p p
Second Solution. Replacing a, b, c, d, e by a, b, c, d, e, we need to show the
homogeneous inequality

a+b+c+d+e 4
p  ‹
2 2 2 2 2
a bcd e(a + b + c + d + e ) ≤ 5 ,
5

L
.M
where a, b, c, d, e are positive real numbers. According to Remark from P 3.57, it suffices
to prove this inequality for b = c = d = e = 1; that is, to show that

D
p a + 4 9/2
 ‹
a(a2 + 4) ≤ 5 .

A
5
Taking logarithms of both sides, we need to prove that f (a) ≥ 0, where
PI
f (a) = 9 ln (a + 4) − 7 ln 5 − ln a − 2 ln (a2 + 4).
M
From
LY

9 1 4a 4(a − 1)(a − 2)2


f 0 (a) = − − 2 = ,
a+4 a a +4 a(a + 4)(a2 + 4)
O

it follows that f (a) is decreasing for 0 < a ≤ 1 and increasing for a ≥ 1; therefore,
f (a) ≥ f (1) = 0.
.M

Remark. The following more general statement holds (Vasile Cirtoaje, 2006):
• If a1 , a2 , . . . , an are positive real numbers such that
W

a1 + a2 + · · · + an = n,
W

then
W

p1
(a1 a2 · · · an ) n−1 (a12 + a22 + · · · + an2 ) ≤ n.

P 3.117. Let a, b, c, d, e be positive real numbers such that

a + b + c + d + e = 5.

Prove that
1 1 1 1 1 20
+ + + + + 2 ≥ 9.
a b c d e a + b + c 2 + d 2 + e2
2

(Vasile Cîrtoaje, 2006)


Symmetric Polynomial Inequalities in Nonnegative Variables 291

Solution. Without loss of generality, assume that a ≤ b ≤ c ≤ d ≤ e. First, we prove


that the expression

1 1 1 1 20
E(a, b, c, d, e) = + + + + 2
a b c e a + b2 + c 2 + d 2 + e2
is minimal when a = d. If this is true, then E(a, b, c, d, e) is minimal when a = b = c = d,
and it suffices to prove the desired inequality for a = b = c = d, when it is equivalent
to the obvious inequality
(a − 1)2 (6a − 5)2 ≥ 0.
Therefore, it remains to show that

L
a+d a+d
 ‹

.M
E(a, b, c, d, d) ≥ E , b, c, ,e .
2 2

D
This inequality is equivalent to

A
(a − d)2 20(a − d)2
≥ 2 .
PI
ad(a + d) (a + b2 + c 2 + d 2 + e2 )[(a + d)2 + 2b2 + 2c 2 + 2e2 ]
M
Since (a − d)2 ≥ 0 and
LY

1
a2 + b2 + c 2 + d 2 + e2 ≥ (a + b + c + d + e)2 = a + b + c + d + e,
5
O

it suffices to show that


.M

(a + b + c + d + e)[(a + d)2 + 2b2 + 2c 2 + 2e2 ] ≥ 20ad(a + d).

Since
W

a + b + c + d + e ≥ a + a + a + d + d = 3a + 2d
W

and
W

(a + d)2 + 2b2 + 2c 2 + 2e2 ≥ (a + d)2 + 2a2 + 2a2 + 2d 2 = 5a2 + 2ad + 3d 2 ,

it is enough to prove that

(3a + 2d)(5a2 + 2ad + 3d 2 ) ≥ 20ad(a + d).

This is true, since

(3a + 2d)(5a2 + 2ad + 3d 2 ) − 20ad(a + d) = 15a3 − 4a2 d − 7ad 2 + 6d 3


> 5a3 − 4a2 d − 7ad 2 + 6d 3
= (a − d)2 (5a + 6d) ≥ 0.
292 Vasile Cîrtoaje

Thus, the proof is  ‹ equality holds for a = b = c = d = e = 1, and again


completed. The
5 5 5 5 5
for (a, b, c, d, e) = , , , , or any cyclic permutation.
6 6 6 6 3

Remark. The following more general statement holds (Vasile Cirtoaje, 2006):
• If a1 , a2 , . . . , an are positive real numbers such that a1 + a2 + · · · + an = n, then
p
1 1 1 2n n − 1 p
+ + ··· + + 2 ≥ n + 2 n − 1.
a1 a2 an a1 + a2 + · · · + an2
2

L
.M
P 3.118. If a, b, c, d, e ≥ 1, then

D
1 1 1 1 1
 ‹ ‹ ‹ ‹ ‹
a+ b+ c+ d+ e+ + 68 ≥

A
a b c d e

≥ 4(a + b + c + d + e)

1 1 1 1 1
+ + + +
a b c d e
PI ‹
.
M
(Vo Quoc Ba Can and Vasile Cîrtoaje, 2011)
LY

Solution. Write the inequality as E(a, b, c, d, e) ≥ 0, and denote


O

1 1 1 1
 ‹ ‹ ‹ ‹
A= a+ b+ c+ d+ .
a b c d
.M

We claim that
E(a, b, c, d, e) ≥ E(a, b, c, d, 1).
W

If this is true, then (by symmetry)


W

E(a, b, c, d, e) ≥ E(a, b, c, d, 1) ≥ ... ≥ E(a, 1, 1, 1, 1) = 0,


W

and the proof is completed. Since


 ‹
C
E(a, b, c, d, e) − E(a, b, c, d, 1) = (e − 1) B − ,
e
we need to show that
C
B− ≥ 0,
e
where
1 1 1 1
 ‹
B = A− 4 + + + ,
a b c d
C = A − 4(a + b + c + d).
Symmetric Polynomial Inequalities in Nonnegative Variables 293

From A ≥ 16 and
1 1 1 1
+ + + ≤ 4,
a b c d
it follows that B ≥ 0. If C ≤ 0, then B − C/e ≥ 0. If C ≥ 0, then

1 1 1 1
 ‹  ‹  ‹  ‹
C
B− ≥B−C =4 a− +4 b− +4 c− +4 d − ≥ 0.
e a b c d

The equality holds for a = b = c = d = 1 or any cyclic permutation.

L
.M
P 3.119. Let a, b, c and x, y, z be positive real numbers such that

(a + b + c)(x + y + z) = (a2 + b2 + c 2 )(x 2 + y 2 + z 2 ) = 4.

D
A
Prove that
1
a bc x yz <
36
. PI
M
(Vasile Cîrtoaje and Mircea Lascu, 1997)
LY

Solution. Using the given relations and the AM-GM inequality, we have

4(a b + bc + ca)(x y + yz + z x) =
O

= [(a + b + c)2 − (a2 + b2 + c 2 )][(x + y + z)2 − (x 2 + y 2 + z 2 )]


.M

= 20 − (a + b + c)2 (x 2 + y 2 + z 2 ) − (x + y + z)2 (a2 + b2 + c 2 )


W

Æ
≤ 20 − 2(a + b + c)(x + y + z) (a2 + b2 + c 2 )(x 2 + y 2 + z 2 ) = 4,
W

and hence
(a b + bc + ca)(x y + yz + z x) ≤ 1.
W

On the other hand, multiplying the well-known inequalities

(a b + bc + ca)2 ≥ 3a bc(a + b + c)

and
(x y + yz + z x)2 ≥ 3x yz(x + y + z),
we get
(a b + bc + ca)2 (x y + yz + z x)2 ≥ 36a bc x yz,
and hence
1 ≥ (a b + bc + ca)2 (x y + yz + z x)2 ≥ 36a bc x yz.
294 Vasile Cîrtoaje

In order to have 36a bc x yz = 1, it is necessary that (a b + bc + ca)2 = 3a bc(a + b + c)


and (x y + yz + z x)2 = 3x yz(x + y + z). These relations imply a = b = c and x = y = z,
which contradict the hypothesis
(a + b + c)(x + y + z) = (a2 + b2 + c 2 )(x 2 + y 2 + z 2 ) = 4.
1
Consequently, we have a bc x yz < .
36

P 3.120. Let a1 , a2 , · · · , an (n ≥ 3) be positive real numbers such that

L
a1 + a2 + · · · + an = a12 + a22 + · · · + an2 = n − 1.

.M
Prove that
1 1 1 n2 (2n − 3)
+ + ··· + ≥ .

D
a1 a2 an 2(n − 1)(n − 2)

A
(Vasile Cîrtoaje, 2010)
Solution. By the Cauchy-Schwarz inequality, we have PI
(a1 + a2 + · · · + an−1 )2
M
n − 1 = a12 + a22 + · · · + an2 ≥ + an2
n−1
LY

(n − 1 − an )2
= + an2 ,
n−1
O

which provides an ≤ 2(n − 1)/n. Similarly, ai ≤ 2(n − 1)/n for all i. The hint for
proving the given inequality is to apply the Cauchy-Schwarz inequality after we made
.M

the numerators nonnegative and as small as possible. So, since 2n − 2 − nai ≥ 0, we


have
W

X 1 X 1 n2
‹
n
= − +
a1 a1 2n − 2 2n − 2
W

1 X 2n − 2 − na1 n2
= +
W

2(n − 1) a1 2n − 2
[ (2n − 2 − na1 )]2
P
1 n2
≥ ·P +
2(n − 1) a1 (2n − 2 − na1 ) 2n − 2
[n(2n − 2) − n a1 ]2
P
1 n2
= · P 2+
2(n − 1) (2n − 2) a1 − n a1 2n − 2
P

1 n2 (n − 1)2 n2 n2 (2n − 3)
= · + = ,
2(n − 1) (n − 1)(n − 2) 2n − 2 2(n − 1)(n − 2)
from where the conclusion follows. The equality holds for a1 = a2 = · · · = an−1 =
1 − 2/n and an = 2 − 2/n (or any cyclic permutation).
Symmetric Polynomial Inequalities in Nonnegative Variables 295

P 3.121. Let a1 , a2 , · · · , an be positive real numbers such that a1 + a2 + · · · + an = n.


Prove that
1 1 1
 ‹
n2 + + ··· + − n ≥ 4(n − 1)(a12 + a22 + · · · + an2 − n).
a1 a2 an
(Vasile Cîrtoaje, 2004)
Solution. Since
1
n= (a1 + a2 + · · · + an )2 ≤ a12 + a22 + · · · + an2 < (a1 + a2 + · · · + an )2 = n2 ,
n
we can use the substitution

L
.M
a12 + a22 + · · · + an2 = n + n(n − 1)t 2 ,

where 0 ≤ t < 1. On the other hand, from

D
(a2 + · · · + an )2 (n − a1 )2

A
a12 + a22 + · · · + an2 ≥ a12 + = a12 + ,
n−1 n−1
we get
PI
(n − a1 )2
M
n + n(n − 1)t 2 ≥ a12 + ,
n−1
LY

which involves a1 ≤ 1 + (n − 1)t; similarly, we get


O

ai ≤ 1 + (n − 1)t
.M

for any i. The hint for proving the given inequality is to apply the Cauchy-Schwarz
inequality after we made the numerators nonnegative and as small as possible. Since
X 1 1 X• 1 1
˜
W

X
= + −
a1 1 + (n − 1)t a1 1 + (n − 1)t
W

n 1 X 1 + (n − 1)t − a
1
= + ,
1 + (n − 1)t 1 + (n − 1)t a1
W

we can write the desired inequality as


X 1 + (n − 1)t − a 4(n − 1)2 t 2 (1 + (n − 1)t)
1
≥ n(n − 1)t + .
a1 n
By virtue of the Cauchy-Schwarz inequality, we have

[ (1 + (n − 1)t − a1 )]2
P
X 1 + (n − 1)t − a
1
≥ P
a1 a1 (1 + (n − 1)t − a1 )
[n + n(n − 1)t − a1 ]2
P
n(n − 1)t
= P 2= ,
(1 + (n − 1)t) a1 − a1
P
1− t
296 Vasile Cîrtoaje

Therefore, it suffices to prove that

n(n − 1)t 4(n − 1)2 t 2 [(1 + (n − 1)t])


≥ n(n − 1)t + .
1− t n
This inequality is true if

4(n − 1)(1 − t)[1 + (n − 1)t]) ≤ n2 .

Indeed,

4(n − 1)(1 − t)[1 + (n − 1)t] ≤ [(n − 1)(1 − t) + 1 + (n − 1)t]2 = n2 .

L
The equality holds when a1 = a2 = · · · = an = 1, as well as when one of a1 , a2 , . . . , an is

.M
n/2 and the others are n/(2n − 2).

D
A
P 3.122. Let a1 , a2 , · · · , an be nonnegative real numbers such that a1 + a2 + · · · + an = n.
PI
Prove that
M
(n + 1)(a12 + a22 + · · · + an2 ) ≥ n2 + a13 + a23 + · · · + an3 .
(Vasile Cîrtoaje, 2002)
LY

First Solution. If a1 = a2 = · · · = an , then the equality holds. Otherwise, as in the


O

preceding proof, we will use the substitution


.M

a12 + a22 + · · · + an2 = n + n(n − 1)t 2 ,

where 0 < t ≤ 1; in addition, for any i, we have


W

ai ≤ 1 + (n − 1)t.
W

By the Cauchy-Schwarz inequality, we have


W

[ (1 + (n − 1)t − a1 )a1 ]2
P
X
2
[1 + (n − 1)t − a1 ]a1 ≥ P = n(n − 1)t(1 − t)2 ,
[1 + (n − 1)t − a1 ]

hence
X X
a13 ≤ [1 + (n − 1)t] a12 − n(n − 1)t(1 − t)2
= n[(n − 1)(n − 2)t 3 + 3(n − 1)t 2 + 1].

Therefore, it suffices to show that

(n + 1)[n + n(n − 1)t 2 ] ≥ n2 + n[(n − 1)(n − 2)t 3 + 3(n − 1)t 2 + 1],


Symmetric Polynomial Inequalities in Nonnegative Variables 297

which is equivalent to the obvious inequality

n(n − 1)(n − 2)t 2 (1 − t) ≥ 0.

For n = 2, the original inequality is an identity. For n ≥ 3, the equality holds when
a1 = a2 = · · · = an = 1, as well as when n − 1 of a1 , a2 , . . . , an are zero.
Second Solution. Assume that a1 ≥ a2 ≥ · · · ≥ an . Replacing n2 by n(a1 + a2 + · · · + an ),
the desired inequality becomes as follows
X
[(n + 1)a12 − na1 − a13 ] ≥ 0,

L
X
(a1 − 1)(na1 − a12 ) ≥ 0.

.M
Since
a1 − 1 ≥ a2 − 1 ≥ · · · ≥ an − 1

D
and

A
na1 − a12 ≥ na2 − a22 ≥ · · · ≥ nan − an2 ,
we apply Chebyshev’s inequality to get
PI
M
X ”X — ”X —
n (a1 − 1)(na1 − a12 ) ≥ (a1 − 1) (na1 − a12 ) = 0.
LY
O
.M

P 3.123. Let a1 , a2 , · · · , an be nonnegative real numbers such that a1 + a2 + · · · + an = n.


Prove that
W

(n − 1)(a13 + a23 + · · · + an3 ) + n2 ≥ (2n − 1)(a12 + a22 + · · · + an2 ).


W

(Vasile Cîrtoaje, 2002)

Solution. If a1 = a2 = · · · = an , then the equality holds. Otherwise, as in the proof of


W

problem P 3.121, we will use the substitution

a12 + a22 + · · · + an2 = n + n(n − 1)t 2 ,

where 0 < t ≤ 1; in addition, for any i, we have

ai ≥ 1 − (n − 1)t.

By the Cauchy-Schwarz inequality, we have

[ (a1 − 1 + (n − 1)t)a1 ]2
P
X
2
[a1 − 1 + (n − 1)t]a1 ≥ P = n(n − 1)t(t + 1)2 ,
[a1 − 1 + (n − 1)t]
298 Vasile Cîrtoaje

which yields
X X
a13 ≥ n(n − 1)t(t + 1)2 + [1 − (n − 1)t] a12
= n[1 + 3(n − 1)t 2 − (n − 1)(n − 2)t 3 ].

Therefore, it suffices to show that

(n − 1)n[1 + 3(n − 1)t 2 − (n − 1)(n − 2)t 3 ] + n2 ≥ (2n − 1)[n + n(n − 1)t 2 ],

which is equivalent to the obvious inequality

L
n(n − 1)(n − 2)t 2 [1 − (n − 1)t] ≥ 0.

.M
The equality holds for a1 = a2 = · · · = an = 1, and also for a1 = 0 and a2 = a3 = · · · =
n
an = (or any cyclic permutation).

D
n−1

A
P 3.124. Let a1 , a2 , . . . , an (n ≥ 3) be positive real numbers such that
PI
M
a1 + a2 + · · · + an = 1.
LY

Prove that
O

‹n
1 1 1 1
 ‹ ‹ ‹ 
a1 + −2 a2 + −2 an + −2 ≥ n+ −2 .
.M

a1 a2 an n
W

Solution. Applying Popoviciu’s inequality for the convex function f (x) = − ln x, x > 0,
gives
W

 a + a + · · · + a n(n−2)
1 2 n
(b1 b2 · · · bn )n−1 ≥ (a1 a2 · · · an ) ,
n
W

where
1 X
bi = a j , i = 1, 2, · · · , n.
n − 1 j6=i

Under the hypothesis a1 + a2 + · · · + an = 1, this inequality becomes


‹ 2
1 n −n

(1 − a1 ) n−1
(1 − a2 ) n−1
· · · (1 − an ) n−1 n
≥ n 1− a1 a2 · · · an . (*)
n
On the other hand, by the AM-GM inequality, we get in succession:
Æ
(1 − a1 ) + (1 − a2 ) + · · · + (1 − an ) ≥ n (1 − a1 )(1 − a2 ) · · · (1 − an ),
n
Symmetric Polynomial Inequalities in Nonnegative Variables 299

1 n
 ‹
1− ≥ (1 − a1 )(1 − a2 ) · · · (1 − an ).
n
1 n(n−3)
 ‹
1− ≥ (1 − a1 )n−3 (1 − a2 )n−3 · · · (1 − an )n−3 ,
n
1 n(n−3)
 ‹
1− (1 − a1 )2 (1 − a2 )2 · · · (1 − an )2 ≥ (1 − a1 )n−1 (1 − a2 )n−1 · · · (1 − an )n−1 .
n
Multiplying this inequality and (*) yields the desired inequality. The equality holds for
a1 = a2 = · · · = an = 1/n.

L
.M
P 3.125. Let a1 , a2 , . . . , an (n ≥ 3) be positive real numbers such that

a1 + a2 + · · · + an = n.

D
A
Prove that
n
a12 + a22 + · · · + an2 − n ≥ (1 − a1 a2 · · · an ).
PI
n−1
M
Solution. For fixed a12 + a22 + · · · + an2 , according to Remark from P 3.57, the product
LY

a1 a2 · · · an is minimal when one of a1 , a2 , . . . , an is zero or n−1 numbers of a1 , a2 , . . . , an


are equal. Therefore, it suffices to consider these cases.
O

Case 1: a1 = 0. We need to show that a2 + a3 + · · · + an = n involves


.M

n2
a22 + a32 + · · · + an2 ≥ .
n−1
W

Indeed, by the Cauchy-Schwarz inequality, we have


W

1 n2
a22 + a32 + · · · + an2 ≥ (a2 + a3 + · · · + an )2 = .
n−1 n−1
W

Case 2: a2 = a3 = · · · = an . Setting a1 = x and a2 = y, we need to show that

x + (n − 1) y = n

involves
n
x 2 + (n − 1) y 2 − n +

x y n−1 − 1 ≥ 0.
n−1
By Bernoulli’s inequality, we have

1 − x n−1
 ‹
y n−1 = 1 + ≥ 1 + (1 − x) = 2 − x.
n−1
300 Vasile Cîrtoaje

Therefore, it suffices to prove that


n
x 2 + (n − 1) y 2 − n + [x(2 − x) − 1] ≥ 0,
n−1
which is an identity.
The equality holds for a1 = a2 = · · · = an = 1/n, and also for a1 = 0 and a2 = · · · =
n
an = (or any cyclic permutation).
n−1

L
P 3.126. Let a1 , a2 , . . . , an (n ≥ 3) be positive real numbers such that

.M
a1 ≤ a2 ≤ · · · ≤ an ,

D
1 1 1
 ‹
(a1 + a2 + · · · + an ) + + ··· + = k.

A
a1 a2 an

(a) If n2 ≤ k ≤ n2 +
i(n − i)
2
PI
, i ∈ {2, 3, · · · , n−1}, then ai−1 , ai and ai+1 are the lengths
of the sides of a non-degenerate or degenerate triangle;
M
9n2 9n2 − 1
LY

(b) If n2 ≤ k ≤ αn , where αn = for even n, and αn = for odd n, then there


8 8
exist three numbers ai which are the lengths of the sides of a non-degenerate or degenerate
O

triangle.
.M

(Vasile Cîrtoaje, 2010)

Solution. The condition k ≥ n2 follows from the AM-HM inequality


W

1 1 1
 ‹
(a1 + a2 + · · · + an ) + + ··· + ≥ n2 .
W

a1 a2 an
W

(a) For the sake of contradiction, assume that ai−1 , ai and ai+1 are not the lengths of
the sides of a triangle; that is, ai+1 > ai−1 + ai . Let x and y be positive numbers such
that
(i − 1)x = a1 + · · · + ai−1 ,
(n − i) y = ai+1 + · · · + an ,
x ≤ ai−1 ≤ ai < ai+1 ≤ y, x < y.
Let us denote
A(x, y) = (i − 1)x + ai + (n − i) y,
i−1 1 n−i
B(x, y) = + + ,
x ai y
Symmetric Polynomial Inequalities in Nonnegative Variables 301

f (x, y) = A(x, y)B(x, y).


By the Cauchy-Schwarz inequality, we have
1 1 1
B(x, y) ≤ + + ··· + .
a1 a2 an
Since A(x, y) = a1 + a2 + · · · + an , we obtain
1 1 1
 ‹
f (x, y) ≤ (a1 + a2 + · · · + an ) + + ··· + ,
a1 a2 an
and hence
f (x, y) ≤ k.

L
On the other hand, we claim that

.M
f (x, y) > f (ai−1 , ai−1 + ai ).

D
This inequality is equivalent to

A
[ai−1 y − (ai−1 + ai )x][(ai−1 + ai ) y − ai−1 x] ≥ 0,
and is true since y ≥ ai+1 > ai−1 + ai and x ≤ ai−1 imply PI
ai−1 y − (ai−1 + ai )x > ai−1 (ai−1 + ai ) − (ai−1 + ai )ai−1 = 0.
M
Then, we have
LY

k ≥ f (x, y) > f (ai−1 , ai−1 + ai )


O

i−1 1 n−i
 ‹
= [(n − 1)ai−1 + (n − i + 1)ai ] + + .
ai−1 ai ai−1 + ai
.M

Setting ai−1 = 1 and ai = t, t ≥ 1, we get


[n − 1 + (n − i + 1)t][1 + nt + (i − 1)t 2 ] i(n − i)
W

k> ≥ n2 + ,
t(1 + t) 2
W

which contradicts the hypothesis k ≤ n2 + i(n − i)/2. The last inequality is equivalent
to (t − 1)(C t 2 + Dt + E) ≥ 0, where
W

C = 2(i − 1)(n − i + 1), D = (i − 2)(n − i), E = −2(n − 1).


This is true, since
C t 2 + Dt + E ≥ C + D + E = 3(i − 2)(n − i) ≥ 0.
i(n − i)
(b) We apply the result of (a). If n is even, then n2 + attains its maximum
2
2
9n n i(n − i) 9n2 − 1
value for i = . If n is odd, then n2 + attains its maximum value
8 2 2 8
n±1
for i = .
2
302 Vasile Cîrtoaje

P 3.127. Let a1 , a2 , . . . , an (n ≥ 3) be positive real numbers such that

a1 ≤ a2 ≤ · · · ≤ an ,

(a1 + a2 + · · · + an )2 = k(a12 + a22 + · · · + an2 ).

(2n − i)2
(a) If ≤ k ≤ n, i ∈ {2, 3, · · · , n − 1}, then ai−1 , ai and ai+1 are the lengths of
4n − 3i
the sides of a non-degenerate or degenerate triangle;
8n + 1
(b) If ≤ k ≤ n, then there exist three numbers ai which are the lengths of the
9
sides of a non-degenerate or degenerate triangle.

L
.M
(Vasile Cîrtoaje, 2010)

D
Solution. The condition k ≤ n follows from the Cauchy-Schwarz inequality

A
(a1 + a2 + · · · + an )2 ≤ n(a12 + a22 + · · · + an2 ).
PI
(a) For the sake of contradiction, assume that ai−1 , ai and ai+1 are not the lengths of
M
the sides of a triangle; that is, ai+1 > ai−1 + ai . Let x and y be positive numbers such
that
LY

(i − 1)x = a1 + · · · + ai−1 ,

(n − i) y = ai+1 + · · · + an ,
O

x ≤ ai−1 ≤ ai < ai+1 ≤ y, x < y.


.M

Let us denote
A(x, y) = (i − 1)x + ai + (n − i) y,
W

B(x, y) = (i − 1)x 2 + ai2 + (n − i) y 2 ,


W

A2 (x, y)
f (x, y) = .
W

B(x, y)
By the Cauchy-Schwarz inequality, we have

(i − 1)x 2 ≤ a12 + · · · + ai−1


2
,

(n − i) y 2 ≤ ai+1
2
+ · · · + an2 ,
and hence
B(x, y) ≤ a12 + a22 + · · · + an2 .
Since
A(x, y) = a1 + a2 + · · · + an ,
Symmetric Polynomial Inequalities in Nonnegative Variables 303

we obtain
(a1 + a2 + · · · + an )2
f (x, y) ≥ ,
a12 + a22 + · · · + an2
and hence
f (x, y) ≥ k.
On the other hand, from
∂ f (x, y) 2(i − 1)AC
= >0
∂x B2
and
∂ f (x, y) 2(n − i)AD
= < 0,

L
∂y B2

.M
where
C = ai (ai − x) + (n − i) y( y − x) > 0,

D
D = ai (ai − y) + (i − 1)x(x − y) < 0,

A
it follows that f (x, y) is strictly increasing with respect to x > 0 and strictly decreasing
PI
with respect to y > 0. Then, since x ≤ ai−1 and y ≥ ai+1 > ai−1 + ai , we have
M
f (x, y) < f (ai−1 , ai−1 + ai ).
LY

This involves
k < f (ai−1 , ai−1 + ai ),
O

and hence
.M

[(i − 1)ai−1 + ai + (n − i)(ai−1 + ai )]2


k<
(i − 1)ai−1
2
+ ai2 + (n − i)(ai−1 + ai )2
[(n − 1)ai−1 + (n − i + 1)ai ]2
W

=
(n − 1)ai−1
2
+ 2(n − i)ai−1 ai + (n − i + 1)ai2
W

(2n − i)2
≤ ,
W

4n − 3i
which is false. Setting ai−1 = 1 and ai = t, t ≥ 1, the last inequality

[n − 1 + (n − i + 1)t]2 (2n − i)2



n − 1 + 2(n − i)t + (n − i + 1)t 2 4n − 3i
is equivalent to
(t − 1)(E t − F ) ≥ 0,
where
E = (n − i + 1)[(3i − 4)n − 2i 2 + 3i],
F = (n − 1)[(4 − i)n + i 2 − 3i].
304 Vasile Cîrtoaje

Since
E = (n − i + 1)[(3i − 4)(n − i + 1) + i 2 + 2i − 4] ≥ 0,
we get
E t − F ≥ E − F = 2(i − 2)(n − i)(2n − i) ≥ 0.
(b) According to (a), it suffices to show that there exists i ∈ {2, 3, · · · , n − 1} such that

(2n − i)2 8n + 1
≤ ;
4n − 3i 9
that is,
4n − 3i ≥ (2n − 3i)2 .

L
.M
2n − 1 2n 2n + 1
Since one of the numbers , and is integer, it suffices to prove this in-
3 3 3
2n − 1 2n 2n + 1

D
equality for all i ∈ { , , }. Indeed, for these cases, the inequality reduces
3 3 3
to 2n ≥ 0, 2n ≥ 0 and 2n − 2 ≥ 0, respectively.

A
PI
M
LY
O
.M
W
W
W
Bibliography

[1] Andreescu T., Cîrtoaje V., Dospinescu G., Lascu M., Old and New Inequalities, GIL

L
Publishing House, 2004.

.M
[2] Bin X., Boreico I., Can V.Q.B., Bulj A., Lascu M., Opympiad Inequalities, GIL Pub-
lishing House, 2015.

D
A
[3] Bin X., Boreico I., Can V.Q.B., Cîrtoaje V., Lascu M., An Introduction to Inequalities,
GIL Publishing House, 2015. PI
[4] Can V.Q.B., Pohoaţă C., Old and New Inequalities, GIL Publishing House, 2008.
M
[5] Can V.Q.B., Anh T. Q., Su Dung Phuong Phap Cauchy-Schwarz De Chung Minh Bat
LY

Dang Thuc, Nha Xuat Ban Dai Hoc Su Pham, 2010.


O

[6] Cîrtoaje V., Two Generalizations of Popoviciu’s Inequality, Crux Mathematicorum,


Issue 5, 2005, 313 - 318.
.M

[7] Cîrtoaje V., The Equal Variable Method, Journal of Inequalities In Pure and Applied
Mathematics, Volume 7, Issue 3, 2006.
W

[8] Cîrtoaje V., Arithmetic Compensation Method, Math. Reflections, 2, 2006, 1 - 5.


W

[9] Cîrtoaje V., Algebraic Inequalities-Old and New Methods, GIL Publishing House,
W

2006.

[10] Cîrtoaje V., Four Applications of RCF and LCF Theorems, Math. Reflections, 1, 2007.

[11] Cîrtoaje V., On Jensen Type Inequalities with Ordered Variables, Journal of Inequal-
ities In Pure and Applied Mathematics, Volume 9, Issue 1, 2008.

[12] Cîrtoaje V., The Proof of Three Open Inequalities, Crux Mathematicorum, Volume
34, Issue 4, 2008, 231-238.

[13] Cîrtoaje V., Can V. Q. B., Anh T. Q., Inequalities with Beautiful Solutions, GIL Pub-
lishing House, 2009.

305
306 Vasile Cîrtoaje

[14] Cîrtoaje V., On Some Inequalities with Power-Exponential Functions, Journal of In-
equalities In Pure and Applied Mathematics, Volume 10, Issue 1, 2009.

[15] Cîrtoaje V., On the Cyclic Homogeneous Polynomial Inequalities of Degree Four, Jour-
nal of Inequalities in Pure and Applied Mathematics, Volume 10, Issue 3, 2009.

[16] Cîrtoaje V., The Best Lower Bound Depended on Two Fixed Variables for Jensen’s In-
equality with Ordered Variables, Journal of Inequalities and Applications, Volume
2010.

[17] Cîrtoaje V., Baiesu A., An Extension of Jensen’s discrete inequality to half convex
functions, Journal of Inequalities and Applications, Volume 2011.

L
.M
[18] Cîrtoaje V., Proofs of Three Open Inequalities with Power-Exponential Functions,
Journal of Nonlinear Sciences and Applications, Volume 4, no. 2, 2011.

D
[19] Cîrtoaje V., The Best Upper Bound for Jensen’s Inequality, Australian Journal of

A
Mathematical Analysis and Aplications, Volume 7, Issue 2, Art. 22, 2011.
PI
[20] Cîrtoaje V., Zhou Y., Necessary and Sufficient Conditions for Cyclic Homogeneous
M
Polynomial Inequalities of Degree Four in Real Variables, The Australian Journal of
Mathematical Analysis and Applications, Volume 9, Issue 1, Article 15, 2012.
LY

[21] Cîrtoaje V., Zhou Y., Some Strong Sufficient Conditions for Cyclic Homogeneous Poly-
O

nomial Inequalities of Degree Four in Real Variables, Journal of Nonlinear Analysis


and Applications, Volume 2012.
.M

[22] Cîrtoaje V., Can V.Q.B., On Some Cyclic Homogeneous Polynomial Inequalities of De-
gree Four in Real Variables, International Journal of Pure and Applied Mathematics,
W

Volume 80, No. 3, 2012.


W

[23] Cîrtoaje V., On the Arithmetic Compensation Methods, International Journal of Pure
and Applied Mathematics, Volume 80, No. 3, 2012.
W

[24] Cîrtoaje V., Necessary and Sufficient Conditions for Symmetric Homogeneous Poly-
nomial Inequalities of Degree Four and Six in Real Variables, Journal of Nonlinear
Science and Applications, Volume 5, Issue 5, 2012.

[25] Cîrtoaje V., Necessary and Sufficient Conditions for Symmetric Homogeneous Poly-
nomial Inequalities in Nonnegative Real Variables, Mathematical Inequalities and
Applications, Volume 16, No. 2, 2013.

[26] Cîrtoaje V., The Best Lower Bound for Jensen’s Inequality with three fixed ordered
variables Journal, Banach Journal of Mathematical Analysis, Volume 7, Issue 1,
2013.
Symmetric Polynomial Inequalities in Nonnegative Variables 307

[27] Cîrtoaje V., An Extension of Jensen’s discrete inequality to partially convex functions,
Journal of Inequalities and Applications, Volume 2013:54.

[28] Cîrtoaje V., A Strong Method for Symmetric Homogeneous Polynomial Inequalities
of Degree Six in Nonnegative Real Variables, British Journal of Mathematical and
Computers Science, 4(5), 2014.

[29] Cvetkovski, Inequalities: Theorems, Techniques and Selected Problems, Springer-


Verlag Berlin Heidelberg, 2012.

[30] Drimbe M.O., Inegalitati - Idei si Metode, GIL Publishing House, 2003.

L
[31] Engel A., Problem-Solving Strategies, Springer, 1998.

.M
[32] Hung P. K., Secrets in Inequalities, Volume 1: Basic Inequalities, GIL Publishing
House, 2007.

D
[33] Hung P. K., Secrets in Inequalities, Volume 2: Advanced Inequalities, GIL Publishing

A
House, 2008.
PI
[34] Lee Hojoo, Topics in Inequalities-Theorems and Techniques,
M
http://www.artofproblemsolving.com/Forum/viewtopic.php?t=602698
LY

[35] Littlewood G. H., Polya, J. E., Inequalities, Cambridge University Press, 1967.
O

[36] Mitrinovic̆ D. S., Pecaric̆ J. E., Fink A. M., Classical and New Inequalities in Analysis,
Kluwer, 1993.
.M

[37] Panaitopol L., Băndilă V., Lascu M., Inegalităţi, GIL Publishing House, 1995.

[38] Zhou Y., Cîrtoaje V., Some strong sufficient conditions for cyclic homogeneous poly-
W

nomial inequalities of degree four in nonnegative variables, Journal of Nonlinear


W

Science and Applications, Volume 6, Issue 2, 2013.


W
308 Vasile Cîrtoaje

L
D
.M
A
PI
M
LY
O
.M
W
W
W
Chapter 4

Bibliography

L
D
.M
A
PI
M
LY
O
.M
W
W
W

309

Vous aimerez peut-être aussi